Sunteți pe pagina 1din 164

w

w
w
.y
ou
rs
m
ah
bo
ob
.w
or
dp
re
ss
.c
om
Cover:
F SSC CGL Tier I Exam
F SSC CHSL Tier-I Exam
F SSC Sub Inspector Exam
F SSC Multi-Tasking Exam
F SSC Stenographer Exam
F Multiple Sitting
w
w
w
.y
ou
rs
m
ah
• Head Office : B-32, Shivalik Main Road, Malviya Nagar, New Delhi-110017

bo
ob
• Sales Office : B-48, Shivalik Main Road, Malviya Nagar, New Delhi-110017

.w
or
Tel. : 011-26692293 / 26692294

dp
re
ss
.c
om
Typeset by Disha DTP Team

DISHA PUBLICATION
ALL RIGHTS RESERVED

© Copyright Publisher

No part of this publication may be reproduced in any form without prior permission of the publisher. The author and the
publisher do not take any legal responsibility for any errors or misrepresentations that might have crept in. We have tried
and made our best efforts to provide accurate up-to-date information in this book.

For further information about books from DISHA,


Log on to www.dishapublication.com or email to info@dishapublications.com
w
w
w
.y
ou
rs
m
ah
bo
CONTENTS

ob
.w
or
dp
re
ss
.c
om
1. History 1-14
2. Geography 15-28
3. Polity 29-40
4. Economics 41-54
5. General Science 55-88
6. Computers 89-94
7. General Knowledge 95-110

PRACTICE SETS
Practice Set-1 PS-1-3

Practice Set-2 PS-4-6

Practice Set-3 PS-7-9

Practice Set-4 PS-10-12

Practice Set-5 PS-13-16

Current Affairs & GK Update GK-1-34


om
.c
ss
re
dp
or
.w
ob
bo
ah
m
rs
ou
.y
w
w
w
w
w
w
.y
ou
rs
m
1

ah
bo
ob
.w
HISTORY

or
dp
re
ss
CHAPTER

.c
om
1. Arrange the following in chronological order : 12. Simon Commission was boycotted by the nationalist leaders
(SSC CGL 1st Sit. 2010) of India because: (SSC CGL 1st Sit. 2011)
1. Tughlaqs 2. Lodis (a) they felt that it was only an eyewash
3. Saiyids 4. Ilbari Turks (b) all the members of the Commission were English
5. Khiljis (c) the members of the Commission were biased against
(a) 1, 2, 3, 4, 5 (b) 5, 4, 3, 2, 1 India
(c) 2, 4, 5, 3, 1 (d) 4, 5, 1, 3, 2 (d) it did not meet the demands of the Indians
2. Who was the founder of The Servants of India Society? 13. Who among the following British persons admitted the
(SSC CGL 1st Sit. 2010) Revolt of 1857 as a national revolt?
(a) G..K. Gokhale (b) M.G. Ranade (SSC CGL 1st Sit. 2011)
(c) B.G. Tilak (d) Bipin Chandra Pal (a) Lord Dalhousie (b) Lord Canning
3. Mahatma Gandhi was profoundly influenced by the writings (c) Lord Ellenborough (d) Disraeli
of (SSC CGL 1st Sit. 2010) 14. Which of the following is called the 'shrimp capital of India'?
(a) Bernard Shaw (b) Karl Marx (SSC CGL 1st Sit. 2011)
(c) Lenin (d) Leo Tolstoy (a) Mangalore (b) Nagapatnam
4. The monk who influenced Ashoka to embrace Buddhism (c) Kochi (d) Nellore
was (SSC CGL 2nd Sit. 2010)
15. Mention the place where Buddha attained enlightenment
(a) Vishnu Gupta (b) Upa Gupta
(SSC CGL 2nd Sit. 2011)
(c) Brahma Gupta (d) Brihadratha
(a) Sarnath (b) Bodh Gaya
5. The Lodi dynasty was founded by
(SSC CGL 2nd Sit. 2010) (c) Kapilavastu (d) Rajgriha
(a) Ibrahim Lodi (b) Sikandar Lodi 16. Coronation of Shivaji took place in
(c) Bahlol Lodi (d) Khizr Khan (SSC CGL 2nd Sit. 2011)
6. Harshvardhana was defeated by (SSC CGL 2nd Sit. 2010) (a) 1627 A.D. (b) 1674 A.D.
(a) Prabhakaravardhana (c) 1680 A.D. (d) 1670 A.D.
(b) Pulakesin II 17. The System of Dyarchy was introduced in India in
(c) Narasimhasvarma Pallava (SSC CGL 2nd Sit. 2011)
(d) Sasanka (a) 1909 (b) 1935
7. Who among the following was an illiterate ? (c) 1919 (d) 1945
(SSC CGL 2nd Sit. 2010) 18. The Editor of 'Young India' and 'Harijan' was
(a) Jahangir (b) Shah Jahan (SSC CGL 2nd Sit. 2011)
(c) Akbar (d) Aurangazeb (a) Nehru (b) Ambedkar
8. Which Governor General is associated with Doctrine of Lapse ? (c) Mahatma Gandhi (d) Subash Chandra Bose
(SSC CGL 2nd Sit. 2010) 19. Who of the following attended all the Three Round Table
(a) Lord Ripon (b) Lord Dalhousie
Conferences? (SSC CGL 2nd Sit. 2011)
(c) Lord Bentinck (d) Lord Curzon
(a) B.R. Ambedkar (b) M.M. Malavia
9. The Greek ambassador sent to Chandragupta Maurya's Court
was : (SSC CGL 1st Sit. 2011) (c) Vallabhbhai Patel (d) Gandhiji
(a) Kautilya (b) Seleucus Nicator 20. Which king is referred to as Devanampiya Piyadassi (Beloved
(c) Megasthenes (d) Justin of the Gods) in the inscriptions? (SSC CGL 1st Sit. 2012)
10. Identify the European power from whom Shivaji obtained (a) Asoka (b) Harsha
cannons and ammunition: (SSC CGL 1st Sit. 2011) (c) Bindusara (d) Chandragupta Maurya
(a) The French (b) The Portuguese 21. Name the Maratha Saint who was a contemporary of Shivaji.
(c) The Dutch (d) The English (SSC CGL 1st Sit. 2012)
11. The call of "Back to the Vedas" was given by: (a) Saint Eknath (b) Saint Tukaram
(SSC CGL 1st Sit. 2011) (c) Saint Dhyaneshwar (d) Namdev
(a) Swami Vivekananda 22. Which of the following cereals was among the first to be
(b) Swami Dayanand Saraswati used by man? (SSC CGL 1st Sit. 2012)
(c) Aurobindo Ghosh (a) Rye (b) Wheat
(d) Raja Ram Mohan Roy (c) Barley (d) Oat
w
w
w
.y
ou
2 History

rs
m
23. The treaty of Versailles restored Alsace-Lorraine to: 36. The original name of Nana Phadnavis was

ah
(SSC CGL 2nd Sit. 2012) (SSC CGL 2nd Sit. 2012)

bo
ob
(a) Italy (b) Britain (a) Mahadaji Sindhia (b) Tukoji Holkar

.w
(c) France (d) Belgiun (c) Narayan Rao (d) Balaji Janardan Bhanu

or
dp
24. The Asokan Edicts were deciphered first by: 37. Who among the following first propounded the idea of Basic

re
(SSC CGL 2nd Sit. 2012) Education? (SSC CGL 2nd Sit. 2012)

ss
(a) Sir John Marshall (b) Sir William Jones (a) Jawahar Lal Nehru

.c
om
(c) Charles Wilkins (d) James Princep (b) Raja Ram Mohan Roy
25. Who favoured the Artic Home thoery of the Aryans? (c) Mahatma Gandhi
(SSC CGL 2nd Sit. 2012) (d) Dayanand Saraswati
(a) Pargiter (b) A. C. Das 38. Arrange the following in chronological order:
(c) B. G. Tilak (d) Jacobi (SSC CGL 2nd Sit. 2012)
26. Who was the teacher of Gautama Buddha? I. Dandi March
(SSC CGL 2nd Sit. 2012) II. Simon Commission
(a) Panini (b) Alara Kalama III. Poona Pact
(c) Kapila (d) Patanjali IV. Gandhi Irwin Pact
27. The prose collection of the vedic poems are: (a) II, I, III, IV (b) II, I, IV, III
(c) IV, III, I, II (d) IV, III, II, I
(SSC CGL 2nd Sit. 2012)
39. The city of Prayag was named Allahabad - the city of Allah
(a) Samhitas (b) Upanishads
by (SSC CGL 2nd Sit. 2012)
(c) Aranyakas (d) Brahmanas
(a) Aurangzeb (b) Akbar
28. Non-violence as taught and practised by Mahatma Gandhi
(c) Shahjahan (d) Bahadur Shah Zafar
is rooted in the Indian Doctrine of 40. Who gave the title of "Mahamana" to Madan Mohan
(SSC CGL 1st Sit. 2012) Malviya ? (SSC CGL 1st Sit. 2013)
(a) Swaraj (b) Swadeshi (a) Bal Gandadhar Tilak
(c) Satyagraha (d) Ahimsa (b) Dada Bhai Naurozi
29. Who was the founder of the Aligarh Movement ? (c) Gopal Krishna Gokhale
(SSC CGL 1st Sit. 2012) (d) Mahatma Gandhi
(a) Syed Amir Ali 41. Who was the Viceroy of the time of Quit India Movement ?
(b) Maulvi Chiragh Ali (SSC CGL 1st Sit. 2013)
(c) Sir Syed Ahmed Khan (a) Lord Irwin
(d) Abdul Halim Sharar (b) Lord Mountbatten
30. ‘Prince of Pilgrims’ was the name attributed to (c) Lord Wavell
(SSC CGL 1st Sit. 2012) (d) Lord Lin Lithgow
(a) Plutarch (b) Hiuen Tsang 42. Who was the other Congress leader who joined with Motilal
(c) Fa-Hien (d) I-Tsing Nehru to start the Swaraj Party in 1923?
31. Where did Buddha deliver his first sermon ? (SSC CGL 1st Sit. 2013)
(SSC CGL 1st Sit. 2012) (a) G.K. Gokhale (b) B.G. Tilak
(a) Sarnath (b) Rajagriha (c) Chittaranjan Das (d) M.K. Gandhi
(c) Kapilavastu . (d) Bodh-Gaya 43. What is the first sermon of Buddha called as ?
32. Name the Mughal Prince, who translated Bhagavat Gita into (SSC CGL 1st Sit. 2013)
Persian ? (SSC CGL 1st Sit. 2012) (a) Mahaparinivansutta
(a) Dara Shukoh (b) Sulaiman Shukoh (b) Brahmajalasutta
(c) Khusru (d) Murad (c) Dhammachakkapabattanasutta
33. The surgery that was practised in ancient India is known
(d) Kachchayanagottasutta
from the works of which of the following scholars ?
44. From which of the following banks did Madan Mohan
(SSC CGL 1st Sit. 2012) Malaviya take loans for financing "The Hindustan Times"?
(a) Atreya (b) Sushruta (SSC CGL 1st Sit. 2013)
(c) Charaka (d) Vagbhata (a) State Bank of India
34. Who among the following was the First Viceroy of India? (b) Punjab National Bank
(SSC CGL 2nd Sit. 2012) (c) Bank of Maharashtra
(a) Lord Ripon (b) Lord Curzon (d) Bank of Baroda
(c) Lord Mountbatten (d) Lord Canning 45. Where are the Todas found ? (SSC CGL 1st Sit. 2013)
35. Prithvi Raj Chauhan was defeated in the Second Battle of (a) Madhya Pradesh
Tarain by (SSC CGL 2nd Sit. 2012) (b) Tamil nadu
(a) Mahmud Ghazni (b) Muhammad Ghori (c) Rajasthan
(c) Qutbuddin Aibak (d) Yalduz (d) Aruncachal Pradesh
w
w
w
.y
ou
History 3

rs
m
46. Rowlatt Act 1919 was enacted during the period of 57. Which of the following movements saw the biggest peasant

ah
bo
(SSC CGL 2nd Sit. 2013) guerilla war on the eve of independence?

ob
(a) Lord Chelmford (b) Lord William (SSC CGL 1st Sit. 2013)

.w
(c) Lord Minto (d) Lord Bentinck (a) Punnapra Vayalar Movement

or
(b) Telangana Movement

dp
47. Panchayati Raj System was implemented first in the pair of
(c) Noakhali Movement

re
states (SSC CGL 2nd Sit. 2013)

ss
(d) Tebhaga Movement

.c
(a) Andhra Pradesh and Rajasthan 58. The founder of the Lodi Dynasty was

om
(b) Assam and Bihar (SSC CGL 1st Sit. 2013)
(c) Arunachal Pradesh and Uttar Pradesh (a) Bahlol Lodi
(d) Punjab and Chandigarh (b) Sikandar Shah Lodi
48. Which of the following tribes is associated with the “Tana (c) Jalal Khan Lodi
(d) Ibrahim Lodi
Bhagat” movement? (SSC CGL 2nd Sit. 2013)
59. The Gandharva School of art is also known as the
(a) Uraon (b) Munda (SSC CGL 1st Sit. 2013)
(c) Santhal (d) Kondadora (a) Buddhist-Roman art
49. Who founded the Naujawan Bharat Sabha? (b) Dravidian-Roman art
(a) B. C. Pal (c) Greco-Roman art
(b) G. Subramania Iyer (d) Greco-Roman-Buddhist art
(c) Sardar Bhagat Singh 60. The Bolshevik Revolution of Russia symbolized following:
(SSC CGL 1st Sit. 2013)
(d) Rukmani Lakshmipathi
(a) coming of the Communist Rule
50. The Narendra Mandal or Chamber of Princes was (b) establishment of Republic of Russia
inaugurated in 1921 by (SSC CGL 2nd Sit. 2013) (c) establishment of Democracy in Russia
(a) Lord Curzon (d) overthrow of Romanov Dynasty
(b) Lord Wellesley 61. Which General, who commanded the British forces against
(c) Duke of Cannaught the Americans in their War of Independence later became
(d) Duke of Wellingdon Governor-General of India ? (SSC CGL 1st Sit. 2013)
(a) Dalhousie (b) William Bentinck
51. Buddha, Dhamma and Sangha together are known as
(c) Wellesley (d) Cornwallis
(SSC CGL 2nd Sit. 2013) 62. Who established the Indian Civil Liberties Union in 1936?
(a) Triratna (b) Trivarga (SSC CGL 1st Sit. 2013)
(c) Trisarga (d) Trimurti (a) Subhash Chandra Bose
52. Who was called Lichchavi Dauhitra? (b) Bal Gangadhar Tilak
(SSC CGL 2nd Sit. 2013) (c) Jawahar Lal Nehru
(a) Chandragupta I (b) Skandagupta (d) Rajendra Prasad
63. Which of the following was established first?
(c) Kumaragupta (d) Samudragupta
(SSC CGL 1st Sit. 2013)
53. Pulakesin II was the greatest ruler of the (a) Banaras Hindu University
(SSC CGL 1st Sit. 2013) (b) University of Bombay
(a) Chalukyas of Kalyani (c) Aligarh Muslim University
(b) Pallavas of Kanchi (d) University of Allahabad
(c) Cholas of Tamil Nadu 64. The first Indian Satellite Aryabhatta was launched in
(d) Chalukyas of Badami (SSC CGL 1st Sit. 2013)
(a) 1972 (b) 1975
54. The Uttaramerur inscription provides information on the
(c) 1977 (d) 1979
administration of the (SSC CGL 1st Sit. 2013) 65. Where did Aurangzeb die? (SSC CGL 1st Sit. 2013)
(a) Pallavas (b) Cholas (a) Pune (b) Aurangabad
(c) Chalukyas (d) Satavahanas (c) Ahmad Nagar (d) Mumbai
55. Pitts India Act of 1784 was a/an (SSC CGL 1st Sit. 2013) 66. Who gave the title of "Sardar" to Ballabh Bhai Patel?
(a) Ordinance (b) Resolution (SSC CGL 1st Sit. 2013)
(c) White paper (d) Regulation Act (a) Mahatma Gandhi (b) Vinoba Bhave
56. Which one of the following statements is not true in respect (c) Women of Bardoli (d) Peasants of Gujrat
67. What Satyagraha was held at Nagpur in 1923?
of A.O. Hume? (SSC CGL 1st Sit. 2013)
(SSC CGL 1st Sit. 2013)
(a) He founded the Indian National Congress.
(a) Salt Satyagraha
(b) He presided over the Congress Annual Sessions twice. (b) Individual Satyagraha
(c) He was an ornithologist. (c) Ryots Satyagraha
(d) He was a member of the Indian Civil Service. (d) Flag Satyagraha
w
w
w
.y
ou
4 History

rs
m
68. Which one of the following is not a sect of Buddhism? 80. The South East trade winds are attracted towards the Indian

ah
(SSC CGL 1st Sit. 2013)

bo
sub continent in the rainy season due to

ob
(a) Mahayana (b) Hinayana (SSC CGL 1st Sit. 2015)

.w
(c) Digambar (d) Theravad (a) the effect of easterlies

or
(b) the effect of Northern–East trade winds

dp
69. Who was the viceroy when Delhi became the capital of British

re
India? (SSC CGL 1st Sit. 2013) (c) the presence of low atmospheric pressure over North–

ss
(a) Lord Curzon (b) Lord Minto West India

.c
81. What is 'Reformation'? (SSC CGL 1st Sit. 2015)

om
(c) Lord Hardinge (d) Lord Waveli
70. Multan was named by the Arabs as (SSC CGL 2014) (a) Revival of classical learning
(a) City of beauty (b) City of wealth (b) The revolt against authority of pope
(c) City of gold (d) Pink city (c) Rise of absolute monarchy
71. Which one of the following was the book written by (d) Change in attitude of man
82. Swaraj is my Birth Right and I shall have it. This was
Amoghvarsha, the Rashtrakuta King? (SSC CGL 2014)
advocated by : (SSC CGL 1st Sit. 2015)
(a) Adipurana
(a) Mahatma Gandhi (b) Lala Lajpat Rai
(b) Ganitasara Samgraha
(c) Sardar Patel (d) Lokmanya Tilak
(c) Saktayana
83. Gandhiji's Famous Quit India Movement call to the British
(d) Kavirajamarga
was given in : (SSC CGL 1st Sit. 2015)
72. Who built the Kailasanatha Temple at Ellora?
(a) 1940 (b) 1942
(SSC CGL 2014)
(c) 1941 (d) 1943
(a) Rajendra I
84. In which year was the Indian National Congress formed :
(b) Mahendra Varman I
(SSC CGL 1st Sit. 2015)
(c) Krishna I
(a) 1901 (b) 1835 (c) 1875 (d) 1885
(d) Govinda I
85. Bangladesh was created in : (SSC CGL 1st Sit. 2015)
73. The land measures of the Second Pandyan Empire was
(a) 1973 (b) 1970 (c) 1972 (d) 1971
mentioned in (SSC CGL 2014)
86. Raja Ram Mohan Roy was the founder of :
(a) Thalavaipuram Copper Plates
(SSC CGL 1st Sit. 2015)
(b) Uttirameru Inscription
(a) Brahmo Samaj
(c) Kudumiyammalai Inscription
(b) Prathna Samaj
(d) Kasakudi Copper Plates
(c) Ram Krishna Mission
74. Who was the greatest ruler of the Satavahanas?
(d) Arya Samaj
(SSC CGL 2014)
87. Who initiated the movement to form the Indian National
(a) Satkarni I
Congress : (SSC CGL 1st Sit. 2015)
(b) Gautamiputra Satkarni
(a) Annie Besant (b) A.O. Hume
(c) Simuka
(c) W.C. Banerjee (d) Gandhi ji
(d) Hala
88. Kamarup is an ancient name of which region of India?
75. The greatest king of the Pratihara dynasty was
(SSC CGL 1st Sit. 2016)
(SSC CGL 1st Sit. 2015)
(a) Bihar (b) Rajasthan
(a) Bhoj (Mihir–Bhoj) (b) Dantidurga (c) Karnataka (d) Assam
(c) Nagbhatta II (d) Vatsaraj 89. Which battle led to the downfall of the Vijayanagar empire?
76. In 1939 Subhash Chandra Bose was elected as President of (SSC CGL 1st Sit. 2016)
the Congress Party defeating (SSC CGL 1st Sit. 2015) (a) Battle of Takkolam (b) Battle of Talikota
(a) Jawaharlal Nehru (c) Battle of Kanwah (d) Battle of Panipat
(b) Maulana Abul Kalam Azad 90. Who introduced Mansabdari system in India?
(c) V.B. Patel (SSC CGL 1st Sit. 2016)
(d) Pattabhi Sitharamayya (a) Babur (b) Humayun
77. Jallianwala incident took place at (SSC CGL 1st Sit. 2015) (c) Akbar (d) Jahangir
(a) Lucknow (b) Surat 91. The ‘Cabinet Mission’ of 1946 was led by 1946
(c) Amritsar (d) Allahabad
(SSC CGL 1st Sit. 2016)
78. Who was the founder of Lodhi dynasty ?
(a) Lord Linlithgow (b) Lord Mountbatten
(a) Sikandar Lodhi (b) Bahlol Lodhi
(c) Sir Pethic Lawrence (d) Sir Mountford
(c) Ibrahim Lodhi (d) Daulat Khan Lodhi
92. What did Gandhiji meant by ‘Sarvodaya’?
79. Which one of the following pair is not correctly matched ?
(SSC CGL 1st Sit. 2016)
(SSC CGL 1st Sit. 2015)
(a) Non-violence
(a) Akbar – Todarmal
(b) Upliftment of untouchables or dalits
(b) Chanakya – Chandragupta
(c) The birth of a new society based on ethical values
(c) Vikramaditya – Chaitanya
(d) Satyagraha
(d) Harshvardhan – Hiuen Tsang
w
w
w
.y
ou
History 5

rs
m
93. Harsha moved his capital from _____ to ____. 104. Which one of the following is the most lasting contribution

ah
(SSC CGL 1st Sit. 2016) of the Rastrakutas ? (SSC CHSL 2014)

bo
ob
(a) Thanesar, Kanauj (b) Delhi, Deogiri (a) Kailasha Temple

.w
(c) Kamboj, Kanauj (d) Valabhi, Delhi (b) Pampa, Ponna, Ranna, the three writers of Kannada

or
Poetry and Kailasha Temple

dp
94. A collective term used by the Jains for their sacred books is
(c) Patronage of Jainism

re
(SSC CGL 1st Sit. 2016) (d) Conquests

ss
(a) Prabandhas (b) Agamas

.c
105. Ravikirti, a Jain, who composed the Aihole Prashasti, was

om
(c) Nibandhas (d) Charits patronized by (SSC CHSL 2014)
95. Who propounded the "Doctrine of Passive Resistance"? (a) Pulakeshi I (b) Harsha
(SSC CGL 1st Sit. 2016) (c) Pulakeshi II (d) Kharavela
(a) Balgangadhar Tilak 106. When did the reign of Delhi Sultanate came to an end ?
(SSC CHSL 2014)
(b) Aurobindo Ghosh
(a) 1498 A.D. (b) 1526 A.D.
(c) Lala Lajpat Rai
(c) 1565 A. D. (d) 1600 A.D.
(d) Bipin Chandra Pal 107. The original founder of the Manuscripts and Editor of
96. The first Muslim to be elected President of 'Indian National Kautilya's Arthashastra was (SSC CHSL 2014)
Congress' was? (SSC CGL 1st Sit. 2016) (a) Srikanta Shastri (b) Srinivasa lyangar
(a) Maulana Azad (b) Mohammed Ali (c) R. Shamashastri (d) William Jones
(c) Badruddin Tyabji (d) Shah Wali-ullah 108. Whose army did Alexander, the Greek ruler confront on the
97. The Dandi March of Gandhi-is an example of banks of the river Jhelum ? (SSC CHSL 2015)
(a) Ambi (b) Chandragupta Maurya
(SSC CHSL 2013) (c) Porus (d) Dhanananda
(a) Non-Coopefation (b) Direct Action 109. When was RTI Act enacted in India ? (SSC CHSL 2015)
(c) Boycott (d) Civil Disobedience (a) 15th August 2005 (b) 15th March 2005
th
(c) 15 June 2005 (d) 15th July 2005
98. Which one of the following inscriptions relate to the
Chalukya king, Pulakesin II ? (SSC CHSL 2013) 110. The famous activist Medha Patakar is associated with which
movement ? (SSC CHSL 2015)
(a) Nasik (b) Maski (a) Narmada Bachao Andolan
(c) Hathigumpha (d) Aihole (b) Save the Tiger
99. Who among the following introduced the Mansabdari (c) Preserve the wet lands
system ? (SSC CHSL 2013) (d) Beti Padao Andolan
111. Who built the "Purana Quilla"? (SSC CHSL 2015)
(a) Jahangir (b) Shah Jahan
(a) Babar (b) Shershah
(c) SherShah (d) Akbar (c) Aurangzeb (d) Akbar
100. Which one of the following cities and the personalities 112. Where did Chandragupta maurya spent his last days ?
associated with their establishment is wrongly matched ? (SSC CHSL 2015)
(SSC CHSL 2013) (a) Thaneshwar (b) Kanchi
(a) Calcutta - Robert Clive (c) Patliputra (d) Shravanabelagola
113. Who is the author of 'Indica'? (SSC CHSL 2015)
(b) Pondicherry - Francis Martin (a) Fa–Hien (b) Hiuen Tsang
(c) Ahmedabad Ahmad Shah I (c) Megasthanes (d) Seleucus
(d) Madras - Francis Day 114. Who built the famous Shiva temple at Ellora ?
101. Which one of the following wars decided the fate of the (SSC CHSL 2015)
French in India ? (SSC CHSL 2013) (a) Mauryan Emperor Ashoka
(b) Gupta King Samudra Gupta
(a) Battle of Plassey
(c) Chalukyan King Pulikeshi II
(b) Battle of Wandiwash
(d) Rashtrakuta Ruler Krishna I
(c) First Carnatic War 115. Who composed the Allahabad Pillar inscription ?
(d) Battle of Buxar (SSC CHSL 2015)
102. The Crimean War came to an end by the (a) Mahasena (b) Veerasena
(SSC CHSL 2013) (c) Vishnusena (d) Harisena
(a) Treaty of St. Germain 116. Todar Mal, the brilliant revenue officer served under :
(b) Treaty of Trianon (SSC CHSL 2015)
(c) Treaty of Versailles (a) Bhagwan Das (b) Humayun
(d) Treaty of Paris (c) Baz Bahadur (d) Sher Shah
103. Tulsidas wrote Ramcharitmanas in the reign of 117. When was the League of Nations established ?
(SSC CHSL 2014) (SSC CHSL 2015)
(a) Babar (b) Akbar (a) In 1920 (b) In 1939
(c) Aurangzeb (d) Jahangir (c) In 1914 (d) In 1918
w
w
w
.y
ou
6 History

rs
m
118. In which of the Round Table Conference Mahatma Gandhi 129. Where was Christopher Columbus from?

ah
bo
participated ? (SSC CHSL 2015) (SSC Multitasking 2013)

ob
(a) First Round Table Conference, 1930 (a) Portugal (b) Venice

.w
(b) Second Round Table Conference, 1931 (c) Genoa (d) Spain

or
130. To make tools and weapons, the earliest inhabitants of India

dp
(c) Third Round Table Conference, 1932

re
used (SSC Multitasking 2014)
(d) All of the above

ss
(a) clay (b) wood

.c
119. The use of which of the following regional languages was (c) stones (d) bronze

om
popularised by the Bhakti leader, Shankaradeva ? 131. Name an Indian Saint (priest) who, attended the "World
(SSC CHSL 2012) Congress of Religions" held at Chicago (U.S.) in 1893.
(a) Bengali (b) Brijbhasha (SSC Multitasking 2014)
(c) Avadhi (d) Assamese (a) Basaveshwara (b) Swami Vivekananda
120. The Mansabdari system was introduced by: (c) Ramanuja (d) Madhavacharya
(SSC CHSL 2012) 132. Chhatrapati Shivaji was a follower of
(a) Shah Jahan (b) Jahangir (SSC Multitasking 2014)
(c) Babur (b) Akbar (a) Madhava (b) Kabir
121. When and by whom were the Asokan inscriptions (c) Basava (d) Ramadasa
deciphered for the first time ? (SSC CHSL 2012) 133. Who was the greatest Kushan ruler?
(a) 1787 - John Tower (b) 1825 - Charles Metcalfe (SSC Multitasking 2014)
(c) 1837 - James Prinsep (d) 1810 - Harry Smith (a) Vashiska (b) Vasudeva
122. Amuktamalyada is the work of : (SSC CHSL 2012) (c) Huvishka (d) Kanishka
(a) Krishnadeva Raya (b) Vachcharaj 134. Who was the founder of Arya Samaj?
(c) Kharavela (d) Allasani Peddana (SSC Multitasking 2014)
(a) Kabir Das (b) Shankaracharya
123. Name the Commander of the Arab army who conquered the
(c) Ranade (d) Dayanand Saraswati
Sindh. (SSC Multitasking 2013)
135. The French challenge to British in India came to an end with
(a) Muhammad bin Qasim
(SSC Sub. Ins. 2012)
(b) Al Hazzaz
(a) Battle of Wandiwash
(c) Qutbuddin Aibak
(b) Battle of Srirangapattinam
(d) Allauddin Khilji
(c) Battle of Plassey
124. Who was the chairperson of the Chinese Communist Party
(d) Battle of Buxar
at the time of liberation of China?
136. Identify the medical trio of Ancient India from the following
(SSC Multitasking 2013)
names. (SSC Sub. Ins. 2012)
(a) Liu Shaoqi (b) Zhou Enlai
(a) Charaka, Susruta and Vagbhata
(c) Deng Xiaoping (d) Mao Zedong
(b) Charaka, Vatsyayana and Vagbhata
125. Who was regarded by Gandhiji as his political Guru?
(c) Charaka, Susruta and Bharata
(SSC Multitasking 2013)
(d) Charaka, Susruta and Patanjali
(a) Gopal Krishna Gokhale
137. Match the medieval travellers with their countries :
(b) Lala Lajpat Rai
(SSC Sub. Ins. 2012)
(c) Bipin Chandra Pal
A. Marco Polo 1. Spain
(d) Bal Gangadhar Tilak
B. Ibn Battuta 2. Balkh
126. Which among the following movements was not led by
C Antonio Monserrate 3. Italy
Mahatma Gandhi? (SSC Multitasking 2013)
D. Mahmud Wali Balkhi 4. Morocco
(a) Civil Disobedience Movement
A B C D A B C D
(b) Quit India Movement
(a) 4 3 1 2 (b) 3 1 4 2
(c) Swadeshi-Movement
(c) 3 4 1 2 (d) 1 3 2 4
(d) Non-Cooperation Movement
138. Match the following : (SSC Sub. Ins. 2012)
127. In which session of the Indian National Congress was the
A. Sarojini Naidu 1. Muslim League
“Poorna Swaraj” resolution adopted?
B. M.A. Jinnah 2. Indian National Congress
(SSC Multitasking 2013)
C. Sir Tej Bahadur Sapru 3. Hindu Mahasabha
(a) Karachi Session in 1931
D. V.D. Savarkar 4. Liberal Party
(b) Lucknow Session in 1916
A B C D A B C D
(c) Belgaum Session in 1924
(a) 2 1 4 3 (b) 2 1 3 4
(d) Lahore Session in 1929
(c) 2 4 1 3 (d) 4 1 3 2
128. The school of arts developed during the Kushan Period
139. Which Sultan received a robe of honour from the caliph?
with the mixture of Indian and Greek style is known as
(SSC Multitasking 2013) (SSC Sub. Ins. 2013)
(a) Mughal art (b) Kushan art (a) Ala-ud-din Khilji (b) lltutmish
(c) Persian art (d) Gandhara art (c) Balban (d) Qutub-ud-din Albak
w
w
w
.y
ou
History 7

rs
m
140. When Margaret Thatcher was P.M. with which country 149. Which was the second capital of Akbar ?

ah
bo
Britain waged a war to regain Control of Falkland Islands? (SSC Sub. Ins. 2014)

ob
(SSC Sub. Ins. 2013) (a) Delhi (b) Agra

.w
(a) Chile (b) Argentina (c) Fatehpur-Sikri (d) Patna

or
150. The first country which discovered sea route to India was

dp
(c) Brazil (d) None of the above

re
141. Match the following (SSC Sub. Ins. 2013) (SSC Sub. Ins. 2014)

ss
(a) Chalukyas (b) Hoysalas (a) Portugal (b) Dutch

.c
(c) French (d) Britain

om
(c) Rashtrakutas (d) Kakatiyas
151. The unification of Karnataka was achieved in the year
(i) Malkhed (ii) Vatapi
(SSC Sub. Ins. 2014)
(iii) Warangal (iv) Dwarasamudra
(a) 1956 (b) 1957
(a) (a)-(ii), (b )-(iv), (c)-(i), (d)-(iii) (c) 1958 (d) 1960
(b) (a)-(iv), (b)-(iii), (c)-(i), (d)-(ii) 152. Who introduced the Indian University Act?
(c) (a)-(i), (b)-(ii), (c)-(iii), (d)-(iv) (SSC Sub. Ins. 2014)
(d) (a)-(iii), (b)-(ii), (c)-(iv),(d)-(i) (a) Lord Curzon (b) Lord Minto
142. The world's tallest statue of Mahatma Gandhi is in (c) Lord Morelay (d) Lord Rippon
(SSC Sub. Ins. 2013) 153. Chinese travellers visited India primarily because
(a) Champaran (b) Patna (SSC Sub. Ins. 2014)
(c) Lucknow (d) Rajkot (a) they were interested in Buddhism
143. Which one of the following is the principal source of (b) they were invited by the Indian kings
information on Asoka's campaign against Kalinga ? (c) they were interested to study Indian culture
(SSC Sub. Ins. 2013) (d) they were interested to stay in India
(a) Pillar Edict VII (b) Mahavamsa 154. Development that meets the needs of the present, without
(c) Divyavadana (d) Rock Edict XIII compromising the ability of future generations to meet their
144. Gandhi's concept of Trusteeship: (SSC Sub. Ins. 2013) own needs was the focal point of Brundtland commission is
(a) Recognises right of private ownership of property (SSC Sub. Ins. 2014)
(a) sustainable development
(b) Transforms the capitalistic society into an egalitarian
(b) mitigation
one
(c) disaster management
(c) Excludes legislative regulation of the ownership and
(d) capacity building
use of wealth
155. Name the Kingdom which first used elephants in wars?
(d) Does not fix minimum or maximum income
(SSC Sub. Ins. 2015)
145. Match the following: (SSC Sub. Ins. 2013)
(a) Avanti (b) Champa
(a) Hunter's Commission
(c) Magadha (d) Kosala
(b) Wardha Scheme
156. Tulsidas wrote Ramacharitamans during the reign of :
(c) University's Act
(SSC Sub. Ins. 2015)
(d) Radhakrishnan Commission
(a) Akbar (b) Krishnadeva Raya
(i) 1948 (ii) 1904
(c) Jahangir (d) Rama Raya
(iii) 1937 (iv) 1882
157. Cripps Mission came to India in: (SSC Sub. Ins. 2015)
(a) (a)-(iii), (b)-(ii), (c)-(iv), (d)-(i)
(a) 1945 (b) 1946
(b) (a)-(iv), (b)-(ii), (c)-(iii), (d)-(i)
(c) 1940 (d) 1942
(c) (a)-(iii) (b)-(iv), (c)-(i), (d)-(ii)
158. Who among the following Sultans tried to prohibit Sati?
(d) (a)-(iv), (b)-(iii), (c)-(ii), (d)-(i)
(SSC Sub. Ins. 2015)
146. The Gandhara art nourished under:
(a) Muhammad Bin Tuglaq (b) Firoz Tuglaq
(SSC Sub. Ins. 2013) (c) Jalaluddin Khilji (d) Alauddin Khilji
(a) the Kushanas (b) the Satavahanas 159. Who constructed Humayun’s Tomb in Delhi ?
(c) the Guptas (d) the Mauryas (SSC Sub. Ins. 2015)
147. Most important safeguard of liberty is (a) Haji Begam (b) Babar
(SSC Sub. Ins. 2014) (c) Humayun (d) Akbar
(a) bold and impartial judiciary 160. Who among the following Mughal rulers banned music
(b) well-knit party system and dancing ? (SSC Sub. Ins. 2015)
(c) decentralisation of power (a) Jahangir (b) Babar
(d) declaration of rights (c) Aurangzeb (d) Humayun
148. What is the ancient school of law ? (SSC Sub. Ins. 2014) 161. Who was the governor-general during the Second Anglo-
(a) The Philosophical School Mysore War? (SSC Sub. Ins. 2015)
(b) The Historical School (a) Lord Wellesley (b) Sir John Shore
(c) The Analytical School (c) Lord Cornwallis (d) Warren Hastings
(d) The Sociological School
w
w
w
.y
ou
8 History

rs
m
162. Which of the following Pacts sought to be resolved the 173. which of the following was not built by Firoz Shah Tughlaq?

ah
bo
Hindu - Muslim differences? (SSC Sub. Ins. 2016) (SSC Stenographer 2013)

ob
(a) Lucknow Pact (b) Lahore Pact (a) Firozabad (b) Fatehabad

.w
(c) Gandhi - Irwin Pact (d) Poona Pact (c) Tughlaqabad (d) Jaunpur

or
163. In which of the following year the Gandhi-Irwin Pact was 174. who among the follwing granted permission to the English

dp
re
signed? (SSC Sub. Ins. 2016) to establish their factory in India?

ss
(a) 1932 (b) 1935 (SSC Stenographer 2013)

.c
(a) Akbar (b) Jehangir

om
(c) 1931 (d) 1929
164. The Vedic literature was composed between (c) Shah Jahan (d) Aurangzeb
(SSC Sub. Ins. 2016) 175. The permanent Land Settlement in Bengal was introduced
(a) 3500 B.C. and 1000 B.C. by (SSC Stenographer 2013)
(b) 2500 B.C. and 500 B.C. (a) Warren Hestings (b) Thomas Munro
(c) 3000 B.C. and 1000 B.C. (c) Thomas Reid (d) Lord Cornwallis
(d) 3500 B.C. and 2500 B.C. 176. Din-I-llahi was founded by (SSC Stenographer 2014)
165. Who among the following was the Prime Minister of England (a) Kabir (b) Akbar
when India was given independence? (SSC Sub. Ins. 2016) (c) Shah jahan (d) Guru Nanak Dev
(a) Mountbatten (b) Churchill 177. 'Purna Swaraj' was announced in the Indian National
(c) Attlee (d) Wavell Congress Session of (SSC Stenographer 2014)
166. How many times did Babur invade India before 1526 A.D.? (a) Calcutta (b) Lahore
(SSC Sub. Ins. 2016) (c) Nagpur (d) Karachi
(a) None of these (b) Four times 178. Who chaired the Boundary Commission in 1947, to demarcate
(c) Five times (d) Two times the boundary line between India and pakistan?
167. Which of the following countries has become the first (SSC Stenographer 2014)
country in the world to receive funds from United Nations (a) Krishna Menon (b) McMohan
for its fast growing Solar Home Systems? (c) J. Nehru (d) Henderson Brooks
(SSC Sub. Ins. 2016) 179. Ashoka embraced Buddism (SSC Stenographer 2014)
(a) before the Kalinga war
(a) Afghanistan (b) Burma
(b) after the Kalinga war
(c) Bangladesh (d) Switzerland
(c) just before his death
168. In Indian architecture 'Surkhi' was introduced by
(d) when he was a Crown Prince
(SSC Sub. Ins. 2016)
180. The original name of 'Mahabharata' is
(a) Mughals (b) Kushans
(SSC Stenographer 2016)
(c) Guptas (d) Sultanate Sultans
(a) Rajtarangini (b) Bharat Katha
169. which one of the following land reform measures can be
(c) Kathasaritasagar (d) Jai Samhita
said to have been fully implemented by now in India?
181. Sir Eyre Coote was associated with which of the following?
(SSC Stenographer 2013)
(SSC Stenographer 2016)
(a) Tenancy reforms
(a) Battle of Ambur (b) Battle of Arcot
(b) Abolition of intermediaries (c) Battle of Wandiwash (d) Battle of Adyar
(c) Land ceilings 182. Which among the following is related to Sadr-us-Sadr?
(d) Consolidation of holdings (SSC Stenographer 2016)
170. Which of the following recommended reservation for the (a) Ecclesiastical matters
Other Backward Classes (OBCs)? (SSC Stenographer 2013) (b) Judicial administration
(a) Mandal Commission (c) Military administration
(b) Kothari Commission (d) Land revenue
(c) Sachar Committee 183. Which among the following inscription is known as Prayaga
(d) None of these Prashasti ? (SSC Stenographer 2016)
171. Which of the following was not an aspect of Later Vedic (a) Hathigumpha Inscription
Age? (SSC Stenographer 2013) (b) Aihole Inscription
(a) Importance of Kingship in political life (c) Mehroli Inscription
(b) Discovery of iron (d) Allahabad Pillar Inscription
(c) Polygamy 184. Which of the following pair is NOT correctly matched?
(d) Simple, non-ritualistic worship (SSC Stenographer 2016)
172. Who among the follwing presided over the fourth Buddhist (a) Ibn Batuta - Morocco
Council? (SSC Stenographer 2013) (b) Nikitin - Samarkand
(a) Ashoka (b) Kanishka (c) Marco Polo - Italy
(c) Ashvaghosha (d) Vasumitra (d) Alberuni - Uzbekistan
w
w
w
.y
ou
History 9

rs
m
ah
HINTS & SOLUTIONS

bo
ob
.w
or
dp
re
1. (d) 27. (a) As Samhita is the collection of the mantras, so

ss
.c
2. (d) The Servants of India Society was formed in Pune, sometimes Samhitas are referred to as Mantras. Most

om
Maharashtra, on June 12, 1905 by Gopal Krishna of these mantras or hymns are concerned with nature
Gokhale, who left the Deccan Education Society to and deities.
form this association. 28. (d) 29. (c) 30. (b)
3. (d) 31. (a) 32. (a) 33. (b)
4. (b) Upagupta (c. 3rd Century BC) was a Buddhist monk. 34. (d) Lord Canning was the Governor General of India from
According to some stories in the Sanskrit text 1856 - 1862 and the first Viceroy in India from 1
Ashokavadana, he was the spiritual teacher of the November 1858. Lord Mountbatten was the First
Mauryan emperor Ashoka. Governor General of Independent India.
5. (c) 6. (b) 7. (c)
35. (b) 1191 - First Battle of Tarain in which Prithviraj Chauhan
8. (b) The doctrine of lapse was an annexation policy defeated Mohd. Ghori.1192 - Second Battle of Tarain
purportedly devised by Lord Dalhousie, who was the in which Mohd.Ghori defeated Prithviraj Chauhan.
Governor General for the East lndia Company in lndia
36. (d)
between 1848 and 1856.
37. (c) The first major attempt in curriculum reconstruction in
9. (c) 10. (b) 11. (b) 12. (b) 13. (d)
India was made in 1937 when Gandhiji propounded the
14. (d) 15. (b) 16. (b) 17. (c) 18. (c)
idea of Basic Education.
19. (a)
38. (b) Simon Commission (1927) > Dandi March (1930) >
20. (a) King Asoka assumed the title Devanampiya Piyadasi
Gandhi Irwin Pact (1931) > Poona Pact (1932)
which means "Beloved-of-the-Gods, He who Looks
on with Affection". 39. (b) Emperor Akbar named Prayag as Allahabad - City of
God- also called Allahabad in 1575 AD. The city of
21. (b) Shivaji, the great Maratha King and founder of a
nationalist tradition was contemporary of Tukaram. Allahabad is situated at the confluence of three rivers
Tukaram introduced Shivaji to Ramdas for his spritual - Ganga, Yamuna and the invisible Saraswati. Every
education. 12th year when the waters are felt to be especially
22. (b) Wheat was the first cereal to be cultivated by man. In purifying, Allahabad holds a much greater festival called
several places in the Middle East it was sowed, tended Kumbh Mela. Built by Emperor Akbar in 1583 AD, the
and reaped soon after 8000 BC. The people of Jericho Allahbad fort stands on the banks of the river Yamuna
are the first known to have lived mainly from the near the confluence site i.e SANGAM.
cultivation of crops. 40. (d) Madan Mohan Malaviya was an Indian educationist
23. (c) After approximately 200 years of French rule, Alsace and politician notable for his role in the Indian
and the German-speaking part of Lorraine were ceded independence movement and as the two time president
to Germany in 1871 under the Treaty of Frankfurt. In of Indian National Congress. He was respectfully
1919, both regions were returned to France. addressed as Pandit Madan Mohan Malaviya and also
24. (d) James Princep was an English scholar and antiquary. addressed as 'Mahamana' by Mahatama Gandhi.
From 1832 to 1838 he was assay-master in the India 41. (d) Lord Linlithgow was Viceroy of India from 1936 to 1944
Government Mint, Kolkata. He is most noted as a and this eight years period was longest reign as Viceroy
philologist for fully deciphering and translating the of India.
rock edicts of Asoka from the Brahmi script. 42. (c)
25. (c) Tilak propounded the theory of the Arctic home of the 43. (c) The Dhammacakkappavattana Sutta is considered to
Aryans, meaning that the Aryans originated in the be a record of the first teaching given by Gautama
Arctic region, and later, on the journey south, divided Buddha after he attained enlightenment. The main topic
into two branches. One branch went to Europe, while of this sutra is the Four Noble Truths, which are the
the other branch came to India.
central teachings of Buddhism that provide a unifying
26. (b) Alara Kalama was a hermit saint and a teacher of yogic theme, or conceptual framework, for all of Buddhist
meditation who lived near Rajagriha. According to the
thought.
Pali Canon scriptures, he was one of the teachers of
44. (b) 45. (b)
Gautama Buddha.
w
w
w
.y
ou
10 History

rs
m
46. (a) The Rowlatt Act was passed by the Imperial Legislative Indian National Congress. Thought it was he who

ah
bo
Council in London on 18 March 1919, indefinitely emerged as the founder of the Congress, it had

ob
extending "emergency measures" enacted during the somehow never happened to Hume to preside over a

.w
First World War in order to control public unrest and plenary session even once.

or
dp
root out conspiracy. Lord Chelmsford (1916 to 1921) 57. (b) The Telangana Movement was the biggest peasant

re
was the Viceroy of India then. guerrilla war of modern Indian history affecting 3000

ss
.c
47. (a) Jawaharlal Nehru inaugurated the first generation of villages and 3 million population. The uprising began

om
panchayat raj at Nagaur in Rajasthan on 2 October in july 1946 and was at its greatest intensity between
1959. It was also implemented in Andhra Pradesh in August 1947 and September 1948.
the same year as per the recommendations of the 58. (a) Bahlul Khan Lodi was the founder of Lodi dynasty of
Balwant Rai Mehta Committee. the Delhi Sultanate in India upon the abdication of the
48. (a) Tana Bhagat movement was apparently initiated in 1914 last claimant from the previous Sayyid rule.
by a young Oraon tribesman known as Jatra Oraon. 59. (d) Greco-Roman-Buddhist Art
49. (c) The Naujawan Bharat Sabha was founded by Bhagat 60. (a) the Bolshevik Revolution, was a seizure of state power
Singh in March 1926 and was declared illegal under instrumental in the larger Russian Revolution of 1917.
the Criminal Law Amendment Act of 1908 in September It took place with an armed insurrection in Petrograd
1934. Its purpose was to forment revolution against traditionally dated to 25 October 1917.
the British Raj by gathering together worker and 61. (b) Lieutenant-General Lord William Henry Cavendish-
peasant youths. Bentinck, GCB, GCH, PC, known as Lord William
50. (c) The chamber of Princes was instituted on 8 February Bentinck, was a British soldier and statesman. He
1921 . The inauguration ceremony was performed by served as Governor-General of India from 1828 to 1835.
His Royal Highness the Duke of Connaught in the 62. (c) 63. (b) 64. (b) 65. (c)
Diwan–i–am of Red Fort on be; half of His Majesty the 66. (a) 67. (d) 68. (c) 69. (c)
King Emperor. 70. (c) During the early period, Multan was known as the city
51. (a) The Three Jewels (triratna) are the three things that of gold for its large and wealthy temples. The Sun
Buddhists take refuge in, and look toward for guidance, temple, Suraj Mandir, was considered one of the largest
in the process know as taking refuge. The Three Jewels and wealthiest temples in the entire sub-continent.
are: Buddha, Dhamma and Sangha. 71. (d) Amoghavarsha I was a follower of the Digambara
52 (a) Chandragupta I was known as 'Lichchavidauhitra.' His branch of Jainism. His own writing Kavirajamarga is a
marriage with the Lichchavi Princess Kumaradevi was landmark literary work in the Kannada language and
one of the significant events in the Gupta rule. The became a guide book for future poets and scholars for
importance of this marriage can be known further from centuries to come.
Samudragupta's Allahabad inscription in which he has 72. (c) The Kailasa temple is a famous rock cut monument,
described himself as "Lichchhavis–dauhitra or one of the 34 monasteries and temples known
daughter's son of the Lichchavis. collectively as the Ellora Caves, extending over more
53. (b) Pulakesin II (610 – 642 AD) was the most famous ruler than 2 km.The temple was commissioned and completed
of the Chalukya dynasty. In his reign the Chalukyas of between dated 757-783 CE, when Krishna I ruled the
Badami saw their kingdom extend over most of the Rashtrakuta dynasty. It is designed to recall Mount
Deccan. Kailash, the home of Lord Shiva. It is a megalith carved
54. (b) An inscription of the 8th century AD at Uttaramerur out of one single rock. It was built in the 8th century
temple describes the constitution of the local council, by the Rashtrakuta king Krishna I.
eligibility and disqualifications for the candidates, the 73. (a) the Thalavaipuram copper plate, belonging to the period
method selection, their duties and delimits their power between 1018 and 1054 brought out by the Pandya
in Chola dynasty. kings, describes giant waves, most possibly a tsunami.
55. (b) The Pitt’s India Act of 1784 was in the nature of a 74. (b) Gautamiputra Satakarni was the greatest of the
regulating act as it was intended to address the Satavahana rulers. His reign period is noted by some
shortcomings of the Regulating Act of 1773 by bringing scholars as 80 to 104 and by others from 106 to 130; in
the East India Company’s rule in India under the control any case he is credited with a rule of 24 years.
of the British Government. 75. (a) Mihira Bhoja was a ruler of the Gurjara Pratihara
56. (b) Allan Octavian Hume was a civil servant, political dynasty 2of India. Bhoja's empire extended to Narmada
reformer and amateur ornithologist and horticulturalist River in the South, Sutlej River in the northwest, and
in British India. He was one of the founders of the up to Bengal in the east.
w
w
w
.y
ou
History 11

rs
m
76. (d) Bose appeared at the 1939 Congress meeting and was Deccan sultanates, resulted in a defeat of Vijayanagara,

ah
bo
elected president over Gandhi's preferred candidate and ended in greatly weakening one of the greatest

ob
Pattabhi Sitaramayya. Indian Empires originating from Southern India before

.w
77. (c) The Jallianwala Bagh Massacre happened in Amritsar, the Maratha Empire.

or
dp
in 1919. It is named after the Jallianwala Bagh (Garden) 90. (c) The mansabdari system was of Central Asian origin

re
atAmritsar. On April 13, 1919, British, Indian Army and it was first introduced by Babur in North India.

ss
.c
soldiers started shooting an unarmed gathering of men, But it was Akbar who institutionalized it in Mughal

om
women and children. military set up and civil administration.
78. (b) Bahlol Lodi was the founder of the Lodi dynasty.He 91. (c) Cabinet Mission was composed of three Cabinet
ruled for long thirty-nine years (1451-89). He was the Ministers of England Sir Pethick Lawrence, Secretary
governor of Lahore and Sirhind during the rule of of State for India, Sir Stafford Cripps, President of the
Muhammad Shah of Sayyid dynasty. Board of Trade;
79. (c) Sri Chaitanya Mahaprabhu arrived in the empire at the Alexander, the First Lord of the Admiralty. The mission
time of Emperor Prataparudra (Gajapatis)and stayed arrived on March 24, 1946. The objective of this mission
for 18 long years at Puri. was to Devise a machinery to draw up the constitution
80. (c) The instance heat that prevails in the Indian Sub of Independent India. Make arrangements for interim
continent causes a low pressure region over the Government. Thus the mission was like a declaration
northern plains. It is intense enough to attract the of India's independence.
moisture bearing winds from the Indian Ocean .Thus 92. (c) Sarvodaya is a Sanskrit term meaning 'universal uplift'
the south east trade winds from the southern or 'progress of all'. The term was used by Mahatma
hemisphere are attracted towards India. Gandhi as the title of his 1908 translation of John
81. (b) Reformation was a 16th-century movement in Western Ruskin's tract on political economy, Unto This Last,
Europe that aimed at reforming some doctrines and and Gandhi came to use the term for the ideal of his
practices of the Roman Catholic Church and resulted own political philosophy.
in the establishment of the Protestant churches. 93. (a)
82. (d) Bal Gangadhar Tilak is considered as "Father of Indian 94. (b) The collective term given by the Jainas to their Sacred
National Movement". He was a social reformer, freedom literature is called Agamas written in Prakrt.
fighter, national leader and a scholar of Indian history, 95. (b) Doctrine of Passive Resistance is authored by
sanskrit, hinduism, mathematics and astronomy. During Aurobindo Ghosh. It is based on a series of articles by
freedom struggle, his slogan "Swaraj is my birthright Aurobindo Ghosh which were published in April 1907
and I shall have it" inspired millions of Indians. in the journal Bande Matram. The articles were written
83. (b) The Quit India Movement(August Kranti), was a civil when the Bengal was burning with indignation after
disobedience movement launched in India on 9 August its partition October 16, 1905 by Viceroy Curzon. The
1942 by Mohandas Karamchand Gandhi. main theme of the articles is the methods and ideology
84. (d) The Indian National Congress was founded on which could be adopted in face of oppression and
December 28, 1885, by members of the Theosophical injustice as perpetrated by the contemporary British
Society. The founders included a prominent member rule in India.
of the Theosophical Society, Allan Octavian Hume as 96. (c) Badruddin Taiyabji became the Ist Indian Barrister in
well as Dadabhai Naoroji and Dinshaw Wacha. Bombay; became the 2nd Indian Chief Justice; was the
85. (d) Modern Bangladesh emerged as an independent nation founding member of Bombay presidency association
in 1971 after achieving independence from Pakistan in and INC and also presided over the 3rd congress
the Bangladesh Liberation War. The Provisional session in Madras in 1887.
Government of Bangladesh was formed on 17 April 97. (d) The Dandi March of Gandhi was an important part of
1971. the Indian Independence Movement.It was a direct
86. (a) 'Brahmo Samaj' was founded by Raja Ram Mohan Roy action campaign of tax resistance and non-violent
in 1828. protest against British saltmonopoly and triggered the
87. (b) Allan Octavian Hume was a civil servant, political wider Civil Disobedience Movement.
reformer in British India who initiated the movement to 98. (d) Aihole inscription is found at Aihole in Karnataka state
form the Indian National Congress. India, was written by the Ravikriti,court poet of
88. (c) Chalukya king,Pulakeshin II who reigned from 610 to
89. (b) The Battle of Talikota (26 January 1565), a watershed 642 CE.The poetic verses of Ravikirti,in praise of the
battle fought between the Vijayanagara Empire and the king, can be read in the Meguti temple,dated 634CE.
w
w
w
.y
ou
12 History

rs
m
99. (d) Akbar introduced the Mansabdari system. This system 107. (c) R. Shamashastri transcribed, edited and published the

ah
bo
came under the military reforms of Akbar. Under this Sanskrit edition in 1909. He proceeded to translate it

ob
system each officer was assigned a rank(mansab). into English, publishing it in 1915.

.w
Varying from 10 to 10,000, the mansab carried the Zat(the

or
108. (c) Porus fought Alexander the Great in the Battle of the

dp
personal status and salary) and Sawar (the number of Hydaspes (also known as Jhelum) in 326 BC and is

re
cavalry men to be maintained. believed to be defeated.

ss
.c
100. (a) Calcutta with Robert Clive is wrongly matched. Lord 109. (c) Right to Information Act (RTI) was enacted on 15th

om
Curzon was associated with Calcutta. Lord Curzon felt June 2005 and came fully into force on 12th October
that the Bengal province was too big to be administered 2005.
efficiently and so he wanted to split it into two 110. (a) Indian Social activist Medha Patkar is the founder
provinces, one of which had Dacca as its capital. member of Narmada Bachao Andolan.
101. (b) Battle of Wandiwash decided the fate of French in 111. (b) Purana Qila was built by the Afghan king Sher Shah
India. Battle of Wandiwash, (Jan. 22, 1760), in the history Suri.
of India, a confrontation between the French, under the
112. (d) Chandragupta Maurya is said to have lived as an ascetic
comte de Lally, and the British, under Sir Eyre Coote. It
at Shravanabelagola for several years before starving
was the decisive battle in the Anglo-French struggle
himself to death, as per Jain Practice of Sallekhana.
in southern India during the Seven Years' War (1756-63).
113. (c) Megasthenes gave an account of India in his book
102. (d) The Crimean War came to an end by the treaty of Paris.
‘Indica’.
Crimean War, (October 1853-February 1856), war fought
114. (d) The construction of the famous Shiva temple at Ellora
mainly on the Crimean Peninsula between the Russians
is often attributed to the Rashtrakuta king Krishna I.
and the British, French, and Ottoman Turkish, with
support from January 1855 by the army of Sardinia- 115. (d) Harisena was a 4th century Sanskrit poet. His most
Piedmont.The resulting Treaty of Paris, signed on famous poem written in 345 AD is inscribed on the
March 30, 1856, guaranteed the integrity of Ottoman Allahabad Pillar.
Turkey and obliged Russia to surrender southern 116. (d) Todar Mal started his career as a revenue officer at the
Bessarabia, at the mouth of the Danube. court of Sher Shah Suri. After the Sur dynasty was
103. (b) Tulsidas wrote Shri Ram-Charit-Manas during the reign overthrown by the Mughals, Todar Mal continued his
of Mughal Emperor Akbar.Tulsidas started writing this service to the then Mughal Emperor Akbar.
greatest Hindu 'Granth' on the birth day of Lord Shree 117. (d) The League of Nations was an intergovernmental
Ram, i.e. Chaitra Navmi (9th day of Hindu month Chaitra) organisation founded on 10th January 1920 as a result
in year 1574. The life span of Akbar is 1556 1605. of the Paris Peace Conference that ended the first World
104. (a) Architecture reached a milestone in the Dravidian style War.
during the reign of Rashtrakutas, the finest example of 118. (b) A settlement between Mahatma Gandhi and Viceroy
which is seen in the Kailasanath Temple or Kailasa Lord Irwin known as the Gandhi-Irwin pact was reached
Temple at Ellora. and Gandhi was appointed as the sole representative
105. (c) The famous Badami Chalukyas King Pulakeshi II of the Congress to the second Round table conference,
(610-642 A.D.) was a follower of Vaishnavism. The 1931.
inscription of Ravikirti, his court poet, is a eulogy of 119. (d) Sankaradeva was an erudite scholar, a prolific writer, a
the Pulakeshi II and is available at the Meguti temple. versatile saint-poet of unlimited merit, a lyricist of
It is dated 634 CE and is written in Sanskrit language universal acceptance, a musician of high calibre, a
and old Kannada script. The Aihole inscription pioneer in the field of Assamese prose, drama and
describes the achievements of Pulakeshi II and his dramatic performances, a painter and above all the
victory against King Harshavardhana. greatest religious teacher-preacher-leader of the
106. (b) The Delhi Sultanate was the name of Delhi-based medieval Vaisnava movement in Assam which is rightly
Muslim kingdoms that ruled over large parts of India known as the Sankaradeva Movement.
for 320 years (1206 – 1526). Five dynasties ruled over 120. (d) 121. (c) 122. (a)
Delhi Sultanate sequentially, the first four of which 123. (a) Muhammad bin Qasim Al-Thaqafi (c. 31 December 695-
were of Turkic origin and the last was the Afghan Lodi. 18 July 715) was an Umayyad general who conquered
The Lodi dynasty was replaced by the Mughal the Sindh and Punjab regions along the Indus River
dynasty. The five dynasties were the Mamluk dynasty (now a part of Pakistan) for the Umayyad Caliphate.
(1206–90); the Khilji dynasty (1290–1320); the Tughlaq He was born and raised in the city of Taif (in modern
dynasty (1320–1414); the Sayyid dynasty (1414–51); day Saudi Arabia). Qasim's conquest of Sindh and
and the Afghan Lodi dynasty (1451–1526). Punjab enabled further Islamic expansion into India.
w
w
w
.y
ou
History 13

rs
m
124. (d) Mao Zedong, commonly referred to as Chairman Mao aspect of the fight for independence and establishing

ah
bo
(December 26, 1893 - September 9, 1976), was a Chinese his own kingdom. Shivaji Maharaj's assembly of eight

ob
communist revolutionary, politician and socio-political ministers was formed based on Hindu ideals. One

.w
theorist. The founding father of the People's Republic comes across this concept of eight ministers in the

or
dp
of China from its establishment in 1949, he governed Ramayan and the Mahabharat.

re
the country as Chairman of the Communist Party of 133. (d) Kanishka was the greatest ruler of the Kushan Empire,

ss
.c
China until his death. a realm that covered much of present-day India,

om
125. (a) Gandhi calls Gokhale his mentor and guide. Gandhi Pakistan, Iran and other parts of central Asia and China
also recognised Gokhale as an admirable leader and during the first and second centuries.
master politician, describing him as 'pure as crystal, 134. (d) Arya Samaj is a Hindu reform movement founded by
gentle as a lamb, brave as a lion and chivalrous to a Swami Dayananda on 7 April 1875. He was a sannyasi
fault and the most perfect man in the political field'. who promoted the Vedas. Dayananda emphasised the
126. (c) The Swadeshi movement started with the partition of ideals of brahmacharya. The group found most of its
Bengal by the Viceroy of India, Lord Curzon, 1905 and support in Punjab.
continued up to 1911. It was the most successful of 135. (a) 136. (d) 137. (c) 138. (a)
the pre-Gandhian movements. Its chief architects were 139. (b) Iltutmish received contlnnation of his robe of honour
Aurobindo Ghosh, Lokmanya Bal Gangadhar Tilak, and title Nasir amir al-muminin (Helper of the
Bipin Chandra Pal and Lala Lajpat Rai. Commander of the Faithful) from the 'Abbasid Caliph
127. (d) The Purna Swaraj declaration, or Declaration of the al-Mustansir in 626 (1229) and remained on the throne
Independence of India was promulgated by the Indian for twenty-six years, This added an element of strength
National Congress on January 26, 1930, resolving the to Iltutmish's authority and gave him a status in the
Congress and Indian nationalists to fight for Purna Muslim world .
Swaraj, or complete self-rule independent of the British 140. (b) As a British Overseas Territory, the Falkland Islands
Empire. The flag of India had been hoisted by Congress enjoy a large degree of internal self-governance with
President Jawaharlal Nehru on December 31, 1929, on the United Kingdom guaranteeing good government
the banks of the Ravi river in Lahore, modern-day and taking responsibility for their defence and foreign
Pakistan. affairs. In 1982, following Argentina's invasion of the
128. (d) Gandhra is noted for the distinctive Gandhra style of islands, the two-month-long undeclared Falklands War
Buddhist art, which developed out of a merger of Greek, between both countries resulted in the surrender of all
Syrian, Persian, and Indian artistic influence. This Argentine forces and the return of the islands to British
development began during the Parthian Period (50 BC administration.
- AD 75). Gandhran style flourished and achieved its 141. (a) Chalukyas : Vatapi (Badami): Hoysalas : Dwarasamudra:
peak during the Kushan period, from the 1st to the 5th Rashtrakutas : Malkhed; and Kakatiyas: Warangal.
centuries. It declined and suffered destruction after 142. (b) With a towering height of 70 feet, the world's tallest
invasion of the White Huns in the 5th century. statute of Mahatma Gandhi was recently unveiled in
129. (c) Christopher Columbus was an Italian explorer, Patna in Bihar. Funded by the state government, the
navigator, and colonizer, born in the Republic of Genoa, Rs 10 crore statue, inclusive of a 30-feet-high pedestal,
in what is today northwestern Italy. has been built by Delhi-based sculptor Ramsutar and
130. (c) the earliest inhabitants of India used stones to make Sons.
tools and weapons. 143. (d) The vivid description of Kalinga war is given in 13th
131. (b) Swami Vivekanand was the Indian saint to attend ' Rock Edict of Asoka. The edict gives description of
World Congress of Religion' held at Chicago in 1893. the devastation caused to Kalinga due to war and how
He is perhaps best known for his inspiring speech the Mauryan emperor felt remorse for it.
beginning with "Sisters and Brothers of America," 144. (b) Trusteeship principle advocated by Gandhiji provides
through which he introduced Hinduism at the opening a means of transforming the present capitalist order of
session of the Parliament on 11 September. Thereafter society into an egalitarian one. He envisaged
he conducted hundreds of public and private lectures trusteeship as a post capitalist arrangement which gives
and classes, disseminating tenets of Hindu philosophy no quarter to capitalism, but gives the present owning
in America, England and Europe. In America class a chance of reforming itself (Gandhi Nehru And
Vivekananda became India's spiritual ambassador. Globalization, p 7).
132. (d) Chhatrapati Shivaji was a follower of Ramayana and 145. (d) Hunter's Commission: 1882; Wardha Scheme: 1937;
Mahabharta from his childhood. Shivaji Maharaj did University's Act; 1904; and Radhakrishnan
everything possible to promote Hinduism in every Commission: 1948.
w
w
w
.y
ou
14 History

rs
m
146. (a) Gandhara style flourished and achieved its peak during 152. (a) Lord Curzon introduced the Indian University Act. The

ah
bo
the Kushan period. from the 1st to the 5th centuries. It Indian Universities Act of 1904, passed on March, 21

ob
declined and suffered destruction after invasion of the was formulated on the basis of the recommendations

.w
White Huns in the 5th century. of the Indian University Commission of 1902. Curzon

or
dp
147. (a) Bold and impartial judiciary is the most important gave importance on improving the standard and quality

re
safeguard of liberty and no one can restrain it. One of of higher education.

ss
.c
the most important safeguards is that the person has 153. (a) After the spread of the Buddhist religion, Chinese

om
someone appointed with legal powers to represent them travelers came to India in big numbers to collect
even in extreme case of deprivation of liberty. religious books and to visit holy places of Buddhism.
148. (a) The philosophical school of law is the ancient school 154. (a) Sustainable development is development that meets
of law. It came into existence in 3rd century in Roman the needs of the present without compromising the
empire. ability of future generations to meet their own needs.
149. (c) Akbar celebrated his conquest of Rajputana by laying 155. (c) 156. (a)
the foundation of a new capital, 23 miles (37 km) W.S.W 157. (d) The Cripps mission was an attempt in late March 1942
of Agra in 1569. It was called Fatehpur Sikri ("the city by the British government to secure full Indian
of victory"). cooperation and support for their efforts in World War
150. (a) Vasco da Gama was a Portuguese explorer who II.
discovered the sea route to India from Europe through 158. (a)
the Cape of Good Hope. 159. (a) Hamida Banu Begam, also known as Haji Begam,
151. (a) The Unification of Karnataka refers to the formation of commenced the construction of Humayun tomb in 1569.
the Indian state of Karnataka, then named Mysore 160. (c) 161. (d) 162. (a) 163. (c) 164. (b)
State, in 1956 when several Indian states were created 165. (c) 166. (c) 167. (c) 168. (d) 169. (d)
by redrawing borders based on linguistic
170. (a) 171. (b) 172. (d) 173. (d) 174. (d)
demographics
175. (d) 176. (b) 177. (b) 178. (b) 179. (b)
180. (d) 181. (c) 182. (b) 183. (d) 184. (b)
w
w
w
.y
ou
Geography 15

rs
m
2

ah
bo
ob
.w
GEOGRAPHY

or
dp
re
ss
CHAPTER

.c
om
1 Seismic sea waves which approach the coasts at greater 12. The age of the Earth can be determined by
force are known as (SSC CGL 1st Sit. 2010) (SSC CGL 2nd Sit. 2010)
(a) Tides (b) Tsunami (a) Geological Time Scale
(c) Current (d) Cyclone (b) Radio-Metric Dating
2. The land of maximum biodiversity is(SSC CGL 1st Sit. 2010) (c) Gravity method
(a) Tropical (b) Temperate (d) Fossilization method
(c) Monsoonal (d) Equatorial 13. How much of the Earth's land surface is desert?
3. Indian Standard Time relates to (SSC CGL 1st Sit. 2010) (SSC CGL 1st Sit. 2011)
(a) 75.5°E longitude (b) 82.5°E longitude (a) 1/10th (b) 1/5th
(c) 1/3 rd (c) 1/6th
(c) 90.5°E longitude (d) 0° longitude
14. River Indus originates from: (SSC CGL 1st Sit. 2011)
4. Which is the second nearest star to the Earth after the Sun?
(a) Hindukush range (b) Himalayan range
(SSC CGL 1st Sit. 2010)
(c) Karakoram range (d) Kailash range
(a) Vega (b) Sirius
15. The lowest layer of the atmosphere is:
(c) Proxima Centauri (d) Alpha Centauri
(SSC CGL 1st Sit. 2011)
5. The forest in Sundarban is called (SSC CGL 1st Sit. 2010)
(a) Stratosphere (b) Thermosphere
(a) Scrub jungle (b) Mangrove
(c) Troposphere (d) Mesosphere
(c) Deciduous forest (d) Tundra 16. The Konkan Railway connects:
6. Which of the following Indonesian regions was a victim of (SSC CGL 1st Sit. 2011)
massive earthquake in 2004 ? (SSC CGL 1st Sit. 2010) (a) Goa – Mangalore
(a) Irian Jaya (b) Sumatra (b) Roha – Mangalore
(c) Kalibangan (d) Java (c) Kanyakumari – Mangalore
7. The first non-stop air-conditioned 'DURANTO' train was (d) Kanyakumari – Mumbai
flagged off between (SSC CGL 2nd Sit. 2010) 17. Bark of this tree is used as a condiment–
(a) Sealdah – New Delhi (SSC CGL 1st Sit. 2011)
(b) Mumbai – Howrah (a) Cinnamon (b) Clove
(c) Bangalore – Howrah (c) Neem (d) Palm
(d) Chennai – New Delhi 18. The atmospheric air is held to the Earth by:
8. Which one of the following states does not form part of (SSC CGL 1st Sit. 2011)
Narmada River basin ? (SSC CGL 2nd Sit. 2010) (a) gravity
(a) Madhya Pradesh (b) Rajasthan (b) winds
(c) Gujarat (d) Maharashtra (c) clouds
9. Soil erosion on hill slopes can be checked by (d) rotation of the Earth
(SSC CGL 2nd Sit. 2010) 19. The common tree species in nilgiri hills is
(a) Afforestation (b) Terrace cultivation (SSC CGL 2nd Sit. 2010)
(c) Strip cropping (d) Contour ploughing (a) Sal (b) Pine
10. Who coined the word 'Geography'? (SSC CGL 2nd Sit. 2010) (c) Eucalyptus (d) Teak
20. Which is the largest living bird on Earth?
(a) Ptolemy (b) Eratosthenese
(a) Emu (b) Ostrich
(c) Hacataus (d) Herodatus
(c) Albatross (d) Siberian Crane
11. Which of the following is called the 'ecological hot spot of
21. Rihand Dam Project provides irrigation to
India'? (SSC CGL 2nd Sit. 2010)
(SSC CGL 2nd Sit. 2011)
(a) Western Ghats
(a) Gujarat and Maharashtra
(b) Eastern Ghats (b) Orissa and West Bengal
(c) Western Himalayas (c) Uttar Pradesh and Bihar
(d) Eastern Himalayas (d) Kerala and Karnataka
w
w
w
.y
ou
16 Geography

rs
m
22. Which is the longest irrigation canal in India? 34. Earth is a very big magnet. In which direction does it

ah
bo
(SSC CGL 2nd Sit. 2011) magnetic field extend? (SSC CGL 2nd Sit. 2012)

ob
(a) Sirhind Canal (b) Yamuna Canal (a) west to east (b) north to south

.w
(c) Indira Gandhi Canal (d) East Kosi Canal (c) south to north (d) east to west

or
dp
23. 'Loktak' is a (SSC CGL 2nd Sit. 2011) 35. The most densely populated state in India is:

re
(a) Valley (b) Lake (SSC CGL 2nd Sit. 2012)

ss
.c
(c) River (d) Mountain Range (a) Kerala (b) Uttar Pradesh

om
24. Which city receives the highest cosmic radiation amongst (c) West Bengal (d) Tamil nadu
the following? (SSC CGL 2nd Sit. 2011) 36. The biggest planet in the solar system is:
(a) Chennai (b) Mumbai (SSC CGL 2nd Sit. 2012)
(c) Kolkata (d) Delhi (a) Venus (b) Jupiter
25. The HYV programme in India is also called as (c) Saturn (d) Uranus
(SSC CGL 1st Sit. 2012) 37. Peninsular India has the following zonal soil types:
(a) Traditional Agriculture (SSC CGL 2nd Sit. 2012)
(b) New Agricultural Strategy (a) Red and yellow soil (b) Forest soil
(c) White Revolution (c) Saline soil (d) Alluvial soil
(d) Blue Revolution 38. The land between two rivers is called
26. Railway coaches are manufactured at (SSC CGL 1st Sit. 2012)
(SSC CGL 1st Sit. 2012) (a) Natural Levees (b) Alluvial Cones
(a) Jamshedpur (b) Chittaranjan (c) Braided Stream (d) Doab
(c) Perambur (d) Varanasi 39. Which year is called as the ‘Demographic divide’ ?
27. A series of lines connecting places having a quake at the (SSC CGL 1st Sit. 2012)
same time are called (SSC CGL 1st Sit. 2012) (a) 1941 (b) 1921
(a) Homoseismal lines (b) Seismolines (c) 1901 (d) 1931
(c) Coseismal lines (d) Isoseismal lines 40. Which country has a high density of population?
(SSC CGL 1st Sit. 2012)
28. What would be the impact of global warming on mangrove
(a) India (b) Canada
forests? (SSC CGL 1st Sit. 2012)
(c) Sweden (d) Greenland
(a) They will grow more luxurious
41. The iron and steel plant in Chhattisgarh is at
(b) Large areas of mangroves will be submerged
(SSC CGL 1st Sit. 2012)
(c) Their role as carbon sinks will become more important
(a) Burnpur (b) Salem
(d) Both (a) and (c) above
(c) Bhilai (d) Bokaro
29. The brightest planet is (SSC CGL 1st Sit. 2012)
42. The leading sesame producing country in the world is
(a) Venus (b) Mercury
(SSC CGL 1st Sit. 2012)
(c) Jupiter (d) Mars
(a) Mexico (b) U.S.A.
30. Wheat, Barley, Lemon, Orange, rye and pearl millet belong
(c) China (d) India
to: (SSC CGL 2nd Sit. 2012)
43. Which one of the following is not correctly matched?
(a) the same plant family
(SSC CGL 2nd Sit. 2012)
(b) two plant families
(a) Darjeeling - West Bengal
(c) three plant families
(b) Mount Abu - Rajasthan
(d) four plant families (c) Kodaikanal - Tamil Nadu
31. A plant known only in cultivation having arisen under (d) Simla - Uttar Pradesh
domestication is referred to as: 44. The earth is at its maximum distance from the Sun on
(SSC CGL 2nd Sit. 2012) (SSC CGL 2nd Sit. 2012)
(a) Scion (b) Cultigen (a) January 30th (b) December 22nd
(c) Cultivar (d) Clone (c) September 22nd (d) July 4th
32. The iron and steel plant in Bihar is at: 45. Consider the following pairs:
(SSC CGL 2nd Sit. 2012) (SSC CGL 2nd Sit. 2012)
(a) Visakhapatnam (b) Bokaro Tributary Main River
(c) Burnpur (d) Vijay Nagar 1. Chambal : Yamuna
33. The state which has registered the highest population 2. Sone : Narmada
growth rate according to 2001 census is: 3. Manas : Brahmputra
(SSC CGL 2nd Sit. 2012) Which one of the pairs given above is/are correctly matched?
(a) Kerala (b) Uttar Pradesh (a) 1, 2 and 3 (b) 1 and 2
(c) Nagaland (d) Sikkim (c) 2 and 3 (d) 3 only
w
w
w
.y
ou
Geography 17

rs
m
46. The total population divided by available arable land area is 57. The programme of ‘Operation Flood’ was concentrated on

ah
bo
referred to as (SSC CGL 2nd Sit. 2012) (SSC CGL 2nd Sit. 2013)

ob
(a) Population density (b) Nutritional density (a) increasing irrigation facilities.

.w
or
(c) Agricultural density (d) Industrial density (b) flood control.

dp
47. The eastward continuation of the Brazil current is called : (c) increasing the milk production.

re
(SSC CGL 1st Sit. 2013)

ss
(d) increase the flood grains production.

.c
(a) North Atlantic drift 58. According to Ferrel’s law (Coriolis Force) winds change their

om
(b) South Atlantic drift direction (SSC CGL 2nd Sit. 2013)
(c) Counter Equatorial drift (a) Towards left in Northern hemisphere and towards right
(d) West Atlantic drift in Southern hemisphere.
48. Ice glacier’s melting is a common phenomenon linked to the (b) Towards right in Northern hemisphere and towards left
rise in a seawater level. The glaciers are predominantly in Southern hemisphere.
present in (SSC CGL 2nd Sit. 2013) (c) Towards right in both the hemisphere.
(a) Greenland (b) Antarctica (d) Towards left in both the hemisphere.
(c) Himalayas (d) Arctic 59. Which one of the following atmospheric layers absorb
49. Which hill station is called as the ‘Queen of the Satpuras’ ultraviolet rays of the sun? (SSC CGL 2nd Sit. 2013)
(SSC CGL 2nd Sit. 2013) (a) Troposphere (b) Stratosphere
(a) Pachmarhi (b) Nilgiri (c) Ionosphere (d) Ozonosphere
(c) Mahenderagiri (d) Cardamom 60. The drainage pattern developed on folded sedimentary rock
50. Which national highway connects Delhi and Kolkata via is termed as (SSC CGL 2nd Sit. 2013)
Mathura and Varanasi? (SSC CGL 2nd Sit. 2013) (a) Trellis (b) Dendritic
(a) NH 4 (b) NH 2 (c) Radial (d) Deranged
(c) NH 10 (d) NH 6 61. Which one of the following is not a line of demarcation
51. The country where drip irrigation is more efficiently used is between two countries ? (SSC CGL 2nd Sit. 2013)
(SSC CGL 2nd Sit. 2013) (a) Durand Line (b) Mac Mahon Line
(a) India (b) Israel (c) Plimsoll Line (d) Maginot Line
(c) Sri Lanka (d) England 62. Water potential remains lowest in (SSC CGL 2nd Sit. 2013)
52. Which river in India flows in a rift-valley? (a) Water plants (b) Woody plants
(SSC CGL 1st Sit. 2013) (c) Succulents (d) Halophytes
(a) Narmada (b) Krishna 63. The free living bacterium in the soil which increases the the
(c) Cauvery (d) Tapti yield of rice is (SSC CGL 2nd Sit. 2013)
53. A narrow strip of land that connects two larger land masses (a) Rhizobium (b) Azotobacter
is called (SSC CGL 1st Sit. 2013) (c) Acetobacter (d) Anabaena
(a) Cape (b) Isthmus 64. Frontal cyclones occur characteristically in
(c) Strait (d) Peninsula (SSC CGL 2nd Sit. 2013)
54. Maps on large scale, representing both natural and man- (a) Equatorial region
made features are called (SSC CGL 1st Sit. 2013) (b) Tropical region
(a) Thematic maps (c) Mid-latitudinal region
(b) Atlas maps (d) Polar region
(c) Wall maps 65. "Yosemite" is a (SSC CGL 1st Sit. 2013)
(d) Topographic maps (a) River (b) Peak
55. The angle between the magnetic meridian and the (c) Waterfall (d) Dam
geographical meridian at a place is (SSC CGL 1st Sit. 2013) 66. Where is the shore based steel plant located?
(a) Declination (b) Latitude (SSC CGL 2nd Sit. 2013)
(c) Azimuth (d) Dip (a) Tuticorin (b) Salem
56. An irrigation project is categorized as a major project if it (c) Vishakhapatnam (d) Mangalore
covers a cultivable command area of 67. Which two of the following are connected by the North
(SSC CGL 2nd Sit. 2013) South corridor? (SSC CGL 2nd Sit. 2013)
(a) less than 2,000 hectares (a) Srinagar and Kanyakumari
(b) 2,000 to 10,000 hectares (b) Mumbai and Chennai
(c) above 10,000 hectares (c) Amritsar and Kolkata
(d) all the above (d) Hyderabad and Bhopal
w
w
w
.y
ou
18 Geography

rs
m
68. Which of the following statements is correct? 78. Which of the following river does not originate in Indian

ah
bo
(SSC CGL 2014) territory ? (SSC CGL 2015)

ob
(a) Mahadeo hills are in the west of Maikala hills. (a) Mahanadi (b) Brahmaputra

.w
(b) Mahadeo hills are the part of Karnataka Plateau. (c) Satluj (d) Ganga

or
dp
(c) Mahadeo hills are in the east of Chhotanagpur Plateau. 79. Which one among the following industries in the maximum

re
(d) Mahadeo hills are the part of Aravalli ranges. consumer of water in India ? (SSC CGL 2015)

ss
.c
69. Which one of the following pairs is not correctly matched? (a) Textile (b) Engineering

om
(SSC CGL 2014) (c) Paper and Pulp (d) Thermal Power
(a) Hevea Tree—Brazil 80. Choose the correct option which represents the arrangement
(b) Sumatra Storm—Malaysia of atmospheric layers. (SSC CGL 2015)
(c) Kajan River—Borneo (a) Troposphere, Stratosphere, Mesosphere, Ionosphere,
(d) Dekke Toba fish—Brazil Exosphere
70. Which of the following resources is renewable one? (b) Mesosphere, Ionosphere, Exosphere, Troposphere,
(SSC CGL 2014) Stratosphere
(a) Uranium (b) Coal (c) Ionosphere, Exosphere, Mesosphere, Troposphere,
(c) Timber (d) Natural Gas Stratosphere
(d) Exosphere, Troposphere, Ionosphere, Mesosphere,
71. Soil erosion can be prevented by (SSC CGL 2014)
Stratosphere
(a) Increasing bird population
81. Which of the following options correctly explains the term
(b) Afforestation
‘heat budget’? (SSC CGL 2015)
(c) Removal of vegetation
(a) It is the amount of heat which the surface of earth
(d) Overgrazing
receives form the sun.
72. Natural sources of air pollution are (SSC CGL 2014)
(b) It is the radiation from the earth in the form of long
(a) Forest fires
waves
(b) Volcanic eruptions
(c) It is a mode of transfer of heat through matter by
(c) Dust storm
molecular activity.
(d) Smoke from burning dry leaves
(d) It is the balance between incoming and outgoing
73. The 'graded profile' of a river course is a radiation.
(SSC CGL 2015) 82. The layer of atmosphere close to the earth's surface is called:
(a) smooth curve in the upper course (SSC CGL 2015)
(b) smooth curve in the middle course (a) Exosphere (b) Ionosphere
(c) smooth curve in the lower course (c) Stratosphere (d) Troposphere
(d) smooth curve from source to mouth 83. Which of the following plant shows chloroplast dimorphism?
74. Sink hole is a phenomenon of _______ topography. (SSC CGL 2015)
(SSC CGL 2015) (a) Sugarcane (b) Sugar beet
(a) Desert (b) Tundra (c) Rice (d) Wheat
(c) Karst (d) Plain 84. Day and Night are equal at the : (SSC CGL 2015)
75. Kerala is famous for the cultivation of (SSC CGL 2015) (a) Prime Meridian (b) Poles
1. Coconut 2. Black pepper (c) Equator (d) Antarctic
3. Rubber 4. Rice 85. Evergreen type forests are found in : (SSC CGL 2015)
(a) 1, 2 and 4 (b) 2, 3 and 4 (a) Mediterranean region
(c) 1 and 4 (d) 1, 2 and 3 (b) Monsoon climatic area
76. The longest continental Railway in the world is (c) Desert region
(SSC CGL 2015) (d) Equatorial region
(a) Trans Siberian Railway 86. The ash–grey soils of high latitude coniferous forests are
(b) Canadian Pacific Railway known as : (SSC CGL 2015)
(c) Canadian National Railway (a) Grey–Brown soils (b) Red and Yellow soils
(d) Trans Atlantic Railway (c) Tundra soils (d) Podsols
77. Which of the following is FALSE with respect to rain water 87. Equinox occurs when the sun is vertically above
harvesting? (SSC CGL 2015) (SSC CGL 1st Sit. 2016)
(a) It helps raising water table (a) Tropic of Capricorn
(b) It helps meet rising water demand (b) Tropic of Cancer
(c) It increases run–off losses (c) Poles
(d) It is a device of water conservation (d) Equator
w
w
w
.y
ou
Geography 19

rs
m
88. Among the world oceans, which ocean is having the widest 97. With which country, India has the longest international

ah
bo
continental shelf ? (SSC CGL 1st Sit. 2016) boundary? (SSC CGL 1st Sit. 2016)

ob
(a) Antarctic ocean (b) Arctic Ocean (a) Nepal (b) Pakistan

.w
or
(c) Indian Ocean (d) Atlantic ocean (c) China (d) Bangladesh

dp
98. Which State in India has the largest coastline?

re
89. Which is largest peninsular river in India?

ss
(SSC CGL 1st Sit. 2016)
(SSC CGL 1st Sit. 2016)

.c
(a) Tamil Nadu (b) Andhra Pradesh

om
(a) Krishna (b) Godavari
(c) Gujarat (d) West Bengal
(c) Cauvery (d) Mahanadi
99. Jog falls in Karnataka is located over which river?
90. Red soil is normally found in India in which regions?
(SSC CGL 1st Sit. 2016)
(SSC CGL 1st Sit. 2016)
(a) Kaveri (b) Godavari
(a) Eastern Region only (c) Sharavati (d) Krishna
(b) Southern Region only 100. Which one of the following areas of India is covered by
(c) Eastern & Southern part of the Deccan Plateau tropical evergreen forest? (SSC CGL 1st Sit. 2016)
(d) None of these (a) Semi-arid areas of Gujarat
91. Limestone is a raw material used by which industry? (b) Eastern Ghats
(SSC CGL 1st Sit. 2016) (c) Western Ghats
(a) Aluminium (b) Fertilizers (d) Madhya Pradesh
(c) Cement (d) Petrochemicals 101. Which of the following State is surrounded by Bangladesh
from three sides? (SSC CGL 1st Sit. 2016)
91. Mount Abu is a hill station located in _____ ranges.
(a) Nagaland (b) Assam
(SSC CGL 1st Sit. 2016)
(c) Arunachal Pradesh (d) Tripura
(a) Vindhya (b) Satpuda
102. Earth received heat from the sun is known as:
(c) Aravalli (d) Sahyadri
(SSC CHSL 2012)
92. The only perennial river in Peninsular India is _____ . (a) Insolation (b) Infrared heat
(SSC CGL 1st Sit. 2016) (c) Solar radiation (d) Thermal radiation
(a) Godavari (b) Kaveri 103. Spot the odd item in the following: (SSC CHSL 2012)
(c) Krishna (d) Bhima (a) Red sea (b) Black sea
93. When does solar eclipse take place ? (c) Caspian sea (d) Dead sea
(SSC CGL 1st Sit. 2016) 104. Match correctly the following, deserts and their location by
(a) When the sun is between the moon and earth choosing the correct response: (SSC CHSL 2013)
(b) When the earth is between the moon and sun Desert Location
(c) When the moon is between the sun and earth a. Kalahari 1. South America
(d) When the moon does not lie on the line joining the sun b. Atacama 2. Australia
and earth c. Thar 3. Africa
d. Great Victoria 4. Asia
94. Strait of Gibraltar connects which of the following?
(a) a-3, b-1, c-4, d-2 (b) a-2, b-3, c-1, d-4
(SSC CGL 1st Sit. 2016)
(c) a-4, b-3, c-2, d-1 (d) a-3, b-2, c-1, d-4
(a) Red Sea-Mediterranean Sea
105. The longest river of peninsular India is
(b) Red Sea-Arabian Sea
(SSC CHSL 2013)
(c) Atlantic Ocean-Mediterranean Sea
(a) Godavari (b) Krishna
(d) Mediterranean Sea-Black Sea (c) Kaveri (d) Narmada
95. The largest solar power plant in India is located at 106. The Himalayan mountain range is an example of
(SSC CGL 1st Sit. 2016) (SSC CHSL 2013)
(a) Nagercoil (b) Jaisalmer (a) Fold mountain (b) Volcanic mountain
(c) Madhapur (d) Rann of Kutch (c) Residual mountain (d) Block mountain
96. On which river is the Tehri dam built? 107. Which one of the following is a warm ocean current ?
(SSC CGL 1st Sit. 2016) (SSC CHSL 2013)
(a) Alakananda (b) Bhagirathi (a) Gulf Stream (b) Kurile
(c) Ganga (d) Hooghly (c) Canary (d) Labrador
w
w
w
.y
ou
20 Geography

rs
m
108. The main advantage of Rain Water Harvesting (RWH) is 119. How much of world's surface is covered by water ?

ah
bo
(SSC CHSL 2013) (SSC CHSL 2015)

ob
(a) Avoid soil erosion (a) 70% (b) 80%

.w
or
(b) Recharge ground water (c) 25% (d) 55%

dp
(c) Avoid floods

re
120. Which of the following Scientist proved that the path of

ss
(d) Reduce the loss of water each planet around the Sun is elliptical ?(SSC CHSL 2015)

.c
om
109. India is the largest producer and exporter of (a) Galileo (b) Newton
(SSC CHSL 2014)
(c) Copernicus (d) Kepler
(a) Cotton (b) Copper
121. Which of the following rivers originates from Trans
(c) Tea (d) Mica
Himalayas ? (SSC CHSL 2015)
110. The soils which are rich in Calcium are known as
(a) Sindu (b) Saraswathi
(SSC CHSL 2014)
(c) Ganga (d) Yamuna
(a) Pedocals (b) Pedalfers
122. The most suitable soil for the production of cotton is ?
(c) Podsols (d) Laterits
111. Cultivable land is defined as (SSC CHSL 2014) (SSC CHSL 2015)
(a) land actually under crops (a) Black lava soil (b) Loamy soil
(b) Culitivable waste land + fallow land (c) Well drained soil (d) Alluvial soil
(c) Old fallow lands + current fallow lands 123. The largest producer of Lignite in India is:
(d) Total fallow lands + net sown area (SSC CHSL 2015)
112. Which of the following is the largest Biosphere Reserves of (a) Kerala (b) Rajasthan
India ? (SSC CHSL 2014) (c) Tamil Nadu (d) Gujarat
(a) Nilgiri (b) Nandadevi 124. Sex–ratio is calculated as : (SSC CHSL 2015)
(c) Sundarbans (d) Gulf of Mannar (a) No of females per 1,000 males in a Country
113. With what bio-region is the term "Steppe" associated ? (b) No of males per 1,000 females in a Country
(SSC CHSL 2014)
(c) No. of children per 1,000 people in a Country.
(a) Grasslands (b) Tropical Forests
(d) No of people per 1,000 children in a Country.
(c) Savanna (d) Coniferous Forests
125. Which state of India has made rain water harvesting
114. About how much of the world's land area is tropical
compulsory for all houses ? (SSC CHSL 2015)
rainforest ? (SSC CHSL 2014)
(a) 2 percent (b) 7 percent (a) Tamil Nadu (b) Punjab
(c) 10 percent (d) 15 percent (c) Haryana (d) Maharashtra
115. The world's growing appetite for what food product is a 126. The five key indicators of global climate change of our
leading cause of tropical deforestation ? (SSC CHSL 2014) planet are : (SSC CHSL 2015)
(a) Pork (b) Sugar (a) Sea–level, Rising temperatures, Rainfall, Nitrogen and
(c) Lamb (d) Beef Actic Sea ice
116. What is Kyoto Protocol ? (SSC CHSL 2015) (b) None of the options
(a) It is an agreement among countries to takes steps for (c) Arctic sea ice, carbon dioxide, Global temperature, Sea
planting trees to control pollution level and land ice.
(b) It is an agreement among countries to start using (d) Antartic sea ice, Oxygen, Rainfall, Drought and Sea level
nuclear energy 127. The area reserved for the welfare of wild life is called
(c) It is an agreement among countries to take steps for
(SSC CHSL 2015)
reducing global warming
(a) Sanctuary (b) Botanical garden
(d) It is an agreement among countries to takes steps for
reducing acid rain. (c) Forest (d) National pak
117. The largest reservoir of fresh water is : (SSC CHSL 2015) 128. The National Green Tribunal deals with cases relating to ?
(a) Ground Water (b) Ponds (SSC CHSL 2015)
(c) Lakes (d) Glaciers (a) Criminal offenses
118. The most abundant element by number in the living system (b) Issues relating to protection and conservation of
is : (SSC CHSL 2015) historical monuments.
(a) Hydrogen (b) Oxygen (c) Civil cases
(c) Carbon (d) Nitrogen (d) Environmental protection and conservation of forests.
w
w
w
.y
ou
Geography 21

rs
m
139. Consider the following sea-ports:

ah
129. The biggest oil spill in world history took place in the ?

bo
(SSC CHSL 2015) (SSC Multitasking 2014)

ob
1. Chennai 2. Machilipatnam

.w
(a) Persian Gulf (b) Caspian Sea

or
(c) Mediterrarean Sea (d) South China sea 3. Nagapattinam 4. Tuticorin

dp
The correct sequence of these ports from north to south is

re
130. Among the foollowing which country has the highest life

ss
(a) 1, 3, 2, 4 (b) 2, 1, 4, 3

.c
expectancy ? (SSC CHSL 2015)

om
(c) 1, 2, 4, 3 (d) 2, 1, 3, 4
(a) USA (b) Switzerland
140. Santa Cruz is (SSC Multitasking 2014)
(c) Japan (d) Denmark
(a) an International airport in Chennai
131. The redness in atmosphere at Sunrise and Sunset is due to:
(b) a Domestic airport in Chennai
(SSC CHSL 2015)
(c) an International airport in Mumbai
(a) Dispersion of light (b) Scattering of light (d) a Domestic airport in Mumbai
(c) Refraction of light (d) Reflection of light 141. The Almatti Dam is constructed on the river
132. Which one of the following tribes practices pastoral (SSC Multitasking 2014)
nomadism ? (SSC CHSL 2015) (a) Tungabhadra (b) Krishna
(a) Boro (b) Masai (c) Kaveri (d) Sileru
(c) Pygmies (d) Eskimo 142. The project similar to T.V.A. (Tennessee Valley Authority)
133. Select the High Yielding Varieties of seed-crops developed of U.S.A. in India is (SSC Sub. Ins. 2012)
under Green Revolution in India. (a) Damodar Valley Project
(SSC Multitasking 2013) (b) Mahanadi Delta Project
(a) Wheat, Rice, Sugarcane, Pulses and Maize. (c) Ramganga Multipurpose Project
(b) Rice, Wheat, Pulses, Oil seeds and Sugarcane (d) Idukki Project
(c) Maize, Black-gram, Jowar, Coffee and Tea. 143. Petrology is the study of (SSC Sub. Ins. 2012)
(a) rocks (b) soils
(d) Rice, Wheat, Jowar, Bajra and Maize.
(c) earth (d) minerals
134. India is the largest producer and consumer of
144. Match the following : (SSC Sub. Ins. 2012)
(SSC Multitasking 2013)
A. Rosewood 1. Mangrove forests
(a) Sugar (b) Paddy
B. Bushes 2. Alpine forests
(c) Tea (d) Coffee C. Birch 3. Deciduous forests
135. The busiest rail section in respect to goods transportation is D. Sundari tree 4. Dry forests
(SSC Multitasking 2013) A B C D A B C D
(a) Mumbai – Chennai section (a) 4 3 2 1 (b) 3 4 2 1
(b) Delhi – Kolkata section (c) 1 2 3 4 (d) 2 3 4 1
(c) Kolkata – Chennai section 145. During the period of South-West monsoon, Tamil Nadu
(d) Delhi – Mumbai section remains dry because : (SSC Sub. Ins. 2012)
136. The Tropic of Cancer does not pass through (a) the winds do not reach this area
(SSC Multitasking 2013) (b) there are no mountains in this area
(c) it lies in the rain-shadow area
(a) China (b) Myanmar
(d) the temperature is too high to let the winds cool down
(c) Nepal (d) Bangladesh
146. Which one of the following is not a line of demarcation
137. The country in East Asia which is most conspicuous for its
between two countries? (SSC Sub. Ins. 2013)
decreasing population growth since 1981 is
(a) International Date Line
(SSC Multitasking 2013) (b) MacMahon Line
(a) Japan (b) South Korea (c) Radcliffe Line
(c) Thailand (d) China (d) Durand Line
138. The recently discovered field with oil potential in Krishna- 147. Which state in India occupies the same ranking position in
Godavari Basin is called (SSC Multitasking 2014) respect of area and population? (SSC Sub. Ins. 2013)
(a) Ravva Offshore Block (b) Golkunda Block (a) Manipur (b) Meghalaya
(c) Bombay High (d) Telangana Block (c) Nagaland (d) Gujarat
w
w
w
.y
ou
22 Geography

rs
m
148. Lake formed in a cut off river, meander is called: 159. The total number of planets revolving around the sun are:

ah
bo
(SSC Sub. Ins. 2013) (SSC Sub. Ins. 2015)

ob
(a) Playa Lake (b) Meteoric Lake (a) Seven (b) Eight

.w
or
(c) Ox-Bow Lake (d) Crater Lake (c) Five (d) Six

dp
160. Which of the following is the major copper producing

re
149. Which is the leading state in India in "Ship Breaking"

ss
industry? (SSC Sub. Ins. 2013) country? (SSC Sub. Ins. 2015)

.c
om
(a) Tamil Nadu (b) Maharashtra (a) Indonesia (b) Russia
(c) Sri Lanka (d) Chile
(c) Gujarat (d) West Bengal
161. The largest forest which covers 25 percent of the world’s
150. Contour line is the imaginary line joining places of equal:
Forest land is: (SSC Sub. Ins. 2015)
(SSC Sub. Ins. 2013)
(a) Monsoon Forest
(a) Rainfall (b) Humidity
(b) Temperate forests of Europe
(c) Elevation (d) Temperature
(c) The tropical Rain forest
151. Which is the first state in India to enact the Food Security (d) Taiga forest of Siberia
Act? (SSC Sub. Ins. 2013)
162. The method of age determination called "radioactive uranium
(a) Chhattisgarh (b) Gujarat dating" cannot be used to determine the age of
(c) Tamil Nadu (d) Bihar (SSC Sub. Ins. 2016)
152. The first short based integrated steel plant in the country is (a) the earth (b) rocks
(SSC Sub. Ins. 2014) (c) minerals on earth (d) fossils and plant bodies
(a) Vijaynagar (b) Salem 163. Altocumulus clouds occur at altitude of
(c) Vishakapatnam (d) Bhadravati (SSC Sub. Ins. 2016)
153. Name the condition which influences the development of (a) 10000 m - 12000 m (b) 6000 m - 10000 m
plants into distinctive forms. (SSC Sub. Ins. 2014) (c) 1000 m - 1800 m (d) 2000 m - 6000 m
(a) Climatic conditions 164. The minor irrigation schemes have Cultural Command Area
(b) Soil conditions (CCA) upto (SSC Sub. Ins. 2016)
(c) Environmental conditions (a) 3000 hectares (b) 2500 hectares
(d) Social conditions (c) 1500 hectares (d) 2000 hectares
154. Agriculture should serve as an instrument of income, 165. Which of the following Islands is under direct threat of
livelihood and opportunity to the local community - this rising sea levels due to climate change has appealed for
statement is given by (SSC Sub. Ins. 2014) help from European leaders? (SSC Sub. Ins. 2016)
(a) Palau (b) Tuvalu Islands
(a) Dr Madhavan Nair (b) Dr Manmohan Singh
(c) Marshall Islands (d) Nauru
(c) Dr Abdul Kalam (d) Dr M S Swaminathan
166. Which of the following regions is called the 'granary of the
155. The forests which act as barriers against cyclones are:
world'? (SSC Steno. 2013)
(SSC Sub. Ins. 2015)
(a) Temperate grasslands
(a) Evergreen forests (b) Mangrove forests
(b) British type vegetation
(c) Monsoon forests (d) Alpine Forest (c) Laurentian type vegetation
156. The resources which are obtained from biosphere and have (d) Tropical grasslands
life are : (SSC Sub. Ins. 2015) 167. Spot the odd item in the following: (SSC Steno. 2013)
(a) Renewable resources (b) Potential resources (a) Prairies (b) Pampas
(c) Biotic resources (d) Abiotic resources (c) Selvas (d) Sundarban
157. In a solar or lunar eclipse, the region of earth’s shadow is 168. Which of the following rovers of India flows in rift-valley?
divided into. (SSC Sub. Ins. 2015) (SSC Steno. 2013)
(a) Four parts (b) Five parts (a) Kaveri (b) Tapti
(c) Two parts (d) Three parts (c) Son (d) Ken
158. What does the word ‘amphibian’ mean ? 169. The largest coral reef in the world is found near the Eastern
(SSC Sub. Ins. 2015) coast of (SSC Steno. 2013)
(a) Two lives (b) Four lives (a) Japan (b) China
(c) Three lives (d) One life (c) Cuba (d) Australia
w
w
w
.y
ou
Geography 23

rs
m
170. The strong, cold, ley wind blowing in the polar region is 179. Which of the following is related to horticulture ?

ah
bo
known as (SSC Steno. 2013) (SSC Steno. 2016)

ob
(a) Chinook (b) Bora (a) Green revolution (b) White revolution

.w
or
(c) Blizzard (d) Harmattan (c) Operation flood (d) Golden revolution

dp
171. The lines joining places of equal values of horizontal

re
180. The island of Honshu in Japan is famous for

ss
component of Earth's magnetic field are called

.c
(SSC Steno. 2016)

om
(SSC Steno. 2014)
(a) Oil (b) Diamonds
(a) isogonic lines (b) isodynamic lines
(c) Coal (d) Iron ore
(c) aclinic lines (d) isoclinic lines
172. Which one among the following states is smallest in area? 181. 40°N latitude acts as demarcation line between
(SSC Steno. 2014) (SSC Steno. 2016)
(a) Gujarat (b) Karnataka (a) North and South Korea
(c) Tamil Nadu (d) Andhra Pradesh (b) USA and Canada
173. Highest coffee growing state in India is(SSC Steno. 2014) (c) North add South Vietnam
(a) Karnataka (b) Uttar Pradesh (d) Egypt and Sudan
(c) Maharashtra (d) Tamil Nadu 182. Lakes which contain high concentration of humic acid in
174. The rootless plant is (SSC Steno. 2014) water is (SSC Steno. 2016)
(a) lemna (b) banana (a) Desert salt lakes (b) Volcanic lakes
(c) ginger (d) lemon (c) Dystrophic lakes (d) Deep ancient lakes
175. The highest peak of the Eastern ghats of India is 183. The first person ever to reach the South Pole was
(SSC Steno. 2014) (SSC Steno. 2016)
(a) Shevroy hills (b) Mahendragiri (a) Magellan (b) merigo Vespucci
(c) Javadi hills (d) Annaimudi (c) Amundsen (d) Peary
176. Which country is the largest producer of raw jute? 184. It is reported that there is an ongoing decrease in the pH
(SSC Steno. 2014) value of ocean water because of global warming. It happens
(a) Bangladesh (b) Pakistan due to (SSC Steno. 2016)
(c) Myanmar (d) India (a) Larger uptake of atmospheric nitrogen by ocean water.
177. The largest proven oil reserve of the world lies in
(b) Lesser uptake of atmospheric nitrogen by ocean water.
(SSC Steno. 2016)
(c) Larger uptake of CO2 by ocean water
(a) Iran (b) Iraq
(d) Lesser uptake of CO2 by ocean water.
(c) Venezuela (d) Saudi Arabia
185. The largest tea growing country in the World is
178. Kundankulam Project is located in which state ?
(SSC Steno. 2016)
(SSC Steno. 2016)
(a) Telangana (b) Kerala (a) India (b) Brazil
(c) Karnataka (d) Tamil Nadu (c) Sri Lanka (d) China
w
w
w
.y
ou
24 Geography

rs
m
ah
HINTS & SOLUTIONS

bo
ob
.w
or
dp
re
1. (b) A tsunami, also known as a seismic sea wave, is a 32. (b) Bokaro Steel Plant – the fourth integrated plant in the

ss
.c
series of waves in a water body caused by the Public Sector – started taking shape in 1965 in

om
displacement of a large volume of water, generally in collaboration with the Soviet Union. Once in Bihar, it
an ocean or a large lake. is now in Jharkhand.
2. (a) 3. (b) 4. (c) 33. (c) Nagaland had recorded the country’s highest decadal
5. (b) The Sundarbans is a natural region comprising population growth of 64.41 per cent in 2001 and 56.08
southern Bangladesh and a part in the Indian state of in 1991 respectively.
West Bengal. It is the largest single block of tidal 34. (b) The inclination of the Earth's field is 90°at the North
halophytic mangrove forest in the world. Magnetic Pole and –90° at the South Magnetic Pole.
6. (b) 7. (a) 8. (b) 35. (c) Bihar with 1,102 persons per square kilometre is the
9. (b) Terrace cultivation, method of growing crops on sides most densely populated state of India as per the 2011
of hills or mountains by planting on graduated terraces Census. West Bengal comes second with 1,029
built into the slope. Though labour–intensive, the persons per kilometre as per the option.
method has been employed effectively to maximize 36. (b) Jupiter is the fifth planet from the sun and the largest
arable land area in variable terrains and to reduce soil planet within the Solar System.
erosion and water loss. 37. (a) Red and Yellow Soil in India is largely available in the
10. (b) 11. (a) 12. (b) 13. (b) 14. (d) Deccan Plateau.
15. (c) 16. (b) 17. (a) 18. (a) 19. (c) 38. (d) 39. (b) 40. (a) 41. (c) 42. (d)
20. (b) 21. (c) 22. (c) 23. (b) 24. (a) 43. (d) 44. (d) 45. (a) 46. (a)
25. (b) The New Agricultural Strategy is based on 47. (b) South Atlantic Current is an eastward ocean current,
concentration of high-yielding varieties of seeds and fed by the Brazil Current. That fraction of it which
complementary inputs on selected water-as-sured reaches theAfrican coast feeds the Benguela Current.
areas. It is continuous with the northern edge of the Antarctic
26. (c) Passenger coaches are manufactured at three principal Circumpolar Current.
places : Integral Coach Factory (ICF) at Perambur, 48. (b) It is mainly cause crbyo phenomena: global warming
Railway Coach Factory (RCF) at Kapurthala, And and ozone depletion. Polar ice caps in Antarctica and
Bharat Earth Movers Ltd. (BEML) at Bangalore. other places are–melting fast. This has led to increase
27. (a) Homoseismal lines is the line on the Earth's surface in sea level over the years.
connecting points where the seismic wave arrives, 49. (a) Panchmarhi a beautiful hill station in Madhya Pradesh
generated by an earthquake, at the same time. is known as Queen of Satpura because of its mystic
28. (d) Mangrove wetlands are possible sinks/sources for beauty, it is a known destination for Bollywood film
carbon dioxide and other related greenhouse gases. shootings.
Global warming may promote expansion of mangrove 50. (b) National Highway 2, commonly referred as Delhi–
forests to higher latitudes. Elevated CO2 concentration Kolkata Road, is a busy Indian National Highway that
may increase mangrove growth by stimulating runs through the states of Delhi, Haryana, Uttar
photosynthesis or improving water use efficiency. Pradesh, Bihar, Jharkhand, and West Bengal. It
29. (a) There are droplets of sulphuric acid and acidic crystals in constitutes a major portion of the historical Grand
the atmosphere of Venus. The smooth surfaces of these Trunk. Road along with NH 91 and NH 1 in India.
droplets of sulfuric acid and crystals reflect light very 51. (b) Drip irrigation is a form of irrigation that saves water
well, which is one reason why Venus is so bright. and fertilizer by allowing water to drip slowly to the
30. (b) Most cereals, including wheat, rye, rice, oats, barley, roots of many different plants, either onto the soil
corn, sorghum, millet, green bristlegrass and pearl millet surface or directly onto the root zone, through a network
belong to the botanical family Gramineae. Orange and of valves, pipes, tubing, and emitters.
lemon are citrus fruits. Drip irrigation was developed in Israel by Simcha Blass
31. (b) A cultigen is a plant that has been deliberately altered and his son Yeshayahu.
or selected by humans; it is the result of artificial 52. (a) Narmada is the only river in India that flows in a rift
selection. These "man-made" or anthropogenic plants valley, flowing west between the Satpura and Vindhya
are, for the most part, plants of commerce that are used ranges. The Tapti River and Mahi River also flow
in horticulture, agriculture and forestry. through rift valleys, but between different ranges.
w
w
w
.y
ou
Geography 25

rs
m
53. (b) An Isthmus is a narrow strip of land connecting two 68. (a) The Mahadeo Hills are a range of hills in Madhya

ah
bo
larger land areas, usually with water on either side. Pradesh state of central India. The hills form the central

ob
54. (d) A topographic map is a type of map characterized by part of the Satpura Range.

.w
large scale detail and quantitative representation of 69. (d) Dekke Toba fish in found in Indonesia. Lake Toba

or
dp
relief, usually using contour lines in modern mapping, (Indonesian: Danau Toba) is a lake and supervolcano.

re
but historically using a variety of methods. The lake is 100 kilometres long, 30 kilometres wide, and

ss
.c
55. (a) Magnetic declination is the angle between compass up to 505 metres (1,666 ft) deep. The fauna includes

om
north (the direction the north end of a compass needle several species of zooplankton and benthic animals.
points) and true north (the direction along the earth’s Since the lake is oligotrophic (nutrient-poor), the native
surface towards the geographic North Pole. fish fauna is relatively scarce, and the only endemics
56. (c) Above 10,000 hectares are Rasbora tobana.
57. (c) Operation Flood in India, a project of the National Dairy 70. (c) Timber means wood that we obtain from plants are
Development Board (NDDB) was the world's biggest called renewable in the sense that after cutting a tree if
dairy development program which made India, a milk- we plant another tree then it will grow up and again
deficient nation, the largest milk producer in the world, give us wood and timber whereas all other things given
surpassing the USA in 1998, with about 17 percent of are non-renewable, once the stock is finished we cannot
global output in 2010-11, which in 30 years doubled get more of it.
the milk available per person, and which made dairy 71. (a) Revegetation is often used to join up patches of natural
farming India's largest self-sustainable rural habitat that have been lost, and can be a very important
employment generator. All this was achieved not merely tool in places where much of the natural vegetation
by mass production, but by production by the masses. has been cleared. It is therefore particularly important
58. (b) the law explains that wind is deflected to the right in in urban environments, and research in Brisbane has
the Northern Hemisphere and to the left in the Southern shown that revegetation projects can significantly
Hemisphere, derived from the application of the Coriolis improve urban bird populations. The Brisbane study
effect to air masses. showed that connecting a revegetation patch with
59. (d) ozone layer, also called ozonosphere, region of the existing habitat improved bird species richness, while
upper atmosphere, between roughly 15 and 35 km (9 simply concentrating on making large patches of habitat
and 22 miles) above Earth's surface, containing was the best way to increase bird abundance.
relatively high concentrations of ozone molecules (O3). 72. (c) A dust storm or sand storm is a meteorological
60. (b) A dendritic drainage pattern refers to the pattern formed phenomenon common in arid and semi-arid regions.
by the streams, rivers, and lakes in a particular drainage Dust storms arise when a gust front or other strong
basin. It usually looks like the branching pattern of wind blows loose sand and dirt from a dry surface.
tree roots and it mainly develops in regions underlain 73. (d) A river profile is a curve which shows the slop of a
by homogeneous material. river from source to mouth.A stream flowing over
61. (c) Plimsol line is not a line of demarcation between two irregular terrain may have waterfalls, rapids and lakes
countries. along its course.Though the stream will wear away the
62. (d) A halophyte is a plant that grows in waters of high irregularities to leave a smoothly curving profile called
salinity, coming into contact with saline water through a graded profile.
its roots or by salt spray, such as in saline semi-deserts, 74. (c) Karst is a landscape formed from the dissolution of
mangrove swamps, marshes and sloughs, and soluble rocks including limestone, dolomite and
seashores. An example of a halophyte is the salt marsh gypsum. It is characterized by sinkholes, caves, and
grass Spartina alterniflora (smooth cordgrass). underground drainage systems.
63. (b) Azotobacter is a genus of usually motile, oval or 75. (d) Kerala is famous for the cultivation of coconut, tea,
spherical bacteria that form thick-walled cysts and may coffee, cashew and spices.
produce large quantities of capsular slime. 76. (a) The Trans-Siberian Railway is the longest railway line
64. (c) Extratropical cyclones, sometimes called mid-latitude in the world. It has a length of 9,289 km which connects
cyclones or wave cyclones, are a group of cyclones Moscow to Vladivostok.
defined as synoptic scale low pressure weather 77. (c) One of the Major objectives and advantages of
systems that occur in the middle latitudes of the Earth rainwater harvesting is to reduce run off loss.
(outside the tropics) not having tropical characteristics, 78. (b) Brahmaputra River is one of the largest rivers in the
and are connected with fronts and horizontal gradients world. The origin of Brahmaputra River is in
in temperature and dew point otherwise known as southwestern Tibet as the Yarlung River.
"baroclinic zones". 79. (d) Thermal Power Plants consumes maximum industrial
65. (c) 66. (c) 67. (a) water in India.
w
w
w
.y
ou
26 Geography

rs
m
80. (a) The current sequence is Troposphere-Stratosphere- 95. (c)

ah
bo
Mesosphere-Ionosphere-Exosphere 96. (b) Tehri Dam is a multi-purpose rock and earth-fill

ob
81. (d) The Earth and the atmosphere are heated by energy embankment dam on the Bhagirathi River near Tehri in

.w
from the sun. The atmospheric heat budget of the Earth Uttarakhand, India.

or
dp
depends on the balance between insolation and 97. (d) India has 15,106.7 km of land border and a coastline of

re
outgoing terrestrial radiation. 7,516.6 km including island territories. The length of

ss
.c
82. (d) The troposphere is the first layer above the surface our land borders with neighbouring countries is as

om
and contains half of the Earth's atmosphere. Weather follows : Bangladesh : 4,096.7 China : 3,488 Pakistan :
occurs in this layer. This is closest to the Earth's surface, 3,323 Nepal : 1,751 Myanmar : 1,643 Bhutan : 699
extending up to about 10-15 km above the Earth's Afghanistan : 106.
surface. 98. (c) Gujarat is strategically located with largest share in
83. (a) Sugarcane shows chloroplast dimorphism. The India’s coastline, followed by Andhra Pradesh and
presence of bundle sheath with chloroplast in the Tamil Nadu.
leaves is granulated and lacking in starch. Such feature 99. (c) Jog Fall located at southern state of Karnataka is
is known as chloroplast dimorphism. India’s highest waterfall. Its height is 830 feet and is
84. (c) The equator receives equal day and night throughout located on Sharavati River.
the year because it does not tilt in relation to the sun's 100. (c) 101. (d) 102. (a) 103. (a)
location. Because of the tilted axis of the Earth, the poles 104. (a) Kalahari desert is present in Africa while Atacama
and locations away from the equator lean towards or away
Desert is in South America. Thar Desert is in Australia
from the sun as an orbit is completed, while the equator
and Great Victoria is in Australia.
stays in essentially the same location relative to the sun.
105 . (a) Godavari is the longest river of peninsular India. From
85. (b) The tropical rainforest is earth's most complex biome
its source to the Eastern Ghats, the Godavari River
in terms of both structure and species diversity. It
occurs under optimal growing conditions, abundant flows through gentle, somewhat monotonous terrain,
precipitation and year round warmth. Mean monthly along the way receiving the Darna, Purna, Manjra,
temperatures are above 64 ° F; precipitation is often in Pranhita, and Indravati rivers. Upon entering the
excess of 100 inches a year. There is usually a brief Eastern Ghats region, however, the river flows between
season of reduced precipitation. In monsoonal areas, steep and precipitous banks, its width contracting until
there is a real dry season but that is more than it flows through a deep cleft only 600 feet (180 metres)
compensated for with abundant precipitation the rest wide, known as the Gorge.
of the year. 106. (a) The Himalayan Mountain Range is an example of fold
86. (a) podsol soil is a soil that develops in temperate to cold mountain.They are known as fold mountains because
moist climates underconiferous or heath vegetation; the mountains extend for 2500 km in length in a series
an organic mat over a grey leached layer. of parallel ridges or folds and consist of three folds
87. (d) namely Himadri, Himachal, Shiwalik.
88. (b) The largest shelf - the Siberian Shelf in the Arctic 107. (a) Gulf Stream is a warm ocean current. It flows along the
Ocean - stretches to 1,500 kilometers (930 mi) in width. North America and drifts towards western Europe, thus
The South China Sea lies over another extensive area raising the temperature of western coast considerably.
of continental shelf, the Sunda Shelf, which joins
108. (b) Recharging of ground water is the main advantage of
Borneo, Sumatra, and Java to the Asian mainland.
rain water harvesting. Rainwater harvesting provides
89. (b) In terms of length, catchment area and discharge, the
an independent water supply during regional water
Godavari river is the largest in peninsular India and
restrictions and in developed countries is often used
had been dubbed as the 'Dakshina Ganga' - the South
to supplement the main supply. It provides water when
Ganges river.
there is a drought, can help mitigate flooding of low-
90. (c) 91. (c) 91. (c) 92. (b)
lying areas, and reduces demand on wells which may
93. (c) A solar eclipse occurs when the Moon passes between
Earth and the Sun, thereby totally or partly obscuring enable ground water levels to be sustained.
the image of the Sun for a viewer on Earth. 109. (c) India is the largest producer and exporter of tea. The
94. (c) Approximately 9 miles (14 km) wide at its narrowest Indian tea industry has grown to own many global tea
point, the Strait of Gibraltar is the entry point into the brands and has evolved into one of the most
Mediterranean Sea from the Atlantic Ocean. It is technologically equipped tea industries in the world.
bordered by the continents of Africa and Europe, and Tea production, certification, exportation, and all other
the countries of Morocco, Spain, the British colony of facts of the tea trade in India is controlled by the Tea
Gibraltar, and the Spanish exclave of Ceuta. Board of India.
w
w
w
.y
ou
Geography 27

rs
m
110. (a) Pedocal is a subdivision of the zonal soil order. It is a 128. (d) The National Green Tribunal deals with cases relating

ah
bo
class of soil which forms in semiarid and arid regions. to environmental protection and conservation of

ob
It is rich in calcium carbonate and has low soil organic forests and other natural resources.

.w
matter. 129. (a) Kuwait Spill in Persian Gulf, Kuwait of 19th January

or
dp
111. (d) Land able to be used for farming is called "cultivable 1991 was the biggest oil spill in world history.

re
land". It includes total fallow land and net sown area.

ss
130. (c) Japan has the highest life expectancy among the

.c
112. (d) Gulf of Mannar with a area of 10500 sq.km. is the largest countries across the world.

om
biosphere reserve of India.
131. (b) Scattering of light causes redness in atmosphere at
113. (a) In physical geography, a steppe is an ecoregion,
Sunrise and Sunset.
characterized by grasslandplains without trees apart
132. (c) Pygmies follow practices of pastoral nomadism.
from those near rivers and lakes.
114. (a) Covering around 2 percent of the Earth's total surface 133. (d) High Yielding Variety (HYV) seeds played vital role in
area, the world's tropical rainforests are home to the progress of agriculture. They are considered as
50 percent of the Earth's plants and animals. Rainforests `miracle seeds'. The high yielding programme was
can be found all over the world from as far north as launched in the Kharif season of 1966 in selected areas
Alaska and Canada to Latin America, Asia and Africa. having assured rainfall. Seed is the basic and crucial
115. (d) The world's growing appetite for beef is a leading cause input for attaining sustained growth in agricultural
of tropical deforestation. production.
116. (c) Kyoto Protocol is an international treaty among 134. (c) Tea is an aromatic beverage commonly prepared by
countries to reduce greenhouse gases emissions based pouring hot or boiling water over cured leaves of the
on the premise that (i) global warming exists and (ii) tea plant, Camellia sinensis. After water, tea is the most
man-made CO2 emissions.
widely consumed beverage in the world.
117. (d) Glacial ice or Glaciers are the largest reservoir of
freshwater on Earth. 135. (b) Delhi-Kolkata section
118. (a) 136. (c) The Tropic of Cancer, also referred to as the Northern
119. (a) About 70 percent of the Earth’s surface is water- tropic, is the circle of latitude on the Earth that marks
covered and the oceans hold about 96.5 percent of all the most northerly position at which the Sun may
Earth’s water. appear directly overhead at its zenith.
120. (d) Johannes Kepler was a German mathematician, 137. (d) The demographics of the People's Republic of China
astronomer and astrologer. He is best known for his are identified by a large population with a relatively
laws of planetary motion which stated that the orbit of small youth division, which is partially a result of
a planet is an ellipse with the Sun at one of the two
China's one-child policy. Chinese population reached
foci.
the billion mark in 1982.
121. (a) The Trans-Himalayan rivers originate beyond the great
138. (a) The Ravva oil and gas field in the Krishna- Godavari
Himalayas. The name of the rivers are the Indus, the
Sutlej and the Brahmaputra river. Basin was developed in partnership with Cairn India,
122. (a) Black soil or Black Lava Soil is considered most suitable ONGC, Videocon and Ravva Oil, under a production
for sowing cotton crops. sharing contract (PSC) that runs until 2019.
123. (c) Neyveli lignite field in Tamil Nadu is the largest lignite 139. (d) Starting from north, Machallipatnam is located in
coal mine in India. Andhra Pradesh. Chennai, Nagapatnam and Tuticorin
124. (a) Sex ratio is used to describe the number of females per are present in Tamil Nadu.
1000 males in country. In the Population Census of 140. (d) Santa Cruz is a domestic airport in Mumbai.
2011 it was revealed that the population ratio in India 141. (b) The Almatti Dam is a dam project on the Krishna River
2011 is 940 females per 1000 of males. in North Karnataka, India which was completed in July
125. (a) In Tamil Nadu, rain water harvesting was made 2005. The Almatti Dam is the main reservoir of the Upper
compulsory for every building to avoid ground water Krishna Irrigation Project.
depletion.
142. (a) Damodar Valley Corporation, commonly known as
126. (c) NASA has developed a series of interactive a maps
DVC, was incorporated on July 7, 1948 by an Act of the
and graphs to describe the global climte and how it
Constituent Assembly of India (Act No. XIV of 1948).
change over time. The focus of 5 key climate indications
It was the oldest versatile river basin project of
include Arctic sea ice, carbon dioxide concentration,
Global surface temperature, Sea level and land ice. sovereign India. Damodar Valley Corporation has been
127. (d) The area reserved for the welfare of wildlife is called planned like the Tennessee Valley Authority of the
National Park. USA.
w
w
w
.y
ou
28 Geography

rs
m
143. (a) Petrology is the branch of geology that studies the 151. (a) The Chhattisgarh Assembly, in December 2012,

ah
bo
origin, composition, distribution and structure of rocks. enacted a landmark legislation by unanimously passing

ob
144. (b) the Food Security Act, becoming the first state in the

.w
145. (c) The eastern coast of India, particularly in Tamilnadu, country to introduce such a law. The Act makes food

or
dp
remains relatively dry during the south-west monsoon entitlements a right and its non-compliance has been

re
period. This is because the Tamilnadu coast lies in the made an offence.

ss
.c
rain-shadow area of the Arabian Sea current and is 152. (c) Visakhapatnam Steel Plant (VSP) is the first shore based

om
parallel to the Bay of Bengal current. integrated steel plant Public sector in the country
146. (a) The International Date Line (IDL) is an imaginary line located at Visakhapatnam in Andhra Pradesh. The plant
on the surface of the Earth from the north to the south was commissioned in August 1992 with a capacity to
pole and demarcates one calendar day from the next. It produce 3 million tonnes per annum (MTPA) of liquid
passes through the middle of the Pacific Ocean, roughly steel. The plant has been built to match international
following the 180° longitude but it deviates to pass standards with state-of-the-art technology,
around some territories and island groups. incorporating extensive energy saving and pollution
control measures. VSP has an excellent layout capable
147. (b) The position of Meghalaya is 22nd in terms of both
of expanding up to 16 MTPA.
area as well as population. The position of rest of the
153. (c) Environmental conditions play a key role in defining
states in terms of area and population respectively are:
the function and distribution of plants, in combination
Nagaland: 25, 24; Manipur: 23, 23 (excludes Mao-
with other factors which influence the development of
Maram. Paomata. and Purul sub-divisions of Senapati
plants into distinctive forms.
district of Manipur); and Gujarat: 7, 10.
154. (d) Dr. M S Swaminanthan said the given lines.
148. (c) An oxbow lake is a U-shaped body of water formed Swaminathan is known as "Indian Father of Green
when a wide meander from the main stem of a river is Revolution" for his leadership and success in
cut off to create a lake. This landform is called an oxbow introducing and further developing high-yielding
lake for the distinctive curved shape, named after part varieties of wheat in India.
of a yoke for oxen. 155. (b) Mangrove trees act as a natural barrier against
149. (c) Gujarat is the leading state in ship breaking industry cyclones and storms.
because of its long coastline and a well developed 156. (c) 157. (d) 158. (a) 159. (b)
infrastructure for the industry-specific works. Alang 160. (d) Chile is the world's largest copper producer, producing
in Gujarat, considered to be the world's largest, is a 1.79 million tons in 2013.
famous ship breaking centre. 161. (d) 162. (d) 163. (d) 164. (d) 165. (b)
150. (c) Contour line is a line on a map or chart joining points 166. (a) 167. (d) 168. (b) 169. (d) 170. (c)
of equal height (elevation) or depth. Closely spaced 171. (b) 172. (c) 173. (a) 174. (a) 175. (b)
contour lines indicate a steeper grade than the more 176. (a) 177. (d) 178. (d) 179. (a) 180. (a)
loosely spaced lines. 181. (c) 182. (c) 183. (c) 184. (c) 185. (d)
w
w
w
.y
ou
rs
m
3

ah
bo
ob
.w
POLITY

or
dp
re
ss
CHAPTER

.c
om
1. The term 'Caste' was derived from 12. Which one of the following is the guardian of Fundamental
(SSC CGL 1st Sit. 2010) Rights ? (SSC CGL 2nd Sit. 2010)
(a) Portuguese (b) Dutch (a) Legislature (b) Executive
(c) German (d) English (c) Political parties (d) Judiciary
2. The term 'Greater India' denotes 13. Sarkaria Commission was concerned with
(SSC CGL 1st Sit. 2010) (SSC CGL 2nd Sit. 2010)
(a) Political unity (b) Cultural unity (a) Administrative Reforms
(c) Religious unity (d) Social unity (b) Electoral Reforms
3. Who is rightly called the "Father of Local Self Government" (c) Financial Reforms
in India ? (SSC CGL 1st Sit. 2010) (d) Centre-State relations
(a) Lord Mayo (b) Lord Ripon 14. The speaker of the Lok-Sabha has to address his/her letter
(c) Lord Curzon (d) Lord Clive of resignation to (SSC CGL 2nd Sit. 2010)
4. The two forms of democracy are (SSC CGL 1st Sit. 2010) (a) Prime Minister of India
(a) Parliamentary and Presidential (b) President of India
(b) Direct and Indirect (c) Deputy Speaker of Lok Sabha
(c) Monarchical and Republican (d) Minister of Parliamentary Affairs
(d) Parliamentary and King 15. A Presidential Ordinance can remain in force
5. Which is an extra-constitutional body ? (SSC CGL 2nd Sit. 2010)
(a) Language Commission (SSC CGL 1st Sit. 2010) (a) For Three months (b) For six months
(b) Planning Commission (c) For nine months (d) Indefinitely
(c) Election Commission 16. Judicial review in the Indian Constitution is based on:
(a) Rule of Law (SSC CGL 1st Sit. 2011)
(d) Finance Commission
(b) Due process of Law
6. The Prime Minister of India is (SSC CGL 1st Sit. 2010)
(c) Procedure established by Law
(a) Elected (b) Appointed
(d) Precedents and Conventions
(c) Nominated (d) Selected
17. The Drafting of the Constitution was completed on:
7. Which is not an All India Service ? (SSC CGL 1st Sit. 2010)
(SSC CGL 1st Sit. 2011)
(a) Indian Administration Service th
(a) 26 January, 1950 (b) 26th December, 1949
(b) Indian Police Service
(c) 26th November, 1949 (d) 30th November, 1949
(c) Indian Foreign Service
18. Who was the President of the Constituent Assembly?
(d) Indian Forest Service
(SSC CGL 1st Sit. 2011)
8. The declaration that Democracy is a Government 'of the
(a) Pt. Jawahar Lal Nehru (b) Sardar Patel
people' by the people; for the people' was made by
(c) Dr. Rajendra Prasad (d) Dr. B.R. Ambedkar
(SSC CGL 2nd Sit. 2010)
19. Which innovative discussion process is introduced by the
(a) George Washington (b) Winston Churchill Indian parliament to the World Parliamentary systems?
(c) Abraham Lincoln (d) Theodore Roosevelt (SSC CGL 1st Sit. 2011)
9. India attained 'Dominion Status' on (a) Question hour (b) Zero hour
(SSC CGL 2nd Sit. 2010) (c) Resolutions (d) Presidential Speech
(a) 15th January, 1947 (b) 15th August, 1947 20. The judges of the Supreme Court retire at the age of :
(c) 15th August, 1950 (d) 15th October, 1947 (SSC CGL 1st Sit. 2011)
10. Despotism is possible in a (SSC CGL 2nd Sit. 2010) (a) 60 years (b) 65 years
(a) One party state (b) Two party state (c) 62 years (d) 58 years
(c) Multi party state (d) Two and multi party state 21. Who was the architect of North and South Blocks of the
11. Marx belonged to (SSC CGL 2nd Sit. 2010) Central Secretariate in Delhi? (SSC CGL 1st Sit. 2011)
(a) Germany (b) Holland (a) Sir Edward Lutyens (b) Herbert Baker
(c) France (d) Britain (c) Robert Tor Russell (d) Antonin Raymond
w
w
w
.y
ou
30 Polity

rs
m
22. If the Anglo–Indian community does not get adequate (a) Commissioner for Scheduled Castes and Tribes

ah
bo
representation in the Lok Sabha, two members of the (b) Prime Minister

ob
community can be nominated by the (c) President

.w
(a) Prime Minister (SSC CGL 2nd Sit. 2011)

or
(d) Governor

dp
(b) President 33. The seat of Kerala High Court is located at:

re
(c) Speaker (SSC CGL 2nd Sit. 2012)

ss
.c
(d) President in consultation with the Parliament (a) Kottayam (b) Thiruvananthapuram

om
23. For the election of President of India, a citizen should have (c) Kollam (d) Ernakulam
completed the age of (SSC CGL 2nd Sit. 2011) 34. The first speaker of Lok Sabha was:
(a) 25 years (b) 30 years (a) S. Radhakrishnan (SSC CGL 2nd Sit. 2012)
(c) 35 years (d) 18 years (b) M. Ananthasayanam Ayyangar
24. A member of Parliament will lose his membership of (c) Sardar Hukum Singh
Parliament if he is continuously absent from Sessions for
(d) G. V. Mavlankar
(SSC CGL 2nd Sit. 2011)
35. The Social Contract theory deals with
(a) 45 days (b) 60 days
(SSC CGL 1st Sit. 2012)
(c) 90 days (d) 365 days
(a) The Purpose of State
25. In India, the Residuary Powers are vested with
(b) The Origin of the State
(a) Union Government (SSC CGL 2nd Sit. 2011)
(c) The Nature of State
(b) State Government
(d) The Functions of State
(c) Both the Union Government and the State Government
36. The Supreme Court of India offers advice to the President
(d) Local Government
on matters of Legal, Public or Constitutional importance
26. The National Commission for Minorities was constituted in
based on (SSC CGL 1st Sit. 2012)
the year (SSC CGL 1st Sit. 2012)
(a) Article-148 (b) Article-129
(a) 1990 (b) 1992
(c) Article-147 (d) Article-143
(c) 1980 (d) 1989
37. Which is the most effective means of executive control of
27. In which of the following systems of government is bi–
cameralism an essential feature? (SSC CGL 1st Sit. 2012) administration ? (SSC CGL 1st Sit. 2012)
(a) Federal system (b) Unitary system (a) Financial administration
(c) parliamentary system (d) Presidential system (b) Political direction
28. Socialism succeeds in achieveing (SSC CGL 1st Sit. 2012) (c) Appointment and removal of top officials
(a) higher standard of living of the people (d) Subordinate legislation
(b) equal distribution of income in the society 38. Violation of‘Rule of Law’ arises mostly from
(c) higher individual welfare in the society (SSC CGL 1st Sit. 2012)
(d) maximum social welfare in the society (a) Lack of Checks and Balances
29. The seat of Madhya Pradesh High Court is located at (b) Executive’s Quasi-Judicial Powers
(SSC CGL 1st Sit. 2012) (c) Limited Franchise
(a) Gwalior (b) Indore (d) Delegated Legislation
(c) Bhopal (d) Jabalpur 39. The Constitutional Amendment Act that has introduced
30. Who said that the Directive Principles of State Policy are safeguards against the misuse of proclamation of national
just like “a cheque on bank payable at the convenience of emergency is the (SSC CGL 2nd Sit. 2012)
the bank”. (SSC CGL 2nd Sit. 2012) (a) 42nd Amendment Act (b) 43rd Amendment Act
(a) Pandit Nehru (b) K. T. Shah (c) 44th Amendment Act (d) 45th Amendment Act
(c) B. R. Ambedkar (d) N. G. Ranga 40. The Fundamental Rights can be suspended by the
31. The proposal for the creation of new All-India Services can (SSC CGL 2nd Sit. 2012)
be considered only: (SSC CGL 2nd Sit. 2012) (a) Governor (b) President
(a) if majority of State Legislatures make such demand (c) Law Minister (d) Prime Minister
(b) if Lok Sabha passes a resolution by two-thirds majority 41. The main reason for the growth of communalism in India is
(c) if the Rajya Sabha passes a resolution by two-thirds (SSC CGL 2nd Sit. 2012)
majority (a) Educational and economic backwardness of minority
(d) None of the above groups
32. The authority to specify which castes shall be deemed to (b) Political consciousness
be scheduled castes rests with the: (c) Social inequalities
(SSC CGL 2nd Sit. 2012) (d) Imposing ban on communal organisations
w
w
w
.y
ou
Polity 31

rs
m
42. A Retired Judge of a High Court is not permitted to practice 53. Who presides over the Joint Session of Indian Parliament?

ah
bo
as a lawyer in (SSC CGL 2nd Sit. 2012) (SSC CGL 1st Sit. 2013)

ob
(a) Supreme Court (a) Chairperson of Rajya Sabha

.w
or
(b) Any Court in India (b) Seniormost Member of Parliament

dp
(c) High Courts (c) Speaker of Lok Sabha

re
ss
(d) Except the High Court where he retired (d) President of India

.c
om
43. Which one of the following does not match? 54. Under which Article of the Constitution of India, can the
(SSC CGL 2nd Sit. 2012) fundamental rights of the members of the Armed Forces be
(a) Hindu Marriage Act : 1955 specifically restricted? (SSC CGL 1st Sit. 2013)
(b) Medical Termination of Pregnancy Act : 1971 (a) Article 21 (b) Article 25
(c) Domestic Violence on women Act : 1990 (c) Article 33 (d) Article 19
(d) Cruelty against Women : 1995 55. Which Article of the Indian Constitution did Dr. B. R.
44. The vacancy of the office of the President must be filled Ambedkar term as the “Heart and Soul of the Indian
within : (SSC CGL 1st Sit. 2013) Constitution”? (SSC CGL 1st Sit. 2013)
(a) 3 months (b) 6 months (a) Article 356 (b) Article 32
(c) 12 months (d) 1 month (c) Article 14 (d) Article 19
45. In which part of the Indian Constitution, the fundamental 56. Who was the first to use the term ‘State’?
duties are enshrined ? (SSC CGL 1st Sit. 2013) (SSC CGL 1st Sit. 2013)
(a) IV (b) IV A (a) Aristotle (b) Machiavelli
(c) IV B (d) V (c) Hobbes (d) Plato
46. Who described the Government of India Act, 1935 as a new 57. Who is the Chairman of the 20th Law Commission?
charter of bondage ? (SSC CGL 1st Sit. 2013) (SSC CGL 1st Sit. 2013)
(a) B.R. Ambedkar (b) Mahatma Gandhi (a) Justice K.G. Balakrishnan
(c) Rajendra Prasad (d) Pt. Jawaharlal Nehru (b) Justice D. K. Jain
47. Which of the following is not a fundamental right as per (c) Justice Usha Mehra
the Indian Constitution? (SSC CGL 2nd Sit. 2013) (d) Justice J. S. Verma
(a) Right to Education (b) Right to Information 58. Which one of the following is not a function of Election
(c) Right to Speech (d) Right to Life Commission ? (SSC CGL 1st Sit. 2013)
48. Who is custodian of the Indian Constitution? (a) Allotment of symbols
(SSC CGL 2nd Sit. 2013) (b) Fixation of election dates
(a) President of India (b) Chief Justice of India (c) Maintaining fairness of election
(c) Prime Minister of India (d) Chairman of Rajya Sabha (d) Selecting the candidates for election
49. Constituent Assembly of India was formulated on the 59. ‘State is a necessary evil’ is associated with
recommendation of (SSC CGL 2nd Sit. 2013) (SSC CGL 1st Sit. 2013)
(a) Wavel Plan (b) Cripps Mission (a) Individualism (b) Idealism
(c) August Offer (d) Cabinet Mission (c) Marxism (d) Constructivism
50. Which of the following is an essential element of the state? 60. Who was the first President of the All India Trade Union
(SSC CGL 2nd Sit. 2013) Congress (AITUC) ? (SSC CGL 1st Sit. 2013)
(a) Sovereignty (b) Government (a) C.R. Das (b) V.V. Giri
(c) Territory (d) All these (c) Lala Lajpat Rai (d) Sarojini Naidu
51. Which has become a legal right under 44th Amendment? 61. If there is a deadlock between Rajya Sabha and Lok Sabha
(a) Right to Education (SSC CGL 2nd Sit. 2013) over an ordinary bill, it will be resolved by
(b) Right to Property (a) The President (SSC CGL 1st Sit. 2013)
(c) Right to Judicial Remedies (b) The Council of Ministers
(d) Right to work (c) The Joint Session of Parliament
52. By which Constitution Amendment Act, Right to Property (d) The Supreme Court
ceased to remain a fundamental right? 62. Provisions of citizenship in Indian Constitution, became
(SSC CGL 2nd Sit. 2013) applicable in (SSC CGL 1st Sit. 2013)
(a) 44th (d) 42nd (a) 1950 (b) 1949
(c) 43rd (d) 45th (c) 1951 (d) 1952
w
w
w
.y
ou
32 Polity

rs
m
63. The National Emergency in India declared by the President 75. The South East trade winds are attracted towards the Indian

ah
bo
of India due to the external aggression or armed revolt sub continent in the rainy season due to

ob
through (SSC CGL 1st Sit. 2013) (SSC CGL 1st Sit. 2015)

.w
(a) Article–352 (b) Article–356 (a) the effect of easterlies

or
dp
(c) Article–360 (d) Article–368 (b) the effect of Northern–East trade winds

re
64. The most important feature of Cabinet system of (c) the presence of low atmospheric pressure over North–

ss
Government is (SSC CGL 2014)

.c
West India

om
(a) Individual responsibility (d) the development of cyclone over the equator
(b) Collective responsibility 76. The 'graded profile' of a river course is a
(c) Responsibility to none (SSC CGL 1st Sit. 2015)
(d) Non-responsibility (a) smooth curve in the upper course
65. Direct legislation in Switzerland has (SSC CGL 2014) (b) smooth curve in the middle course
(a) a natural growth (b) a haphazard growth (c) smooth curve in the lower course
(c) an artificial growth (d) None of the above (d) smooth curve from source to mouth
66. Who gave the idea of "Cabinet Dietatorship"? 77. Sink hole is a phenomenon of _______ topography.
(SSC CGL 2014) (SSC CGL 1st Sit. 2015)
(a) Muir (b) Lowell (a) Desert (b) Tundra
(c) Marriot (d) Laski (c) Karst (d) Plain
67. In which of the following countries are the judges of the 78. Kerala is famous for the cultivation of
federal court elected by the two Houses of the Federal
(SSC CGL 1st Sit. 2015)
Legislature? (SSC CGL 2014)
1. Coconut 2. Black pepper
(a) Switzerland (b) Germany
3. Rubber 4. Rice
(c) Canada (d) Both (a) and (b)
(a) 1, 2 and 4 (b) 2, 3 and 4
68. The President of the USA appoints Supreme Court Judges
(c) 1 and 4 (d) 1, 2 and 3
(a) with Senate's consent (SSC CGL 2014)
79. The longest continental Railway in the world is
(b) at his discretion
(a) Trans Siberian Railway (SSC CGL 1st Sit. 2015)
(c) with consent of the House of Representatives
(b) Canadian Pacific Railway
(d) None of these
69. Which of the following is not a Union Territory ? (c) Canadian National Railway
(SSC CGL 1st Sit. 2015) (d) Trans Atlantic Railway
(a) Lakshadweep (b) Puducherry 80. Who was the first Speaker of the Lok Sabha :
(c) Nagaland (d) Dadra and Nagar Haveli (SSC CGL 1st Sit. 2015)
70. The greatest king of the Pratihara dynasty was (a) B.R. Ambedkar (b) G.V. Mavalankar
(SSC CGL 1st Sit. 2015) (c) N. Sanjeev Reddy (d) Dr S.P. Mukherjee
(a) Bhoj (Mihir–Bhoj) (b) Dantidurga 81. What is the plural volting system?
(c) Nagbhatta II (d) Vatsaraj (SSC CGL 1st Sit. 2015)
71. In 1939 Subhash Chandra Bose was elected as President of (a) All the citizens caste three votes each
the Congress Party defeating (SSC CGL 1st Sit. 2015) (b) Eligible voter exercises one vote and some voters with
(a) Jawaharlal Nehru specific qualifications cast more than one vote.
(b) Maulana Abul Kalam Azad (c) Only the higher officials caste more than one votes
(c) V.B. Patel (d) Candidates themselves caste more than one vote.
(d) Pattabhi Sitharamayya 82. Voting is : (SSC CGL 1st Sit. 2015)
72. Jallianwala incident took place at (SSC CGL 1st Sit. 2015) (a) The unit of area who constitute a unit for electing
(a) Lucknow (b) Surat representative
(c) Amritsar (d) Allahabad (b) The process by which voters exercise their right to vote
73. Who was the founder of Lodhi dynasty ? (c) The process of selecting representatives
(SSC CGL 1st Sit. 2015) (d) Universal adult franchise.
(a) Sikandar Lodhi (b) Bahlol Lodhi 83. An amendment of the constitution may be initiated.
(c) Ibrahim Lodhi (d) Daulat Khan Lodhi (SSC CGL 1st Sit. 2016)
74. Which one of the following pair is not correctly matched ? (a) by introduction by the President of India.
(a) Akbar – Todarmal (SSC CGL 1st Sit. 2015) (b) by introduction of a Bill in Rajya Sabha.
(b) Chanakya – Chandragupta (c) by the Governors of States.
(c) Vikramaditya – Chaitanya (d) by the introduction of a bill in either House of
(d) Harshvardhan – Hiuen Tsang Parliament.
w
w
w
.y
ou
Polity 33

rs
m
84. The Directive Principles of State Policy has been adopted 95. Which one of the following items comes under the

ah
(SSC CGL 1st Sit. 2016)

bo
from which Constitution? Concurrent List ? (SSC CHSL 2012)

ob
(a) U.S. Constitution (b) British Constitution (a) Trade Unions (b) Citizenship

.w
(c) Irish Constitution (d) French Constitution

or
(c) Local Government (d) Inter-State rivers

dp
85. Which type of democracy do we follow in India?
96. Which Amendment of the Constitution deals with Political

re
(SSC CGL 1st Sit. 2016)

ss
defections ? (SSC CHSL 2012)

.c
(a) Direct (b) Presidential

om
(a) 50th (b) 52th
(c) Representative (d) Dictatorship
86. The Chairman of the Drafting Committee of the Constituent (c) 60th (d) 44th
Assembly of India was (SSC CGL 1st Sit. 2016) 97. Which schedule of the Indian Constitution prescribes
(a) K.M. Munshi (b) D.P. Khaitan distribution of seats in Rajya Sabha ? (SSC CHSL 2012)
(c) Dr. B.R. Ambedkar (d) T.T. Krishnamacharia (a) 4th schedule (b) 5th schedule
87. Who has the right to decide whether a Bill is a money bill or (c) 6th schedule (d) 3rd schedule
not? (SSC CGL 1st Sit. 2016) 98. Who said in the Constituent Assembly that the Directive
(a) Speaker of Lok Sabha (b) Prime Minister Principles of the State Policy are like a 'Cheque on a bank
(c) President (d) Finance Minister payable at the convenience of the bank' ?
88. The discretionary powers of a Governor is limited in (SSC CHSL 2012)
(SSC CGL 1st Sit. 2016) (a) K.M. Munshi (b) B.R. Ambedkar
(a) Appointment of Chief Minister
(c) Austin (d) K. T. Shah
(b) Dismissal of the Ministry
99. How many members can be nominated to both the Houses
(c) Dissolution of the Legislative Assembly
of the Parliament by the President ? (SSC CHSL 2013)
(d) Assent to Bills
89. Who is the first law officer of the country? (a) 14 (b) 16
(SSC CGL 1st Sit. 2016) (c) 10 (d) 12
(a) Chief Justice of India (b) Attorney General 100. Under which Article of the Constitution can an individual
(c) Law Minister (d) Solicitor General move to the Supreme Court directly in case of any violation
90. Which one of the following was established with a definite of Fundamental Rights ? (SSC CHSL 2013)
provision under an Article of the Constitution of India? (a) Article 31 (b) Article 32
(SSC CGL 1st Sit. 2016) (c) Article 28 (d) Article 29
(a) Union Public Service Commission 101. Presidential form of government consists of the following :
(b) National Human Rights Commission (SSC CHSL 2013)
(c) Election Commission
(a) Popular election of the President
(d) Central Vigilance Commission
(b) No overlap in membership between the executive and
91. The President can advance money to meet unforeseen
expenses from the (SSC CGL 1st Sit. 2016) the legislature
(a) Consolidated Fund of India (c) Fixed term of office
(b) Grants of the Central Government (d) All the above
(c) Aid from the Union Government 102. The concept of Concurrent List in Indian Constitution is
(d) Contingency Fund borrowed from the Constitution of (SSC CHSL 2013)
92. How many cricketers, who have represented India in test (a) U.S.A. (b) Japan
matches, are presently Lok Sabha members ? (c) Canada (d) Australia
(SSC CHSL 2012) 103. Who admits a new State to the Union of India ?
(a) Two (b) One (SSC CHSL 2014)
(c) Nil (d) Three
(a) President (b) Supreme Court
93. Which one of the following was created by the 'Pitt's India
Act' ? (SSC CHSL 2012) (c) Prime Minister (d) Parliament
(a) Board of Control (b) Board of Revenue 104. Who has got the power to create All India Services ?
(c) Standing Council (d) Court of Directors (SSC CHSL 2014)
94. Term 'Federal' has been used in the Indian Constitution in: (a) Supreme Court (b) The Parliament
(SSC CHSL 2012) (c) Council of Ministers (d) Prime Minister
(a) Part-III 105. In the provisional Parliament of India, how many members
(b) Article-368 were there ? (SSC CHSL 2014)
(c) Nowhere in the Constitution (a) 296 (b) 313
(d) Preamble (c) 318 (d) 316
w
w
w
.y
ou
34 Polity

rs
m
106. The 73rd Constitutional amendment act is related to ? 117. According to Karl Marx, the change economic system results

ah
bo
(SSC CHSL 2015) in inevitable changes in (SSC Sub. Ins. 2012)

ob
(a) Panchayat Raj (b) Foreign Exchange (a) political system only (b) social system only

.w
(c) Finance Commission (d) RBI

or
(c) the entire systems (d) religious system only

dp
107. Impeachment Proceedings against the President for 118. The model code of conduct for political parties and

re
Violation of the Constitution can be initiated in :

ss
candidates to be followed during the elections is

.c
(SSC CHSL 2015)

om
(SSC Sub. Ins. 2012)
(a) The Supreme Court (a) laid down in the Consitution of India
(b) The Rajya Sabha
(b) specified in The Representation of the People Act, 1951
(c) Either House of Parliament
(c) enjoined by the Supreme Court
(d) The Lok Sabha
(d) agreed through a voluntary agreement among the
108. Who was the First Speaker of the Lok Sabha ?
recognised political parties
(SSC CHSL 2015)
119. A citizen can directly move the Supreme Court for any
(a) K.S. Hegde (b) Hukum Singh
violation of Fundamental Rights under
(c) Ganesh Vasudev (d) Neelam Sanjeeva Reddy
109. Which Article of the Indian Constitution deals with Election (SSC Sub. Ins. 2012)
Commission ? (SSC CHSL 2015) (a) Article 31 (b) Article 32
(a) Article 356 (b) Article 360 (c) Article 33 (d) Article 34
(c) Article 324 (d) Article 352 120. Which one of the following sitting Vice-Presidents of India
110. If the President wants to resign, he shall address his letter contested for the post of President and lost the election?
of resignation to : (SSC CHSL 2015) (a) S. Radhakrishnan (SSC Sub. Ins. 2012)
(a) Vice-President of India (b) Speaker of Lok Sabha (b) V.V. Giri
(c) Chief Justice of India (d) Prime Minister of India (c) Bhairon Singh Shekhawat
111. Constitution of India came into force in (d) Both (B) and (C)
(SSC Multitasking 2013) 121. Which article of the Indian Constitution provides for equal
(a) 1949 (b) 1951 opportunities for all citizen in Public employment?
(c) 1956 (d) 1950 (SSC Sub. Ins. 2013)
112. Cold War refers to (SSC Multitasking 2013)
(a) Article-22 (b) Article-16
(a) tension between East and West
(c) Article-20 (d) Article-25
(b) ideological rivalry between Capitalist and Communist
122. Which one of the following is opposite to democratic
world
state? (SSC Sub. Ins. 2013)
(c) tension between Superpowers
(a) Despotism (b) Republic
(d) All of the above
113. The concept of Judicial Review has been borrowed from (c) Socialism (d) Monarchy
the Consitution of (SSC Multitasking 2014) 123. Who said: "Where there is no law, there is no freedom"?
(a) France (b) Great Britain (SSC Sub. Ins. 2013)
(c) U.S.A. (d) U.S.S.R. (a) Bentham (b) Lenin
114. In the Constitution of India, the budget is known as (c) Marx (d) Locke
(SSC Multitasking 2014) 124. Which one of the following statements about the Chief
(a) Annual Financial Statement Justice of India (CJI) is not correct ?
(b) Annual Revenue Statement (SSC Sub. Ins. 2013)
(c) Annual Budget Statement (a) He appoints the Chief Justice of all High Courts.
(d) Annual Expenditure Statement (b) The CJI administers the oath or-office to the President
115. Which of the following countries has a Parliamentary form (c) When both the offices of the President and Vice-
of Government? (SSC Multitasking 2014) President fall vacant simultaneously, the CJI discharges
(a) New Zealand the duties of the President.
(b) Cuba
(d) The CJI can hold his office till he attains the age of 65
(c) United States of America
years.
(d) France
125. A philosophy that the worker should share in industrial
116. Which of the following systems in independent India goes
decisions is termed as (SSC Sub. Ins. 2014)
against the very basis of democracy? (SSC Sub. Ins. 2012)
(a) industrial democracy (b) worker sovereignty
(a) Caste system (b) Economic system
(c) industrial socialism (d) worker dictatorship
(c) Party system (d) Parliamentary system
w
w
w
.y
ou
Polity 35

rs
m
126. Liberty stands for (SSC Sub. Ins. 2014) 135. To whom the President has to submit his resignation ?

ah
bo
(a) absence of restraint (SSC Sub. Ins. 2015)

ob
(b) consists in the presence of restraint (a) Speaker (b) Chief Justic

.w
(c) feeling enjoyed in a cogenial atmosphere

or
(c) Prime Minister (d) Vice President

dp
(d) the eager maintenance of that atmosphere in which 136. The total number of planets revolving around the sun are:

re
men have the opportunity to be their best selves - Laski

ss
(SSC Sub. Ins. 2015)

.c
127. When the Vice-President is acting as President he

om
(a) Seven (b) Eight
(SSC Sub. Ins. 2014)
(c) Five (d) Six
I. will have all powers and functions of both President
and Vice-President 137. Immunization technique was developed by:
II. gets all the allowances and privileges of the President (SSC Sub. Ins. 2015)
III. should continue to work as the Chairman of the Rajya (a) Louis Pasteur (b) Robert Koch
Sabha (c) Joseph Lister (d) Edward jenner
(a) I, II and III (b) I and III 138. Which one of the following chapters in the Indian
(c) I and II (d) II only Constitution guarantees fundamental rights to the people?
128. In Presidential Government, the President is (SSC Sub. Ins. 2015)
(SSC Sub. Ins. 2014) (a) Part III (b) Part IV
(a) independent of the Legislature (c) Part I (d) Part II
(b) dependent on the Legislature 139. Mac Iver says ‘Kinship creates society and society at length
(c) dependent on the Judiciary creates ________’. (SSC Sub. Ins. 2015)
(d) bound by the advice of the Council of Ministers
(a) Association (b) City State
129. The method of amending the Constitution by popular veto
(c) Nation (d) State
is found in (SSC Sub. Ins. 2014)
140. If the Union Parliament is to assume legislative power over
(a) Britain (b) Switzerland
and subject included in the State List, the resolution to the
(c) Russia (d) India
effect has to be passed by which of the following ?
130. Which of the following is the inalienable attribute of the
parliamentary system of government ? (SSC Sub. Ins. 2014) (SSC Sub. Ins. 2015)
(a) Flexibility of the Constitution (a) Lok, Sabha, Rajya Sabha and legislatures of the
(b) Fusion of Executive and Legislature Concerned States.
(c) Judicial Supremacy (b) Both Lok Sabha and Rajya Sabha
(d) Parliamentary Sovereignty (c) Lok Sabha
131. Which one of the following kinds of equality is not (d) Rajya Sabha
compatible with the liberal notion of equality ? 141. Which Article of the Indian constitution defines the duties
(SSC Sub. Ins. 2014) of the Chief Minister? (SSC Sub. Ins. 2015)
(a) Legal Equality (b) Political Equality (a) Article 164 (b) Article 166
(c) Social Equality (d) Economic Equality (c) Article 167 (d) Article 163
132. The states recorganisation in 1956 created : 142. Political Science is a science of (SSC Sub. Ins. 2016)
(SSC Sub. Ins. 2015) (a) Economic Engineering (b) Cultural Engineering
(a) 17 States and 6 Union Territories.
(c) Social Engineering (d) Political Engineering
(b) 17 States and 9 Union Territories.
143. Which of the following is not the essential element of the
(c) 14 States and 6 Union Territories.
State? (SSC Sub. Ins. 2016)
(d) 15 States and 9 Union Territories.
(a) Government (b) Sovereignty
133. Reservation for the Scheduled castes and Scheduled tribes
in the service has been provided in the Indian Constitution (c) Population (d) Institutions
under : (SSC Sub. Ins. 2015) 144. Parliamentary Government is a form of Constitutional
(a) Article 365 (b) Article 375 democracy in which (SSC Sub. Ins. 2016)
(c) Article 315 (d) Article 335 (a) the legislature emerge from and is responsible to the
134. The Members of the Constituent Assembly which enacted executive.
the constitution of India were: (b) the executives emerge from and is responsible to the
(a) Nominated by Governor-General. judiciary.
(b) Nominated by the Political Parties. (c) the executive emerge from and is responsible to the
(c) Elected by the Legislative Assemblies of various legislature.
Provinces. (d) the legislatures emerge from and is responsible to the
(d) Directly elected by the People. judiciary.
w
w
w
.y
ou
36 Polity

rs
m
145. The concept of Liberty, Equality and Fraternity enshrined 152. Fundamental Rights are borrowed from the Constitution of

ah
bo
in the Indian Constitution is inspired from (SSC Stenographer 2014)

ob
(SSC Sub. Ins. 2016) (a) America (b) Ireland

.w
(a) US Constitution (b) Irish Constitution

or
(c) Australia (d) Russia

dp
(c) French Constitution (d) UK Constitution 153. The term "politics" was first used by

re
ss
146. The authority to alter the boundaries of states in India rests (SSC Stenographer 2016)

.c
with the (SSC Stenographer 2013) (a) Plato (b) John Locke

om
(a) President (b) Prime minister (c) Aristotle (d) Socrates
(c) Parliament (d) State Government 154. Arrange the names of the President in the order they served.
147. Schedule VII of Indian Constitution contains (SSC Stenographer 2016)
(a) Presidential election (SSC Stenographer 2013) (a) N.S. Reddy, Dr. Shankar Dayal Sharma, R.
(b) Acts beyond judicial review Venkataraman, Gyani Zail Singh
(c) States and Union territories (b) R. Venkataraman, Dr. Shankar Dayal Sharma, Gyani Zail
(d) Division of Powers into 3 lists Singh, N.S. Reddy
148. Which Article of the Indian Constitution mentions about (c) N.S. Reddy, Gyani Zail Singh, R. Venkataraman, Dr
financial emergency? (SSC Stenographer 2013) Shankar Dayal Sharma
(a) 360 (b) 350 (d) N.S. Reddy, R. Venkataraman, Gyani Zail Singh, Dr
(c) 340 (d) 330 Shankar Dayal Sharma
149. Maximum age of superannuation for the judges of the 155. Who first gave the concept of "Distributive Justice"?
Supreme Court of India is (SSC Stenographer 2013) (SSC Stenographer 2016)
(a) 62 years (b) 65 years
(a) Machiavelli (b) Locke
(c) 60 years (d) 70 years
(c) Plato (d) Aristotle
150. Name the portfolio held by Sri Pranab Mukherjee before he
156. The reorganization of states on linguistic basis was done in
was elevated as President of India (SSC Stenographer 2014)
(SSC Stenographer 2016)
(a) Railway Minister
(a) 1952 (b) 1956
(b) Finance Minister
(c) 1950 (d) 1951
(c) External Affairs Minister
157. Bodo and Dogri were added in the 8th Schedule by the
(d) Home Minister
following amendment : (SSC Stenographer 2016)
151. Who addresses the joint sessions of the parliament? st
(a) 91 Amendment
(a) The President (SSC Stenographer 2014)
(b) 92nd Amendment
(b) The Prime Minister
(c) 81st Amendment
(c) The Chairman of Rajya Sabha
(d) 85th Amendment
(d) The Lok Sabha Speaker
w
w
w
.y
ou
Polity 37

rs
m
ah
HINTS & SOLUTIONS

bo
ob
.w
or
dp
re
1. (a) 2. (b) 40. (b) The Fundamental Rights can be suspended during the

ss
.c
3. (b) The real benchmarking of the government policy on Emergency under Article 359 of the Constitution by

om
decentralisation can, however, be attributed to Lord the President of India.
Ripon who, in his famous resolution on local self- 41. (a) 42. (d)
government on May 18, 1882, recognised the twin 43. (c) Domestic Violence on Women Act 2005 is the first
considerations of local government: (i) administrative significant attempt in India to recognise domestic abuse
efficiency and (ii) political education. as a punishable offence, to extend its provisions to
4. (a) 5. (b) 6. (b) 7. (c) 8. (c) those in live-in relationships, and to provide for
emergency relief for the victims, in addition to legal
9. (b) 10. (a) 11. (a) 12. (d) 13. (d)
recourse.
14. (c) 15. (b) 16. (c) 17. (c) 18. (c) 44. (b) 45. (b) 46. (d)
19. (b) 20. (b) 21. (b) 22. (a) 23. (b) 47. (*) Right to Information is a part of fundamental rights
24. (b) 25. (a) under Article 19 (1) of the Constitution which states
26. (b) The Union Government set up the National that every citizen has freedom of speech and expression.
Commission for Minorities (NCM) under the National The 86th Constitutional amendment making education
Commission for Minorities Act, 1992. a fundamental right was passed by Parliament in 2002.
27. (a) Some countries, Such as Argentina, Austria, Australia, Right to speech comes under freedom of speech and
expression which is a fundamental right. The
Belgium, Brazil, Canada, Germany, India, Malaysia,
constitution guarantees the right to life and personal
Mexico, Pakistan, Russia, Switzerland and the United
liberty as a fundamental right under article 21.
States, link their bicameral systems to their federal
48. (b) The Constitution has made the Supreme Court as the
political structure.
custodian and protector of the Constitution. The
28. (b) Socialism is an economic system characterised by Supreme Court decides disputes between the Centre
social ownership and/or cannot of the means of and the Units as well as protects the Fundamental
production and cooperative management of the Rights of the citizens of India.
economy. 49. (d) May preceded by the Shimla Conference of 1945.
29. (d) The Court was established in Nagpur, but after the 50. (d) The state has four essential elements: population,
reorganisation of states on 1 November 1956, it was territory, government and sovereignty. Absence of any
moved to Jabalpur. of these elements denies to it the status of statehood.
30. (b) K.T. Shah said that Dr. Rajendra Prasad won his first 51. (b) The 44th amendment eliminated the right to acquire,
election with 507,400 votes over his nearest rival K.T. hold and dispose of property as a fundamental right.
Shah who got 92,827 votes. However, in another part of the Constitution. Article
300 (A) was inserted to affirm that no person shall be
31. (c) Article 312 provides that an All India Service can be
deprived of his property save by authority of law.
created only if the Council of State declares by a
52. (a) The 44th amendment of 1978 eliminated the right to
resolution supported by not less than a two-thirds acquire, hold and dispose of property as a fundamental
majority that it is necessary in the national interest to right. The Constitution originally provided for the right
create one or more such All india Services. to property under Articles 19 and 31.
32. (a) In the original Constitution, Article 338 provided for a 53. (c) The Speaker of Lok Sabha presides over the joint
Special officer, called the Commissioner for SCs and sittings of both Houses of Parliament. The joint session
STs, to have the responsibility of monitoring the of parliament is convened by the President of India.
effective implementation of various safeguards for SCs/ 54. (c) Parliament may restrict the application of the
STs in the Constitution as well as other related Fundamental Rights to members of the Indian Armed
legislations and to report to the President. Forces and the Police, in order to ensure paper
33. (d) The High Court of Kerala is headquartered at Kochi. It discharge of their duties and the maintenance of
is located in Ernakulam. discipline, by a law made under Article 33.
55. (b) Dr. B R Ambedkar, the chairman of the Drafting
34. (d) Ganesh Vasudev Mavalankar, popularly known as
committee called the fundamental right to
Dadasaheb, was an independence activist, the President
constitutional remedies as the heart and soul of the
(from 1946 to 1947) of the Central Legislative Assembly,
Indian constitution. According to this right, a person
then Speaker of the Constituent Assembly of India, an can move the Supreme Court in case of violation of
later the first Speaker of the Lok Sabha. their fundamental rights. In the Constitution, this right
35. (b) 36. (d) 37. (c) 38. (a) 39. (c) is enshrined in Article 32.
w
w
w
.y
ou
38 Polity

rs
m
56. (b) Machiavelli has been credited with formulating for the 68. (a) The Court consists of the Chief Justice of the United

ah
bo
first time the “modern concept of the state”. The term States and eight associate justices who are nominated

ob
‘lo state’ appears widely in Machiavelli’s writings, by the President and confirmed by the Senate. Once

.w
especially in The Prince, in connection with the appointed, justices have life tenure unless they resign,

or
dp
acquisition and application of power in a coercive retire, take senior status, or are removed after

re
sense. impeachment (though no justice has ever been

ss
.c
57. (b) Justice D.K Jain, Judge, Supreme Court of India, is the removed).

om
Chairman of the Twentieth Law Commission of India. 69. (c) Nagaland is a state in Northeast India. The state capital
The Twentieth Law Commission was constituted is Kohima. Nagaland became the 16th state of India
through a Government Order with effect from 1st from 1 December 1963.
September, 2012. 70. (a) Mihira Bhoja was a ruler of the Gurjara Pratihara
58. (d) Selecting the candidate of election is the function of dynasty of India. Bhoja's empire extended to Narmada
the political party River in the South, Sutlej River in the northwest, and
59. (a) This quote is given by the theory of Individualism up to Bengal in the east.
60. (c) The All India Trade Union Congress is the oldest trade 71. (d) Bose appeared at the 1939 Congress meeting and was
union of India, established in 1920, History of AITUC elected president over Gandhi's preferred candidate
is coterminous with the history of organised labour Pattabhi Sitaramayya.
movement in India. Since its birth, AITUC has had a 72. (c) The Jallianwala Bagh Massacre happened in Amritsar,
major role to play in mass movement phase in India's in 1919. It is named after the Jallianwala Bagh (Garden)
freedom struggle. atAmritsar. On April 13, 1919, British, Indian Army
61. (c) The Joint Session of Parliament resolves the deadlock soldiers started shooting an unarmed gathering of men,
between Lok Sabha and Rajya Sabha over an ordinary women and children.
bill. 73. (b) Bahlol Lodi was the founder of the Lodi dynasty.He
62. (a) 63. (a) ruled for long thirty-nine years (1451-89). He was the
64. (b) Cabinet collective responsibility is constitutional governor of Lahore and Sirhind during the rule of
convention in governments is that members of the Muhammad Shah of Sayyid dynasty.
Cabinet must publicly support all governmental 74. (c) Sri Chaitanya Mahaprabhu arrived in the empire at the
decisions made in Cabinet, even if they do not privately time of Emperor Prataparudra (Gajapatis)and stayed
agree with them. This support includes voting for the for 18 long years at Puri.
government in the legislature. Cabinet collective 75. (c) The instance heat that prevails in the Indian Sub
responsibility is related to the fact that, if a vote of no continent causes a low pressure region over the
confidence is passed in parliament, the government is northern plains. It is intense enough to attract the
responsible collectively, and thus the entire moisture bearing winds from the Indian Ocean .Thus
government resigns.
the south east trade winds from the southern
65. (a) Switzerland is a small country located in the heart of
hemisphere are attracted towards India.
western Europe, at the intersection of German, French
76. (d) A river profile is a curve which shows the slop of a
and Italian language and culture. Switzerland has been
river from source to mouth.A stream flowing over
multicultural in its own way for centuries. Direct
irregular terrain may have waterfalls, rapids and lakes
Democracy in particular, has a long, but not undisputed
along its course.Though the stream will wear away the
tradition in this country. Switzerland's unique political
irregularities to leave a smoothly curving profile called
system is today world's most stable democratic system,
a graded profile.
offering a maximum of participation to citizens.
77. (c) Karst is a landscape formed from the dissolution of
66. (a) John Muir was a Scottish-American naturalist, author,
soluble rocks including limestone, dolomite and
and early advocate of preservation of wilderness in
gypsum. It is characterized by sinkholes, caves, and
the United States.
underground drainage systems.
67. (a) The Federal Assembly is bicameral, being composed
78. (d) Kerala is famous for the cultivation of coconut, tea,
of the 200-seat National Council and the 46-seat Council
of States. The houses have identical powers. Members coffee, cashew and spices.
of both houses represent the cantons, but, whereas 79. (a) The Trans-Siberian Railway is the longest railway line
seats in the National Council are distributed in in the world. It has a length of 9,289 km which connects
proportion to population, each canton has two seats Moscow to Vladivostok.
in the Council of States, except the six 'half-cantons' 80. (b) G. V Mavalankar was the first speaker of the Lok Sabha.
which have one seat each. Both are elected in full once 81. (d) Plural voting is the practice whereby one person might
every four years, with the last election being held in be able to vote multiple times in an election.
2011. 82. (c) Voting is the process of selecting representatives.
w
w
w
.y
ou
Polity 39

rs
m
83. (d) As per the procedure laid out by article 368 for Congress and Muslim League.The first official meeting

ah
bo
amendment of the Constitution, an amendment can be of this Constituent Assembly was held on 9 Dec, 1946

ob
initiated only by the introduction of a Bill in either while the last meeting was held on 24 Jan, 1950. On

.w
House of Parliament. The Bill, passed by the required 26th Jan, 1950, the day when Constitution of India

or
dp
majority, is then presented to the President who shall finally took in effect, Constitutional Assembly was

re
give his assent to the Bill. renamed as Provisional Parliament of India.This

ss
84. (c) The concept of Directive Principles of State Policy was Provisional Parliament was dissolved after the first

.c
om
borrowed from the Irish Constitution. The makers of general election of India in 1952.
the Constitution of India were influenced by the Irish 106. (a) 73rd Constitutional amendment act is related to provide
nationalist movement. Hence, the Directive Principles 3-tier system of Panchayati Raj for all states having
of the Indian constitution have been greatly influenced population of over 20 lakh.
by the Directive Principles of State Policy. 107. (c) Impeachment of the president for violation of
85. (c) 86. (c) 87. (a) 88. (d) 89. (b) consititution of India may start in either of the two
90. (c) 91. (d) 92. (d) 93. (a) 94. (c) houses of the Parliament.
95. (a) 96. (b) 97. (a) 98. (d) 108. (c) Ganesh Vasudev Mavalankar was the first speaker of
99. (a) According to the Indian Constitution, 14 members can Lok Sabha.
be nominated to both the houses of parliament by the 109. (c) The Article 324 of the Indian constitution basically
President. This is the legislative power of the President empowers the Election Commission to exercise its
where he nominates 12 members to the Rajya Sabha power to prevent criminals and corrupt persons from
and if not adequately represented 2 Anglo-Indian entering politics.
members to the Lok Sabha. 110. (a) If the President wants to resign he/she shall address
100. (b) Under Article 32 of the Constitution, an individual can his letter of resignation to the Vice-President.
directly move to the Supreme Court in Case of any 111. (d) The constitution of India came into effect from 26th
violation of fundamental rights. Fundamental Rights January 1950
are those rights which are essential for the growth of 112. (d) The Cold War, often dated from 1947 to 1991, was a
an individual's personality and are enjoyed by every sustained state of political and military tension between
citizen irrespective of caste, color, creed, race and sex. powers in the Western Bloc, dominated by the United
101. (d) A presidential system is a republican system of States with NATO among its allies, and powers in the
government where a head of government is also head Eastern Bloc, dominated by the Soviet Union along
of state and leads an executive branch that is separate with the Warsaw Pact. This began after the success of
from the legislative branch. The United States, for their temporary wartime alliance against Nazi Germany,
instance, has a presidential system. Popular election leaving the USSR and the US as two superpowers with
of President, no overlap in membership and fixed term profound economic and political differences.
of office are the main criteria of Presidential form of 113. (c) The concept of Judicial Review in the Constitution of
Government. India is borrowed from USA. In the Indian constitution,
102. (d) The concept of Concurrent List in Indian Constitution Judicial review is dealt with under Article 13. Judicial
is borrowed from the Constitution of Australia. The Review refers that the Constitution is the supreme
Concurrent List or List-III is a list of 52 items(though power of the nation and all laws are under its
the last item is numbered 47) given in Part XI of the supremacy.
Constitution of India, concerned with relations between 114. (a) In the Constitution of India, the budget is known as
the Union and States. This part is divided between Annual Financial Statement. The Union Budget of
legislative and administrative powers. The legislative India, referred to as the Annual Financial Statement[1]
section is divided into three lists: Union List, State List in Article 112 of the Constitution of India, is the annual
and Concurrent List. budget of the Republic of India, presented each year
103. (d) The parliament of India has power to add a new state on the last working day of February by the Finance
to the Union of India. This is done by collecting votes Minister of India in Parliament. The budget, which is
of Members of Parliament in the favour of new state. presented by means of the Financial Bill and the
104. (b) the Parliament has the power to create all India Services. Appropriation bill has to be passed by the House
105. (b) The Provisional Parliament of India was consisted of before it can come into effect on April 1, the start of
313 members. The Constitutional Assembly of India India's financial year.
was introduced in 1934. This will become the major 115. (a) New Zealand has parliamentary form of government. A
assembly to draft constitution for India (Including parliamentary system of government means that the
present day Pakistan and Bangladesh). Members of executive branch of government must have the direct
this assembly was indirectly elected representatives or indirect support of the parliament. This support is
from across the India. It consists of the members of usually shown by a vote of confidence. The
w
w
w
.y
ou
40 Polity

rs
m
relationship between the executive and the legislature determined by Parliament by law and, until provision

ah
bo
in a parliamentary system is called responsible in that behalf is so made, such emoluments, allowances

ob
government. Parliamentary systems usually have a and privileges as are specified in the Second Schedule.

.w
head of government and a head of state. The head of When the President is unable to discharge his functions

or
dp
government is the prime minister, who has the real power. owing to absence, illness or any other cause, the Vice-

re
The head of state often is an elected (either popularly President shall discharge his functions until the date

ss
or through parliament) president or, in the case of a on which the President resumes his duties. The Vice-

.c
om
constitutional monarchy, hereditary. President shall not be a member of either House of
116. (a) 117. (c) Parliament or of a House of the Legislature of any State,
118. (b) The Representation of The People Act , 1951 : An Act and if a member of either House of Parliament or of a
to provide for the conduct of elections to the Houses House of the Legislature of any State be elected Vice-
of Parliament and to the House or Houses of the President, he shall be deemed to have vacated his seat
Legislature of each State. in that House on the date on which he enters upon his
119. (b) A citizen has the right to 'move the supreme court' office as Vice-President.
(under article 32) directly in case s/he faces any violation 128. (a) a system of government in which the powers of the
of his/her fundamental rights. president are constitutionally separate from those of
120. (c) Bhairon Singh Shekhawat was the 11th Vice-President the legislature.
of India. He served in that position from August 2002, 129. (b) The method of amending the Constitution by popular
when he was elected to a five-year term, until he veto is found in Switzerland.
resigned on July 21, 2007, after losing the presidential 130. (b) A parliamentary form of government is a democratic
election to Pratibha Patil. one where the head of state and head of government
121. (b) Article 16 of the Indian Constitution deals with equality are held by different people and the executive and
of opportunity in matters of public employment. It states legislature branches of government are linked. The
that no citizen shall, on grounds only of religion, race, executive and legislature branch are linked because
caste, sex, descent. place of birth, residence or any of the executive branch gets its power from the legislative
them, be ineligible for, or discriminated against in branch and is held accountable for their actions by
respect or, any employment or office under the State. them as well. The leaders of the majority party or alliance
122. (a) Democracy is a political system whereby people elect in the legislature become the members of the Cabinet
their representatives who then administer the region/ or Ministry. Naturally, the ministers can easily extend
nation. Despotism is the opposite of democracy as the their influence on the legislature. Consequently, the
principle of representation is absent in it. It is mostly programs and policies of the Cabinet are backed by a
associated with autocratic/authoritarian set ups. majority inside the legislature. This feature is inalienable
123. (d) John Locke in his Second Treatise of Civil Government, in Parliamentary form of government.
Chapter 6, said that the end of law is not to abolish or 131. (d) Economic equality is not compatible with the liberal
restrain, but to preserve and enlarge freedom. notion of equality. Liberal means free of restraints and
According to him. in all the states of created beings economic equality is about a level playing field where
capable of laws, where there is no law, there is no everyone has the same access to the same wealth;
freedom. which is not possible.
124. (a) Judges in a High Court are appointed by the President 132. (c) 133. (d) 134. (c) 135. (d) 136. (b)
of India in consultation with the Chief Justice of India 137. (d) Edward Jenner is often called "the father of
and the governor of the state. The Constitution confers immunology".
the power of appointment of judges on the President 138. (a) The Fundamental Rights are defined in Part III of the
of India. Constitution which apply to every Indian citizen
125. (a) Industrial democracy is an arrangement which involves irrespective of race, place of birth, religion, caste, creed
workers making decisions, sharing responsibility and or gender.
authority in the workplace. 139. (d) "Kinship" says MacIver, creates society and society
126. (a) Liberty means absence of restraint and assures freedom at length creates the state."
of thought, expression, belief, faith and worship. 140. (d)
127. (d) The Vice-President shall, during, and in respect of, the 141. (c) Article 167 defines the duties of the chief minister.
period while he is so acting as, or discharging the 142. (a) 143. (d) 144. (c) 145. (c) 146. (c)
functions of, President have all the powers and 147. (c) 148. (a) 149. (b) 150. (b) 151. (a)
immunities of the President and be entitled to such 152. (a) 153. (c) 154. (d) 155. (c) 156. (b)
emoluments, allowances and privileges as may be 157. (a)
w
w
w
.y
ou
Economics 41

rs
m
4

ah
bo
ob
.w
ECONOMICS

or
dp
re
ss
CHAPTER

.c
om
1. Formalised system of trading agreements with groups of 11. Which among the following agencies released the report,
countries is known as (SSC CGL 1st Sit. 2010) Economic Outlook for 2009–10 ? (SSC CGL 2nd Sit. 2010)
(a) Trading blocks (b) Trade ventures (a) Planning Commission
(c) Trade partners (d) Trade organisations (b) PM's Economic Advisory Council
2. Depression formed due to deflating action of winds are called (c) Finance Commission
(SSC CGL 1st Sit. 2010) (d) Reserve Bank of India
(a) Playas (b) Yardang 12. India and U.S. have decided to finalise agreements related
(c) Ventifacts (d) Sand dunes to which of the following ? (SSC CGL 2nd Sit. 2010)
3. Which one of the following is not a method of estimating (a) Trade and Investment (b) Intellectual Property
National Income ? (SSC CGL 1st Sit. 2010) (c) Traditional Knowledge (d) All of the above
(a) Expenditure method (b) Product method 13. The exchange of commodities between two countries is
(c) Matrix method (d) Income method referred as (SSC CGL 2nd Sit. 2010)
4. The monetary policy is India is formulated by (a) Balance of trade (b) Bilateral trade
(SSC CGL 1st Sit. 2010) (c) Volume of trade (d) Multilateral trade
(a) Central Government 14. A want becomes a demand only when it is backed by the
(b) Industrial Financial Corporation of India (SSC CGL 2nd Sit. 2010)
(c) Reserve Bank of India (a) Ability to purchase (b) Necessity to buy
(d) Industrial Development Bank of India (c) Desire to buy (d) Utility of the product
5. A short-term government security paper is called 15. The terms 'Micro Economics' and "Macro Economics" were
(SSC CGL 1st Sit. 2010) coined by (SSC CGL 2nd Sit. 2010)
(a) Share (b) Debenture (a) Alfred Marshall (b) Ragner Nurkse
(c) Mutual fund (d) Treasury bill (c) Ragner Frisch (d) J.M. Keynes
6. WTO basically promotes (SSC CGL 1st Sit. 2010) 16. During periods of inflation, tax rates should
(a) Financial support (b) Global peace (SSC CGL 2nd Sit. 2010)
(c) Unilateral trade (d) Multilateral trade (a) increase (b) decrease
7. Under which market condition do firms have excess (c) remain constant (d) fluctuate
capacity? (SSC CGL 1st Sit. 2010) 17. Which is the biggest tax paying sector in India ?
(a) Perfect competition (SSC CGL 2nd Sit. 2010)
(b) Monopolistic competition (a) Agriculture sector (b) Industrial sector
(c) Duopoly (c) Transport sector (d) Banking sector
(d) Oligopoly 18. "Economics is what it ought to be" – This statement refers
8. Price theory is also known as (SSC CGL 1st Sit. 2010) to (SSC CGL 2nd Sit. 2010)
(a) Macro Economics (b) Development Economics (a) Normative economics (b) Positive economics
(c) Public Economics (d) Micro Economics (c) Monetary economics (d) Fiscal economics
9. At present, India is following (SSC CGL 1st Sit. 2010) 19. The excess of price a person is to pay rather than forego the
(a) Fixed exchange rate consumption of the commodity is called
(b) Floating exchange rate (SSC CGL 2nd Sit. 2010)
(c) Pegged up exchange rate (a) Price (b) Profit
(d) Pegged down exchange rate (c) Producer's surplus (d) Consumer's surplus
10. National Income is the (SSC CGL 1st Sit. 2010) 20. When there is an official change in the exchange rate of
(a) Net National Product at market price domestic currency, then it is called:
(b) Net National Product at factor cost (SSC CGL 1st Sit. 2011)
(c) Net Domestic Product at market price (a) Appreciation (b) Depreciation
(d) Net Domestic Product at factor cost (c) Revaluation (d) Deflation
w
w
w
.y
ou
42 Economics

rs
m
21. Inflation redistributes income and wealth in favour of: 32. The study of population is known as.

ah
bo
(SSC CGL 1st Sit. 2011) (SSC CGL 2nd Sit. 2012)

ob
(a) Pensioners (b) Poor (a) Demography (b) Climatology

.w
(c) Middle class (d) Rich (c) Petrology (d) Hydrology

or
dp
22. The fringe benefit tax was introduced in the budget of 33. The BSE Sensex is based on how many stocks ?

re
(SSC CGL 1st Sit. 2011) (SSC CGL 1st Sit. 2012)

ss
.c
(a) 2003-04 (b) 2004-05 (a) 80 (b) 100

om
(c) 2005-06 (d) 2006-07 (c) 30 (d) 50
23. The purchase of shares and bonds of Indian companies by 34. The funds raised by the Government within the country is
Foreign Institutional Investors is called known as (SSC CGL 1st Sit. 2012)
(SSC CGL 2nd Sit. 2011) (a) Domestic budgetary source
(a) Foreign Direct Investment (b) Domestic banking source
(b) Portfolio Investment (c) Domestic saving source
(c) NRI Investment (d) Domestic monetary source
(d) Foreign Indirect Investment 35. Sick units are referred to (SSC CGL 1st Sit. 2012)
24. Monopolist resorts to price discrimination depending upon (a) SIA (b) DGTD
the (SSC CGL 1st Sit. 2012) (c) FIPB (d) BIFR
(a) Elasticity of supply (b) Elasticity of demand 36. The concept ‘Duopoly’ was introduced by
(c) Law of demand (d) Law of supply (SSC CGL 1st Sit. 2012)
25. Economic planning is an essential feature of (a) Sweezy (b) Malthus
(SSC CGL 1st Sit. 2012) (c) Ricardo (d) Cournot .
(a) Socialist economy (b) Capitalist economy 37. Which of the following is the feature of monopolistic
(c) Mixed economy (d) Dual economy competition ? (SSC CGL 1st Sit. 2012)
26. The Rashtriya Barh Ayog (RBA) is related with (a) Single firm (b) Large number of firms
(SSC CGL 1st Sit. 2012) (c) Group of firms (d) None of the above
(a) Droughts and Floods (b) Poverty Alleviation 38. Which one of the following is an example for a non-economic
(c) Floods (d) Disaster Management good? (SSC CGL 2nd Sit. 2012)
27. Low cost housing is an example for: (a) Doctor’s service (b) Teacher’s service
(SSC CGL 2nd Sit. 2012) (c) Mother’s service (d) Banker’s service
(a) Mixed wants (b) Social wants 39. Which one of the following committee is associated with
(c) Private wants (d) Merit wants banking sector reforms in India? (SSC CGL 2nd Sit. 2012)
28. Consumption for the sake of enjoying social (a) L. C. Gupta (b) Narsimhan
acknowledgement is called: (SSC CGL 2nd Sit. 2012) (c) Chakravarty (d) Kelkar
(a) Rational consumption 40. Which one of the following is not a qualitative credit control
(b) Social consumption measure of the RBI? (SSC CGL 2nd Sit. 2012)
(c) Conspicuous consumption (a) Fixing margin requirements
(d) Demonstration consumption (b) Variable interest rates
29. Of the following economists, whom do you consider to be (c) Open market operations
the Master of “Partial Analysis”? (SSC CGL 2nd Sit. 2012) (d) Credit rationing
(a) Leon Walras (b) Alfred Marshall 41. The 13th Five Year Plan will be operative for the period.
(c) J. M. Keynes (d) Lionel Robbins (SSC CGL 2nd Sit. 2012)
30. India making ‘Double Taxation Avoidance Agreements’ (a) 2010 - 2015 (b) 2011- 2016
(DTAA) with other countries for the promotion of: (c) 2012 - 2017 (d) 2013 - 2018
(SSC CGL 2nd Sit. 2012) 42. The national income of a nation is the
(a) Bilateral trade (SSC CGL 2nd Sit. 2012)
(b) External commercial borrowings (a) Government’s annual revenue
(c) Foreign direct investments (b) Sum total of factor incomes
(d) Foreign institutional investment (c) Surplus of public sector enterprises
31. Brain drain has been caused by: (SSC CGL 2nd Sit. 2012) (d) Exports minus imports
(a) failure to recognise talent in the originating country. 43. Externality theory is the basic theory of the following branch
(b) the lure of high living standards of Economics : (SSC CGL 1st Sit. 2013)
(c) lack of employment opportunities (a) Macro Economics (b) Environomics
(d) socio-economic instability (c) Fiscal Economics (d) International Economics
w
w
w
.y
ou
Economics 43

rs
m
44. Diamonds are priced higher than water because : 53. Forced Savings refer to (SSC CGL 1st Sit. 2013)

ah
bo
(SSC CGL 1st Sit. 2013) (a) Compulsory deposits imposed on income tax payers

ob
(a) consumers do not buy them at lower prices. (b) Provident fund contribution of private sector

.w
or
(b) they are sold by selected firms with monopolistic employees

dp
powers. (c) Reduction of consumption consequent to a rise in

re
ss
(c) their marginal utility to buyers is higher than that of prices

.c
om
water (d) Taxes on individual income and wealth
(d) their total utility to buyers is higher than that of water 54. High powered money is (SSC CGL 1st Sit. 2013)
45. "Functional Finance" is associated with : (a) Banks reserves at Central Bank
(SSC CGL 1st Sit. 2013) (b) All loans and advances of banks
(a) Abba 'P' Lerner (b) Adolph Wogner (c) Money held by banks
(c) Adam Smith (d) Adams (d) Currency held by public and reserves with the Central
46. Of the following land uses, which is restricted to Special Bank
Economic Zones ? (SSC CGL 1st Sit. 2013) 55. Hardening interest rate means (SSC CGL 1st Sit. 2013)
(a) Information Technology Companies (a) interest rate is remaining sticky
(b) Educational Institutions (b) interest rate is very low
(c) Free trade Centres (c) interest rate is increasing
(d) Marketing Centres (d) interest rate is falling
47. As per the TRIPS Agreement-1994, a good originating from 56. “Dumping” is a situation when the seller
a region with specific character/quality/reputation is (SSC CGL 1st Sit. 2013)
covered/to a protected under the IPR as (a) supplies mote than the demand for products in the
(SSC CGL 2nd Sit. 2013) World Market.
(a) Patent (b) supplies more in the Domestic Market.
(b) Trademark (c) sells a commodity at a lower price in the World Market
(c) Trade secret and charges a higher price in the Domestic Market.
(d) GI (Geographical Indicator) (d) sells a commodity at a higher price in the World Market
48. ‘Gold’ is mainly related to (SSC CGL 2nd Sit. 2011) and charges a lower price in the Domestic Market.
(a) Local market (b) National market 57. Tick the correct option with regards to the contribution
(c) International market (d) Regional market towards GDP (Gross Domestic Product) from Agriculture
49. The demand for labour is called (SSC CGL 1st Sit. 2013) (SSC CGL 1st Sit. 2013)
(a) Derived demand (b) Factory demand (a) Durin g 1950–51 (GDP 51-88%) and 2011–12
(c) Market demand (d) Direct demand (GDP 14-01%)
50. Which of the following is not an investment expenditure in (b) During 1950–51 (GDP 11-00%) and 2011–12 (GDP 25%)
goods and services? (SSC CGL 1st Sit. 2013) (c) Durin g 1990–91 (GDP 29-53%) and 2011–12
(a) Purchase of machinery (GDP 66-77%)
(b) An increase in business inventories (d) Durin g 1980–81 (GDP 35-69%) and 2011–12
(c) Expansion of the main plant of a company (GDP 20-69%)
(d) Purchase of a house 58. The principle of maximum social advantage is the basic
51. Which one of the following represents the Savings of the principle of (SSC CGL 1st Sit. 2013)
Private Corporate Sector? (SSC CGL 1st Sit. 2013) (a) Micro Economics
(a) Undistributed profits (b) Macro Economics
(b) Excess of income over expenditure (c) Fiscal Economics
(c) Dividends paid to shareholders (d) Environmental Economics
(d) Total profits of a company 59. Which Five Year Plan is not correct among the following?
52. Social accounting system in India is classified into (SSC CGL 1st Sit. 2013)
(SSC CGL 1st Sit. 2013) (a) First 1951–56 (b) Second 1956–61
(a) Assets, liabilities and debt position (c) Third 1961–66 (d) Fourth 1966–71
(b) Public sector, Private sector and Joint sector 60. An economic theory is a/an (SSC CGL 1st Sit. 2013)
(c) Income, product and expenditure (a) Axion (b) Proposition
(d) Enterprise, households and government (c) Hypothesis (d) Tested hypothesis
w
w
w
.y
ou
44 Economics

rs
m
61. Indian Special Economic Rules amendment came in the year 73. Which of the following method is not used in determining

ah
bo
(SSC CGL 1st Sit. 2013) National Income of a country? (SSC CGL 1st Sit. 2014)

ob
(a) 2000 (b) 2002 (a) Income Method (b) Output Method

.w
or
(c) 2004 (d) 2006 (c) Input Method (d) Investment Method

dp
62. According to Marx, the source of value is 74. What does the letter 'e' denotes in the term 'e-banking'?

re
(SSC CGL 1st Sit. 2013)

ss
(SSC CGL 1st Sit. 2014)

.c
(a) Capital (b) Land

om
(a) Essential Banking (b) Economic Banking
(c) Labour (d) None of the above (c) Electronic Banking (d) Expansion Banking
63. The Community Development Programme was launched in 75. Which among the following is not a Bretton Woods
the year (SSC CGL 1st Sit. 2013) Institution ? (SSC CGL 1st Sit. 2015)
(a) 1950 (b) 1952 (a) International Monetary Fund (IMF)
(c) 1951 (d) 1953 (b) World Bank
64. Prof. Milton Friedman was the leader of
(c) Organisaiton of Economic Cooperation an d
(SSC CGL 1st Sit. 2014) Development (O.E.C.D.)
(a) Ohio school (b) Chicago school
(d) None of these
(c) Cambridge school (d) London school
76. Equilibrium price in the market is determined by the
65. Which one of the following is not a qualitative control of
(SSC CGL 1st Sit. 2015)
credit by the Central Bank of a country?
(a) equality between total cost and total revenue
(SSC CGL 1st Sit. 2014)
(b) equality between average cost and average revenue.
(a) Rationing of credit
(b) Regulation of consumer credit (c) equality between marginal cost and marginal revenue
(c) Variation of the reserve ratio (d) equality between marginal cost and average cost.
(d) Regulation of margin requirements 77. In the national context which of the following indicates
66. The market in whcih loans of money can be obtained is Macro Approach ? (SSC CGL 1st Sit. 2015)
called (SSC CGL 1st Sit. 2014) (a) Sales of Bata Shoe Company
(a) Reserve market (b) Institutional market (b) Exports of Mangoes to U.K.
(c) Money market (d) Exchange market (c) Income from Railways
67. If the marginal return increases at a diminishing rate, the (d) Inflation in India
total return (SSC CGL 1st Sit. 2014) 78. Internal economies (SSC CGL 1st Sit. 2015)
(a) increases (b) decreases (a) arise in an economy as it makes progress
(c) remains constant (d) becomes zero (b) accrue to a firm when it expands its output
68. The law of Increasing Returns means (c) arise when there is expansion in internal trade
(SSC CGL 1st Sit. 2014) (d) arise when there is expansion in an industry
(a) increasing cost (b) decreasing cost 79. One of the features of a free market economy is
(c) increasing production (d) increasing income (SSC CGL 1st Sit. 2015)
69. Which of the following is the Regulator of the credit rating (a) public ownership of factors of production
agencies in India? (SSC CGL 1st Sit. 2014) (b) rationing and price control
(a) RBI (b) SBI
(c) consumer's sovereignty
(c) SIDBI (d) SEBI
(d) active state intervention
70. Which is the first Indian Company to be listed in NASDAQ?
80. Gross National Product – Depreciation Allowance = ?
(SSC CGL 1st Sit. 2014)
(SSC CGL 1st Sit. 2015)
(a) Reliance (b) TCS
(a) Gross Domestic Product
(c) HCL (d) Infosys
(b) Personal Income
71. RRBs are owned by (SSC CGL 1st Sit. 2014)
(c) Net National Product
(a) Central Government (b) State Government
(c) Sponsor Bank (d) Jointly by all of the above (d) Per Capita Income
72. The Monetary and Credit Policy is announced by which of 81. The Panchayat Samiti remains accountable for its functions
the following? (SSC CGL 1st Sit. 2014) to (SSC CGL 1st Sit. 2015)
(a) Ministry of Finance of Centre (a) The Gram Panchayats and Gram Sabhas
(b) Reserve Bank of India (b) Zilla Parishads
(c) State Bank of India (c) Anchal Panchayats
(d) Planning Commission of India (d) Janpad Panchayats
w
w
w
.y
ou
Economics 45

rs
m
82. The one rupee note bears the signature of : (c) NRI deposits

ah
bo
(SSC CGL 1st Sit. 2015) (d) External commercial borrowing

ob
(a) Governor, Reserver Bank of India 93. Which one of the following is not an instrument of Fiscal

.w
(SSC CGL 1st Sit. 2016)

or
(b) Finance Minister policy?

dp
(c) Secretary, Ministry of Finance (a) Open Market Operations

re
ss
(d) None of these (b) Taxation

.c
83. NABARD stands for (SSC CGL 1st Sit. 2015) (c) Public borrowing

om
(a) National business for Accounting and Reviewing (d) Public expenditure
(b) National Bank for Agriculture and Rural Development 94. In which of the following market forms, a firm does not
(c) National Bank for Aeronautics and Radar Development exercise control over price? (SSC CGL 1st Sit. 2016)
(d) National Bureau for Air and Road Transport (a) Monopoly
84. Surplus budget is recommended during : (b) Perfect competition
(SSC CGL 1st Sit. 2015) (c) Oligopoly
(a) Depression (b) Boom (d) Monopolistic competition
(c) War (d) Famines 95. What is the name of portal launched by RBI recently to
85. Economic profit or normal profit is the same as : check illegal money collection? (SSC CGL 1st Sit. 2016)
(SSC CGL 1st Sit. 2015) (a) Sahyog (b) Sahayata
(a) accounting profit (b) optimum profit (c) Sampark (d) Sachet
(c) net profit (d) maximum profit 96. Fiscal policy in India is formulated by?
86. CENVAT is related to which of the following ? (SSC CGL 1st Sit. 2016)
(SSC CGL 1st Sit. 2016) (a) Reserve Bank of India
(a) Sales Tax (b) Excise Duty
(b) Planning Commission
(c) Custom Duty (d) Service Tax
(c) Finance Ministry
87. Malthusian theory is associated with which of the following ?
(d) SEBI
(SSC CGL 1st Sit. 2016)
97. Open market operation refers to : (SSC CHSL 2012)
(a) Poverty (b) Employment
(a) borrowing by scheduled banks to industry and trade
(c) Diseases (d) Population
(b) purchase and sale of government securities by the rbi
88. Which is the parameter for the economic development ?
(c) deposit mobilisation
(SSC CGL 1st Sit. 2016)
(a) Per capita monetary income (d) borrowing by scheduled banks from the RBI
(b) National income 98. Who are the price-takers under Perfect Competition ?
(c) Per capita rural income (SSC CHSL 2012)
(d) Population (a) industry (b) government
89. A ‘Market Economy’ is one which (SSC CGL 1st Sit. 2016) (c) firms (d) buyers
(a) is controlled by the Government 99. An employment situation where the marginal productivity
(b) is free from the Government control of agricultural labour is zero is known as :
(c) in influenced by international market forces (SSC CHSL 2012)
(d) All of these (a) Seasonal unemployment
90. Regional Rural Banks are sponsored by (b) Cyclical unemployment
(SSC CGL 1st Sit. 2016) (c) Disguised unemployment
(a) Nationalised Commercial Bank (d) Disguised unemployment
(b) Reserve Bank of India 100. Which one of the following is a raw material oriented
(c) State Bank of India industry ? (SSC CHSL 2012)
(d) Government of India (a) Sugar industry
91. Who is the Chief Economic Advisor to the Government of (b) Petroleum refinery
India? (SSC CGL 1st Sit. 2016) (c) Light engineering industry
(a) Raghu Ram Rajan (b) Arvind Subramanian (d) Ship building
(c) Rajiv Mahirishi (d) Arvind Pangariya 101. The basic object of all production is to (SSC CHSL 2013)
92. Which type of foreign investment is considered as unsafe? (a) increase physical output
(SSC CGL 1st Sit. 2016) (b) satisfy human wants
(a) Foreign Direct Investment (FDI) (c) provide employment
(b) Portfolio investment (d) make profits
w
w
w
.y
ou
46 Economics

rs
m
102. Inflation is caused by (SSC CHSL 2013)

ah
110. Other things being equal, a decrease in quantity demanded

bo
(a) decrease in production of a commodity can be caused by (SSC CHSL 2014)

ob
(b) increase in money supply and decrease in production (a) a rise in the price of the commodity

.w
or
(c) increase in money supply (b) a rise in the income of the consumer

dp
(d) increase in production (c) a fall in the price of a commodity

re
ss
103. Dumping is a form of price discrimination at (d) a fall in the income of the consumer

.c
(SSC CHSL 2013)

om
111. Lender of the Last Resort is : (SSC CHSL 2015)
(a) local level (b) within industry (a) IDBI (b) NABARD
(c) national level (d) international level (c) SBI (d) RBI
104. The equilibrium of a firm under perfect competition will be
112. Perfectly inelastic demand is equal to : (SSC CHSL 2015)
determined when (SSC CHSL 2013)
(a) One (b) Zero
(a) Marginal Cost > Average Cost
(c) Infinite (d) Greater than one
(b) Marginal Revenue > Average Cost
113. When price of a substitute of commodity 'x' falls, the demand
(c) Marginal Revenue > Average Revenue
for 'x' (SSC CHSL 2015)
(d) Marginal Revenue = Marginal Cost
(a) remains unchanged
105. Expenditure on advertisement and public relations by an
enterprise is a part of its (SSC CHSL 2013) (b) Increases at increasing rate
(a) fixed capital (c) rises
(b) consumption of fixed capital (d) falls
(c) final consumption expenditure 114. Mixed Economy means : (SSC CHSL 2015)
(d) intermediate consumption (a) Co-existence of public and private sectors
106. Which of the following is not an economic problem ? (b) Co-existence of rich and poor
(SSC CHSL 2014) (c) Co-existence of small and large Industries
(a) Deciding between paid work and leisure. (d) Promoting both agriculture and Industries in the
(b) Deciding between expenditure on one good and the economy
other. 115. By whom was the autonomous investment separated from
(c) Deciding between alternative methods of personal induced investment ? (SSC CHSL 2015)
saving. (a) Malthus (b) Joan Robinson
(d) Deciding between different ways of spending leisure (c) Adam Smith (d) Schumpeter
time. 116. A demand curve will not shift: (SSC CHSL 2015)
107. Which of the following occurs when labour productivity (a) When only prices of substitute products change
rises ? (SSC CHSL 2014) (b) When there is a change in advertisement expenditure
(a) The equilibrium nominal wage falls
(c) When only price of the commodity changes
(b) The equilibrium quantity of labour falls
(d) When only income changes.
(c) Competitive firms will be induced to use more capital
117. The time element in price analysis was introduced by
(d) The labour demand curve shifts to the right
(SSC CHSL 2015)
108. Which of the following are consumer semi-surable goods ?
(a) Alfred Marshall (b) J.S. Mill
(SSC CHSL 2014)
(a) Cars and television sets (c) J.R. Hicks (d) J.M. Keynes
(b) Milk and milk products 118. A camera in the hands of a professional photographer is a
(c) Foodgrains and other food products ___________ good. (SSC Multitasking 2013)
(d) Electrical appliance like fans and electic irons (a) Capital (b) Free
109. Which of the following statements is correct ? (c) Intermediary (d) Consumer
(SSC CHSL 2014) 119. A portion of an individual's total income is spent on
(a) Most workers will work for less then their reservation consumption. The remaining part is called
wage. (SSC Multitasking 2014)
(b) The reservation wage is the maximum amount any firm (a) Savings (b) Deposits
will pay for a worker. (c) Surplus (d) Excess
(c) Economic rent is the difference between the market 120. Phase of increasing returns is otherwise called
wage and the reservation wage. (SSC Multitasking 2014)
(d) Economic rent is the amount one must pay to enter a (a) increasing cost (b) decreasing cost
desirable labour market. (c) increasing revenue (d) decreasing revenue
w
w
w
.y
ou
Economics 47

rs
m
121. Monopoly refers to (SSC Multitasking 2014) 128. Which of the following is not a function of the Exim Bank of

ah
bo
(a) competition among monopolists India? (SSC Sub. Ins. 2012)

ob
(b) absence of competition (a) Financing of export and import of goods and services

.w
(b) Inspection of exported goods for quality assurance

or
(c) a firm charging different prices to different customers

dp
(d) a market situation in which there is only one buyer of a (c) Financing of joint ventures in foreign countries

re
ss
commodity and one seller (d) Loans to Indian parties for contribution to share capital

.c
122. Primary sector of an economy includes of joint ventures abroad

om
(SSC Multitasking 2014) 129. The Social Forestry Scheme was introduced during
(a) Service sector rendering services like banking, (SSC Sub. Ins. 2012)
transport, etc. which one of the primary importance (a) Fourth Five Year Plan (b) Eighth Five Year Plan
(b) Important enterprises of the manufacturinig sector (c) Second Five Year Plan (d) Sixth Five Year Plan
(c) Indsutrial sector which is of primary importance for 130. Which one of the following iron and steel works in India is
the economy not under public sector? (SSC Sub. Ins. 2012)
(d) Agriculture and allied activities (a) Bokaro (b) Jamshedpur
123. Liquidity preference theory of interest is propounded by (c) Bhilai (d) Durgapur
(SSC Multitasking 2014) 131. If a firm is operating at loss in the short-period in perfect
combination. it should; (SSC Sub. Ins. 2013)
(a) J.S. Mill (b) A. Marshall
(a) decrease the production and the price.
(c) I. Fisher (d) J.M. Keynes
(b) increase the production and the price
124. Which one of the following statements is correct?
(c) continue to operate as long as it covers even the
(SSC Sub. Ins. 2012)
variable costs.
(a) A commodity will value if it is wanted by somebody.
(d) shut-down and leave the industry
(b) A commodity will have value only if it is scarce relative
132. Which of the following is an indirect tax ?
to demand.
(SSC Sub. Ins. 2013)
(c) The value of a commodity depends upon its price.
(a) Capital Gains Tax (b) Excise Duty
(d) The value of a commodity is entirely dependent upon
(c) Wealth Tax (d) Estate Duty
the substitutes.
133. Taxes on professions can be levied by:
125. Which of the following formulae could be used for
(SSC Sub. Ins. 2013)
calculating the per capita income of a country?
(a) State government only
(SSC Sub. Ins. 2012)
(b) both by state and union government
Total familyincome (c) by panchayats only
(a) Number of family members (d) Union government only
134. Which one is not a function of money?
Nationalincome (SSC Sub. Ins. 2013)
(b) Total population (a) Transfer of value (b) Store of value
(c) Price stabilisation (d) Value measurement
Totalincomeof various industries
(c) 135. "Closed Economy" means: (SSC Sub. Ins. 2013)
Total number of industrial wor kers (a) no provision for public sector
(b) no provision for private sector
TotalGovernment Revenue
(d) (c) economy policy not well defined
Totalpopulation
(d) a country having no imports and exports
126. For controlling inflation, the central bank should 136. At "Break-even point", (SSC Sub. Ins. 2013)
(SSC Sub. Ins. 2012) (a) the industry is in equilibrium in the long-run.
(a) sell Government securities in the open market (b) the producers suffers the minimum losses
(b) lower the bank rate (c) the seller earns maximum profit
(c) purchase Government securities in the open market (d) the firm is at zero-profit point
(d) lower the reserve ratio of the banks 137. In the balance of payments account, unrequited receipts
127. There is no selling cost under (SSC Sub. Ins. 2012) and payments are also regarded as (SSC Sub. Ins. 2014)
(a) Perfect competition (a) bilateral transfers
(b) Monopolistic competition (b) unilateral transfers
(c) Oligopoly (c) capital account transfers
(d) Duopoly (d) invisible transfers
w
w
w
.y
ou
48 Economics

rs
m
138. Price and output are determinates in market structure other 146. Basic problems of an economy is/are

ah
bo
than (SSC Sub. Ins. 2014) (SSC Sub. Ins. 2016)

ob
(a) monopoly (b) perfect competition (a) Providing Social Security and employment to all.

.w
or
(c) oligopoly (d) monopsony (b) Elimination of poverty and reduction of inequalities of

dp
139. If average cost falls, marginal cost (SSC Sub. Ins. 2014) income and wealth.

re
ss
(a) increases at a higher rate (c) Providing basic requirements of life to all.

.c
(d) Decide as to what, how and for whom to produce.

om
(b) falls at the same rate
(c) increases at a lower rate 147. Demand curve is indeterminate under
(d) falls at a higher rate (SSC Sub. Ins. 2016)
140. Which one of the following disburses long term loans to (a) duopoly (b) monopoly
private industry in India ? (SSC Sub. Ins. 2014) (c) pure competition (d) oligopoly
(a) Food Corporation of India 148. Marginal Revenue is (SSC Sub. Ins. 2016)
(b) Life Insurance Corporation of India (a) Revenue realized on the sale of an extra units.
(c) Primary Credit Society (b) Revenue realized from the sale of all units.
(d) Land Development Banks (c) The average revenue of a firm.
141. A low interest policy is also known as : (d) Revenue realized on every unit sold.
(SSC Sub. Ins. 2015) 149. All economic goods are considered are ................in
economics. (SSC Steno. 2013)
(a) investment policy
(a) Wealth (b) Money
(b) income generating policy
(c) Capital (d) Materials
(c) dear money policy
150. Total demand for goods and services at various levels of
(d) cheap money policy
employment is called as (SSC Steno. 2013)
142. The market equilibrium for a commodity is determined by:
(a) Effective demand (b) National demand
(SSC Sub. Ins. 2015)
(c) Market Demand (d) Employment demand
(a) the balancing of the forces of demand and supply for
151. Relationship between price of related goods and quantity
the commodity.
of a particular commodity is called as (SSC Steno. 2013)
(b) the intervention of the Government.
(a) Income - demand (b) Market - demand
(c) the market demand of the commodity. (c) Cross - demand (d) Price - demand
(d) the market supply of the commodity. 152. Devaluation means (SSC Steno. 2014)
143. “Economies” of a firm are: (SSC Sub. Ins. 2015) (a) reduction in the external value of currency
(a) an increase in its profits (b) fall in valuation of the essentials in an economy
(b) its dominance of the market (c) depreciation of the fixed assets
(c) saving in its production costs (d) increase in the currency value in terms of foreign
(d) a reduction in its selling expenses currency
144. Regarding money supply situation in India it can be said 153. Disguised unemployment means (SSC Steno. 2014)
that the: (SSC Sub. Ins. 2015) (a) willing to work and not getting the work
(a) Currency with the public is almost equal to the deposits (b) not getting work on all days
with the banks. (c) insufficiency of capital structure to absorb the large
(b) Currency with the public is inconvertible only. force
(c) Currency with the public is more than the deposits (d) more people working, than required
with the banks. 154. The '3 B 's' - brokers, bankers and businessmen were
(d) Currency with the public is less than the deposits with responsible for (SSC Steno. 2014)
the banks. (a) The Great Depression
145. The equilibrium price of a commodity will definitely rise of (b) Russian Revolution
there is a/an: (SSC Sub. Ins. 2015) (c) French Revolution
(a) increase in supply combined with a decrease in (d) First world war
demand. 155. ATM means (SSC Steno. 2014)
(b) increase in demand accompanied by a decrease in (a) All Time Money
supply. (b) Automatic Teller Machine
(c) decrease in both demand and supply. (c) Automated Teller Machine
(d) increase in both demand and supply. (d) Any Time Money
w
w
w
.y
ou
Economics 49

rs
m
156. Which market forms allow free entry and exit of firms? 160. An indirect instrument of monetary policy is

ah
bo
(SSC Steno. 2014) (SSC Steno. 2016)

ob
(a) Perfect and Monopolistic (a) Open market operations

.w
or
(b) Perfect and Oligopoly (b) Statutory liquidity ratio

dp
(c) Oligopoly and Monopoly (c) Bank rate

re
ss
(d) Monopoly and Monopolistic (d) Cash reserve ratio

.c
157. GNP differs from NNP due to (SSC Steno. 2014) 161. One of the following is NOT a component of foreign

om
(a) net indirect taxes (b) Direct taxes exchange reserves in India (SSC Steno. 2016)
(c) interest on public debt (d) Depreciation (a) Gold stock of RBI
158. Which one of the following is an example of joint supply? (b) SDR holdings of government
(SSC Steno. 2014) (c) Foreign exchange assets of RBI
(a) Petrol and Bus (d) Foreign exchange assets of government
(b) Ink and Fountain pen 162. The market system in which there are only two buyers facing
(c) Sugar and Coffee a large number of sellers is called (SSC Steno. 2016)
(d) Wool and Cotton (a) monopsony (b) duopsony
159. Who fixes the REPO rate in India ? (SSC Steno. 2016) (c) duopoly (d) oligopoly
(a) WTO - World Trade Organization 163. 'Investing opportunities model' was proposed by
(b) SEBI - Securities and Exchange Board of India (SSC Steno. 2016)
(c) RBI - Reserve Bank of India (a) Revenstein (b) Davis
(d) IMF - International Monetary Fund (c) E.S. Lee (d) S.A. Stouffer
w
w
w
.y
ou
50 Economics

rs
m
ah
HINTS & SOLUTIONS

bo
ob
.w
or
dp
re
1. (a) A trade bloc is a type of intergovernmental agreement, 25. (a) Economic planning is an essential feature of socialism.

ss
often part of a regional intergovermental organization, The most prominent example of a planned economy

.c
om
where regional barriers to trade, (tariffs and non-tariff was the economic system of the Soviet Union.
barriers) are reduced or eliminated among the 26. (c) The Government of India decide to set up the Rashtriya
participating states. Barh Ayog (National Flood Commission) in 1976 to
2. (b) 3. (c) evolve a coordinated, integrated and scientific
4. (c) The Reserve Bank of India is India’s central banking approach to the flood control problems in the country.
institution, which controls the monetary policy of the 27. (d) The concept of a merit good introduced in economics
indian rupee. It commenced its operations on 1 April by Richard Musgrave (1957, 1959) is a commodity
1935 during the British Rule in accordance with the which is judged that an individual or society should
provisions of the Reserve Bank of India Act, 1934. have on the basis of some concept of need, rather
5. (d) T- bills are issued to meet short-term mismatches in than ability and willingness to pay. Examples include
receipts and expenditure. Bonds of longer maturity the provision of food stamps to support nutrition, the
are called dated securities. delivery of health services to improve quality of life
6. (d) 7. (b) 8. (d) 9. (b) and reduce morbidity, subsidized housing and
10. (b) National income is the total value a country’s final arguably education.
output of all new goods and services produced in one 28. (c) Conspicuous consumption is the purchase of goods
year.
or services for the specific purpose of displaying one's
11. (b) Economic Advisory Council to the Prime Minister
wealth. It is a means to show ones social status.
(PMEAC) is a non–constitutional, non–permanent and
29. (b) Alfred Marshall was committed to partial equilibrium
independent body constituted to give economic advice
models over general equilibrium on the grounds that
to the Government of India, specifically the Prime
the inherently dynamical nature of economics made
Minister. The council serves to highlight key economic
the former more practically useful.
issues facing the country to the government of India
30. (a) These agreements give the right of taxation in respect
from a neutral viewpoint.
of the income of the nature of interest, dividend, royalty
12. (d)
and fees for technical services to the country of
13. (c) A bilateral trade is the exchange of goods between two
residence. It promotes bilateral trade and investment.
countries that facilitates trade and investment by
reducing or eliminating tariffs, import quotas, export 31. (c) Brain drain can be described as the process in which a
restraints and other trade barriers. country loses its most educated and talented workers
14. (d) to other countries through migration. The main causes
15. (d) Ragnar Anton Kittil Frisch was a Norwegian economist include seeking employment or higher paying jobs,
and the co-recipient of the first Nobel Memorial Prize political instability and to seek a better quality of life.
in Economic Sciences in 1969 (with Jan Tinbergen). He 32. (a) Demography is the statistical study of human
is known for having founded the discipline of populations and sub-populations. It encompasses the
econometrics, and for coining the widely used term study of the size, structure, and distribution of these
pair macroeconomics /microeconomics in 1933. populations, and spatial and/or temporal changes in
16. (a) 17. (b) them in response to birth, migration, aging and death.
18. (a) Normative economics (as opposed to positive 33. (c) 34. (a) 35. (d) 36. (d) 37. (c)
economics) is a part of economics that expresses value 38. (c)
or normative judgments about economic fairness or 39. (b) In order to raise the standards of the banks
what the outcome of the economy or goals of public internationally, a number of committees were appointed
policy ought to be. by RBI. Among them Narasimham committee I (1991),
19. (c) 20. (c) 21. (d) 22. (c) 23. (d) Narasimham Committee II (1998) and Verma Committee
24. (b) The act of selling the same article, product under a (1999) were influential in improving international
single control, at different prices to different buyers is standards, and led to banking sector reforms, globally
known as price discrimination. Information on the price flexible to its deregulation, norms and conditions etc.
elasticity of demand can be used by business as part The above said committees have basically identified
of a policy of price discrimination (also known as yield the causes for the weak banks and guidelines have
management). been given to improve their efficiency.
w
w
w
.y
ou
Economics 51

rs
m
40. (c) 52. (c) Social accounting is a method by which a firm seeks

ah
bo
41. (c) Here, statement of the question is wrong. The tenure to place a value on the impact on society of its

ob
from 2012 to 2017 is designated as12th Five Year Plan operations. One social accounting system primarily

.w
not 13th Five Year Plan. The tenure of 13th Five-Year attempts to measure National Income, final product,

or
dp
Plan would be 2018 - 2022. consumption and accumulation of capital.

re
42. (b) 53. (c) Forced saving is an economic situation in which

ss
consumers spend less than their disposable income,

.c
43. (d) In economics, an externality is the cost or benefit that

om
affects a party who did not choose to incur that cost or not because they want to save but because the goods
benefit. Economists often urge governments to adopt they seek are not available or because goods are too
policies that "internalize" an externality, so that costs expensive.
and benefits will affect mainly parties who choose to 54. (a) Bank's reserves at Central Bank
incur them. 55. (a) Interest rate is remaining sticky
56. (c) Sells a commodity at lower price in the world market
44. (c)
and charges a higher price in the domestic market
45. (a) Functional finance is an economic theory proposed
57. (a) 58. (c) 59. (d) 60. (b) 61. (d)
by Abba P. Lerner, based on effective demand principles
62. (c) 63. (b)
and chartalism. It states that government should
64. (b) Milton Friedman (July 31, 1912 - November 16, 2006)
finance itself to meet explicit goals, such as taming the
was an American economist, statistician, and writer
business cycle, achieving full employment, ensuring
who taught at the University of Chicago for more than
growth, and low inflation. three decades. He was a recipient of the 1976 Nobel
46. (c) Prize in Economic Sciences, and is known for his
47. (d) Geographical Indication (GI) means the name of a research on consumption analysis, monetary history
region or a locality, a specific place, or, in exceptional and theory, and the complexity of stabilization policy.
cases, a country, used to describe a product originating 65. (c) The qualitative or selective methods of credit control
in that region, locality, specific place or country, which are adopted by the Reserve Bank in its pursuit of
possesses a specific quality, reputation or other economic stabilization an d as part of credit
characteristics attributable to that geographical origin, management. The four important methods are Margin
an d the production and/or processin g and or Requirements, Credit Rationing, Regulation of
preparation of which take place in the defined Consumer Credit, Moral Suasion.
geographical area. 66. (c) A segment of the financial market in which financial
48. (c) Gold is mainly related to the international market as of instruments with high liquidity and very short
all the precious metals, it is the most popular as an maturities are traded. The money market is used by
investment. Gold has been used throughout history participants as a means for borrowing and lending in
as money and has been a relative standard for currency the short term, from several days to just under a year.
equivalents specifc to economic regions or countries, 67. (a) In economics, diminishing returns (also called law of
until recent times. Gold price has shown a long term diminishing returns, law of variable proportions,
correlation with the price of crude oil. principle of diminishing marginal productivity, or
49. (a) The demand for labour is “derived” from the diminishing marginal returns is the decrease in the
production and demand for the product being marginal (incremental) output of a production process
demanded. If the demand for the product increases, as the amount of a single factor of production is
either the price will increase or the demand for incrementally increased, while the amounts of all other
production labour will increase until the equilibrium factors of production stay constant.
price and production numbers are met. 68. (b) The law of increasing returns is the opposite of the law
50. (d) Investment expenditure refers to the expenditure of decreasing returns. Where the law of diminishing
incurred either by an individual or a firm or the returns operates, every additional investment of capital
and labour yields less than proportionate returns. But,
government for the creation of new capital assets like
in the case of the law of increasing returns, the return
machinery, building etc. The purchase of house cannot
is more than proportionate.
be considered as investment exienditure as it may be for
69. (d) The Securities and Exchange Board of India (SEBI) is
personal use.
the regulator for the securities market in India. It was
51. (a) For private corporate sector, retained profits adjusted
established in the year 1988 and given statutory powers
for non operating surplus/ deficit is considered as its
on 12 April 1992 through the SEBI Act, 1992.
Net Saving. Retained profits are those which are
70. (d) Infosys Ltd is an Indian multinational corporation that
ploughed back into business after making provides business consulting, information technology,
commitments to depreciation provision for various software engineering and outsourcing services. It is
fixed assets, debts, government and to share-holders. headquartered in Bangalore, Karnataka.
w
w
w
.y
ou
52 Economics

rs
m
71. (d) Regional Rural Banks are local level banking 82. (d) The one rupee note bears the signature of secretary

ah
bo
organizations operating in different States of India. ministry of Finance.

ob
They have been created with a view to serve primarily 83. (b) NABARD is set up as an apex Development Bank with

.w
the rural areas of India with basic banking and financial a mandate for facilitating credit flow for promotion and

or
dp
services. The main purpose of RRB's is to mobilize development of agriculture, small-scale industries,

re
financial resources from rural / semi-urban areas and cottage and village industries, handicrafts and other

ss
grant loans and advances mostly to small and marginal

.c
rural crafts. It was established on 12 July 1982 by a

om
farmers, agricultural labourers and rural artisans. special act by the parliament
72. (b) The Reserve Bank of India is India's Central Banking 84. (b) Surplus budget is the order of the economies in boom
Institution, which controls the Monetary Policy of the time.
Indian Rupee. It commenced its operations on 1 April 85. (b) Economic profit is the difference between total revenue
1935 during the British Rule in accordance with the and total opportunity cost. If a firm's total opportunity
provisions of the Reserve Bank of India Act, 1934. cost is less than the total revenues then the firm is
73. (d) The national income of a country can be measured by
making economic profit. If a firm's total opportunity
three alternative methods: (i) Product Method (ii)
cost is greater than the total revenues then the firm is
Income Method, and (iii) Expenditure Method.
making economic losses. It is similar to optimum profit.
74. (c) For many people, electronic banking means 24-hour
86. (c) Concept relating to Central Excise Duty and Service
access to cash through an automated teller machine
Tax:- For. a proper understanding what is CENVAT,
(ATM) or Direct Deposit of paychecks into checking
some basic knowledge about Central Excise duty,
or savings accounts. But electronic banking involves
many different types of transactions, rights, Service Tax etc, is necessary.
responsibilities - and sometimes, fees. 87. (d) 88. (b) 89. (b) 90. (a) 91. (b)
75. (c) The Bretton Woods Institutions are the World Bank, 92. (b)
and theInternational Monetary Fund (IMF). They were 93. (a) The major instruments of fiscal policy are as follows:
set up at a meeting of 43 countries in Bretton Woods, A. Budget B. Taxation C. Public Expenditure D. Public
New Hampshire, USA in July 1944. Works E. Public Debt.
76. (c) The price and output under monopoly are determined 94. (b)
by equality between marginal cost and marginal 95. (d) The Reserve Bank of India (RBI) Governor,
revenue and not by the intersection of demand and RaghuramRajan, on 4 August 2016 launched Sachet
supply curves. portal, sachet.rbi.org.in, tocheck illegal money
77. (d) Macroeconomics is a branch of economics dealing with collection.
the performance, structure, behavior, and decision- 96. (c) 97. (b) 98. (c) 99. (c) 100. (a)
making of an economy as a whole, rather than individual 101. (b) The basic object of all production is to satisfy human
markets. Macroeconomists develop models that explain wants. In terms of Economics, satisfaction of human
the relationship between such factors as national income, wants comes first, second comes the provision of
output, consumption, unemployment, inflation etc. employment, third comes the profit motive and at last
78. (d) Internal economies arise within the firm because of the comes the increase of physical outputs.
expansion of the size of a particular firm. They are called 102. (b) Inflation is caused by increase in money supply and
the economies of scale. decrease in production. In economics, inflation is a
79. (c) In a free market economy there is a freedom of choice sustained increase in the general price level of goods
for the consumers to buy goods and services which and services in an economy over a period of time. It
suit their tastes and preferences. This is generally called can be defined as too much money chasing too few
the principle of consumer sovereignty. This means in goods. When the general price level rises, each unit
a market economy the consumers are just like a king or
of currency buys fewer goods and services.
sovereign who dictate what goods and services and
103. (d) Dumping is a form of price discrimination at the
what quantities of them are produced.
international level.In economics, "dumping" is a kind
80. (c) Net national product (NNP) refers to gross national
of predatory pricing, especially in the context of
product (GNP), i.e. the total market value of all final
international trade. It occurs when manufacturers
goods and servicesproduced by the factors of
export a product to another country at a price either
production of a country or other polity during a given
time period, minus depreciation. below the price charged in its home market or below
NNP = GNP – Depreciation its cost of production.
81. (b) Zilla Parishad is the apex body of the PR(Panchayati 104. (d) The equilibrium of a firm under perfect competition
Raj) system located at the district level. Chairpersons/ will be determined when marginal revenue equals
Presidents of Panchayat Samitis come within its marginal cost. In the short run, perfectly-competitive
jurisdiction. markets are not productively efficient as output will
w
w
w
.y
ou
Economics 53

rs
m
not occur where marginal cost is equal to average cost 120. (d) Phase of increasing returns is otherwise called

ah
bo
(MC=AC). They are allocatively efficient, as output decreasing revenue. If output increases by more than

ob
will always occur where marginal cost is equal to that proportional change in inputs, there are increasing

.w
marginal revenue (MC=MR). returns to scale (IRS).

or
dp
105. (d) Expenditure on advertisement and public relation by 121. (b) A monopoly exists when a specific person or enterprise

re
an enterprise is a part of its intermediate consumption. is the only supplier of a particular commodity.

ss
.c
Visibility through advertising and promotion builds Monopolies are thus characterized by a lack of

om
an enterprise' reputation with the customer. Although economic competition to produce the good or service
most advertising is through word of mouth, that word and a lack of viable substitute goods.
of mouth starts with awareness that customers have 122. (d) The primary sector of the economy is the sector of an
gained about the enterprise through their advertising economy making direct use of natural resources. This
and promotions. includes agriculture, forestry, fishing and mining. This
106. (d) Deciding between different ways of spending leisure is contrasted with the secondary sector, producing
time is not an economic problem. manufactured goods, and the tertiary sector, producing
107. (d) When labour productivity rises, the labour demand services.
curve shifts to the right. As the productivity increases, 123. (d) J.M. Keynes propounded what has come to be known
the production function shifts up and simultaneously as the liquidity preference theory of interest. According
the labour demand curve shifts out and right. At a to this theory, the rate of interest is determined by the
given real wage, more workers are hired and output demand for and supply of money.
increases. 124. (b)
108. (c) Semi durable consumer goods are products with 125. (b) Per capita income, more simply known as income per
durability of about one year. E.g., food grains and food person, is the mean income within an economic
products. aggregate such as a country or city. It is calculated by
109. (c) Economic rent is the difference between the market taking a measure of all sources of income in the
wage and the reservation wage. aggregate (such as GDP or Gross national income) and
dividing it by the total population.
110. (a) Other things being equal, a decrease in quantity
126. (a) The Central Bank can use various different methods
demanded of a commodity can be caused by a rise in
for reducing inflation. To control inflation, central bank
the price of the commodity.
sells the government securities to the public through
111. (d) As a Banker to Banks, the Reserve Bank of India acts
the banks. This results in transfer of a part of bank
as the “lender of the last resort”.
deposits to central bank account and reduces credit
112. (b) When the price elasticity of demand for a good is
creation capacity of the commercial banks.
perfectly inelastic i.e. Ed = 0.
127. (a) In perfect competition, there must be large number of
113. (d) It is because demand for goods which have substitutes
buyers and sellers. According to Scitovsky buyers and
is more elastic because when price of a substitute falls
sellers are price takers in the purely competitive market.
in relation to its commodity, the demand for the
Each seller (or firm) sells its products at the price
commodity also falls.
determined by the market. Similarly, each buyer buys
114. (a) Mixed economy means an economic system in which
the commodity at the price determined by the market.
both the private enterprise and a degree of state
128. (b)
monopoly (usually in public services, defense,
129. (d) Social forestry means the management and protection
infrastructure, and basic industries) coexist. of forests and afforestation on barren lands with the
115. (d) 116. (b) purpose of helping in the environmental, social and
117. (a) Alfred Marshall Propounded the theory that price is rural development. This scheme was introduced in Sixth
determined by both demand and supply and also gave Five Year Plan.
great importance to the time element in the 130. (b) At present all important steel plants except TISCO (Tata
determination of price. Iron and Steel co. Ltd) which is located in Jamshedpur
118. (a) Camera is a capital for a photographer because he earns are under public sector.
his livelihood as it is his occupation 131. (c) The situation when a firm is operating at loss in the
119. (a) If a portion of individuals income is spent on short period in perfect competition arises when the
consumption, the remaining portion will be saving. price is so low that total revenue is not even enough to
Since whatever is not consumed must be saved, as cover the variable cost of production. Shut down point
soon as we specify a consumption function we have is that point at which the price is equal to average
necessarily specified a savings function. "Function" variable costs or the firm covers its variable costs. So
just means that one thing depends on another thing or it should operate as long as it covers even the variable
things. costs.
w
w
w
.y
ou
54 Economics

rs
m
132. (b) Some examples of indirect taxes include value added 137. (b) In the balance of payment accounts, unrequited

ah
bo
tax, excise duty, sales tax, stamp duty and custom duty payments or receipts (gifts) are regarded s unilateral

ob
levied on imports. These are taxes levied by the state transfers. In the balance-of-payments statement of a

.w
on expenditure and consumption, but not on property country, the accounts that show the amount of money

or
dp
or income. sent or received as gifts.

re
133. (a) In India, the professional tax is imposed at the state 138. (b) Pr ice, output and perfect competition are the

ss
.c
level. However, not all the states impose this tax. determinants of market structure. The demand curve

om
Business owners, working individuals, merchants and of an individual firm under a purely competitive
people carrying out various occupations comes under industry is perfectly elastic. This is the increase or
the purview of this tax. Professional tax is levied by decrease of the output of a single seller has no effect
on the total supply and market place.
particular Municipal Corporations.
139. (b) Marginal cost and average cost curves are related. If
134. (c) Generally, economists have defined four types of
average costs falls, marginal cost falls at the same rate.
functions of money which are as follows: (i) Medium
140. (d) Land development bank (LDB) disburses long term
of exchange (transfer of value) (ii) Measurement of
loans to private industry in India.The main objective
value. (iii) Standard of deferred payments. and (iv) Store
of the LDB is to promote the development of land,
of value. Price stabilization is a function of monetary agriculture and increase the agricultural production.
policy. The LDB provides long-term finance to members
135. (d) Closed economy is an economy in which no activity is directly through its branches
conducted with outside economies. A closed economy 141. (d) Cheap Money policy is a monetary policy through
is self-sufficient, meaning that no imports are brought which a bank sets low interest rates so that credit is
in and no exports are sent out. The goal is to provide easily achievable.
consumers with everything that they need from within 142. (a) Market equilibrium is a market state where the supply
the economy's borders. in the market is equal to the demand in the market.
136. (d) The break-even point (BEP) is the point at which cost 143. (d) 144. (d) 145. (b) 146. (d) 147. (d)
or expenses and revenue are equal: there is no net loss 148. (a) 149. (d) 150. (c) 151. (c) 152. (a)
or gain, and one has "broken even." For businesses, 153. (b) 154. (a) 155. (c) 156. (a) 157. (d)
reaching the break-even point is the first major step 158. (d) 159. (c) 160. (a) 161. (c) 162. (c)
towards profitability. 163. (d)
w
w
w
.y
ou
rs
m
5

ah
bo
ob
.w
GENERAL SCIENCE

or
dp
re
ss
CHAPTER

.c
om
1. A concave lens always forms an image which is 11. In which of the following processes is energy released ?
(SSC CGL 1st Sit. 2010) (SSC CGL 1st Sit. 2010)
(a) real and erect (b) virtual and erect (a) Respiration (b) Photosynthesis
(c) real and inverted (d) virtual and inverted (c) Ingestion (d) Absorption
2. A vitamin requires cobalt for its activity. The vitamin is 12. Animals living in the three trunks are known as
(SSC CGL 1st Sit. 2010) (SSC CGL 1st Sit. 2010)
(a) Vitamin B12 (b) Vitamin D (a) Arboreal (b) Volant
(c) Vitamin B2 (d) Vitamin A (c) Amphibious (d) Aquatic
3. One of the constituents of tear gas is 13. In input frequency of a full wave rectifier be n, then output
(a) Ethane (b) Ethanol frequency would be (SSC CGL 1st Sit. 2010)
(c) Ether (d) Chloropicrin
n
4. The modulus of rigidity is the ration of (a) (b) n
2
(SSC CGL 1st Sit. 2010)
(a) longitudinal stress to longitudinal strain 3n
(b) Volume stress to volume strain (c) (d) 2n
2
(c) shearing stress to shearing strain
14. Heat transfer horizontally within the atmosphere is called
(d) tensile stress to tensile strain
(SSC CGL 1st Sit. 2010)
5. The propagation of sound waves in a gas involves
(SSC CGL 1st Sit. 2010) (a) Conduction (b) Convection
(a) adiabatic compression and refraction (c) Absorption (d) Advection
(b) isothermal compression and rarefaction 15. Noise is measured in (SSC CGL 1st Sit. 2010)
(c) isochoric compression and rarefaction (a) Watt (b) REM
(d) isobaric compression and rarefaction (c) Centigrade (d) Decibel
6. An stomic clock is based on transitions in 16. The bats can fly in the dark because
(SSC CGL 1st Sit. 2010) (SSC CGL 1st Sit. 2010)
(a) Sodium (b) Caesium (a) they can see the objects in darkness
(c) Magnesium (d) Aluminium (b) they have weak legs and are likely to be attacked by
7. Plasma membrane in eukaryotic celle is made up of predators
(SSC CGL 1st Sit. 2010)
(c) they generate flashes of light
(a) Phospholipid (b) Lipoprotein
(d) they generate ultrasonic sound waves
(c) Phospholpo-protein (d) Phospho-protein
17. What changes will happen to a bow of ice and water kept at
8. Which one of the following is also called the 'power plants'
of the cell ? (SSC CGL 1st Sit. 2010) exactly zero degree Celsius? (SSC CGL 1st Sit. 2010)
(a) Golgi body (b) Mitochondrion (a) All ice will melt
(c) Ribosome (d) Lysosome (b) All water will become ice
9. What is the chemical name of vinegar ? (c) No change will happen
(SSC CGL 1st Sit. 2010) (d) Only some ice will melt.
(a) Citric acid (b) Acetic acid 18. The art and science of map making is called
(c) Pyruvic acid (d) Malic acid (SSC CGL 2nd Sit. 2010)
10. Which of the following is not a property of heavy water ? (a) Remote Sensing (b) Cartography
(SSC CGL 1st Sit. 2010) (c) Photogrammetry (d) Mapping
(a) Boiling point of heavy water is lower than that of 19. Silver halides are used in photographic plates because they
ordinary water
are (SSC CGL 2nd Sit. 2010)
(b) Density of heavy water is higher than that of ordinary
(a) oxidised in air
water
(c) Freezing point of heavy water is higher than that of (b) soluble in hyposolution
ordinary water (c) reduced by light
(d) It produces corrosion (d) totally colourless
w
w
w
.y
ou
56 General Science

rs
m
20. Tetra ethyle lead (TEL) is (SSC CGL 2nd Sit. 2010) 32. The longest bone in the human body is:

ah
bo
(a) a catalyst in burning fossil fuel (SSC CGL 1st Sit. 2011)

ob
(b) an antioxidant (a) Ulna (b) Humerus

.w
(c) a reductant (c) Femur (d) Tibia

or
dp
(d) an antiknock compound 33. The time period of a pendulum when taken to the Moon

re
21. Curie point is the temperature at which would: (SSC CGL 1st Sit. 2011)

ss
(SSC CGL 2nd Sit. 2010)

.c
(a) remain the same (b) decrease

om
(a) Matter becomes radioactive (c) become zero (d) increase
(b) A metal loses magnetic properties 34. The function of ball hearings in a wheel is:
(c) A metal loses conductivity (SSC CGL 1st Sit. 2011)
(d) Transmutation of metal occurs.
(a) to increase friction
22. The isotope used for the production of atomic energy is
(b) to convert kinetic friction into rolling friction
(SSC CGL 2nd Sit. 2010)
(c) to convert static friction into kinetic friction
(a) U-235 (b) U-238
(d) just for convenience
(c) U-234 (d) U-236
35. 'Shock–absorbers' are usually made of steel as it:
23. The acceleration due to gravity at the equator
(SSC CGL 2nd Sit. 2010) (SSC CGL 1st Sit. 2011)
(a) is less than that at the poles (a) is not brittle
(b) is greater than that at the poles (b) has lower elasticity
(c) is equal to that at the poles (c) has higher elasticity
(d) does not depend on the earth's centripetal acceleration (d) has no ductile property
24. Which of the following is not a nucleon ? 36. Which of the following could be used as fuel in propellant or
(SSC CGL 2nd Sit. 2010) rockets? (SSC CGL 1st Sit. 2011)
(a) Proton (b) Neutron (a) Liquid Hydrogen + Liquid Nitrogen
(c) Electron (d) Positron (b) Liquid Oxygen + Liquid Argon
25. The material used in the manufacture of lead pencil is (c) Liquid Nitrogen + Liquid Oxygen
(SSC CGL 2nd Sit. 2010) (d) Liquid Hydrogen + Liquid Oxygen
(a) Graphite (b) Lead 37. The addition of gypsum to portland cement helps in:
(c) Carbon (d) Mica (SSC CGL 1st Sit. 2011)
26. Angle of friction and angle of repose are (a) increasing the strength of cement
(SSC CGL 2nd Sit. 2010) (b) rapid setting of cement
(a) equal to each other (c) preventing rapid setting of cement
(b) not equal to each other (d) reduction in the cost of cement
(c) proportional to each other 38. White lung disease is prevalent among the workers of:
(d) None of the above (SSC CGL 1st Sit. 2011)
27. What happens to a person who receives the wrong type of (a) Paper industry (b) Cement industry
blood? (SSC CGL 2nd Sit. 2010)
(c) Cotton industry (d) Pesticide industry
(a) All the arteries constrict
39. lodoform is used as an: (SSC CGL 1st Sit. 2011)
(b) All the arteries dialates
(a) antipyretic (b) analgesic
(c) The RBCs agglutinate
(c) antiseptic (d) anaesthetic
(d) The spleen and lymphnodes deteriorate
40. An artificial ecosystem is represented by:
28. If all bullets could not be removed from gun shot injury of a
man, it may cause poisoning by (SSC CGL 2nd Sit. 2010) (SSC CGL 1st Sit. 2011)
(a) Mercury (b) Lead (a) pisciculture tank (b) agricultural land
(c) Iron (d) Arsenic (c) zoo (d) aquarium
29. Ringworm is a ................ disease. (SSC CGL 2nd Sit. 2010) 41. The constituents of automobile exhaust that can cause
(a) Bacterial (b) Protozoan cancer is are: (SSC CGL 1st Sit. 2011)
(c) Viral (d) Fungal (a) Oxides of nitrogen
30. Pituitary gland is situated in (SSC CGL 2nd Sit. 2010) (b) Carbon monoxide
(a) the base of the heart (c) Polycyclic hydrocarbons
(b) the base of the brain (d) Lead
(c) the neck 42. The optimum dissolved oxygen level (in mg/litre) required
(d) the abdomen for survival of aquatic organisms is:
31. Saliva helps in the digestion of: (SSC CGL 1st Sit. 2011) (SSC CGL 1st Sit. 2011)
(a) Fats (b) Starch (a) 4 – 6 (b) 2 – 4
(c) Proteins (d) Vitamins (c) 8 – 10 (d) 12 – 16
w
w
w
.y
ou
General Science 57

rs
m
43. The world's only floating national park is situated in: (c) refraction and total internal reflection of light

ah
(SSC CGL 1st Sit. 2011)

bo
(d) dispersion of light only

ob
(a) Manipur (b) Kuala Lumpur 56. The phenomenon of light associated with the apprearance

.w
(c) Bilaspur (d) Dispur of blue colour of the sky is (SSC CGL 2nd Sit. 2011)

or
dp
44. Who invented vaccination for 'Small Pox'? (a) Interference (b) Reflection

re
(SSC CGL 2nd Sit. 2011) (c) Refraction (d) Scattering

ss
(a) Sir Fredrick Grant Banting 57. Lens is made up of (SSC CGL 2nd Sit. 2011)

.c
om
(b) Sir Alexander Fleming (a) Pyrex glass (b) Flint glass
(c) Edward Jenner (c) Ordinary glass (d) Cobalt glass
(d) Louis Pasteur 58. The element which is used for vulcanizing rubber
45. BT seed is associated with (SSC CGL 2nd Sit. 2011) (SSC CGL 2nd Sit. 2011)
(a) Rice (b) Wheat (a) Sulphur (b) Bromine
(c) Cotton (d) Oil seeds (c) Silicon (d) Phosphorus
46. Which one of the following minerals is found in Monazite 59. Which of the following is responsible for the extra strength
sand? (SSC CGL 2nd Sit. 2011) of pyrex glass? (SSC CGL 2nd Sit. 2011)
(a) Potassium (b) Uranium (a) Potassium Carbonate (b) Lead Oxide
(c) Thorium (d) Sodium (c) Borax (d) Ferric Oxide
47. In coriander, the useful parts are (SSC CGL 2nd Sit. 2011) 60. The noble gas used for the treatment of cancer is
(a) roots & leaves (SSC CGL 2nd Sit. 2011)
(b) leaves & flowers (a) Helium (b) Argon
(c) leaves & dried fruits (c) Krypton (d) Radon
(d) flowers & dried fruits 61. Fertilization occurs normally in the
48. Which plant is called Herbal Indian Doctor? (SSC CGL 1st Sit. 2012)
(SSC CGL 2nd Sit. 2011) (a) Cervix (b) Vagina
(a) Amla (b) Mango (c) Fallopian tube (d) Uterus
(c) Neem (d) Tulsi 62. People consuming alcohol in heavy quantities generally die of
(SSC CGL 1st Sit. 2012)
49. The pH of human blood is (SSC CGL 2nd Sit. 2011)
(a) liver or stomach cancer
(a) 7.2 (b) 7.8
(b) weakening of heart muscles leading to cardiac arrest
(c) 6.6 (d) 7.4
(c) blood cancer
50. Which amongst the following is largest endocrine gland in
(d) Cirrhosis
the body? (SSC CGL 2nd Sit. 2011)
63. The organisms at the base of the grazing food-chain are
(a) Thyroid (b) Parathyroid
(SSC CGL 1st Sit. 2012)
(c) Adrenal (d) Pituitary
(a) Carnivores (b) Decomposers
51. Which amongst the following is the largest mammal?
(c) Producers (d) Herbivores
(SSC CGL 2nd Sit. 2011)
64. The mass of 10 moles of water is (SSC CGL 1st Sit. 2012)
(a) Elephant (b) Whale
(a) 90 g (b) 45 g
(c) Dinosaur (d) Rhinoceros
(c) 18 g (d) 180 g
52. Which part becomes modified as the tusk of elephant? 65. Vitamin A is rich in (SSC CGL 1st Sit. 2012)
(SSC CGL 2nd Sit. 2011) (a) Carrot (b) Lime
(a) Canine (b) Premolar (c) Beans (d) Rice
(c) Second incisor (d) Molar 66. The high boiling point of water compared to hydrogen
53. Optical fibres are based on the phenomenon of sulphide or hydrogen chloride is due to
(SSC CGL 2nd Sit. 2011) (SSC CGL 1st Sit. 2012)
(a) Interference (a) Dipole insulation
(b) Dispersion (b) Van der Waal's attraction
(c) Diffraction (c) Polar covalent bonding
(d) Total Internal Reflection (d) Hydrogen bonding
54. Now–a–days yellow lamps are frequently used as street 67. Which of the following determines the chemical properties
lights. Which of the following gases is used in these lamps? of an element? (SSC CGL 1st Sit. 2012)
(SSC CGL 2nd Sit. 2011) (a) Number of electrons
(a) Sodium (b) Neon (b) Number of neutrons
(c) Hydrogen (d) Nitrogen (c) Number of protons
55. 'Mirage' is an example of (d) All of the above
(SSC CGL 2nd Sit. 2011) 68. 'Lumen' is the unit of (SSC CGL 1st Sit. 2012)
(a) refraction of light only (a) Illuminance (b) Brightness
(b) total internal reflection of light only (c) Luminous flux (d) Luminous intensity
w
w
w
.y
ou
58 General Science

rs
m
69. Which one of the following forces is a 'dissipative force'? 83. We receive sunlight on earth surface. What type of light

ah
(SSC CGL 1st Sit. 2012) (SSC CGL 2nd Sit. 2012)

bo
beams are these?

ob
(a) Electrostatic force (b) Magnetic force (a) Random (b) Parallel

.w
(c) Gravitational force (d) Frictional force (c) Converging (d) Diverging

or
(SSC CGL 2nd Sit. 2012)

dp
70. If a resistive wire is elongated, its resistance 84. Polar-bears hold cures for:

re
(SSC CGL 1st Sit. 2012) (a) Type II diabetes (b) Osteoporosis

ss
(a) decreases (b) remains constant (c) Breast-cancer (d) Kidney failure

.c
om
(c) increases (d) All of the above 85. Which colour/colours of light has the highest velocity
71. If a magnet has a third pole, then the third pole is called through vacuum? (SSC CGL 2nd Sit. 2012)
(SSC CGL 1st Sit. 2012) (a) Blue (b) Red
(a) defective pole (b) consequent pole (c) Green (d) All of the above
(c) extra pole (d) arbirary pole 86. The ultimate source of energy in a hydroelectric power
72. The sweet taste of fruits is due to (SSC CGL 1st Sit. 2012) station is: (SSC CGL 2nd Sit. 2012)
(a) Lactose (b) Fructose (a) solar energy
(c) Maltose (d) Ribose (b) the potential energy of water
73. Which is NOT a correct statement? (c) the kinetic energy of water
(SSC CGL 2nd Sit. 2012) (d) the electro-chemical energy of water
(a) Phenols are acidic 87. The disease that kills more people than lung cancer as a
(b) In benzene all the atoms lie in one plane consequence of air pollution is: (SSC CGL 2nd Sit. 2012)
(c) Methylated spirit contains only methanol (a) chronic bronchitis (b) asthma
(d) Dilute solutions contain less amount of solute (c) emphesema (d) heart attack
74. The infective stage of Malaria is: 88. Which of the following pairs is correctly matched?
(SSC CGL 2nd Sit. 2012) (SSC CGL 2nd Sit. 2012)
(a) Gametocyte (b) Ring stage (a) Milk of lime–sodium sulphate
(c) Sporozoite (d) Merozoite (b) Glauber’s salt–calcium sulphate
75. Which of the following is meant for the ex-situ conservation (c) Salt petre–potassium nitrate
of various species? (SSC CGL 2nd Sit. 2012) (d) Gypsum–calcium hydroxide
(a) Sperm bank (b) Blood bank 89. ‘Eutrophication’ is associated with
(c) Germplasm bank (d) Herbarium (SSC CGL 1st Sit. 2012)
76. An algae type ocean deposit is: (SSC CGL 2nd Sit. 2012) (a) Nitrates and Phosphates
(a) Neritic remains (b) Diatom Ooze (b) Sewage
(c) Pteropod Ooze (d) Pelagic deposits (c) Silt load
77. Photosynthetic vesicle found in bacteria is called a: (d) Vegetation
(SSC CGL 2nd Sit. 2012) 90. The best way to maintain a natural equilibrium between the
(a) Mesosome (b) Chromatophore pest and predator is by using (SSC CGL 1st Sit. 2012)
(c) Genophore (d) Pneumatophore (a) insecticides (b) biological control
78. What type of mirror is used in a view finding mirror of a (c) pesticides (d) herbicides
vehicle? (SSC CGL 2nd Sit. 2012) 91. ‘Dry ice’ is the condensed form of
(a) Convex mirror (b) Plane mirror (SSC CGL 1st Sit. 2012)
(c) Concave mirror (d) Paraboloidal mirror (a) sulphur tri-oxide
79. Pyroligneous acid obtained from wood contains: (b) carbon dioxide
(SSC CGL 2nd Sit. 2012) (c) highly cooled water
(a) 10% Formaldehyde (b) 10% Acetic acid (d) oxygen
(c) 10% Formic acid (d) 10% ethanol 92. The materials which are strongly attracted by magnet are
80. Union Carbide India Ltd. manufactured essentially: called (SSC CGL 1st Sit. 2012)
(SSC CGL 2nd Sit. 2012) (a) ferro-magnetic substances
(a) Heavy water (b) Petrochemicals (b) universal substances
(c) Fertilizers (d) Leather goods (c) para-magnetic substances
81. Drying oils contain a fairly large proportion of: (d) dia-magnetic substances
(a) Unsaturaled fatty acids (SSC CGL 2nd Sit. 2012)
93. In a pin-hole camera, we usually get
(b) Fats
(SSC CGL 1st Sit. 2012)
(c) Proteins
(a) erect impression (b) inverted impression
(d) Saturated fatty acids
(c) erect image (d) inverted image
82. The red, orange and yellow colours of leaves are due to:
94. What happens to the kinetic energy of gas molecules with
(SSC CGL 2nd Sit. 2012)
rise of temperature ? (SSC CGL 1st Sit. 2012)
(a) Carotenoids (b) Aldehydes
(a) Remains same (b) Fluctuates
(c) Tannins (d) Lignins
(c) Increases (d) Decreases
w
w
w
.y
ou
General Science 59

rs
m
95. The sex of a child is determined (SSC CGL 1st Sit. 2012) 108. During respiration, the gases enter into the blood and leave

ah
bo
(a) six to seven weeks after conception the same by the process of (SSC CGL 2nd Sit. 2012)

ob
(b) in the third month of pregnancy (a) Active transport

.w
(c) at the time of sperm’s entry (b) Diffusion

or
dp
(d) at the time of fertilisation of ovum (c) Diffusion and active transport

re
96. The food chain of the ecosystem is maintained by the (d) Osmosis

ss
(SSC CGL 1st Sit. 2012) (SSC CGL 2nd Sit. 2012)

.c
activities of 109. Heart is devoid of

om
(a) Decomposers (b) Predators (a) Cardiac muscle (b) Involuntary muscle
(c) Producers (d) Consumers (c) Voluntary muscle (d) Smooth muscle
97. Which one of these primates is closest to the modern man? 110. The soil salinity is measured by (SSC CGL 2nd Sit. 2012)
(SSC CGL 1st Sit. 2012) (a) Conductivity meter (b) Hygrometer
(a) Orang-utan (b) Chimpanzee (c) Psychrometer (d) Auxanometer
(c) Gorilla (d) Gibbon 111. Which of the following is a fungal disease?
98. A gram of fertile agricultural soil may contain bacteria upto (SSC CGL 2nd Sit. 2012)
(SSC CGL 1st Sit. 2012) (a) Leucoderma (b) Eczema
(a) five million (c) Ringworm (d) Elephantiasis
(b) one billion and above 112. Chickenpox is caused by (SSC CGL 2nd Sit. 2012)
(c) fifty thousand (a) DNA virus (b) Variola virus
(d) five hundred thousand (c) Streptococcus (d) Vibrio cholerae
99. A reduction reaction involves (SSC CGL 1st Sit. 2012) 113. Instruments can be shielded from outside magnetic effects
(a) addition of oxygen by surrounding them with (SSC CGL 2nd Sit. 2012)
(b) addition of nitrogen (a) Iron shield (b) Rubber shield
(c) addition of hydrogen (c) Brass shield (d) Glass shield
(d) None of the above 114. Find the odd one. (SSC CGL 2nd Sit. 2012)
100. An antiknock for petrol is (SSC CGL 1st Sit. 2012) (a) Marble (b) Chalk
(a) Sodium hydroxide (b) Ethanol (c) Limestone (d) Slaked lime
(c) Sodium benzoate (d) Lead tetraethyl 115. The following is a pseudo-force: (SSC CGL 2nd Sit. 2012)
101. Which one of the following pairs is not correctly matched ? (a) Centrepetal force
(SSC CGL 1st Sit. 2012) (b) Centrifugal reaction force
(a) Vitamin C - Scurvy (c) Centrifugal force
(b) Vitamin K - Clotting of blood (d) Strong nuclear force
(c) Vitamin A - Night blindness 116. The hydraulic brake used in automobiles is a direct
(d) Vitamin E - Rickets application of (SSC CGL 2nd Sit. 2012)
102. The fibre that will yield ammonia when destructively distilled (a) Archimedes’ principle
is (SSC CGL 1st Sit. 2012) (b) Torricellean law
(a) Wool (b) Cellulose acetate (c) Bernoulli’s Theorem
(c) Cotton (d) Silk (d) Pascal’s law
103. During respiration, energy is derived from 117. Amides can be converted to amines by the reaction named
(SSC CGL 1st Sit. 2012) (SSC CGL 2nd Sit. 2012)
(a) ATP (b) Chlorophyll (a) Perkin (b) Claisen
(c) RNA (d) DNA (c) Hoffman (d) Clemmesen
104. When was a global network of daily temperature records 118. The base used as an antacid is (SSC CGL 2nd Sit. 2012)
created ? (SSC CGL 1st Sit. 2012) (a) Calcium hydroxide
(a) Around 1890 (b) Around 1920 (b) Barium hydroxide
(c) Around 1800 (d) Around 1850 (c) Magnesium hydroxide
105. Sex hormones are (SSC CGL 1st Sit. 2012) (d) Silver hydroxide
(a) Vitamins (b) Alkanes 119. A process which is not helpful in the prevention of rusting
(c) Carbohydrates (d) Steroids of iron is (SSC CGL 2nd Sit. 2012)
106. A form of condensation that reduces visibility and causes (a) annealing (b) applying grease
breathing problems is (SSC CGL 2nd Sit. 2012) (c) galvanising (d) painting
(a) Dew (b) Frost 120. Denatured alcohol (SSC CGL 2nd Sit. 2012)
(c) Smog (d) Mist (a) is a form of alcohol
107. Green glands are associated with (SSC CGL 2nd Sit. 2012) (b) is unfit for drinking as it contains poisonous substances
(a) Reproduction (b) Excretion (c) contains coloured impurities
(c) Respiration (d) Digestion (d) is sweet to taste
w
w
w
.y
ou
60 General Science

rs
m
121. Phenolics as pollutants can be removed from waste water 132. The disease that has been eradicated from the world, is :

ah
(SSC CGL 2nd Sit. 2012) (SSC CGL 1st Sit. 2013)

bo
by use of

ob
(a) Ion exchange resin technique (a) Small pox (b) Leprosy

.w
(b) Electrolyte decomposition technique (c) Poliomyelitis (d) Chicken pox

or
dp
(c) Reverse osmosis method 133. In dicots the pollen grains possess :

re
(d) Polymeric adsorbents (SSC CGL 1st Sit. 2013)

ss
122. The stability of a pond ecosystem depends on (a) one germ pore (b) two germ pores

.c
om
(SSC CGL 2nd Sit. 2012) (c) three germ pores (d) four germ pores
(a) micro-organisms and fishes 134. 'Farad' is the unit of : (SSC CGL 1st Sit. 2013)
(b) micro-organisms and zoo planktons (a) Conductnce (b) Capacitance
(c) fishes and reptiles (c) Inductance (d) Resistance
(d) producers and consumers 135. Steel is more elasitc than rubber because it :
123. Supersonic air planes create a shock wave called (SSC CGL 1st Sit. 2013)
(SSC CGL 2nd Sit. 2012) (a) is harder than rubber
(a) Transition wave (b) Ultrasound (b) requires larger deforming force
(c) Transverse wave (d) Sonic boom (c) is never deformed
124. The danger signals are red while the eye is more sensitive to (d) is deformed very easily
yellow because (SSC CGL 2nd Sit. 2012) 136. Identify the old term amongst the following group :
(a) absorption in red is less than yellow and hence red is (SSC CGL 1st Sit. 2013)
visible from a distance (a) Coaxial cable (b) Optical fibre
(b) scattering in yellow light is less than red (c) Twisted pair wire (d) Microwaves
(c) the wavelength of red light is more than yellow light 137. Stains of rust on clothes can be removed by :
(d) none of the above reasons (SSC CGL 1st Sit. 2013)
125. Transboundary pollution (or) Acid rain is caused by : (a) H2O2 (b) Oxalic acid
(SSC CGL 1st Sit. 2013) (c) Petrol (d) Alcohol
(a) Nitrogen oxide and sulphur dioxide 138. The percentage of nitrogen present in ammonium sulphate
(b) Carbon monoxide is : (SSC CGL 1st Sit. 2013)
(c) Carbon dioxide (a) 18% (b) 21%
(d) Hydrocarbon (c) 25% (d) 30.5%
126. Which of the following is an endemic species? 139. Ethanol containing 5% water is known as :
(SSC CGL 1st Sit. 2013) (SSC CGL 1st Sit. 2013)
(a) Nicobar pigeon (b) Horn bill (a) Absolute alcohol (b) Dilute alcohol
(c) Indian Rhino (d) Pink head duck (c) Power alcohol (d) Rectified spirit
127. The natural disaster in which carbon-di-oxide suddenly 140. The hormone used as an oral contraceptive is :
erupts from a deep lake water is known as __________. (SSC CGL 1st Sit. 2013)
(SSC CGL 1st Sit. 2013) (a) Aldesterone (b) Cortisone
(a) Liminic (b) Lacaustrine (c) Progesterone (d) Testosterone
(c) Fluvial (d) Glacial 141. Expand the term IPCC: (SSC CGL 1st Sit. 2013)
128. Blood group was discovered by : (a) International Pollution Control Council
(a) Alexander Fleming (b) William Harvey (b) International Panel of Climate Control
(c) Landsteiner (d) Pavlov (c) Interim Panel of Climate Change
129. Pick out the correct match : (SSC CGL 1st Sit. 2013) (d) Intergovernmental Panel on Climate Change
(a) Egg yolk Protein and Fat 142. Acceptable "Noise Pollution Level" in India range between:
(b) Fleshy foods Calcium and Protein (SSC CGL 1st Sit. 2013)
(c) Fish Starch and Vitamin (a) 10-15 dec (b) 16-35 dec
(d) Milk Fibre and Minerals (c) 40-45 dec (d) 70-100 dec
130. Blood is a : (SSC CGL 1st Sit. 2013) 143. Endosulfan spray on cashew crop resulted in the pollution
(a) reproductive tissue (b) connective tissue to the tune of tragedy in : (SSC CGL 1st Sit. 2013)
(c) epithelial tissue (d) muscular tissue (a) Tamil Nadu (b) Kerala
131. DPT vaccine is administered to prevent diseases like : (c) Andhra Pradesh (d) Karnataka
(SSC CGL 1st Sit. 2013) 144. Hypothensmia occurs due to loss of excessive heat from
(a) Diphtheria, Pertussis and Typhoid body due to sudden low body temperature in :
(b) Diphtheria, Pertussis and Tetanus (SSC CGL 1st Sit. 2013)
(c) Dengue, Pertussis and Typhoid (a) Snakes (b) Frogs
(d) Dengue, Polio and Tetanus (c) Human beings (d) Lizards
w
w
w
.y
ou
General Science 61

rs
m
145. Solids which conduct electricity at higher temperature but 157. Of the following man-made disasters, which is socially

ah
bo
not at lower temperature are called induced? (SSC CGL 2nd Sit. 2013)

ob
(SSC CGL 2nd Sit. 2013) (a) Debris Avalanche (b) Salt Water Intrusion

.w
(a) super-conductor (d) metallic-conductor (c) Arson (d) Ozone depletion

or
dp
(c) semi-conductor (d) insulator 158. Which one of the following endocrine gland is situated in

re
146. Which one of the following has greatest mass? the neck? (SSC CGL 2nd Sit. 2013)

ss
.c
(SSC CGL 2nd Sit. 2013) (a) Pancreas (b) Thyroid

om
(a) electron (d) proton (c) Pituitary (d) Adrenals
(c) neutron (d) hydrogen nucleus 159. The seat of intelligence is situated in the
147. A television channel is characterised by (SSC CGL 2nd Sit. 2013)
(SSC CGL 2nd Sit. 2013) (a) cerebrum (b) cerebellum
(a) frequency of transmitted signal (c) medulla (d) thalamus
(b) velocity of transmitted signal 160. What is the Normal Blood Volume in human adult?
(c) physical dimension of television screen (SSC CGL 2nd Sit. 2013)
(d) size of picture tube (a) One litre (b) Three litres
148. The density of water is 1 g/cc. This is strictly valid at (c) Five litres (d) Seven litres
(SSC CGL 2nd Sit. 2013) 161. The fasting blood glucose level in adults in mg/100 ml is
(a) 0°C (b) 4°C (SSC CGL 2nd Sit. 2013)
(c) 25°C (d) 100°C (a) 200 (b) 160
149. The process of photosynthesis involves conversion of (c) 100 (d) 60
(SSC CGL 2nd Sit. 2013) 162. Entomology is the study of (SSC CGL 2nd Sit. 2013)
(a) chemical energy into radiant energy (a) Birds (b) Insects
(b) chemical energy into mechanical energy (c) Fossils (d) Fungi
(c) solar energy into chemical energy 163. Exobiology is a science that deals with
(d) mechanical energy into solar energy (SSC CGL 2nd Sit. 2013)
(a) extinct forms
150. A colloidal system in which a liquid is dispersed in a liquid
(b) life in other planets
is called. (SSC CGL 2nd Sit. 2013)
(c) life in the outer space
(a) gel (b) emulsion
(d) life in marine habitat
(c) sol (d) precipitate
164. In radio-communication, the signals emitted by transmitting
151. The antiseptic compound present in dettol is
antenna are reflected on (SSC CGL 2nd Sit. 2013)
(SSC CGL 2nd Sit. 2013)
(a) stratosphere (b) ozonosphere
(a) Iodine (b) Enloroxylenol
(c) ionosphere (d) troposphere
(c) Biothional (d) Cresol
165. Tiny marine animals which constitute limestone skeletons
152. Genomic (DNA) studies in camel have been completed
are called (SSC CGL 1st Sit. 2013)
recently by the scientists of (SSC CGL 2nd Sit. 2013)
(a) Coral reefs (b) Diatoms
(a) South Africa (b) India
(c) Clamitomonous (d) Foraminifera
(c) China (d) Pakistan
166. Which of the following plant pigments absorbs in red and
153. Air quality depicting PM 2.5 is more hazardous to
far-red region of light? (SSC CGL 1st Sit. 2013)
(SSC CGL 2nd Sit. 2013)
(a) Phytochrome (b) Cryptochrome
(a) Archaeological Monuments (c) Carotenoide (d) Chlorophyll
(b) National Parks 167. The process through which excess of light energy is
(c) Botanical Gardens dissipated in photosynthesis is known as
(d) Old Men and Women (SSC CGL 1st Sit. 2013)
154. Piped Natural Gas (PNG) is used for (a) Photolysis (b) Photophosphorylation
(SSC CGL 2nd Sit. 2013) (c) Quenching (d) Scavenging
(a) Mining (b) Welding 168. AIDS virus destroys (SSC CGL 1st Sit. 2013)
(c) Anaesthesia (d) Cooking (a) Neutrophils (b) Basophils
155. Greenpark Stadium is in (SSC CGL 2nd Sit. 2013) (c) Lymphocytes (d) Monocytes
(a) Bengaluru (b) Dehradun 169. The device used for measuring the wavelength of X-rays is
(c) Chandigarh (d) Kanpur (SSC CGL 1st Sit. 2013)
156. Which of the following is an endangered species? (a) Bragg Spectrometer
(SSC CGL 2nd Sit. 2013) (b) Mass Spectrometer
(a) Black buck (b) Blue sheep (c) G. M. Counter
(c) Gangetic dolphin (d) Mithun (d) Cyclotron
w
w
w
.y
ou
62 General Science

rs
m
170. Alpha particle is the nucleus of an atom of 183. Reduction of nitrates to ammonia can be achieved through

ah
bo
(SSC CGL 1st Sit. 2013) one of the following methods : (SSC CGL 1st Sit. 2013)

ob
(a) Helium (b) Oxygen (a) in alkaline medium using Devarda’s alloy.

.w
(c) Lithium (d) Hydrogen (b) in neutral medium using Devarda’s alloy.

or
dp
171. Teeth and Bones acquire strength and rigidity from (c) in acidic medium using Devarda’s alloy.

re
(SSC CGL 1st Sit. 2013) (d) in neutral medium using Cupric oxide.

ss
184. Grits of sewage are removed in (SSC CGL 1st Sit. 2013)

.c
(a) Chlorine (b) Sodium

om
(c) Calcium (d) Fluorine (a) Grit chamber (b) Detritus tank
172. The type of tail found in Shark is (c) Skimming tank (d) Trickling filter
(SSC CGL 1st Sit. 2013) 185. The most affected sulphur containing amino acid by PAN is
(SSC CGL 1st Sit. 2013)
(a) Heterocercal (b) Diphycercal
(a) Cysteine (b) Methonine
(c) Protocercal (d) Homocercal
(c) Proline (d) Globuline
173. The Sigmoid Colon is part of (SSC CGL 1st Sit. 2013)
186. Which of the following has zero electron affinity ?
(a) Large Intestine (b) Ileum
(SSC CGL 1st Sit. 2013)
(c) Small Intestine (d) Anal Canal (a) Oxygen (b) Fluorine
174. A good conductor while carrying current is (c) Nitrogen (d) Neon
(SSC CGL 1st Sit. 2013) 187. The human body’s largest blood vessel is
(a) positively charged (SSC CGL 1st Sit. 2013)
(b) electrically neutral (a) Pulmonary artery (b) Aorta
(c) alternately charged positive and negative (c) Renal artery (d) Coronary artery
(d) negatively charged 188. In human body, which one of the following hormones regulates
175. Coating of solid waste with imperviouos material is known blood calcium and phosphate ? (SSC CGL 1st Sit. 2013)
as (SSC CGL 1st Sit. 2013) (a) Glucagon
(a) Landfill (b) Capping (b) Growth hormone
(c) Encapsulation (d) Chemical fixation (c) Parathyroid hormone
176. Ultraviolet rays can be used in water treatment as (d) Thyroxine
(SSC CGL 1st Sit. 2013) 189. How do most insects respire ? (SSC CGL 1st Sit. 2013)
(a) Hydrolyser (b) Disinfectant (a) Through skin (b) Through gills
(c) By tracheal system (d) By lungs
(c) Flocculator (d) Precipitator
190. In nuclear reactions, there is conservation of
177. Thiamidine dimer formation in DNA is caused by
(SSC CGL 1st Sit. 2013)
(SSC CGL 1st Sit. 2013)
(a) mass only (b) momentum only
(a) b and g-rays (b) UV-rays (c) energy only (d) mass, energy and momentum
(c) IR-rays (d) X-rays 191. When a particle and an antiparticle come in contact with
178. Silicone is a polymer of (SSC CGL 1st Sit. 2013) each other, they (SSC CGL 1st Sit. 2013)
(a) Dialkyl dichloro silane (a) repell each other (b) annihilate each other
(b) Silane (c) go undisturbed (d) spin about a common axis
(c) Tetraalkyl silane 192. Photoelectric effect is (SSC CGL 1st Sit. 2013)
(d) Silicon tetrachloride (a) an instantaneous process
179. Which is a natural colloid? (SSC CGL 1st Sit. 2013) (b) delayed process
(a) Cane-sugar (b) Blood (c) emission of protons
(c) Sodium chloride (d) Urea (d) emission of neutrons
180. Which one of the following does not contain Silver? 193. For a particle moving with a constant speed along a straight
(SSC CGL 1st Sit. 2013) line PQ, the hodograph is (SSC CGL 1st Sit. 2013)
(a) German Silver (b) Horn Silver (a) a straight line parallel to PQ
(c) Ruby Silver (d) Lunar Caustic (b) a straight line perpendicular to PQ
181. The presence of Cobalt in Vitamin B12 was established for (c) a point
(d) a circle
the first time by (SSC CGL 1st Sit. 2013)
194. Aluminium is obtained by the electrolysis of pure Al2O3
(a) Borax-Bead test (b) Sodium Nitroprusside test
dissolved in (SSC CGL 1st Sit. 2013)
(c) Hydrolysis test (d) Spectroscopy
(a) Bauxite (b) Cryolite
182. Which bacterial strain developed from natural isolates by (c) Feldspar (d) Alumina
genetic manipulations can be used for treating oil spills? 195. Complete hydrolysis of cellulose gives
(SSC CGL 1st Sit. 2013) (SSC CGL 1st Sit. 2013)
(a) Clostridium (b) Nitrosomonas (a) D-fructose (b) L-glucose
(c) Pseudomonas (d) Agrobacterium (c) D-glucose (d) L-fructose
w
w
w
.y
ou
General Science 63

rs
m
196. Each body segment of Earthworm is called 209. Rutherford's scattering experiment proved the presence of

ah
(SSC CGL 1st Sit. 2013) (SSC CGL 1st Sit. 2013)

bo
ob
(a) Proglottid (b) Metamere (a) atoms in all matter

.w
(c) Scolex (d) Rostellum (b) electrons in atoms

or
dp
197. The pollutants which move downward with percolating (c) neutrons in atoms

re
ground water are called (SSC CGL 1st Sit. 2013) (d) nucleus in atoms

ss
(a) Leachates (b) Pollutates 210. When a metal is heated in a flame, the electrons absorb energy

.c
om
(c) Earthites (d) Percolates and jump to higher energy state. On coming back to the
198. Lungs are located in the (SSC CGL 1st Sit. 2013) lower energy state, they emit light, which we can observe in
(a) abdominal cavity (b) pericardial cavity (SSC CGL 1st Sit. 2013)
(c) peritoneal cavity (d) pleural cavity (a) Raman spectra (b) Absorption spectra
199. Which one of the following is the ideal food for newborn (c) Emission spectra (d) Fluorescence
babies? (SSC CGL 1st Sit. 2013) 211. Blood pressure may be increased by the excessive secretion
(a) Water (b) Sugar of (SSC CGL 1st Sit. 2013)
(c) Honey (d) Milk (a) Thyroxine (b) Testosterone
200. Transcription means the synthesis of (c) Estradiol (d) Estrol
(SSC CGL 1st Sit. 2013) 212. The Concept of 'Green House Gases" was postulated by
(a) Lipids (b) Protein (SSC CGL 1st Sit. 2013)
(c) DNA (d) RNA (a) Joseph Furier (b) Abdul Kalam
201. Hydrochloric acid is secreted by the cells lining the (c) M. S. Swaminathan (d) Richael Carlson
(SSC CGL 1st Sit. 2013) 213. "Bhopal gas tragedy" 1984 is related to
(a) Oral cavity (b) Stomach (SSC CGL 1st Sit. 2013)
(c) Ileum (d) Colon (a) Aluminium Phosphide
202. Emulsification is (SSC CGL 1st Sit. 2013) (b) Methyl bromide
(a) breaking fats into small globules
(c) Methyl isocyanate
(b) digestion of fats
(d) Carbon dioxide
(c) absorption of fats
214. The Particulate Matter (PM–10) exhaled from the polluted
(d) storage of fats
atmosphere is often filtered out during the process of
203. Taxonomy is a science that deals with
(SSC CGL 1st Sit. 2013)
(SSC CGL 1st Sit. 2013)
(a) Coughing (b) Sneezing
(a) Morphology (b) Anatomy
(c) A and B (d) Urination
(c) Classification (d) Economic uses
215. The animal who can consume more salt among the following
204. Which of the following is responsible for the working of
is (SSC CGL 1st Sit. 2013)
Newton's colour disc experiment? (SSC CGL 1st Sit. 2013)
(a) Sheep (b) Camel
(a) Formation of pure spectra
(c) Donkey (d) Dog
(b) Formation of impure spectra
(c) Persistence of vision 216. Cactus is referred to as (SSC CGL 2014)
(d) Principle of complementary colour (a) Hydrophyte (b) Mesophyte
205. The dimension MLT–2 corresponds to (c) Xerophyte (d) Epiphyte
(SSC CGL 1st Sit. 2013) 217. Which of the following is not a renewable resource?
(a) force (b) work done (SSC CGL 2014)
(c) acceleration (d) velocity (a) Thorium (b) Geothermal heat
206. Who is the founder of quantum theory of radiation? (c) Tidal power (d) Radiant energy
(SSC CGL 1st Sit. 2013) 218. How many neck canal cells are found in the archegonium of
(a) Einstein (b) Bohr a fern? (SSC CGL 2014)
(c) Plank (d) S.N. Bose (a) One (b) Two
207. Fiber optics cable used in communication, works on the (c) Three (d) Four
principle of (SSC CGL 1st Sit. 2013) 219. Which angiosperm is vesselless? (SSC CGL 2014)
(a) regular reflection of light (a) Hydrilla (b) Trochodendron
(b) diffuse reflection of light (c) Maize (d) Wheat
(c) refraction of light 220. Who was the first child born after operative procedure?
(d) total internal reflection of light (SSC CGL 2014)
208. The outer skin most of the crustaceans are made up of a (a) Caesar (b) Huxley
carbohydrate. This carbohydrate is (c) William (d) Pasteur
(SSC CGL 1st Sit. 2013) 221. Myrmecology is study of (SSC CGL 2014)
(a) cellulose (b) galactose (a) Insects (b) Ants
(c) chitin (d) starch (c) Crustaceans (d) Arthropods
w
w
w
.y
ou
64 General Science

rs
m
222. HIV often changes is shapes due to the presence of an 234. Which of the following Genetically Modified vegetable is

ah
bo
enzyme called (SSC CGL 2014) recently being made available in Indian market?

ob
(a) Reverse Transcriptase (SSC CGL 2014)

.w
(b) Enterokinase (a) Carrot (b) Radish

or
dp
(c) Nucleotidase (c) Brinjal (d) Potato

re
(d) Nucleoditase 235. Electric current is measured using which of the following

ss
instrument ? (SSC CGL 1st Sit. 2015)

.c
223. Fleming's right hand rule is used to find the direction of the

om
(SSC CGL 2014) (a) Voltmeter (b) Anemometer
(a) Alternate current (b) Direct current (c) Wattmeter (d) Ammeter
(c) Induced current (d) Actual current 236. Photoperiodisrn affects (SSC CGL 1st Sit. 2015)
(a) Flowering (b) Vegetative growth
224. The unit of electrical power is (SSC CGL 2014)
(c) Fruiting (d) All of these
(a) Volt (b) Watt
237. Match the following : (SSC CGL 1st Sit. 2015)
(c) Kilowatt hour (d) Ampere
I II
225. The resistance of the human body (dry condition) is of the
A. Ascorbic acid 1. Photosynthetic
order of (SSC CGL 2014)
pigment
(a) 101 Ohm (b) 102 Ohm B. Chlorophyll 2. Quencher
(c) 103 Ohm (d) 104 Ohm C. Carotenoid 3. Enzyme
226. Certain substances loose their electrical resistance D. Superoxide 4. Vitamin–C
completely at super low temperature. Such substances are dismutase
called (SSC CGL 2014) A B C D
(a) super conductors(b) semi conductors (a) 4 2 1 3
(c) dielectrics (d) perfect conductors (b) 2 4 1 3
227. The section of the CPU that selects, interprets and monitors (c) 4 1 3 2
the execution of program instructions is (SSC CGL 2014) (d) 4 1 2 3
(a) Memory (b) Register unit 238. Allantois of Embryo helps in (SSC CGL 1st Sit. 2015)
(c) Control unit (d) ALU (a) respiration (b) excretion
228. Brass contains (SSC CGL 2014) (c) protection (d) digestion
(a) Copper and Zinc 239. Which one of the following animals belongs to mollusca ?
(b) Copper and Tin (SSC CGL 1st Sit. 2015)
(c) Copper and Silver (a) Hare (b) Hydra
(d) Copper and Nickel (c) Hyla (d) Haliotis
229. Which is the purest commercial form of iron? 240. Outside the nucleus DNA is found in
(a) Pig iron (b) Steel (SSC CGL 2014) (SSC CGL 1st Sit. 2015)
(c) Stainless steel (d) Wrought iron (a) Mitochondria
230. In galvanization, iron is coated with (SSC CGL 2014) (b) Ribosome
(c) Endoplasmic reticulum
(a) Copper (b) Zinc
(d) Golgi bodies
(c) Tin (d) Nicked
241. Animal protein is called first class protein because it is
231. Which one of the following is also known as solution?
(SSC CGL 1st Sit. 2015)
(a) A compound (SSC CGL 2014)
(a) delicious in taste
(b) A homogeneous mixture
(b) cheaper in the market
(c) A heterogeneous mixture (c) rich in essential amino acids
(d) A suspension (d) easily digestible
232. The cells which are closely associated and interacting with 242. It is easy to burst a gas filled balloon with a needle than with
guard cells are (SSC CGL 2014) a nail. It is because (SSC CGL 1st Sit. 2015)
(a) Transfusion tissue (a) nail exerts more pressure than needle on the balloon
(b) Complementary cells (b) needle exerts more pressure than nail on the balloon
(c) Subsidiary cells (c) gas is reactive with the needle
(d) Hypodermal cells (d) nail is more longer than needle
233. Conversion of starch to sugar is essential for 243. The velocity of sound in moist air is more than in dry air
(SSC CGL 2014) because the moist air has (SSC CGL 1st Sit. 2015)
(a) Stomatal opening (a) less pressure than dry air
(b) Stomatal closing (b) more pressure than dry air
(c) Stomatal formation (c) more density than dry air
(d) Stomatal growth (d) less density than dry air
w
w
w
.y
ou
General Science 65

rs
m
244. X–rays can be used (SSC CGL 1st Sit. 2015) 256. Reverse transcription was discovered by :

ah
bo
(a) to detect heart diseases. (SSC CGL 1st Sit. 2015)

ob
(b) to detect defects in precious stones and diamonds. (a) Beadle and Tatum

.w
(c) to detect gold under the earth. (b) Watson and Crick

or
dp
(d) for cutting and welding of metals. (c) Temin and Baltimore

re
245. Ice is packed in saw dust because (SSC CGL 1st Sit. 2015) (d) Har Govind Khorana

ss
(a) saw dust is poor conductor of heat.

.c
257. Burns caused by steam are much more severe than those

om
(b) saw dust is a good conductor of heat.
caused by boiling water because: (SSC CGL 1st Sit. 2015)
(c) saw dust does not stick to the ice.
(d) saw dust will not get melted easily. (a) Steam pierces through the pores of body quickly
246. What happens when a drop of glycerol is added to crushed (b) Temperature of steam is higher
KMnO4 spread of a paper ? (SSC CGL 1st Sit. 2015) (c) Steam is gas and engulfs the body quickly
(a) There is a violent explosion (d) Steam has latent heat
(b) There is no reaction 258. Which among the following is the sweetest sugar?
(c) The paper ignites (SSC CGL 1st Sit. 2015)
(d) There is a crackling sound. (a) lactose (b) maltose
247. Most commonly used bleaching agent is (c) glucose (d) fructose
(SSC CGL 1st Sit. 2015) 259. Ultra purification of a metal is done by :
(a) Alcohol (b) Carbon dioxide (SSC CGL 1st Sit. 2015)
(c) Chlorine (d) Sodium chloride (a) smelting (b) leaching
248. The least penetrating power ray is
(c) zone melting (d) slagging
(SSC CGL 1st Sit. 2015)
260. Microbial degradation of nitrates into atmospheric nitrogen
(a) a–Ray (b) b–Ray
is known as : (SSC CGL 1st Sit. 2015)
(c) g–Ray (d) X–Ray
249. Hydrogen peroxide is an effective sterilizing agent. Which (a) Ammonification (b) Denitrification
one of the following product results when it readily loses (c) Putrefacation (d) Nitrifcation
active oxygen ? (SSC CGL 1st Sit. 2015) 261. The best milch breed in the world is :
(a) Water (b) Hydrogen (SSC CGL 1st Sit. 2015)
(c) Ozone (d) Nasant Hydrogen (a) Deoni (b) Holstein – Friesian
250. The maximum fixation of solar energy is done by (c) Sindhi (d) Chittagong
(SSC CGL 1st Sit. 2015) 262. Muddy water is treated with alum in purification process, it
(a) Bacteria (b) Fungi is termed as : (SSC CGL 1st Sit. 2015)
(c) Green plants (d) Protozoa (a) absorption (b) adsorption
251. The term 'brown air' is used for (SSC CGL 1st Sit. 2015) (c) coagulation (d) emulsification
(a) Photochemical smog 263. An enzyme produced by HIV that allows the integration of
(b) Sulfurous smog
HIV DNA into the host cell's DNA is:
(c) Industrial smog
(SSC CGL 1st Sit. 2015)
(d) Acid fumes
252. Peroxyacetyl nitrate is a (SSC CGL 1st Sit. 2015) (a) DNA gyrase (b) Ligase
(a) Plant hormone (c) Integrase (d) Helicase
(b) Vitamin 264. The two specific heats of gases are related by :
(c) Secondary pollutant (SSC CGL 1st Sit. 2015)
(d) Acidic dye (a) Cp / Cv = R (b) Cp – Cv = RJ
253. Which of the following particles has the dual nature of (c) Cp – Cv = R/J (d) Cp + Cv = RJ
particle–wave ? (SSC CGL 1st Sit. 2015) 265. The antibiotic penicillin is obtained from which of the
(a) Neutron (b) Electron following ? (SSC CGL 1st Sit. 2016)
(c) Meson (d) Proton (a) synthetic process (b) a bacterium
254. The metal ion present in vitamin B12 is : (c) fungus (d) virus infected cells
(SSC CGL 1st Sit. 2015) 266. Which of the following is indicated by the colour of a star ?
(a) nickel (b) cobalt (SSC CGL 1st Sit. 2016)
(c) iron (d) zinc
(a) weight (b) distance
255. Who of the following has given the term rhizosphere :
(c) temperature (d) size
(SSC CGL 1st Sit. 2015)
(a) Alexopolus 267. Atomic number of an atom gives the number of which of the
(b) Garret following ? (SSC CGL 1st Sit. 2016)
(c) Hiltner (a) electrons (b) protons
(d) None of the given options (c) neutrons (d) neutrons and protons
w
w
w
.y
ou
66 General Science

rs
m
268. DPT vaccine is categorized as which of the following ? 279. Which of the following acts as best adsorbent?

ah
(SSC CGL 1st Sit. 2016)

bo
(SSC CGL 1st Sit. 2016)

ob
(a) Anti viral vaccine (a) Charcoal

.w
(b) Anti protozoan vaccine (b) Activated Charcoal

or
dp
(c) Anti rickettsial vaccine (c) Activated Coconut Charcoal

re
(d) Carbon black
(d) A combined vaccine

ss
280. The maximum fixation of solar energy is done by

.c
269. Which is the highest quality of hard coal?

om
(SSC CGL 1st Sit. 2016)
(SSC CGL 1st Sit. 2016) (a) Bacteria (b) Protozoa
(a) Anthracite (b) Bituminous (c) Fungi (d) Green plants
(c) Lignite (d) Peat 281. 'Parsec' is the unit measurement of
270. For which one of the following, ‘Diodes’ are generally used (SSC CGL 1st Sit. 2016)
for? (SSC CGL 1st Sit. 2016) (a) Density of stars
(a) Rectification (b) Amplification (b) Astronomical distance
(c) Modulation (d) Filtration (c) Brightness of heavenly bodies
271. An anemometer measures which of the following ? (d) Orbital velocity of giant stars
(SSC CGL 1st Sit. 2016) 282. At room temperature, the metal that remains liquid is:
(a) Speed of light (SSC CGL 1st Sit. 2016)
(a) Mercury (b) Platinum
(b) Speed of wind
(c) Lead (d) Zinc
(c) Speed of water current
283. Which is known as carbolic acid? (SSC CGL 1st Sit. 2016)
(d) Speed of satellites (a) Phenol (b) Ethanol
272. Which cell disorder in our body is responsible for colour (c) Acetic acid (d) Oxalic acid
blindness? (SSC CGL 1st Sit. 2016) 284. Which one of the following substances is normally found in
(a) WBC (b) Cone cell urine? (SSC CGL 1st Sit. 2016)
(c) Rod Cell (d) Neuron (a) Blood proteins (b) Creatinine
273. Which one of the following forms the base of vegetable (c) Red blood cells (d) White blood cells
fibres? (SSC CGL 1st Sit. 2016) 285. The thymus gland produces a hormone called
(a) Cellulose (b) Proteins (SSC CGL 1st Sit. 2016)
(c) Fats (d) Oils (a) thyroxine (b) thymosin
274. A particle is thrown vertically upward. When it reaches the (c) thyronine (d) calcitonin
highest point, it has __________. 286. Blood group AB has (SSC CGL 1st Sit. 2016)
(a) No antigen
(SSC CGL 1st Sit. 2016)
(b) No antibody
(a) a downward acceleration
(c) Neither antigen nor antibody
(b) an upward acceleration (d) Both antigen and antibody
(c) a downward velocity 287. Which vitamins are those, if taken in excess can be dangerous
(d) a horizontal velocity as they are stored in the body? (SSC CGL 1st Sit. 2016)
275. During fermentation of sugar, the compound which is always (a) B Complex (b) E and C
formed is (SSC CGL 1st Sit. 2016) (c) B and C (d) A and D
(a) Methyl Alcohol (b) Ethyl Alcohol 288. Atomic explosion is triggered by (SSC CGL 1st Sit. 2016)
(c) Acetic Acid (d) Ethylene (a) thermo nuclear reaction
276. The quality or tone of a musical sound produced by a (b) chemical reaction
stringed instrument depends on (SSC CGL 1st Sit. 2016) (c) controlled chain reaction
(a) frequency of vibration (d) uncontrolled chain reaction
289. The phenomenon of change in direction of light when it
(b) length of the strings in the instrument
passes from one medium to another is called
(c) Amplitude of vibration
(SSC CGL 1st Sit. 2016)
(d) waveform of the sound (a) Propagation (b) Reflection
277. Removal of carbon particles from air involves the principle (c) Refraction (d) Dispersion
of (SSC CGL 1st Sit. 2016) 290. When ice cubes are made, the entropy of water
(a) Precipitation (b) Filteration (SSC CGL 1st Sit. 2016)
(c) Electrophoresis (d) Sedimentation (a) does not change
278. Which light is least effective in photosynthesis? (b) decreases
(SSC CGL 1st Sit. 2016) (c) increases
(a) Blue light (b) Green light (d) may either increase or decrease depending on the
(c) Red light (d) Sunlight process used
w
w
w
.y
ou
General Science 67

rs
m
291. Two vectors are said to be equal if 304. Which enzyme is obtained from Red gram ?

ah
(SSC CGL 1st Sit. 2016)

bo
(SSC CHSL 2012)

ob
(a) only their magnitudes are same (a) Zymase (b) Maltase

.w
(b) only their directions are same (c) Diastase (d) Urease

or
dp
(c) both magnitude and direction are same 305. A radioactive substance has a half-life of four months.

re
(d) magnitudes are same but directions are opposite Three-fourth of the substance will decay in:

ss
292. The chemical component that is invariably found in all (SSC CHSL 2012)

.c
om
viruses is (SSC CGL 1st Sit. 2016) (a) 8 months (b ) 12 months
(a) proteins (b) lipids (c) 4 months (d) 6 months
(c) DNA (d) RNA 306. Chiropterophily means: (SSC CHSL 2012)
293. Which is used as an Air pollution indicator? (a) production of flowers (b) pollination by wind
(SSC CGL 1st Sit. 2016) (c) pollination by bat (d) production of leaves
(a) Algae (b) Fungi 307. Which one among the following is a solid lubricant ?
(c) Bacteria (d) Lichens (SSC CHSL 2012)
294. Salination of soil is caused by (SSC CGL 1st Sit. 2016) (a) Germanium (b) Sulphur
(a) Pesticides (b) soil erosion (c) Graphite (d) Indium
(c) excess irrigation (d) crop rotation 308. Longest cell in human body is: (SSC CHSL 2012)
295. Which enzyme digests proteins in the stomach? (a) Blood cell (b) Bone cell
(SSC CGL 1st Sit. 2016) (c) Nerve cell (d) Muscle cell
(a) Trypsin (b) Pepsin 309. The acid rain destroys the vegetation because it contains:
(c) Salivary amylase (d) Pancreatic canal (SSC CHSL 2012)
296. Fermentation is a type of _____ _______ process. (a) Ozone (b) Carbon monoxide
(SSC CGL 1st Sit. 2016) (c) Sulphuric acid (d) Nitrates
(a) Aerobic Respiration 310. Activated sludge treatment is called: (SSC CHSL 2012)
(b) Anaerobic Respiration (a) Preliminary treatment (b) Biological treatment
(c) Exothermic Reaction (c) Pre treatment (d) Chemical treatment
(d) Transpiration 311. Caustic soda is : (SSC CHSL 2012)
297. Which part of the plant is used as 'saffron'? (a) deliquescent (b) oxidant
(SSC CHSL 2012) (c) reductant (d) efflorescent
(a) Petals (b) Stamens 312. Red light is used in traffic signal for stopping the traffic
(c) Style and Stigma (d) Sepals because: (SSC CHSL 2012)
298. Suspended colloidal particles in the water can be removed (a) eye is more sensitive to red light.
by the process of : (SSC CHSL 2012) (b) it is least scattered and hence can be easily noticed
(a) Filtration (b) Adsorption from long distance.
(c) Absorption (d) Coagulation (c) it is very pleasant to the eye.
299. Grave's disease is caused due to: (SSC CHSL 2012) (d) it is visible even to longsighted people.
(a) hyperactivity of thyroid 313. What is a neuron ? (SSC CHSL 2013)
(b) hypoactivity of thymus (a) Basic unit of nervous system
(c) hypoactivity of thyroid (b) Basic unit of energy
(d) hyperactivity of thymus (c) Particle released during radioactivity
300. A white solid ‘A’ on heating gives off a gas which turns lime (d) The anti-particle of neutron
water milky. The residue is yellow when hot but turns white 314. A cellulosic wall is found in the cells of (SSC CHSL 2013)
on cooling. The solid A is: (SSC CHSL 2012) (a) plants (b) animals
(a) Zinc Carbonate (b) Lead Sulphate (c) bacteria (d) fungi
(c) Lead Carbonate (d) Zinc Sulphate 315. The filtration unit of kidney is (SSC CHSL 2013)
301. Which of the following is the strongest coagulant ? (a) yellow fiber (b) axon
(SSC CHSL 2012) (c) nephron (d) neuron
(a) Zinc Chloride (b) Aluminium Chloride 316. The nutritive tissue in the seeds of higher plants is known
(c) Barium Chloride (d) Magnesium Sulphate as (SSC CHSL 2013)
302. The property which is seen in light wave but not in sound (a) nucellus (b) hypocotyl
wave is : (SSC CHSL 2012) (c) embryo (d) endosperm
(a) Diffraction (b) Refraction 317. Yeast is an important source of (SSC CHSL 2013)
(c) Polarization (d) Interference (a) protein (b) vitamin B
303. Iron and manganese are removed in water by the process (c) invertase (d) vitamin C
of : (SSC CHSL 2012) 318. Enzymes are (SSC CHSL 2013)
(a) Chlorination (b) Filtration (a) Lipids (b) Steroids
(c) Lime-soda treatment (d) Aeration (c) Carbohydrates (d) Proteins
w
w
w
.y
ou
68 General Science

rs
m
319. Heating of a ore below its melting point in the absence of air 329. Which of the following is a Biological method of soil

ah
bo
is known as (SSC CHSL 2013) conservation ? (SSC CHSL 2014)

ob
(a) Smelting (b) Refining (a) Contour farming (b) Contour terracing

.w
(c) Calcination (d) Roasting (c) Gully control (d) Basin listing

or
dp
320. The most electronegative element among the following is 330. Glucose is a type of (SSC CHSL 2014)

re
(SSC CHSL 2013) (a) Pentose sugar (b) Hexose sugar

ss
(c) Tetrose sugar (d) Diose sugar

.c
(a) Oxygen (b) Fluorine

om
(c) Sodium (d) Chlorine 331. Number of mitochondria in bacterial cell is
321. Mark the compound which possesses ionic, covalent and (SSC CHSL 2014)
co-ordinate bonds. (SSC CHSL 2013) (a) one (b) two
(a) H2O (b) NH4C1 (c) many (d) zero
(c) SO3 (d) S02 332. Rainbow is formed due to (SSC CHSL 2014)
322. The depletion of Ozone layer is mainly due to (a) refraction and dispersion
(b) scattering and refraction
(SSC CHSL 2013)
(c) diffraction and refraction
(a) Chlorofluorocarbons
(d) refraction and reflection
(b) Volcanic eruptions
333. Golden view of sea shell is due to (SSC CHSL 2014)
(c) Aviation fuels
(a) Diffraction (b) Dispersion
(d) Radioactive rays
(c) Polarization (d) Reflection
323. Match correctly the infectious agents given in List I with
334. An object covers distance which is directly proportional to
the diseases caused by them given in List II : the square of the time. Its acceleration is (SSC CHSL 2014)
(SSC CHSL 2013) (a) increasing (b) decreasing
List I List II (c) zero (d) constant
a. Bacterium 1. Kala-azar 335. If the horizontal range of a projectile is four times its maximum
b. Fungus 2. Tuberculosis height, the angle of projection is (SSC CHSL 2014)
c. Protozoan 3 Influenza (a) 30° (b) 45°
d. Virus 4. Ringworm
æ1ö -1 æ 1 ö
(a) a-4, b-2, c-3, d-1 (b) a-1, b-2, c-4, d-3 (c) sin -1 ç ÷ (d) tan ç ÷
(c) a-2, b-4, c-1, d-3 (d) a-3, b-1, c-2, d-4 è4ø è 4ø
324. Matter waves are (SSC CHSL 2013) 336. Which of the following metals has least melting point ?
(a) Longitudinal waves (SSC CHSL 2014)
(b) de Broglie waves (a) Gold (b) Silver
(c) Electromagnetic waves (c) Mercury (d) Copper
(d) Transverse waves 337. The gas produced in marshy places due to decomposition
325. When the milk is churned vigorously the cream from it is of vegetation is (SSC CHSL 2014)
separated out due to (SSC CHSL 2013) (a) Carbon monoxide (b) Carbon dioxide
(a) Centrifugal force (b) Centripetal force (c) Sulphur dioxide (d) Methane
338. In cactue, the spines are the modified (SSC CHSL 2014)
(c) Gravitational force (d) Frictional force
(a) stem (b) sripules
326. Gas thermometers are more sensitive than the liquid
(c) leaves (d) buds
thermometers because the gases (SSC CHSL 2013)
339. The smallest known prokaryotic organism is
(a) have high specific heat (SSC CHSL 2014)
(b) have large coefficient of expansion (a) Microcystis (b) Mycoplasma
(c) are lighter (c) Bacteria (d) Chlorella
(d) have low specific heat 340. According to your text, what can "be thought of as the
327. Which of the following is not caused by atmospheric genetic library that keep life going on Earth" ?
refraction of light ? (SSC CHSL 2013) (SSC CHSL 2014)
(a) Sun becoming visible two or three minutes before actual (a) A bio-engineering lab
sunrise (b) Human genes
(b) Sun appearing red at sunset (c) The human genome project
(c) Twinkling of stars at night (d) Biodiversity
(d) Sun appearing higher in the sky than it actually is 341. The boiling point of water decreases at higher altiudes is
328. From which part of Opium Plant we get morphine ? due to (SSC CHSL 2014)
(SSC CHSL 2014) (a) low tenperature
(a) Leaves (b) Stem (b) low atmospheric pressure
(c) high temperature
(c) Bark (d) Fruit coat
(d) high atmospheric pressure
w
w
w
.y
ou
General Science 69

rs
m
342. The chemical name of "Hypo"commonly used in 354. Maximum oxygen is available from : (SSC CHSL 2015)

ah
bo
photography is (SSC CHSL 2014) (a) Green forests (b) Deserts

ob
(a) Sodium thiosulphate (b) Silver nitrate (c) Grass lands (d) Phytoplanktons

.w
(c) Sodium nitrate (d) Silver iodide 355. In a reaction of the type A + B ® C + D one could ensure it

or
to be a first order reaction by : (SSC CHSL 2015)

dp
343. Soldering of two metals is possible because of the property

re
of : (SSC CHSL 2015) (a) Increasing the concentration of a reactant

ss
(a) Osmosis (b) Viscosity (b) Adding a catalyst

.c
om
(c) Surface tension (d) Cohesion (c) Increasing the temperature
344. Stalactites & Stalagmites form due to the precipitation of : (d) Increasing the concentration of a product
(SSC CHSL 2015) 356. The amount of matter in a ball of steel is its :
(a) CaCl2 (b) MgCO3 (SSC CHSL 2015)
(c) MgCl2 (d) CaCO3 (a) Mass (b) Density
345. Which of the following is a form of sexual reproduction : (c) Volume (d) Weight
357. Transpiration increases in : (SSC CHSL 2015)
(SSC CHSL 2015)
(a) Hot, damp and windy condition
(a) Fission (b) Fragmentation
(b) Cool, damp and windy condition
(c) Budding (d) Harmaphroditism
(c) Cool, dry and still condition
346. In which region of electromagnetic spectrum does the Lyman
(d) Hot, dry and windy condition
series of hydrogen atom lie ?
358. If xylem and phloem are arranged in the same radius, such a
(SSC CHSL 2015) vascular bundle is called : (SSC CHSL 2015)
(a) x–ray (b) Ultraviolet (a) bicollateral (b) concentric
(c) Visible (d) Infrared (c) radial (d) collateral
347. An electrochemcial cell which is used as a source of direct 359. Plank's constant has the dimensions of :
electrical current at constant voltage under standard (SSC CHSL 2015)
conditions is called a : (SSC CHSL 2015) (a) linear momentum (b) angular momentum
(a) Power transistor (c) force (d) energy
(b) Battery 360. The most abundant element by number in the living system
(c) Generator is : (SSC CHSL 2015)
(d) Uninterrupted power supply (UPS) (a) Hydrogen (b) Oxygen
348. A light wave is incident over a plane surface with velocity (c) Carbon (d) Nitrogen
X. After reflection the velocity becomes : 361. Which of the following phenomenon helps to conclude that
(SSC CHSL 2015) light is a transverse wave ? (SSC CHSL 2015)
(a) x (b) 2x (a) diffraction (b) polarisation
(c) refraction (d) interference
x x
(c) (d) 362. Among the following districts of Tamil Nadu, which district
4 2 is unfit for cultivation due to increased salinity :
349. The area reserved for the welfare of wild life is called (SSC CHSL 2015)
(SSC CHSL 2015) (a) Tiruchirapalli (b) Negapattinam
(a) Sanctuary (b) Botanical garden (c) Ramanathapuram (d) Coimbatore
(c) Forest (d) National pak 363. Natural system of classification was proposed by ____
350. The gas dissolved in water that makes it basic is ? botanists. (SSC CHSL 2015)
(SSC CHSL 2015) (a) German (b) Swedish
(a) ammonia (b) hydrogen (c) British (d) Indian
(c) sulphur dioxide (d) carbon dioxide 364. Minamata disease is caused by pollution of water by :
351. Yellow complexion, Medium stature, Oblique eye with an (SSC CHSL 2015)
epicanthic fold is the characteristic feature of : (a) tin (b) methyl isocyanate
(SSC CHSL 2015) (c) mercury (d) lead
(a) Australoids (b) Negroid 365. The "King of Metals" is : (SSC CHSL 2015)
(a) Silver (b) Iron
(c) Mengoloid (d) Cancosoid
(c) Aluminium (d) Gold
352. Chromosome designation of Turner syndrome is :
366. Amino acids are required for the synthesis of :
(SSC CHSL 2015)
(SSC CHSL 2015)
(a) 44A + XO (b) 44A + XXY (a) Lipids (b) Proteins
(c) 44A + XXX (d) 44A + XYY (c) Carbohydrates (d) Alkaloids
353. Distant objects are visible as a little out of focus in this 367. The most suitable unit for expressing nuclear radius is:
condition : (SSC CHSL 2015) (SSC CHSL 2015)
(a) hypermetropia (b) presbiopia (a) fermi (b) angstrom
(c) astigmatism (d) myopia (c) micron (d) nanometre
w
w
w
.y
ou
70 General Science

rs
m
368. Blowing Air with open pipe is an example of : 381. The type of mirror used in automobiles to see the traffic on

ah
bo
(SSC CHSL 2015) the rear side is (SSC Multitasking 2013)

ob
(a) Isochoric Process (b) Isobaric process (a) Convex (b) Concave

.w
(c) Adiabatic process (d) Isothermal process (c) Plano-Convex (d) Plane

or
dp
369. Christmas factor is involved in : (SSC CHSL 2015) 382. The hottest part of the gas flame is known as

re
(a) Excretion (b) Digestion (SSC Multitasking 2013)

ss
(c) Respiration (d) Blood Coagulation (a) dark zone (b) blue zone

.c
om
370. Who is the author of the book “Romancing with Life ”. (c) non-luminous zone (d) luminous zone
(SSC CHSL 2015) 383. Which of the following radiations has the least wavelength?
(a) Dev Anand (b) Shashi Tharoor (SSC Multitasking 2013)
(c) Bill Clinton (d) Kapil Dev (a) b-rays (b) X-rays
371. Which one out of the following helps in burning (c) a-rays (b) g-rays
(SSC Multitasking 2013) 384. The earth is a (SSC Multitasking 2013)
(a) Carbon dioxide (b) Oxygen (a) bad absorber and bad radiator of heat
(c) Carbon monoxide (b) Nitrogen (b) good reflector of heat
372. In organic compounds, nitrogen is estimated by (c) non-absorber of heat
(SSC Multitasking 2013) (d) good absorber and good radiator of heat
(a) Dumas’ method 385. BCG vaccination is given at the age of
(b) Carius method (SSC Multitasking 2013)
(c) Victor-Meyer’s method (a) Within 15 days (b) 2 – 3 years
(d) Liebig’s method (c) 10 years (d) Newborn
373. Master copy of genetic information is 386. Which of the following atmospheric gases constitute
(SSC Multitasking 2013) greenhouse gases? (SSC Multitasking 2013)
(a) DNA (b) Nucleus 1. Carbon dioxide 2. Nitrogen
(c) r-RNA (d) m-RNA 3. Nitrous oxide 4. Water vapour
374. Contraceptive pills in the market contain Select the correct answer using the codes given below.
(SSC Multitasking 2013) (a) 1, 2 and 4 (b) 1, 3 and 4
(c) 1 and 4 (d) 1 and 3
(a) Steroid-hormones
387. Hydrogen bomb is based on the principle of
(b) Inorganic compounds
(SSC Multitasking 2014)
(c) Herbicides
(a) Double decomposition
(d) Antibiotics
(b) Artificial radioactivity
375. Nematocysts are present in (SSC Multitasking 2013)
(c) Nuclear fission
(a) Sea anemone (b) Starfish
(d) Nuclear fusion
(c) Ascaris (d) Centipede
388. The commonly used safety fuse-wire is made of
376. Which of the following micro-organisms is used in milk
(SSC Multitasking 2014)
curdling? (SSC Multitasking 2013)
(a) an alloy of Nickel and Lead
(a) Lactobacillus (b) Acctobacter
(b) an alloy of Tin and Lead
(c) Leuconostoc (d) Bacillus
(c) an alloy of Tin and Nickel
377. Which of the following is present in Chlorophyll molecule? (d) an alloy of Lead and Iron
(SSC Multitasking 2013) 389. At what temperature is the density of water the maximum?
(a) K (b) Mn (SSC Multitasking 2014)
(c) Mn (d) Fe (a) 2°C (b) 4°C
378. When a body falls from an aeroplane, there is increase in its (c) 0°C (d) 1°C
(SSC Multitasking 2013) 390. The linear expansion of a solid rod is independent of its
(a) Potential energy (b) Kinetic energy (SSC Multitasking 2014)
(c) Mass (d) Acceleration (a) increase in temperature
379. What does ‘Ozone Layer’ absorb? (b) time of heat flow
(SSC Multitasking 2013) (c) initial length
(a) g-rays (b) Infrared rays (d) material
(c) Ultraviolet rays (d) X-rays 391. Cathode rays when obstructed by metal cause emission of
380. In a water lifting electric pump, we convert (SSC Multitasking 2014)
(SSC Multitasking 2013) (a) g- rays (b) X-rays
(a) Electrical energy into Kinetic energy (c) a-rays (d) b-rays
(b) Electrical energy into Potential energy 392. Who is the father of biology? (SSC Multitasking 2014)
(c) Kinetic energy into Electrical energy (a) Lamarck (b) Robert Hooke
(d) Kinetic energy into Potential energy (c) Aristotle (d) Pasteur
w
w
w
.y
ou
General Science 71

rs
m
393. The smallest unit of classification is 408. Anticoagulants are not present in

ah
bo
(SSC Multitasking 2014) (SSC Sub. Ins. 2012)

ob
(a) Species (b) Genus (a) Mosquito (b) Bed bug

.w
(c) Family (d) Order (c) Leech (d) Wasp

or
409. Vegetation is effective in absorbing (SSC Sub. Ins. 2012)

dp
394. Arenchyma is present in (SSC Multitasking 2014)

re
(a) Banana stem (b) Palm stem (a) Pollutant gases (b) Polluted water

ss
(c) Aquatic plants (d) Xerophytic plants (c) High frequency sound (d) Pollutant metals

.c
om
395. The deficiency of vitamin A causes 410. When a person cries, there is a watery discharge from the
(SSC Multitasking 2014) nose due to activation of (SSC Sub. Ins. 2012)
(a) Scurvy (b) Night blindness (a) Salivary gland (b) Lachrymal gland
(c) Beri-Beri (d) Dermatitis (c) Thyroid gland (d) Endocrine gland
396. Clove is a (SSC Multitasking 2014) 411. The fundamental role of root hairs in plants is
(a) Dried flower bud (b) Flower (SSC Sub. Ins. 2012)
(c) Fruit (d) Seed (a) to protect the young root from damage by coarse soild
397. On heating, Gypsum loses certain percentage of its water particles
content and becomes (SSC Multitasking 2014) (b) to protect the root from soil microbes
(a) Chalk (b) Calcium sulphate (c) to absorb water and mineral salts from the soil
(c) Plaster of Paris (d) a pearl (d) to bind soil particles to the root for firm fixation of the
398. The name of the scientist who discovered neutron is plant
(SSC Multitasking 2014) 412. Catch crops are (SSC Sub. Ins. 2012)
(a) Fermi (b) Rutherford (a) crops palnted to attract certain insect pests to be
(c) Chadwick (d) Bohr destroyed
399. The bubbles in Champagne and Soda are (b) crops planted to attract certain useful insects to be
(SSC Multitasking 2014) used for biological control of pests
(a) Nitrogen (b) Oxygen (c) crops to be cut and fed green to the cattle
(c) Carbon dioxide (d) Hydrogen (d) substitute crops planted after the regular crop has failed.
400. Gobar gas contains mainly (SSC Multitasking 2014) 413. The pigment that protects plants form UV damage is
(a) Butane (b) Carbon monoxide (SSC Sub. Ins. 2012)
(c) Methane (d) Carbon dioxide (a) Chlorophyll (b) Xanthophyll
401. "Carbon Credit" is a term associated with the (c) Phycocyanin (d) Carotenoids
(SSC Multitasking 2014) 414. A bioenergy source obtained by fermentation to supplement
(a) Global deforestation fossil fuel petrol is (SSC Sub. Ins. 2012)
(b) Offshore banking (a) Kerosene (b) Ethanol
(c) Protection of environment (c) Diesel (d) Methane
(d) Deforestation in India 415. The substance that causes the worst air pollution is
402. The green colour of plant leaves is due to (SSC Sub. Ins. 2012)
(SSC Multitasking 2014) (a) Smoke (b) Sulphur dioxide
(a) Protein (b) Chlorophyll (c) Carbon dioxide (d) Carbon monoxide
(c) Cellulose (d) Starch 416. A liquid is said to boil when its (SSC Sub. Ins. 2012)
403. Temperature is measured by the instrument called (a) vapour pressure equals the surrounding pressure
(SSC Multitasking 2014) (b) vapour pressure vanishes to zero
(a) Voltmeter (b) Calorimeter (c) vapour pressure is greater than the surrounding
(c) Thermometer (d) Ammeter pressure
404. The pH of pure water is (SSC Multitasking 2014) (d) vapour pressure is less than the surrounding pressure
(a) Seven (b) Foruteen 417. Which is not correct regarding covalent compounds?
(c) Zero (d) One (SSC Sub. Ins. 2012)
405. The process of separation of pure water from impurities is (a) The reaction is slow
called (SSC Multitasking 2014) (b) The reaction is fast
(a) Fractional crystallisation (b) Decantation (c) Compounds are usually liquids and gases
(c) Distillation (d) Sublimation (d) Boiling points and melting points are low
406. H2SO4 cannot be used as (SSC Multitasking 2014) 418. Choose the correct statement
(a) Disinfectant (b) Food preservative (SSC Sub. Ins. 2012)
(c) Drying agent (d) Dehydrating agent (a) The components of a mixture cannot be separated
407. The red colour of ripe tomatoes is due to the presence of (b) The properties of a mixture are the same as those of its
(SSC Sub. Ins. 2012) components
(a) Hormones (b) Vitamins (c) Mixtures are homogeneous
(c) Chlorophyll (d) Carotenoids (d) In a mixture the components are present in a fixed ratio
w
w
w
.y
ou
72 General Science

rs
m
419. PVC is obtained by the polymerisation of 430. Which of the following elements is not radio-active?

ah
bo
(SSC Sub. Ins. 2012) (SSC Sub. Ins. 2013)

ob
(a) Propene (b) Vinyl chloride (a) Radium (b) Plutonium

.w
(c) Styrene (d) Acetylene (c) Zirconium (d) Uranium

or
431. Wilting of plants occurs due to excessive :

dp
420. To eliminate the glare of headlights in motor cars,

re
(SSC Sub. Ins. 2012) (SSC Sub. Ins. 2013)

ss
(a) polaroids are used (b) glass prisms are used (a) Respiration (b) Guttation

.c
om
(c) thin films are used (d) filters are used (c) Absorption (d) Transpiration
421. The substances which have infinite electrical resistance are 432. Gypsum is used for improvement of:
(SSC Sub. Ins. 2013)
called (SSC Sub. Ins. 2012)
(a) Alkaline soils (b) Saline soils
(a) insulators (b) condensers
(c) Podsols (d) Acidic soils
(c) conductors (d) resistors
433. Indicate the correct arrangement for electromagnetic
422. Stainless steel is an alloy of (SSC Sub. Ins. 2012)
radiation in order of their increasing wavelength.
(a) iron, chromium and nickel (SSC Sub. Ins. 2013)
(b) iron, chromium and carbon (a) Microwave. infrared, visible. X-rays
(c) iron, carbon and zinc (b) X -rays. visible, infrared, microwave
(d) iron, zinc and manganese (c) Visible. infrared. microwave. X-rays
423. Enriched uranium used in a nuclear reactor is (d) X - rays, infrared, visible, microwave.
(SSC Sub. Ins. 2012) 434. The rapidly growing mass of phytoplankton covering the
(a) uranium free of all impurities surface water of a lake or pond is known as:
(b) uranium treated with radiation (SSC Sub. Ins. 2013)
(c) uranium alloyed with aluminium (a) Eutrophication (b) Water bloom
(d) uranium with a high percentage of a particular isotope (c) Water pollution (d) Water hyacinth
424. In a refrigerator, cooling is produced by 435. Wings of birds are: (SSC Sub. Ins. 2013)
(SSC Sub. Ins. 2012) (a) Modified hind limbs
(a) the ice which deposits in the freezer (b) New structure
(b) the sudden expansion of a compressed gas (c) Integumentary outgrowth
(c) the evaporation of a volatile liquid (d) Modified fore limbs
(d) None of these 436. Spontaneous change is one in there is:
(SSC Sub. Ins. 2013)
425. Which one of the following statements about PYROLYSIS.
(a) A lowering of entropy
which is a process for solid waste treatment is incorrect?
(b) A lowering of free energy
(SSC Sub. Ins. 2013)
(c) Increase in free energy
(a) It converts the waste into solid, liquid and gas of which
(d) An increase in Internal energy
the resultant liquid and gas can be used to produce 437. Sandstone is metamorphosed to : (SSC Sub. Ins. 2013)
energy. (a) Shale (b) Slate
(b) The process occurs at a temperature above 430°C at (c) Quartzite (d) Marble
atmospheric pressure. 438. Instrument used to study the behaviour of a vibrating string
(c) The process occurs under high pressure at temperature is : (SSC Sub. Ins. 2013)
above 430°C. (a) Barometer (b) Hydrometer
(d) It is a thermochemical decomposition of organic waste (c) Hygrometer (d) Sonometer
426. Which of the following supports particle nature of photons? 439. The casual organism of Polio is : (SSC Sub. Ins. 2013)
(SSC Sub. Ins. 2013) (a) A fungi (b) A virus
(a) Diffraction (b) Polarization (c) A worm (d) A bacteria
(c) Photoelectric effect (d) Interference 440. Panda belongs to the same family as that
427. The heaviest naturally occurring element is: (SSC Sub. Ins. 2013)
(SSC Sub. Ins. 2013) (a) Kangaroo (b) Porcupine
(a) Mercury (b) Polonium (c) Whale (d) Bear
(c) Thorium (d) Uranium 441. The pancreas secretes (SSC Sub. Ins. 2014)
428. Haptens are: (SSC Sub. Ins. 2013) (a) Insulin (b) Bile juice
(a) Pseudoantigens (b) Incomplete antigens (c) Peptic juice (d) None of these
442. When we touch leaves of "Touch me not plant", they close,
(c) Antibodies (d) Isoantigens
these movements are called (SSC Sub. Ins. 2014)
429. Sulphur dioxide bleaches colouring matter by :
(a) photonastic movements
(SSC Sub. Ins. 2013)
(b) nyctinastic movements
(a) Reduction (b) Dehydration
(c) seismonastic movements
(c) Decomposition (d) Oxidation
(d) chemonastic movements
w
w
w
.y
ou
General Science 73

rs
m
443. The concept of tissue culture was introduced by 455. The process of imbibition involves (SSC Sub. Ins. 2014)

ah
bo
(SSC Sub. Ins. 2014) (a) Diffusion (b) Capillary action

ob
(a) Halfmeister (b) Hanstein (c) Absorption (d) Both (A) & (B)

.w
(c) Haberlandt (d) Hanning 456. A cell increases in volume when it is placed in

or
dp
444. Beak is formed by (SSC Sub. Ins. 2014) (SSC Sub. Ins. 2014)

re
(a) cheeks (b) jaws (a) Hypertonic solution (b) Hypotonic solution

ss
(c) teeth (d) none

.c
(c) Isotonic solution (d) None of these

om
445. Pinna (external ear) is present in (SSC Sub. Ins. 2014) 457. A reversible and an irreversible engine are working between
(a) Amphibian (b) Fish the same limits of temperature. The efficiency of.
(c) Mammal (d) Reptile (SSC Sub. Ins. 2015)
446. Purity of a metal can be determined with the help of (a) The reversible engine is greater than the irreversible
(a) Pascal's law (SSC Sub. Ins. 2014) engine.
(b) Boyle's law (b) Each engine is 100%.
(c) Archimedes principle (c) The two engines are equal
(d) Conservation of mass principle (d) The irreversible engine is greater than the reversible
447. If both the mass and the velocity of a body is increased to engine.
twice of their magnitude, the kinetic energy will increase by 458. Which of the following is used in the treatment of cancer?
(SSC Sub. Ins. 2014)
(SSC Sub. Ins. 2015)
(a) 2 times (b) 4 times
(a) Electrotherapy (b) Psychotherapy
(c) 8 times (d) 16 times
(c) Chemotherapy (d) Physiotherapy
448. Two bodies kept at a certain distance feel a gravitational
459. The most abundant element is : (SSC Sub. Ins. 2015)
force F to each other. If the distance between them is made
(a) Silicon (b) Calcium
double the former distance, the force will be
(c) Nitrogen (d) Oxygen
(SSC Sub. Ins. 2014)
460. An electron microscope gives higher magnifications than
1
(a) 2F (b) F an optical microscope because : (SSC Sub. Ins. 2015)
2
(a) The electrons have more energy than the light
1 particles.
(c) 4F (d) F
4 (b) The electron microscope uses more powerful lenses.
449. Stationary wave is formed by (SSC Sub. Ins. 2014) (c) The wavelength of electrons is smaller as compared
(a) a transverse wave superposing a longitudinal wave to the wavelength of visible light.
(b) two waves of the same speed superposing (d) The velocity of electrons is smaller than that of light.
(c) two waves of same frequency travelling in the same 461. What does the word ‘amphibian’ mean ?
direction (SSC Sub. Ins. 2015)
(d) two waves of same frequency travelling in the (a) Two lives (b) Four lives
opposite direction (c) Three lives (d) One life
450. In an oxygen molecule, two atoms are united by 462. Nitrogen in water is commonly found in the form of:
(SSC Sub. Ins. 2014) (SSC Sub. Ins. 2015)
(a) the bond (b) two bonds (a) Nitric oxide (b) Nitrous oxide
(c) three bonds (d) four bonds (c) Nitrate (d) Nitrite
451. The inert gas which is substituted for nitrogen in the air 463. Immunization technique was developed by:
used by deep sea divers for breathing is (SSC Sub. Ins. 2015)
(SSC Sub. Ins. 2014) (a) Louis Pasteur (b) Robert Koch
(a) Neon (b) Krypton (c) Joseph Lister (d) Edward jenner
(c) Argon (d) Helium 464. The hydrophilic nature of DNA is due to the presence of.
452. How many neutrons are there in 92U238 atom ? (SSC Sub. Ins. 2015)
(SSC Sub. Ins. 2014) (a) a number of hydrogen bonds
(b) phosphate group
(a) 92 (b) 238
(c) deoxyribose sugar
(c) 146 (d) 330
(d) thymine base
453. Root pressure is measured by (SSC Sub. Ins. 2014)
465. The sense of balance is achieved by:
(a) Barometer (b) Atmometer
(SSC Sub. Ins. 2015)
(c) Manometer (d) Auxanometer
(a) Cerebellum equilibrium
454. Cell becomes turgid because of (SSC Sub. Ins. 2014)
(b) Thalamus equilibrium
(a) Plasmolysis (b) Exosmosis
(c) Cerebrum equilibrium
(c) Endosmosis (d) Diffusion
(d) Spinal cord equilibrium
w
w
w
.y
ou
74 General Science

rs
m
466. Aluminium salt commonly used to stop bleeding is: 475. Antigen presenting cells are specialized cells present in all

ah
bo
(SSC Sub. Ins. 2015) of the following, except (SSC Sub. Ins. 2016)

ob
(a) Aluminium chloride (b) Aluminium nitrate (a) Skin (b) Lymph node

.w
(c) Aluminium sulphate (d) Potash alum (c) Kidney (d) Spleen

or
dp
467. Total internal reflection cannot take place when light goes 476. Which of the following determines whether a group of

re
from: (SSC Sub. Ins. 2015) organisms that is from the same genus and species arise

ss
(a) water to glass (b) water to air

.c
from a common source or from different sources?

om
(c) glass to air (d) glass to water (SSC Sub. Ins. 2016)
468. Interferons are synthesized in response to: (a) Biotyping
(SSC Sub. Ins. 2015) (b) DNA hybridization / DNA
(a) mycoplasma (b) fungi (c) Serotyping
(c) virus (d) bacteria (d) Phage typing
469. The first law of thermodynamics is simply the case of: 477. Starch is insoluble in water but still it is stored in large
(SSC Sub. Ins. 2015) quantity in potato because (SSC Sub. Ins. 2016)
(a) Charle’s law. (a) soil microorganisms deposit it in the tuber.
(b) the law of conservation of energy. (b) it is synthesized in potato root.
(c) the law of heat exchange.
(c) it is useful for human.
(d) Newton’s law of cooling.
(d) it is translocated in the form of sugar from leaves.
470. Red rot of sugarcane is caused by: (SSC Sub. Ins. 2015)
478. Viruses that infect bacteria are called (SSC Sub. Ins. 2016)
(a) Colletotriehum falcatum
(a) Basal body (b) Basidiospores
(b) Cercospora personata
(c) Bacteriophages (d) Basophils
(c) Alternaria alternata
479. A dispersion indicates (SSC Sub. Ins. 2016)
(d) Phylophthora Infestans
(a) the value of standard deviation.
471. A clone is a group of individuals obtained through :
(b) spread of data around central measure.
(SSC Sub. Ins. 2015)
(a) self pollination (b) micropropagation (c) the value of mean
(c) hybridisation (d) cross pollination (d) the value of mode
472. Chemical name of Gammaxane is: (SSC Sub. Ins. 2015) 480. The principle involved in the absorption of water by soil is
(a) Aniline (b) Toluene (SSC Sub. Ins. 2016)
(c) Benzene hexachloride (d) Chloro benzene (a) Suction action (b) Condensation
473. The fleshy thalamus is edible in: (SSC Sub. Ins. 2015) (c) Capillary action (d) Principal of absorption
(a) Mango (b) Orange 481. Which of the following is present in maximum amount in
(c) Tomato (d) Apple acid rain? (SSC Sub. Ins. 2016)
474. A new molecule Heat Shock Protein 90 (HSP90) was (a) HNO3 (b) H2SO4
discovered in 2014 by the Didier Picard. The new discovery (c) H2CO3 (d) HCL
could help in effective treatment of ____ 482. The lethal dose required to kill 50% of the lab animals tested
(SSC Sub. Ins. 2016) under standard is referred as (SSC Sub. Ins. 2016)
(a) TB (b) AIDS (a) MLD (b) ID50
(c) Malaria (d) None of these (c) LD50 (d) ID
w
w
w
.y
ou
General Science 75

rs
m
ah
HINTS & SOLUTIONS

bo
ob
.w
or
dp
re
1. (b) 25. (a) 26. (a) 27. (c) 28. (d) 29. (d)

ss
2. (a) Vitamin B-12, also called cobalamin, is a water-soluble 30. (b) 31. (b) 32. (c) 33. (d) 34. (b)

.c
om
vitamin that has a key role in the normal functioning 35. (c) 36. (d) 37. (c) 38. (d) 39. (c)
of the brain and nervous system, and the formation of 40. (d) 41. (d) 42. (a) 43. (a) 44. (c)
red blood cells. 45. (c) 46. (c) 47. (c) 48. (a) 49. (d)
3. (d) 4. (b) 5. (a) 6. (b) 7. (a) 50. (a) 51. (b) 52. (c) 53. (d) 54. (a)
8. (b) Acetic acid, also known as ethanoic acid, is an organic 55. (c) 56. (d) 57. (b) 58. (a) 59. (c)
chemical compound best recognized for giving vinegar 60. (d)
its sour taste and pungent smell. It is one of the 61. (c) This process usually occurs in the upper third of the
simplest carboxylic acids and has the chemical formula fallopian tube of the woman.
CH3COOH. 62. (d) Cirrhosis is a condition in which the liver does not
9. (b) function properly due to long-term damage. Cirrhosis
10. (a) Boiling point of heavy water is lower than that or is most commonly caused by alcohol, hepatitis B,
ordinary water hepatitis C, and non-alcoholic fatty liver disease.
11. (a) Typically, more than two or three drinks per day over a
12. (a) Arboreal animals are creature who spend the majority number of years is required for alcoholic cirrhosis to
of their lives in trees. They eat, sleep and play in the occur.
63. (c) The food webs we see are grazing food chains since at
tree canopy. There are thousands of species that live
their base are producers which the herbivores then
in trees, including monkeys, koalas, possums, sloths,
graze on.
various rodents, parrots, chameleons, geckos, tree
64. (c) A single mole is set to the number of particles found in
snakes and a variety of insects.
12.000 grams of carbon-12. A mole of water has 6.022 x
13. (a)
1023 water molecules. One mole of water weighs 18.0152
14. (d) Advection is the transfer of heat or matter by the flow
grams.
of a fluid, especially horizontally in the atmosphere or
65. (a) Carrot is rich in Vitamin A and it improves eyesight.
the sea. 66. (d) The reason for this is the hydrogen bonding between
15. (d) neighboring water molecules. Because hydrogen
16. (d) Bats are good at flying at night because they use bonding is a relatively strong intermolecular force, high
sound rather than sight to navigate. Bats send pulses heat energy is required to break up the force.
of sound through their mouths or noses, and these 67. (a) Valence electrons are important in determining how an
pulses echo back outlining the objects in the bats elements reacts chemically with other elements. Since
flight path. The ears of a bat are large and oddly the valence electrons are the electrons in the highest
constructed but they help it to determine where the energy level, they are the most exposed of all the
echoes are coming from. electrons, so they are the electrons that get most
17. (c) No change will happen. involved in chemical reactions.
18. (b) Cartography, the art and science of graphically 68. (c) The lumen is the SI derived unit of luminous flux, a
representing a geographical: area, usually on a flat measure of the total "amount" of visible light emitted
surface such as a map or chart. It may involve the by a source.
superimposition of political, cultural, or other non 69. (d) A dissipative force counteracts motion. Its direction is
geographical divisions onto the representation of a opposite to the direction of the velocity vector.
geographical area. Dynamic friction is a dissipative (non-conservative)
19. (c) force : it dissipates energy (mainly through heat and
20. (d) An antiknock agent is a gasoline additive used to sound), and energy lost by moving in one direction.
reduce engine knocking and increase the fuel's octane 70. (c) Poisson strain is defined as the negative ratio of the
rating by raising the temperature and pressure at which strain in the traverse direction (caused by the
auto ignition occurs. contraction of the bar's diameter) to the strain in the
The typical antiknock agents in use are: Tetraelhyllead longitudinal direction. As the length increases and the
(Still in use as a high octane cross sectional area decreases, the electrical resistance
21. (b) 22. (a) 23. (a) of the wire also rises.
24. (d) A positron is a particle of matter with the same mass as 71. (b) Consequent Poles are magnetic poles that exist where
an electron but an opposite charge. It is a form of the specimen has been successively magnetized in
antimatter because, when a positron encounters an different sections to create more than two poles ; e.g.,
electron, the two completely annihilate to yield energy. two north poles with one south pole between them.
w
w
w
.y
ou
76 General Science

rs
m
72. (b) Fructose, or fruit sugar, a is one of the three dietary 86. (b) In hydroelectric power plants the potential energy of

ah
bo
monosaccharide, along with glucose and galactose, water is utilized to produce electricity. The height of

ob
which is absorbed directly into the bloodstream during water in the reservoir decides how much potential

.w
digestion. energy water possesses.

or
dp
73. (c) Methylated spirit contains mostly ethanol. Ethanol is 87. (c) In people with emphysema, the lung tissue involved

re
slightly soluble in water (mixing 50 ml of ethanol and in exchange of gases (oxygen and carbon dioxide) is

ss
water will make 80 – 90 ml of mixture, not 100ml). impaired or destroyed. Emphysema is included in a

.c
om
74. (c) A sporozoite is the cell form that infects new hosts. In group of diseases called chronic obstructive pulmonary
plasmodium, for instance, the sporozoites are cells that disease of COPD.
develop in the mosquito’s salivary glands, leave the 88. (c) Potassium Nitrate occurs as a mineral niter and is a
mosquito during a blood meal, and enter liver cells natural solid source of nitrogen. Potassium nitrate is
(hepatocytes) where they multiply. one of several nitrogen-containing compounds
75. (c) Ex-situ conservation is the process of protecting an collectively referred to as saltpeter.
endangered species of plant or animal outside of its 89. (a) 90. (b) 91. (b) 92. (a) 93. (b)
natural habitat. Zoos and botanical gardens are the 94. (c) 95. (d) 96. (a) 97. (b) 98. (b)
most conventional methods of ex-situ conservation. 99. (c) 100. (d) 101. (d) 102. (a) 103. (a)
Endangered plants may also be preserved in part 104. (b) 105. (d)
through seed banks or germplasm banks. 106. (c) Two pollutants emitted by motor vehicles react to form
76. (b) Diatom ooze (formed from microscopic unicellular algae
ground-level ozone or smog which can cause
having cell walls consisting of or resembling silica) is
respiratory problems and reduce visibility.
the most widespread deposit in the high southern
107. (b) 108. (b)
latitudes.
109. (c) Cardiac muscle is an involuntary striated muscle tissue
77. (b) Photosynthetic Chromatophores vesicles found in
found only in the organ heart. Involuntary muscles are
some purple bacteria constitute one of the simplest
smooth muscles that are not directly controllable at
light-harvesting systems in nature.
will. For example You don't have to remind yourself to
78. (a) Convex mirrors reflect light outwards; therefore they
are not used to focus light. So, the convex mirror has make your heart beat, so it is involuntary. Voluntary
a wide field of view and hence is used as rear view muscles are controllable like those found in your arms,
mirror as it gives a clear diminished and an erect image legs, hands, etc.
of the traffic that is behind. 110. (b)
79. (b) The principal components of pyroligneous acid are 111. (c) Ringworm is common disease, especially among
acetic acid, acetone and methanol. It was once used as children. It is caused by a fungus, not a worm like the
a commercial source for acetic acid. name suggests. It is a common and highly infectious
80. (b) UCIL produced batteries, carbon products, welding skin infection that causes a ring-like red rash on the
equipment, plastics, industrial chemicals, pesticides, skin.
and marine products. 112. (a) 113. (c) 114. (d)
81. (a) Most drying oils owe their drying properties to the 115. (c) Centrifugal force is an example of a pseudo-force, that
presence of a large percentage of linolenic acid (which is, an apparent force to someone whose frame of
derives its name from "linseed"), which is highly reference is not at rest or moving with a constant
unsaturated. velocity. In the case of centrifugal force, the frame of
82. (a) Carotenoids are tetraterpenoid organic pigments that reference is rotating.
are naturally occurring in the chloroplasts and 116. (d) 117. (c)
chromoplasts of plants and some other photosynthetic 118. (c) An antacid is a substance which neutralizes stomach
organism like algae, some bacteria, and some types of acidity.
fungus. 119. (a) Annealing is the process by which both metal and
83. (a) The variations in sunlight affect different latitudes glass are treated with heat in order to change their
differently, as we might expect from the different angles properties.
at which sunlight falls on the earth's surface at different 120. (b) 121. (d) 122. (d) 123. (d)
latitudes. 124. (c) This is because the scattering in red light is less than
84. (a) Polar bears, which pile on fat to survive hibernation that of yellow colour. The longest visible wavelength
and yet do not become diabetic, hold clues for treating is red and the shortest is violet. The wavelength of red
Type II diabetes, a disease associated with obesity light is more than yellow light.
that afflicts more than 190 million people worlwide, 125. (a) Transboundary pollution is the pollution that
reaching epidemic proportions in many countries. originates in one country but is able to cause damage
85. (d) All electromagnetic waves regardless of their in another country's environment, by crossing borders
wavelengths, including all colors of light, have the through pathways like water or air. Acid rain is a classic
same identical speed in a vacuum. example of a transboundary pollution because it can
w
w
w
.y
ou
General Science 77

rs
m
be blown anywhere by the wind. Sulphur dioxide and body temperature is around 98.6 F (37 C). Hypothermia

ah
bo
nitrous oxides are the two main chemicals that react occurs as your body temperature passes below 95 F

ob
with water to make acid rain. The chemicals are (35 C).

.w
commonly released from power stations, factories and 145. (c) Semiconductors are insulators at low temperatures and

or
dp
transport. reasonably good conductors at higher temperatures.

re
126. (a) The Nicobar pigeon is a pigeon found on small islands As temperature increases, the semi–conductor material

ss
and in coastal regions from the Nicobar Islands, India, becomes a better and better conductor.

.c
om
east through the Malay Archipelago, to the Solomons 146. (c) Mass of proton is 1.672621777 (74) × 10–27 kg. An
and Palau. It is the only living member of the genus electron has a mass (9.1093829140) × 10–31 kg) that is
Caloenas and the closest living relative of the extinct approximately 1/1836 that of the proton. The mass of
dodo.. neutron is slightly larger than that of a proton. The mass
127. (a) A limnic eruption, also referred to as a lake overturn, of the hydrogen nucleus is 1.7 × 10–27 kg. The heaviest
is a rare type of natural disaster in which dissolved of these particles is the neutron.
carbon dioxide (CO2) suddenly erupts from deep lake 147. (a) A television channel is a physical or virtual channel
waters, forming a gas cloud that can suffocate wildlife, over which a television station or television network is
livestock and humans. distributed. Channel numbers represent actual
128. (c) The inventor of blood group is Karl Landsteiner, was frequencies used to broadcast the television signal.
born in Vienna, June 14, 1868. For example, in North America. "Channel 2" refers to
He is a scientist in the field of bio-treatment. He was the broadcast or cable band of 54 to 60 MHz, with
awarded Nobel Prize for Medicine and Fisiologi field in carrier frequencies of 55.25 MHz for NTSC analog video
1930. This is because the classification of all types of (VSB) and 59.75 MHz for analog audio (FM), or 55.31
blood into four namely; A, B, AB, and O in the year MHz for digital ATSC (8VSB).
1909. 148. (b) Water never has an absolute density because its
129. (a) 130. (b)
density varies with temperature. Water has its maximum
131. (b) DPT refers to a class of combination vaccines against
density of lg/cm3 at 4 degrees Celsius. When the
three infectious diseases in humans: diphtheria,
temperature changes from either greater or less than 4
pertussis (whooping cough), and tetanus. The vaccine
degrees, the density will become less than 1 g/cm3.
components include diphtheria and tetanus toxoidsand
149. (c) Photosynthesis is the process by which light energy
kills whole cells of the organism that cause pertussis
is converted into chemical energy by organisms.
(wP).
Carbon dioxide and water are the raw materials of this
132. (a) 133. (c) 134. (b) 135. (b) 136. (d)
process. The light energy comes from the sun and its
137. (b) The oxalic acid is an ideal chemical for cleaning
purposes. Its bleach-like qualities make it perfect for end products are oxygen and glucose.
sterilizing household items. It is also efficient in 150. (b) An emulsion is a mixture of two or more liquids that are
removing rust on various different surfaces. Stains on normally immiscible (nonmixable or unblendable).
counters, bathtubs and kitchen sinks can be removed Emulsions are part of a more general class of two–
through careful application of this chemical. phase systems of matter called colloids. In an emulsion,
138. (b) one liquid (the dispersed phase) is dispersed in the
139. (d) Rectified spirit, also known as neutral spirits, other (the continuous.
rectified alcohol, or ethyl alcohol of agricultural 151. (b) The active ingredient in Dettol that confers its antiseptic
origin is highly concentrated ethanol which has been property is chloroxylenol (C8H9CIO), an aromatic
purified by means of repeated distillation, a process chemical compound. Chloroxylenol comprises 4.8% of
that is called rectification. It is 95.5% alcohol and 4.5% Dettol's total mixture, with the rest composed of pine
water. It is treated with CaO to form lime of alcohol, oil, isopropanol, castor oil soap caramel and water.
which undergoes fractional distillation to give ethanol. 152. (c) Researchers in Mangolia and China completed studies
140. (c) to unravel the genomic Peculiaritces behind the
141. (d) The Intergovernmental Panel on Climate Change physiological tricks that camels use to survive in the
(IPCC) is a scientific and intergovernmental body harshest of conditions. They described the genomes
under the auspices of the United Nations, set up at the of wild and domesticated Bactrian camels.
request of member governments, dedicated to the task 153. (d) Fine particles known as PM10 and PM25can penetrate
of providing the world with an objective, scientific view deep into the lungs, creating health problems. People
of climate change and its political and economic impacts. with heart or lung diseases, older adults, and children
142. (c) 143. (b) are most likely to have problems because of contact
144. (c) Hypothermia is a medical emergency that occurs when with particle pollution. Short–term exposure to PM
your body loses heat faster than it can produce heat, among pregnant women has been associated with
causing a dangerously low body temperature. Normal prematurity and growth retardation.
w
w
w
.y
ou
78 General Science

rs
m
154. (d) PNG is a mixture consisting mainly methane CH4 with a among other functions, it influences radio propagation

ah
bo
small percentage of other higher hydrocarbons. The to distant places on the Earth.

ob
ratio of carbon to hydrogen is least in methane and 165. (a) Coral reefs are underwater structures made from

.w
hence it burns almost completely making it the cleanest calcium carbonate secreted by corals. Coral reefs are

or
dp
fuel. Domestic PNG customers, also known as colonies of tiny animals found in marine waters that

re
Residential customers, use gas for cooking purpose contain few nutrients.

ss
and also for heating water through gas geysers. 166. (a) Phytochrome is a photoreceptor, a pigment that plants

.c
om
155. (d) Green Park Stadium is a 60,000 capacity floodlit multi– use to detect light. It is a protein with a bilin
purpose stadium located in Kanpur, India, and the chromophore. It detects mainly red and fared region
home of the Uttar Pradesh cricket team. of the visible spectrum and regulates germination of
156. (c) According to the Red list of 2012. Ganges River seeds.
Dolphin is one of critically endangered species in lndia 167. (c) Quenching is a process in which absorbed light energy
Endangered species in India comprise large varieties is dissipated as heat and does not take part in
of rare species of wild animals, aquatic animals and photochemistry. The phenomenon involves
insects. quenching of chlorophylla (Chla) fluroescence, which
157. (c) Arson is the crime of intentionally and maliciously is induced under steady-state illumination.
setting fire to buildings, wild land areas, vehicles or 168. (c) AIDS virus destroys the T-cells inside of the immune
other property with the intent to cause damage, Distinct system. T cells or T lymphocytes belong to a group
from spontaneous combustion and natural wildfires, it of white blood cells known as lymphocytes, and play
is considered to be a man–made disaster which is a central role in cell – mediated immunity.
socially induced. 169. (a) Bragg Spectrometer is an instrument used to analyze
158. (b) The thyroid gland, is one of the largest endocrine gland crystal structure by using X– rays. In it, a beam of
in the throat, and consists of two connected lobes. It collimated X–rays strikes the crystal, and a detector
is found at the front of the neck, below the Adam’s measures the angles and intensities of the reflected
apple. The thyroid gland secretes thyroid hormones, beam.
which influence the metabolic rate, protein synthesis, 170. (a) Alpha particles consist of two protons and two
and have a wide range of other effects, including on neutrons bound together into a particle identical to a
development. helium nucleus, which is generally produced in the
159. (b) The cerebrum is the seat of intelligence and provides process of alpha decay.
us with the ability to read, write and speak: make 171. (c) The human body requires more calcium than any other
calculations and compose music; remember the past mineral. At least 99% of the calcium is found in the
and plan for the future; and create works. bones and teeth, giving them strength and rigidity.
160. (c) The average adult has a blood volume of roughly 5 172. (a)
liters, which is composed of plasma and several kinds 173. (a) The sigmoid colon is the part of the large intestine. It
of cells. By volume, the red blood cells constitute about forms a loop that averages about 40 cm in length.
45% of whole blood, the plasma about 54.3%, and white 174. (d) Electrons carry current in a good conductor of
cells about 0.7%. electricity and they are negatively charged. This makes
161. (c) The normal range of, conentration of fasting blood a good electrical conductor negatively charged.
sugar is 80 to 120 mg/ml; in the testing of true blood 175. (c) Containment of toxic or hazardous constituents in
sugar, the normal range of concentration is 70 to 100 industrial solid wastes is curr ently being
mg/ml. so normal glucose levels fall between 70 and accomplished using: (i) coating grains of waste material
150 mg. Higher levels may indicate diseases such as with an inert and non-reactive impervious material
diabetes mellitus. (microencapsulation), and (ii) coating blocks of waste
162. (b) Entomology is the scientific study of insects, a branch material with an inert, non-reactive impervious material
of arthropodology, which in turn is a branch of biology. (microencapsulation).
It is derived from the Greek word 'entomos' which 176. (b) UV radiation can be an effective viricide and
means "that which is cut in pieces or engraved/ bactericide. Disinfection using UV radiation is
segmented", hence "insect". commonly used in wastewater treatment applications
163. (a) and is finding an increased usage in drinking water
164. (c) The ionosphere is a region of Earth's upper atmosphere, treatment.
from about 60 km (37 mi) to 1,000 km (620 mi) altitude 177. (b) When cells are exposed to sunlight, radiant energy
and includes the thermosphere and parts of the can damage the DNA. For example, ultraviolet
mesosphere and exosphere, it is ionized by solar irradiation cause covalent bond formation between
radiation, plays an important part in atmospheric adjacent thymines on the same strand of DNA.
electricity and forms the inner edge of the Ultraviolet light is absorbed by a double bond in
magnetosphere. It has practical importance because, thymine and cytosine bases in DNA.
w
w
w
.y
ou
General Science 79

rs
m
178. (a) In order to manufacture silicones, alkyl-substituted 188. (c) Parathyroid hormone (PTH), parathormone or

ah
bo
chlorosilanes are used as starting material. Since parathyrin, is secreted by the chief cells of the

ob
dimethylsilane contains – OH group at the end of the parathyroid glands as a polypeptide containing 84

.w
chain, polymerization and chain length increases. amino acids. It acts to increase the concentration of

or
calcium (Ca2+) in the blood, whereas calcitonin (a

dp
However, the hydrolysis of alkyl trichlorosilane gives

re
very complex cross linked polymer. hormone produced by the parafollicular cells (C cells)

ss
179. (b) Examples of natural coloids can be found in our body of the thyroid gland) acts to decrease calcium

.c
om
itself : blood consists of colloidal sized red blood concentration.
corpuscles (RBC) which provide the vital oxygen to 189. (c) by tracheal system
all the body tissues. 190. (c) In nuclear physics and nuclear chemistry, a nuclear
180. (a) Nickel silver, also known as German silver, is a copper reaction is semantically considered to be the process
alloy with nickel and often zinc. The usual formulations in which two nuclei, or else a nucleus of an atom and a
is 60% copper, 20% nickel and 20% zinc. Nickel silver subatomic particle (such as a proton, neutron, or high
is named for its silvery appearance, but it contains no energy electron) from outside the atom, collide to
elemental silver unless plated. produce one or more nuclides that are different from
181. (d) Atomic absorption spectroscopy is an analytical the nuclide(s) that began the process.
chemistry technique used for determining 191. (b) annihilate each other
concentration of particular metal element and is widely 192. (a) In the photoelectric effect, electrons are emitted from
used in phar maceutics. It was used for the solids, liquids or gases when they absorb energy from
determination of cobalt in Vitamin B12. light. Electrons emitted in this manner may be called
182. (b) Nitrosomonas is a genus comprising rod shaped photoelectrons.
chemoautotrophic bacteria. This rare bacteria oxidizes 193. (d) a circle
ammonia into nitrite as a metabolic process. 194. (b) Cryolite (Na3AlF6, sodium hexafluoroaluminate) is an
183. (a) Devarda's alloy , is an alloy of aluminium (44% - 46%), uncommon mineral identified with the once large
copper (49% - 51%) and zinc (4% - 6%). Devarda's deposit at Ivigtût on the west coast of Greenland,
depleted by 1987.
alloy is used as reducing agent in analytical chemistry
195. (c) Glucose (C6H12O6, also known as D-glucose, dextrose,
for the determination of nitrates after their reduction to
or grape sugar) is a simple monosaccharide found in
ammonia under alkaline conditions. It owes its name to
plants. It is one of the three dietary monosaccharides,
the Italian chemist Arturo Devarda (1859-1944), who
along with fructose and galactose, that are absorbed
synthezised it at the end of the 19th century to develop
directly into the bloodstream during digestion.
a new method to analyze nitrate in Chile saltpeter.
196. (b) Any of the homologous segments, lying in a
184. (a) Grit chambers are long narrow tanks that are designed
longitudinal series, that compose the body of certain
to slow down the flow so that solids such as sand,
animals, such as earthworms and lobsters. Also called
coffee grounds, and eggshells will settle out of the
somite.
water. Grit causes excessive wear and tear on pumps
197. (a) 198. (d) 199. (d) 200. (c) 201. (b)
and other plant equipment. Its removal is particularly
202. (a) 203. (c) 204. (c) 205. (a) 206. (c)
important in cities with combined sewer systems, which 207. (d) 208. (c) 209. (d) 210. (c) 211. (a)
carry a good deal of silt, sand, and gravel that wash off 212. (a) 213. (c) 214. (b) 215. (b)
streets or land. 216. (c) Succulent plants store water in their stems or leaves.
185. (a) Cysteine (abbreviated as Cys or C) is an a-amino acid They include the Cactaceae family, which has round
with the chemical formula HO2CCH(NH2)CH2SH. It is stems and can store a lot of water. The leaves are often
a semi-essential amino acid, which means that it can be vestigial, as in the case of cacti, wherein the leaves are
biosynthesized in humans. The thiol side chain in reduced to spines, or they do not have leaves at all.
cysteine often participates in enzymatic reactions, Water is stored in the bulbs of some plants, at or below
serving as a nucleophile. ground level. They may be dormant during drought
186. (d) Neon is a chemical element with symbol Ne and atomic conditions and are, therefore, known as drought
number 10. It is in group 18 (noble gases) of the periodic evaders.
table. Neon is a colorless, odorless, inert monatomic 217. (a) Thorium is an element which are used in radioactive
gas under standard conditions, with about two-thirds chemicals where all other three options are power
the density of air. generating systems which are regenerated.
187. (b) The aorta is the largest artery in the human body, 218. (a) chegonium, the female reproductive organ in ferns
originating from the left ventricle of the heart and and mosses. An archegonium also occurs in some
extending down to the abdomen, where it bifurcates gymnosperms, e.g., cycads and conifers. A flask-
into two smaller arteries (the common iliac arteries). shaped structure, it consists of a neck, with one or
The aorta distributes oxygenated blood to all parts of more layers of cells, and a swollen base-the venter-
the body through the systemic circulation. which contains the egg.
w
w
w
.y
ou
80 General Science

rs
m
219. (b) Trochodendron is a genus of flowering plants with 230. (b) Galvanization, or galvanisation, is the process of

ah
bo
one living species, Trochodendron aralioides, and six applying a protective zinc coating to steel or iron, to

ob
extinct species known from the fossil record. prevent rusting. The most common method is hot-dip

.w
220. (a) A Caesarean section (often C-section, also other galvanization, in which parts are submerged in a bath

or
dp
spellings) is a surgical procedure in which one or more of molten zinc.

re
incisions are made through a mother's abdomen 231. (b) A homogeneous mixture is a type of mixture in which

ss
the composition is uniform and every part of the

.c
(laparotomy) and uterus (hysterotomy) to deliver one

om
or more babies. The first modern Caesarean section solution has the same properties. A homogeneous
was performed by German gynecologist Ferdinand mixture in which there is both a solute and solvent
Adolf Kehrer in 1881. But in ancient medical history present is also a solution.
Julius Caesar was the first person to be borne by this 232. (c) The plant epidermis consists of three main cell types:
method and thus the operation named after him. pavement cells, guard cells and their subsidiary cells
that surround the stomata.
221. (b) Study of ants is called Myrmecology.
233. (a) As sugar concentration increases in the guard cells,
222. (a) A Reverse transcriptase (RT) is an enzyme used to
as a result water enters the guard cells. The guard cells
generate complementary DNA (cDNA) from an RNA
become turgid (swollen with water). The thin outer walls
template, a process termed reverse transcription. RT is
bulge out and force the inner wall into a crescent shape.
needed for the replication of retroviruses (e.g., HIV),
In this way a stoma or pore is formed between each
and RT inhibitors are widely used as antiretroviral
pair of guard cell.
drugs.
234. (c) Mahyco, an Indian seed company based in Jalna,
223. (c) Fleming's right-hand rule (for generators) shows the Maharashtra, has developed the Bt brinjal. The
direction of induced current when a conductor moves genetically modified brinjal event is termed Event EE 1
in a magnetic field. The right hand is held with the and Mahyco have also applied for approval of two
thumb, first finger and second finger mutually brinjal hybrids.
perpendicular to each other (at right angles). 235. (d) Electric current is measured using a device called an
224. (b) The watt (symbol: W) is a derived unit of power in the ammeter.
International System of Units (SI), named after the 236. (d) Photoperiodism is the physiological reaction of
Scottish engineer James Watt (1736-1819). organisms to the length of day or night. It occurs in
225. (d) The NIOSH states "Under dry conditions, the plants and animals. It affects Flowering,Vegetative
resistance offered by the human body may be as high growth and fruiting in plants.
as 100,000 Ohms. Wet or broken skin may drop the 237. (d) The correct match is as follows:
body's resistance to 1,000 Ohms," adding that "high- Ascorbic acid- Vitamin C
voltage electrical energy quickly breaks down human Chlorophyll- Photosynthetic pigment
skin, reducing the human body's resistance to 500 Carotenoid- Quencher
Ohms."
Superoxide dismutase- Enzyme
226. (a) The critical temperature for superconductors is the 238. (b) Allantois is a part of a developingamniote's conceptus
temperature at which the electrical resistivity of a metal which primarily involved in nutrition and excretion, and
drops to zero. The transition is so sudden and complete is webbed withblood vessels.
that it appears to be a transition to a different phase of 239. (d) Haliotis belongs to Mollusca.
matter; this superconducting phase is described by
240. (a) Although most DNA is packaged in chromosomes
the BCS theory.
within the nucleus, mitochondria also have a small
227. (c) A central processing unit (CPU) is the electronic amount of their own DNA. This genetic material is
circuitry within a computer that carries out the known as mitochondrial DNA or mtDNA
instructions of a computer program by performing the 241. (c) First class proteins contain all the essential amino acids
basic arithmetic, logical, control and input/output (I/ in sufficient amounts.Animal proteins are obtained from
O) operations specified by the instructions. milk, egg, fish, meat etc. are first classproteins. These
228. (a) Brass is an alloy made of copper and zinc; the are also called adequate proteins.
proportions of zinc and copper can be varied to create 242. (b) Needle exerts more pressure than nail on the balloon.
a range of brasses with varying properties. 243. (c) The speed of sound is greater in moist air than in dry air.
229. (d) Wrought iron is an iron alloy with a very low carbon 244. (b) X rays are frequently used to check the defects in
content, in comparison to steel, and has fibrous Diamonds and other precious stones.
inclusions, known as slag. This is what gives it a "grain" 245. (a) When ice is kept on saw dust then it does not melt
resembling wood, which is visible when it is etched or quickly as it's an insulator of heat and air does not
bent to the point of failure. Wrought iron is tough, circulate in good insulators. So, it prevents ice from
malleable, ductile and easily welded. melting quickly .
w
w
w
.y
ou
General Science 81

rs
m
246. (a) When a drop of Glycerol is added to crushed KMnO4 260. (b) 'Dentrification' is the biological conversion of nitrate

ah
bo
spread on a paper there is a violent explosion. to nitrogen gas, nitric oxide or nitrous oxide. These

ob
247. (c) A bleaching agent is a material that lightens or whitens compounds are gaseous compounds and are not

.w
a substrate through chemical reaction. The most readily available for microbial growth; therefore they

or
dp
common bleaching agents generally fall into two are typically released to the atmosphere.

re
categories: chlorine and its related compounds (such 261. (b) Holstein Friesians are a breed of cattle known today as

ss
.c
as sodium hypochlorite) and the peroxygen bleaching the world's highest-production dairy animals. They are

om
agents, such as hydrogen peroxide and sodium found in Europe and America.
perborate. 262. (c) Coagulation removes dirt and other particles suspended
248. (a) Alpha particles are the least penetrating as they are in water. Alum and other chemicals are added to water
the most densely ionizing. The penetrating power of to form tiny sticky particles called "floc" which attract
nuclear radiation depends upon the ionizing power of the dirt particles. The combined weight of the dirt and
the radiation. The more localised the ionization the less the alum (floc) become heavy enough to sink to the
penetrating power it will possess. bottom during sedimentation.
249. (a) Hydrogen- peroxide is an effective sterilizing agent. 263. (c) Retroviral integrase (IN) is an enzyme produced by a
Water results when it readily loses active oxygen. retrovirus (such as HIV) that enables its genetic material
250. (c) The maximum fixation of solar energy is done by green to be integrated into the DNA of the infected cell.
plants.The energy is stored in the plants as 264. (b) 265. (d) 266. (c) 267. (a) 268. (d)
carbohydrates for their metabolic activities as 269. (a) High grade (HG) and ultra high grade (UHG) anthracite
growth,respiration etc. are the highest grades of anthracite coal.
251. (a) The term 'brown air' is used for photochemical smog. 270. (a)
The brown color is cause by the presence of a various
271. (b) Anemometer is an instrument for measuring the speed
Nitrous Oxides.
of the wind, or of any current of gas.
252. (c) Peroxyacetyl nitrate is a secondary pollutant present
272. (b) 273. (a) 274. (a) 275. (b) 276. (d)
in photochemical smog. It is thermally unstable and
277. (c) 278. (b) 279. (c) 280. (d)
decomposes into peroxyethanoyl radicals and nitrogen
dioxide gas. 281. (b) A parsec (symbol: pc) is a unit of length used to
measure large distances to objects outside our Solar
253. (b) Wave particle duality is a quantum mechanics effect. It
System.
means the electron sometimes acts like a particle and
sometimes it acts like a wave. It depends on the 282. (a)
situations. 283. (a) Phenol, also known as carbolic acid, is an aromatic
254. (b) Vitamin B12 consists of a class of chemically related organic compound with the molecular formula
compounds (vitamers). It contains the biochemically C6H5OH. It is a white crystalline solid that is volatile.
rare element cobalt. The molecule consists of a phenyl group (–C6 H5 )
bonded to a hydroxyl group (OH).
255. (d) In 1904 the German agronomist and plant physiologist
Lorenz Hiltner first coined the term "rhizosphere" to 284. (b)
describe the plant-root interface, a word originating in 285. (b) It grows larger until puberty and then begins to shrink.
part from the Greek word "rhiza", meaning root. The gland produces thymosins, which are hormones
Hiltner described the rhizosphere as the area around a that stimulate the development of antibodies.
plant root that is inhabited by a unique population of 286. (b) 287. (d) 288. (d) 289. (c)
microorganisms influenced, he postulated, by the 290. (b) 291. (c) 292. (a)
chemicals released from plant roots. 293. (d) Lichens can be used asair pollution indicators,
256. (c) Reverse transcription were discovered by Howard especially of the concentration of sulfur dioxide in the
Temin and independently isolated by David Baltimore. atmosphere. Lichens are plants that grow in exposed
257. (d) Steam will produce more severe burns than boiling places such as rocks or tree bark. They need to be
water because steam has more heat energy than water very good at absorbing water and nutrients to grow
due to its latent heat of vaporisation. there.
258. (c) Fructose is the sweetest of all natural sugar types. 294. (c) Soil salinity is the salt content in the soil; the process
259. (c) Zone melting, any of a group of techniques used to of increasing the salt content is known as salinization.
purify an element or a compound or control its Salts occur naturally within soils and water. Salinization
composition by melting a short region (i.e., zone) and can be caused by natural processes such as mineral
causing this liquid zone to travel slowly through a weathering or by the gradual withdrawal of an ocean.
relatively long ingot, or charge, of the solid. 295. (b) 296. (b)
w
w
w
.y
ou
82 General Science

rs
m
297. (c) Saffron, a spice derived from the dried stigmas of the 320. (b) The most electronegative element among the following

ah
bo
saffron crocus. (crocus sativus) a small plant about a is fluorine. Fluorine is a chemical element with symbol

ob
foot tall. Each flower has three female parts. (stigmas) F and atomic number 9. It is the lightest halogen and

.w
two male parts. (stamens) each stigmas is red or dark exists as a highly toxic pale yellow diatomic gas at

or
dp
red in color towards the top and yellow towards the standard conditions. As the most electronegative

re
bottom of the stigma, where it is attached to the flower. element, it is extremely reactive: almost all other

ss
.c
298. (d) 299. (a) 300. (a) 301. (b) 302. (c) elements, in cluding some noble gases, form

om
303. (d) 304. (d) 305. (a) 306. (c) 307. (c) compounds with fluorine.
308. (c) 309. (c) 310. (b) 311. (a) 312. (b) 321. (b) NH4Cl is the compound which possesses ionic,
covalent and coordinate bonds. Ammonium chloride,
313. (a) The basic unit of nervous system is known as neuron.
an inorganic compound with the formula NH?Cl, is a
The nervous system consists of nerves, brain and
white crystalline salt, highly soluble in water. Solutions
spinal cord. They control the working of various
of ammonium chloride are mildly acidic. Sal ammoniac
organs of the body. The part of brain and spinal cord
is a name of the natural, mineralogical form of
is called the Autonomic Nervous System.
ammonium chloride
314. (a) A cellulosic wall is found in the cells of plants.Cellulose
322. (a) The depletion of Ozone layer is mainly due to
is an important structural component of the primary
chlorofluorocarbons. A chlorofluorocarbon is an
cell wall of green plants, many forms of algae and the
organic compound that contains only carbon, chlorine,
oomycetes. Some species of bacteria secrete it to form
and fluorine, produced as a volatile derivative of
biofilms. Cellulose is the most abundant organic methane, ethan e, and propane. They are also
polymer on Earth. commonly known by the DuPont brand name Freon.
315. (c) The filtration unit of kidney is known as nephron. 323. (c) Bacterium is associated with tuberculosis, Fungus
Kidneys filter the nitrogenous waste products of the with ringworm, Protozoan with kaala-azaar and virus
body through nephron and throw them out in the form with influenza.
of urine. Kidneys and skin are the chief organs of
324. (b) Matter waves are de Broglie waves. In quantum
excretion.
mechanics, the concept of matter waves or de Broglie
316. (d) The nutritive tissue in the seeds of higher plants is waves reflects the wave-particle duality of matter. The
known as endosperm. Endosperm is the tissue theory was proposed by Louis de Broglie in 1924 in
produced inside the seeds of most flowering plants his PhD thesis. The de Broglie relations show that the
around the time of fertilization. It surrounds the embryo wavelength is inversely proportional to the momentum
and provides nutrition in the form of starch, though it of a particle and is also called de Broglie wavelength.
can also contain oils and protein. 325 (a) When the milk is churned vigorously, the cream is
317. (b) Yeast is an important source of vitamin B. Yeasts is separated out due to centrifugal force. Centrifugal
eukaryotic microorganisms classified in the kingdom force is the apparent force that draws a rotating body
Fungi, with 1,500 species (estimated to be 1% of all away from the center of rotation. It is caused by the
fungal species). Yeasts are unicellular, although some inertia of the body as the body's path is continually
species with yeast forms may become multicellular redirected.
through the formation of strings of connected budding 326. (b) Gas thermometers are more sensitive than liquid
cells known as pseudohyphae, or false hyphae, as thermometers because the gases have large coefficient
seen in most molds. of expansion. Coefficient of Thermal expansion is the
318 (d) Enzymes are proteins. Enzymes are large biological tendency of matter to change in volume in response
molecules responsible for the thousands of metabolic to a change in temperature, through heat transfer.
processes that sustain life. They are highly selective When a substance is heated, its particles begin moving
catalysts, greatly accelerating both the rate and more and thus usually maintain a greater average
specificity of metabolic reactions, from the digestion separation.
of food to the synthesis of DNA. Most enzymes are 327. (b) Sun appearing red at sunset is not caused by
proteins, although some catalytic RNA molecules have atmospheric refraction of light. Refraction is
been identified. essentially a surface phenomenon. The phenomenon
319. (c) Heating of ore below its melting point in the absence is mainly in governance to the law of conservation of
of air is known as calcinations. Calcinations is a thermal energy and momentum. Due to change of medium, the
treatment process in presence of air or oxygen applied phase velocity of the wave is changed but its
to ores and other solid materials to bring about a frequency remains constant. This is most commonly
thermal decomposition, phase transition, or removal observed when a wave passes from one medium to
of a volatile fraction another at any angle other than 90° or 0°.
w
w
w
.y
ou
General Science 83

rs
m
328. (d) Morphine is the most abundant opiate found in opium, 341. (b) Boiling point of water is lower at higher altitudes due

ah
bo
the dried latex from unripe seedpods of Papaver to the decreased air pressure. Boiling point of water

ob
somniferum (the opium poppy). changes with altitude because atmospheric pressure

.w
329. (a) Efforts by the U.S. Soil Conservation Service to changes with altitude.

or
dp
promote contouring in the 1930s as an essential part 342. (a) Sodium thiosulphate, also called sodium hyposulphite

re
of erosion control eventually led to its widespread or "hypo" is used as a photographic fixer in photography.

ss
adoption. The practice has been proved to reduce

.c
343. (d)

om
fertilizer loss, power and time consumption, and wear 344. (d) Stalactites and Stalagmites are formed due to the
on machines, as well as to increase crop yields and
precipitation of Calcium carbonate (CaCO3).
reduce erosion. Contour farming is most effective
345. (d) Hermaphroditism is a form of sexual reproduction in
when used in conjunction with such practices as strip
cropping, terracing, and water diversion. which an organism can self-fertilize or mate with another
individual of the same species.
330. (b) Glucose has the molecular formula C6H12O6 and is
thus a hexose sugar. 346. (b) Lyman series of hydrogen atom spectral lines in the
Ultraviolet.
331. (d) Bacteria are prokaryotes, which, by definition, are cells
that don't possess membrane-bound organelles. 347. (b) An electric battery is a devices consisting of two or
Mitochondria are membrane-bound organelles. more electrochemical cells that convert stored chemical
332. (d) The rainbow comes from the reflection and refraction energy into electrical energy.
of the sunlight in the falling drops; its colors are mainly
due to dispersion, which means that the refractive index 348. (b) Plane
Incident Mirror
of water depends on the wavelength of light.
speed X
333. (c) When a ray of light falls on sea shell, then its small
amount gets refracted (slightly polarised) and rest
almost gets reflected back (fully polarised).
334. (d) An object covers distance which is directly Reflected
proportional to the square of the time. Its acceleration X
is constant.
335. (b) If the horizontal range of a projectile is four times its Therefore, relative speed= x + x = 2x
maximum height, the angle of projection is 45 degrees. 349. (d) The area reserved for the welfare of wildlife is called
Range is the total horizontal distance covered during National Park.
u 2 sin 2 q 350. (a) 351. (c)
the time of flight. It is calculated as , where
g 352. (a) Turner Syndrome, represented by 44A + XO. This occur
q is the angle of projection. in females in which one of the X-chromosome is
missing.
336. (c) Mercury has the least melting point of the following
metals (–38.83°C). Gold, silver and copper have 1064°C, 353. (d) Myopia also known as short sightedness is a condition
961.8°C and 1085°C respectively. of the eye where one looking at a distant object seems
337. (d) Marsh Gas or methane gas is produced when a little out of focus.
vegetation decomposes in water. Methane, also called 354. (d) In the process of photosynthesis, phytoplankton
marsh gas, colourless, odourless gas that occurs release oxygen into the water. Half of the world’s
abundantly in nature as the chief constituent of natural oxygen is produced from phytoplankton
gas, as a component of firedamp in coal mines, and as photosynthesis. The other half is produced via
a product of the anaerobic bacterial decomposition of photosynthesis on land by trees, shrubs, grasses and
vegetable matter under water. other plants.
338. (c) In cactus, spines are modified leaves, and thorns are 355. (a) 356. (a)
modified branches. 357. (d) Rate of Transpiration increases in hot, dry and windy
339. (b) Mycoplasma are the smallest bacterial cells yet condition.
discovered, can survive without oxygen and are 358. (d) Collateral Bundle is a type of vascular bundle in which
typically about 0.1 µm in diameter. the phloem and xylem lie on the same radius, with the
340. (d) Biodiversity can be thought of as the genetic library phloem located towards the periphery of the stem and
that keeps life going on Earth. Biodiversity, short for xylem towards the centre.
biological diversity, is the term used to describe the
359. (b) The Planck constant has dimensions of physical action,
variety of life found on Earth and all of the natural
these are the same as those of angular momentum. In
processes. This includes ecosystem, genetic and
SI units, the Planck constant is expressed in joule
cultural diversity, and the connections between these
and all species. seconds (J.S)
w
w
w
.y
ou
84 General Science

rs
m
360. (a) Cnidaria (e.g., jellyfish, corals, sea anemones). Several

ah
bo
361. (b) Polarization is a property of waves that can oscillate such capsules occur on the body surface. Each is

ob
with more than one orientation. Electromagnetic waves produced by a special cell called a cnidoblast and

.w
such as light exhibit polarization, as do some other contains a coiled, hollow, usually barbed thread, which

or
dp
types of wave, such as gravitational waves. quickly turns outward (i.e., is everted) from the capsule

re
362. (c) Ramanathapuran district of Tamil Nadu has vast upon proper stimulation. The purpose of the thread,

ss
which often contains poison, is to ward off enemies or

.c
stretches of saline and alkaline soils in the wastal

om
region. The permeability rate of the sandy soil is upto to capture prey.
5 cm/hour and therefore, water availibility for the 376. (a) Lactobacillus, also called Döderlein's bacillus, is a
cultivation of crops is very less. genus of Gram-positive facultative anaerobic or
363. (b) microaerophilic rod-shaped bacteria.
364. (c) Minamata disease is a neurological syndrome caused 377. (c) Chlorophyll is a green pigment found in cyanobacteria
by severe mercury poisoning. and the chloroplasts of algae and plants. Its name is
365. (d) Gold is often referred as king of metals. derived from the Greek words chloros (“green”) and
phyllon (“leaf ”). Chlorophyll is an extremely important
366. (b)
biomolecule, critical in photosynthesis, which allows
367. (a) Fermi is the unit of length used to measure nuclear
plants to absorb energy from light.
distances. The unit is named after the Italian physicist
378. (b) In physics, the kinetic energy of an object is the energy
Enrico Fermi.
which it possesses due to its motion. It is defined as
368. (b)
the work needed to accelerate a body of a given mass
369. (d) Christmas factor is one of the proteases of the from rest to its stated velocity. Having gained this
coagulation system. energy during its acceleration, the body maintains this
370. (a) “Romancing with life” is a memoir written by leading kinetic energy unless its speed changes.
Bollywood star, Dev Anand. 379. (c) The ozone layer is a layer in Earth's atmosphere that
371. (b) Highly concentrated sources of oxygen promote rapid absorbs most of the Sun's UV radiation. It contains
combustion. Fire and explosion hazards exist when relatively high concentrations of ozone (O3), although
concentrated oxidants and fuels are brought into close it is still very small with regard to ordinary oxygen, and
proximity; however, an ignition event, such as heat or is less than ten parts per million, the average ozone
a spark, is needed to trigger combustion. Oxygen itself concentration in Earth's atmosphere being only about
is not the fuel, but the oxidant. 0.6 parts per million. The ozone layer is mainly found in
372. (a) The Dumas method in analytical chemistry is a method the lower portion of the stratosphere from
for the quantitative determination of nitrogen in approximately 20 to 30 kilometres (12 to 19 mi) above
chemical substances based on a method first described Earth, though the thickness varies seasonally and
by Jean-Baptiste Dumas in 1826. geographically.
373. (a) Deoxyribonucleic acid (DNA) is a molecule that 380. (a) Airlift pumps are often used in deep dirty wells where
encodes the genetic instructions used in the sand would quickly abrade mechanical parts. (The
development and functioning of all known living compressor is on the surface and no mechanical parts
organisms and many viruses. DNA is a nucleic acid; are needed in the well). However airlift wells must be
alongside proteins and carbohydrates, nucleic acids much deeper than the water table to allow for
compose the three major macromolecules essential for submergence. Air is generally pumped at least as deep
all known forms of life. under the water as the water is to be lifted. (If the water
374. (a) A steroid hormone (abbreviated as sterone) is a steroid table is 50 ft below, the air should be pumped 100 feet
that acts as a hormone. Steroid hormones can be deep).
grouped into five groups by the receptors to which 381. (a) The passenger-side mirror on a car is typically a convex
they bind: glucocorticoids, mineralocorticoids, mirror. In some countries, these are labeled with the
androgens, estrogens, and progestogens. Vitamin D safety warning “Objects in mirror are closer than they
derivatives are a sixth closely related hormone system appear”, to warn the driver of the convex mirror's
with homologous receptors, though they are distorting effects on distance perception. Convex
technically sterols rather than steroids. Steroid mirrors are preferred in vehicles because they give an
hormones help control metabolism, inflammation, upright, though diminished, image. Also they provide
immune functions, salt and water balance, development a wider field of view as they are curved outwards.
of sexual characteristics, and the ability to withstand 382. (c) non-luminous zone
illness and injury. 383. (d) Gamma radiation, also known as gamma rays, and
375. (a) nematocyst, minute, elongated, or spherical capsule denoted by the Greek letter, refers to electromagnetic
produced exclusively by members of the phylum radiation of extremely high frequency and therefore
w
w
w
.y
ou
General Science 85

rs
m
high energy per photon. Gamma rays are ionizing 394. (c) Aerenchyma refers to spaces or air channels in the

ah
bo
radiation, and are thus biologically hazardous. leaves, stems and roots of some plants, which allows

ob
384. (a) Gives us atmospheric pressure - Ozone layer shields exchange of gases between the shoot and the root.

.w
earth from UV radiation - Ionosphere layer allows radio Aerenchyma is widespread in aquatic and wetland

or
dp
waves to be bounced off and radios to work - Prevents plants which must grow in hypoxic soils.

re
water from vaporising off the earth thanks to 395. (b) Night blindness is the first sign of vitamin A deficiency.

ss
atmospheric pressure.

.c
Night blindness and its worsened condition,

om
385. (d) Bacillus Calmette-Guérin is a vaccine against xerophthalmia, are markers of vitamin A deficiency, as
tuberculosis that is prepared from a strain of the it can also lead to impaired immune function, cancer,
attenuated live bovine tuberculosis bacillus, and birth defects.
Mycobacterium bovis, that has lost its virulence in
396. (a) Cloves are the aromatic flower buds of a tree in the
humans.
family Myrtaceae, Syzygium aromaticum. They are
386. (b) A greenhouse gas (sometimes abbreviated GHG) is a native to the Maluku Islands in Indonesia, and are
gas in an atmosphere that absorbs and emits radiation
commonly used as a spice.
within the thermal infrared range. This process is the
397. (c) Heating gypsum partially dehydrates the mineral by
fundamental cause of the greenhouse effect. The
primary greenhouse gases in the Earth's atmosphere driving off approximately 75% of water contained in its
are water vapour, carbon dioxide, methane, nitrous chemical structure. The partially dehydrated mineral is
oxide, and ozone. called calcium sulfate hemihydrate or calcined gypsum
387. (d) Hydrogen bomb is based on the principle of nuclear commonly known to be Plaster of Paris.
fusion. Hydrogen bomb or H-bomb, weapon deriving 398. (c) Neutron was discovered by a British Physicist named
a large portion of its energy from the nuclear fusion of Sir James Chadwick. In1932, Chadwick showed that
hydrogen isotopes. The hydrogen bomb functions by the radiation from the element beryllium, caused by
the fusion, or joining together, of lighter elements into the bombardment of alpha particles is actually a stream
heavier elements. The end product again weighs less of electrically neutral particles. He called these particles
than its components, the difference once more neutrons. He also studied some other properties of
appearing as energy. Because extremely high these particles. Neutrons directly emitted from atomic
temperatures are required in order to initiate fusion nuclei are termed as fast neutron.
reactions, the hydrogen bomb is also known as a 399. (c) Carbonated beverages like soda get their name from
thermonuclear bomb. the carbon dioxide dissolved in the beverage. Most
388. (b) The commonly used safety fuse wire is made up of an gases will dissolve in water (which is the main
alloy of tin and lead. Fuse is a safety device used in component of soda) but carbon dioxide is particularly
any electrical installation, which forms the weakest link good at dissolving and it adds a nice slightly sour
between the supply and the load. It is a short length of taste to the beverage. It may also be added to make it
wire made of lead / tin /alloy of lead and tin/ zinc having reminiscent of the carbonation found in fermented
a low melting point and low ohmic losses. A fuse wire beverages like champagne.
should always have a high resistance and low melting
400. (c) Bio gas is a clean unpolluted and cheap source of
point. High resistance to heat up soon and low melting
energy in rural areas. It consists of 55-70% methane
point to melt away due to the heat produced by high
resistance so that the circuit is not damaged which is inflammable. Bio gas is produced from cattle
dung in a bio gas plant commonly known as gobar gas
389. (b) When cooled from room temperature liquid water
plant through a process called digestion.
becomes increasingly dense, as with other substances,
but at approximately 4 °C (39 °F), pure water reaches 401. (c) Carbon credit is a permit which allows a country or
its maximum density. As it is cooled further, it expands organization to produce a certain amount of carbon
to become less dense. emissions and which can be traded if the full allowance
390. (b) The linear expansion of a solid rod is independent of is not used. It is associated with protection of
the time of heat flow. It is directly dependent on the environment.
temperature, length of the rod and material of the rod. 402. (b) A green leaf is green because of the presence of a
391. (b) X-rays are produced as a result of obstruction of pigment known as chlorophyll, which is inside an
cathode rays with metal. organelle called a chloroplast. When they are abundant
392. (c) Aristotle is the father of Biology and Zoology. in the leaf's cells, as they are during the growing
393. (a) the smallest unit of classification is species. The season, the chlorophylls' green color dominates and
hierarchy of biological classification's major taxonomic masks out the colors of any other pigments that may
ranks is as follows: species< genus< family < order< be present in the leaf. Thus the leaves of summer are
class< phylum < kingdom < domain<life. characteristically green
w
w
w
.y
ou
86 General Science

rs
m
403. (c) Temperature is measured by a thermometer. One of the material which absorbs it. and then ejects incident

ah
bo
most common devices for measuring temperature is electrons. The reason that the photoelectric effect is

ob
the glass thermometer. This consists of a glass tube evidence for the particle nature of light has to do with

.w
filled with mercury or some other liquid, which acts as how materials absorb that light energy and then eject

or
dp
the working fluid. Temperature increase causes the fluid it in the form of electrons.

re
to expand, so the temperature can be determined by 427. (d) The heaviest element in terms of heaviest per a given

ss
measuring the volume of the fluid. Such thermometers number of atoms is the element with the highest atomic

.c
om
are usually calibrated so that one can read the weight. This is the element with the largest number of
temperature simply by observing the level of the fluid protons, which is presently element 118, ununoctium.
in the thermometer. Unnoctium is the heaviest element, but it is man-made.
404. (a) At 25° C the pH of pure water is very close to 7. Acids The heaviest naturally-occurring element is uranium
have a pH less than 7 while bases have a pH greater (atomic number 92, atomic weight 238.0289).
than 7. Because it has a pH of 7, water is considered to 428. (b) Haptens are small molecules that react with a specific
be neutral. It is neither an acid nor a base, but is the antibody but cannot induce the formation of antibodies
reference point for acids and bases. unless bound to a carrier protein or other large antigenic
405. (c) Distillation is a process of separating the component molecule. They are incomplete or partial antigens.
substances (impurities) from a liquid mixture by 429. (a) Sulphur dioxide gas exhibits bleaching properties in
selective evaporation and condensation. The end presence of moisture. It dissolves in water liberating
result is pure water. nascent hydrogen. Nascent hydrogen removes oxygen
406. (b) H2SO4 is the sulphuric acid which is a highly corrosive atoms from the coloring matter (reduces coloring matter)
strong mineral acid . It cannot be used in food industry and it loses its color.
as a preservative. 430. (c) Zirconium is a lustrous, grey-white. strong transition
407. (d) The deep red colour that is present in tomatoes, pink metal that resembles titanium. Zirconium is mainly used
grapefruit, guava and watermelon is caused by as a refractory and opacifier, although it is used in
lycopene, a carotenoid. small amounts as an alloying agent for its strong
408. (d) 409. (a) resistance to corrosion.
410. (b) When a person cries there is a watery discharge from 431. (d) Plant wilting occurs after excessive loss of water by
the nose due to activation of lachrymal fluid through transpiration and lesser absorption. On a hot day, plants
nasal cavity. transpire heavily and cannot absorb water speedily to
411. (c) The functions of root hairs is to collect water and keep pace with transpiration loss, even when there is
mineral nutrients present in the soil and take this enough water.
solution up through the roots to the rest of the plant. 432. (b) Gypsum is calcium sulphate which is essential to the
As root hair cells do not carry out photosynthesis sustainability of most irrigated soils. Irrigated land
they do not contain chloroplasts. eventually leads to sodicity and salinity unless extreme
412. (d) In agriculture, a catch crop is a fast-growing crop that care is taken. It improves sodic (saline) soils by
is grown simultaneously with, or between successive removing sodium from the soil and replacing it with
plantings of a main crop. calcium.
413. (d) Carotenoids protect the plant from damage from UV 433. (d) The Electromagnetic spectrum in order of decreasing
light and this protection is attributed to their antioxidant energy, thus increasing wavelength : Gamma Rays-
properties: they are able to absorb the harmful oxidative wavelength: 0.01 nm : X-rays -1 nm ; Ultra-violet Rays-
chemicals produced during photosynthesis, the O.l micrometres ; Visible Light-Red light : 0.7
process whereby plants make energy from sunlight. micrometres. Violet Light: 0.4 micrometres ; Infrared
414. (d) Radiation-0.01 mm ; Microwaves -less than 10 cm.
usually 1 cm : Radiowaves- Long, Medium and Short
415. (d) The substance that causes the worst air pollution is
Waves : 2 km-l0 m ; and Very High frequency (VHF)
carbon monoxide (CO).
and Ultra High Frequency (UHF): 10 m-l 0 cm.
416. (a) 417. (b) 418. (b) 419. (a) 420. (a)
434. (a) Eutrophication is the eco-system response to the addi-
421. (a) 422. (a) 423. (d) 424. (c)
tion of artificial or natural substances, such as nitrates
425. (b) Pyrolysis is a thermo-chemical decomposition of and phosphates. through fertilizers or sewage to an
organic material at elevated temperatures in the absence aquatic system. One example is the "bloom" or great
of oxygen (or any halogen). Pyrolysis generally takes increase of phytoplankton in a water body as a
place well above atmospheric pressure at temperatures response to increased levels of nutrients.
exceeding above 430 °C.
435. (d) The wings are modified forelimbs of birds which are
426. (c) Light is made up of photons. so it has some particle the key to bird flight. Each wing composed of three
properties. In the photoelectric effect. light hits some limb bones, the humerus, ulna and radius.
w
w
w
.y
ou
General Science 87

rs
m
436. (b) Entropy and enthalpy together make a new quantity, "glues" the beak to the bones, and the beak, which is

ah
bo
the Free Energy which, under conditions of constant the outer sheath covering the jaw bones. The beak is

ob
temperature and pressure, determines the direction of composed of keratin - the same tough, insoluble protein

.w
spontaneous change. All spontaneous processes have found in fingernails, hoofs, antlers and horns.

or
dp
a negative change in free energy. 445. (c) External ear is present in Mammals. Many mammals

re
437. (c) Sandstone can be turned into a metamorphic rock by can move the pinna (with the auriculares muscles) in

ss
being either heated up to a point where it undergoes a order to focus their hearing in a certain direction in

.c
om
significant change or subjected to high pressures, or much the same way that they can turn their eyes. Most
subjected to chemically active fluids, or some humans, unlike most other mammals, do not have this
combination thereof. A common result of the ability.
metamorphosis of sandstone is the creation of 446. (c) Archimedes used the principle of buoyancy to find
quartzite. the purity of gold. According to which a body immersed
438. (d) A Sonometer is a device for demonstrating the in fluid experiences a buoyant force equal to the weight
relationship between the frequency of the sound of the fluid it displaces. With the help of this principle,
produced by a plucked string, and the tension, length the density of the golden crown and solid gold was
and mass per unit length of the string. These compared by balancing the crown on a scale with a
relationships are usually called Mersenne's laws after reference piece, this set up is then immersed in water. If
Marin Mersenne (1588-1648), who investigated and the crown is less dense than gold, it will displace more
codified them. water, and thereby will experience a greater buoyant
439. (b) Polio is a highly infectious disease caused by a virus. force than the reference piece.
Its causative agent, poliovirus, was identified in 1908 447. (c) The kinetic energy of a non-rotating object of mass m
by Karl Landsteiner. The polio virus invades the
1 2
nervous system, and can cause total paralysis in a traveling at a speed v is mv . If m and v are increased
2
matter of hours.
440. (d) It is generally considered that the Panda belongs to to twice its magnitude, then K.E= ½ × 2m × 2v × 2v =
the bear family, though differences persist. Some DNA 1 2
mv (8) = 8 times kinetic energy..
studies have shown that the giant panda is closer to 2
the bear family while the red panda is indeed closer to
448. (d) The gravitational force is inversely proportional to the
the raccoon family.
square of the distance: If you double the distance
441. (a) The pancreas is a glandular organ in the upper between the two bodies, the force of gravity is reduced
abdomen, but really it serves as two glands in one: a to one-fourth its original value.
digestive exocrine gland and a hormone-producing
449. (d) Stationary wave can occur because the medium is
endocrine gland. Functioning as an exocrine gland,
moving in the opposite direction to the wave, or it can
the pancreas excretes enzymes to break down the
arise in a stationary medium as a result of interference
proteins, lipids, carbohydrates, and nucleic acids in
between two waves travelling in opposite directions
food. Functioning as an endocrine gland, the pancreas
450. (b) Two oxygen atoms can both achieve stable structures
secretes the hormones insulin and glucagon to control
by sharing two pairs of electrons joined in a double
blood sugar levels throughout the day.
bond. O=O. Each line represents one pair of shared
442. (b) Due to the action of motor cells at the base of the
electrons.
petiole and leaflets, the leaves of touch me not plant
451. (d) Helium is used by deep sea divers in their diving tanks
closes. The motor cells are activated by touch of the
as a substitute of nitrogen. The trouble with nitrogen
sensitive hairs present on the surface of touch-me-not
in this situation is that nitrogen is a fairly heavy gas,
plant. Like a number of other plant species, it undergoes
and is soluble in blood at high pressure. Long term use
changes in leaf orientation termed "sleep" or nyctinastic
of nitrogen can cause a strange sense of euphoria, or
movement. The foliage closes during darkness and
well being called nitrogen narcosis. This is a bit like
reopens in light.
being drunk, and makes the diver unable to assess
443. (c) The theoretical basis for plant tissue culture was
dangers. Divers who work at depth or for long periods
proposed by Gottlieb Haberlandt, German Academy of
use a mixture of 20% Oxygen and 80% Helium. Helium
science in1902 on his experiments on the culture of
is used for a number of reasons - It is light, cheap, and
single cell.
does not dissolve in blood the same way that nitrogen
444. (d) Beak is technically only the external surface of a bird's does. Being inert it cannot be toxic to the diver or
mouth. The entire mouth structure of a bird is called corrosive to equipment.
the bill. The bill (or rostrum) consists of a bony
452. (c) The atomic number of uranium is 92, and the mass
framework, a vascular layer containing the blood
number of the isotope is given as 238. Therefore, it has
vessels and nerves, a layer of connective tissue, which
92 protons, 92 electrons, and 238 – 92 = 146 neutrons.
w
w
w
.y
ou
88 General Science

rs
m
453. (d) Auxanometer measure root pressure which is of water in and out of the cell but restricts the flow of

ah
bo
developed in the xylem of the roots . The auxanometer solute under many circumstances. As a result, when a

ob
measures plant growth as well as the pressure cell is placed in a hypotonic solution, water rushes

.w
developed within the xylem cells of roots. into the membrane, increasing the cell's volume.

or
dp
454. (c) Cell becomes turgid because of endosmosis. Plant cells 457. (d)

re
need to be turgid (i.e. rigid) to support plant tissues. 458. (c) Chemotherapy is the use of medicines or drugs to treat

ss
Plant cells become turgid when the concentration inside

.c
cancer.

om
the cell is higher than outside. Therefore water moves 459. (d) Oxygen is the most abundant element, making up about
into the cell by osmosis, and the vacuole swells and 47% of the earth's mass.
pushes against the cell wall. Endosmosis is the 460. (c) 461. (a) 462. (c)
movement of water molecules from outside to inside of
463. (d) Edward Jenner is often called "the father of
a cell through osmosis process.
immunology".
455. (c) Imbibition is a process by which a photographic print
464. (c) 465. (a)
is produced by absorption of a water-soluble dye by a
466. (d) Potash Alum is used to reduce bleeding in minor cuts
relief image or a differentially absorbing image in gelatin
and abrasions, nosebleeds, and haemorrhoids.
or a similar medium or in which a previously formed
dye image is transferred by absorption from one layer 467. (a)
into another layer 468. (c) Interferon is produced by the body's cells as a
456. (b) A physical phenomenon known as osmosis causes defensive response to viruses.
water to flow from an area of low solute, high water 469. (b) The first law of thermodynamics is a version of the law
concentration to an area of high solute, low water of conservation of energy.
concentration, until the two areas have an equal ratio 470. (a) 471. (a) 472. (c) 473. (d) 474. (d)
of solute to water. Normally, the solute diffuses toward 475. (b) 476. (b) 477. (a) 478. (c) 479. (a)
equilibrium as well; however, all cells are surrounded 480. (c) 481. (c) 482. (c)
by a lipid bilayer cell membrane which permits the flow
w
w
w
.y
ou
rs
m
6

ah
bo
ob
.w
COMPUTERS

or
dp
re
ss
CHAPTER

.c
om
1. The messenger satellite launched by NASA is to study (c) North–East
(SSC CGL 1st Sit. 2010) (d) Indo–Pak Border in Punjab and Rajasthan
(a) Mercury (b) Venus 10. The Headquarters of MCF (Master Control Facility) – the
(c) Satrun (d) Jupiter nerve centre of the entire space craft operations in India is at
2. An Intelligent terminal (SSC CGL 1st Sit. 2010) (SSC CGL 2nd Sit. 2011)
(a) has a microprocessor, but can-not be programmed by (a) Hyderabad – Andhra Pradesh
the user (b) Thumba – Kerala
(b) can process small data processing jobs, with the use of (c) Sriharikota – Andhra Pradesh
a large CPU
(d) Hassan – Karnataka
(c) interacts with the user in English
11. In which of the following areas, a spreadsheet software is
(d) cannot take data from the user more useful? (SSC CGL 2nd Sit. 2011)
3. Which of the following countries has recently become the (a) Psychology (b) Publishing
third largest market for Twitter ? (SSC CGL 2nd Sit. 2010)
(c) Statistics (d) Message sending
(a) China (b) India
12. A Group Ware is a (SSC CGL 2nd Sit. 2011)
(c) Brazil (d) Indonesia
(a) Hardware (b) Network
4. Processor's speed of a computer is measured in
(c) Software (d) Firmware
(SSC CGL 2nd Sit. 2010)
13. The transfer of date from a CPU to peripheral devices of
(a) BPS (b) MIPS
computer is achieved through (SSC CGL 1st Sit. 2012)
(c) Baud (d) Hertz
(a) interfaces (b) buffer memory
5. 'C' language is a (SSC CGL 2nd Sit. 2010)
(c) modems (d) computer ports
(a) Low level language
14. Which of the following items is not used in Local Area
(b) High level language
Networks (LANs)? (SSC CGL 1st Sit. 2012)
(c) Machine level language
(a) Interface Card (b) Cable
(d) Assembly level language
(c) Computer (d) Modem
6. Window 7, the latest operating system from Microsoft
Corporation has .............. Indian languages fonts. 15. What is m-commerce? (SSC CGL 2nd Sit. 2012)
(SSC CGL 2nd Sit. 2010) (a) machine commerce (b) mobile commerce
(a) 14 (b) 26 (c) money commerce (d) marketing commerce
(c) 37 (d) 49 16. Who invented the modem ? (SSC CGL 1st Sit. 2012)
7. The first computer made available for commercial use was: (a) Apple Computers Inc.
(SSC CGL 1st Sit. 2011) (b) Digital Equipment Corporation
(a) MANIAC (b) ENIAC (c) Wang Laboratories Ltd.
(c) UNIVAC (d) EDSAC (d) AT&T Information Systems
8. A communication network which is used by large 17. What is the name given to the smallest unit of data space
organizations over regional, national or global area is called: available in a spreadsheet software application ?
(SSC CGL 1st Sit. 2011) (SSC CGL 1st Sit. 2012)
(a) LAN (b) WAN (a) Row (b) Block
(c) MAN (d) VAN (c) Cell (d) Pixel
9. Indian Army's Operation 'Saiyam' was related to : 18. An alternate name for the completely interconnected network
(SSC CGL 1st Sit. 2011) topology is (SSC CGL 2nd Sit. 2012)
(a) Kashmir (a) Mesh (b) Star
(b) Indo–China Border in the Central Region (c) Tree (d) Ring
w
w
w
.y
ou
90 Computer

rs
m
19. Communication satellites are used to : 29. What is used to identify whether a data word has an odd or

ah
(SSC CGL 1st Sit. 2015)

bo
(SSC CGL 1st Sit. 2013) even number of 1's ?

ob
(a) transmit communication signal only (a) Sign bit (b) Zero bit

.w
(c) Parity bit (d) Carry bit

or
(b) receive communication signal only

dp
(c) receive and redirect communication signal 30. Rearranging and allocating space in memory to provide for

re
multiple computing tasks is called

ss
(d) provide information of natural resources only

.c
(SSC CGL 1st Sit. 2015)

om
20. What type of information system would be recognised by
digital circuits ? (SSC CGL 1st Sit. 2013) (a) Multiprogramming
(a) Hexadecimal system (b) Multitasking
(b) Binary system (c) Memory Management
(c) Both hexadecimal and binary system (d) Networking
(d) Only Roman system 31. A computer executes programs in the sequence of :
21. Which of the following is an impact printe ? (SSC CGL 1st Sit. 2015)
(SSC CGL 1st Sit. 2013) (a) Decode, Fetch, Execute
(a) Daisy wheel printer (b) Execute, Fetch, Decode
(b) Ink jet printer (c) Fetch, Decode, Execute
(c) Bubble jet printer (d) Store, Fetch, Execute
(d) Laser printer 32. Which of the following is in the ascending order of Data
22. A television channel is characterised by hierarchy ? (SSC CGL 1st Sit. 2015)
(SSC CGL 2nd Sit. 2013) (a) Bit–Byte – Record – Field – Database – File
(a) frequency of transmitted signal (b) Byte – Bit – File – Record – Database – Field
(b) velocity of transmitted signal (c) Bit– Byte – Field – Record – File – Database
(c) physical dimension of television screen (d) Field – Byte – Bit – Record – File– Database
33. Data in database at a particular point of time is called as?
(d) size of picture tube
(SSC CGL 1st Sit. 2016)
23. Who is the founder of “Facebook” which is currently the
No. 1 social networking website in India? (a) Intension (b) Extension
(SSC CGL 1st Sit. 2013) (c) Back up (d) Application
(a) Orkut Buyukkokten (b) Mark Zuckerberg 34. CPU Scheduler is also known as _____ .
(c) Bill Gates (d) Martin Cooper (SSC CGL 1st Sit. 2016)
24. Which computer was the first to use the magnetic drum for (a) Job Scheduler
memory ? (SSC CGL 2nd Sit. 2013) (b) Resource Scheduler
(a) IBM - 650 (b) IBM - 7090 (c) Short-term Scheduler
(c) IBM - 701 (d) IBM - 360 (d) Process Scheduler
25. Identify the FIFO (First In First Out) structure among the 35. Which of the following is not an operating system?
following : (SSC CGL 2nd Sit. 2013) (SSC CGL 1st Sit. 2016)
(a) Stack (b) Queue (a) Android (b) Vista
(c) De-queue (d) Array (c) iOS (d) Opera
26. Which was the first electronic computer constructed at the 36. In IT, the technique of delaying outgoing acknowledgements
Moore School of Engineering? (SSC CGL 1st Sit. 2013) temporarily is known as (SSC CGL 1st Sit. 2016)
(a) EOVAC (b) ONIVAC (a) AR Acknowledgement
(c) ENIAC (d) EDSAC (b) AR request
27. Which among the following standard protocols is the most (c) Piggybacking
widely used by the Internet? (SSC CGL 1st Sit. 2013) (d) Piggyframing
(a) HTTP (b) TCP/IP 37. CPU performance is often measured in:
(c) SMTP (d) SLIP (SSC CHSL 2012)
28. Who among the following introduced the world's first laptop (a) MHz (b) MIPS
computer in the market? (SSC CGL 2014) (c) Band rate (d) G B
(a) Hewlett-Packard 38. The indentations on CDs and DVDs are called:
(b) Epson (SSC CHSL 2012)
(c) Laplink travelling software Inc (a) pits (b) clusters
(d) Microsoft (c) tracks (d) lands
w
w
w
.y
ou
Computer 91

rs
m
39. Programs which protect a disk from an infection are called 50. Image files can be sent along with the e-mail documents

ah
bo
as : (SSC CHSL 2012) using (SSC Multitasking 2014)

ob
(a) Vaccines (b) Antidotes (a) Attachments (b) Subject

.w
or
(c) Library routines (d) Interpreters (c) Signature (d) CC & BCC

dp
40. A nibble is equal to _____________ bits. 51. The invention of _____________ led to the third generation

re
ss
(SSC CHSL 2013) of computers. (SSC Multitasking 2014)

.c
om
(a) 16 (b) 32 (a) Vacuum tubes
(c) 4 (d) 8 (b) Very Large Scale Integration (VLSI)
41. In HTML, tags consists of keywords enclosed within (c) Transistors
(SSC CHSL 2013) (d) Integrated chips
(a) flower brackets { } 52. In a _____________ network, all devices are connected to
(b) angular brackets < > a device called a hub and they communicate through it.
(c) parentheses ( ) (SSC Multitasking 2014)
(d) square brackets [ ] (a) Ming (b) Mesh
42. printer cannot print more than one character at a time. (c) Bus (d) Star
(SSC CHSL 2013) 53. Which one of the following acts as a mediator between the
user’s program and the hardware?
(a) Line (b) Daisy-wheel
(SSC Sub. Ins. 2012)
(c) Laser (d) Dot-matrix
(a) Compiler (b) Editor
43. The waste generated by end life personal computers is
known as (SSC CHSL 2013) (c) Operating System (d) Browser
(a) E-waste (b) PC-waste 54. If you use a font that is not supported by a browser, then
the text (SSC Sub. Ins. 2012)
(c) Physical waste (d) Computer waste
(a) will be displayed in the default font
44. Telnet stands for (SSC CHSL 2014)
(b) will not be displayed
(a) Telephone Network (b) Television Network
(c) will be displayed using ‘ARIAL’ font only
(c) Teletype Network (d) Telefax Network
(d) willl be displayed with a particular background
45. In operating system, Round Robin Scheduling means:
55. The telephone line is connected through the______on
(SSC CHSL 2015)
the computer. (SSC Sub. Ins. 2013)
(a) A kind of scheduling (a) USB (b) Modem
(b) A process allocation policy (c) Ethernet (d) PS2
(c) A memory allocation policy 56. In the nuclear reactors, moderators are used to :
(d) Repetition policy (SSC Sub. Ins. 2013)
46. FORTRAN is called : (SSC CHSL 2015) (a) Accelerate neutrons
(a) Floppy Translator (b) Formula Translator (b) Absorb the neutrons
(c) File Translator (d) Format Translator (c) Slow down the neutrons
47. All forms of ROM are also known as __________. (d) Generate neutrons
(SSC CHSL 2015) 57. Which is not an extension of a picture file on a computer ?
(a) Middleware (b) Firmware (SSC Sub. Ins. 2014)
(c) Shareware (d) Freeware (a) .jpeg (b) .png
48. The Simplest CUP scheduling algorithm is (c) .gif (d) .mdb
(SSC CHSL 2015) 58. Which measure of memory is the largest ?
(a) Round-robin scheduling algorithm (SSC Sub. Ins. 2014)
(b) Multilevel scheduling algorithm (a) MB Megabyte (b) GB Gigabyte
(c) FCFS scheduling algorithm (c) TB Terabyte (d) KB Kilobyte
(d) SJF scheduling algorithm 59. Which of the following is the maximum number of electrons
49. In MS-Excel, _________ provide a visual representation of that can be present in M-shell?
the values in a worksheet. (SSC Multitasking 2013) (SSC Sub. Ins. 2014)
(a) Views (b) Charts (a) 2 (b) 8
(c) Formulae (d) Templates (c) 18 (d) 32
w
w
w
.y
ou
92 Computer

rs
m
60. RAM is used as a short memory in a computer because it. 64. ..........helps in remote login. (SSC Stenographer 2013)

ah
bo
(SSC Sub. Ins. 2015) (a) Telnet (b) Usenet

ob
(a) is very expensive (b) is programmable (c) Milnet (d) Arpanet

.w
or
(c) is volatile (d) has less capacity 65. What kind of data can you send by e-mail?

dp
61. In a computer system, which device is functionally opposite (SSC Stenographer 2014)

re
ss
of a keyboard ? (SSC Sub. Ins. 2015) (a) Audio (b) Video

.c
om
(a) Joystick (b) Trackball (c) Pictures (d) All of these
(c) Mouse (d) Printer 66. Which of the following units used to measure the speed of
62. One strategy of allocating kernel memory is known as a computer ? (SSC Stenographer 2016)
(SSC Sub. Ins. 2016) (a) BAUD (b) Byte
(a) frames allocation (c) SYPS (d) MIPS
(b) register allocation 67. DHCP is mainly used to (SSC Stenographer 2016)
(c) slab allocation (a) Converting IP address to domain name
(d) resource allocation (b) Multicasting
63. In MS-Excel, ...........is a collection of rows and columns that (c) Provide IP address automatically to the devices
holds text and numbers. (SSC Stenographer 2013) (d) Routing
(a) Workbook (b) Worksheet
(c) File (d) Record
w
w
w
.y
ou
Computer 93

rs
m
ah
HINTS & SOLUTIONS

bo
ob
.w
or
dp
re
1. (a) 2. (a) 3. (c) 28. (b) The Epson HX-20 (also known as the HC-20) is

ss
4. (b) Acronym for million instructions per second. A old generally regarded as the first laptop computer,

.c
om
measure of acomputer's speed and power, MIPS announced in November 1981, although first sold
measures roughly the number of machine instructions widely in 1983. Hailed by BusinessWeek magazine as
that a computer can execute in one second. the "fourth revolution in personal computing", it is
5. (b) 6. (d) 7. (c) 8. (b) generally considered both the first notebook and
9. (d) 10. (d) 11. (c) 12. (c) handheld computer.
29. (c) A parity bit, or check bit is a bit added to the end of a
13. (a) In computing, an interface is a shared boundary across
string of binary code that indicates whether the number
which two separate components of a computer system
of bits in the string with the value one is even or odd.
exchange information. The exchange can be between
Parity bits are used as the simplest form of error
software, computer hardware,peripheral devices,
detecting code.
humans and combinations of these. 30. (c) Memory management is the process of controlling and
14. (c) Interface Card, Ethernet Cable and Routers are used in coordinating computer memory, assigning portions
setting up a LAN. The router can be plugged directly called blocks to various running programs to optimize
into the modem via an Ethernet cable, and all other overall system performance.It involves components
computers are eventually connected in some fashion that physically store data, such as RAM (random
to the route. In terms of LAN, a computer cannot be access memory) chips, memory caches, and flash-
said to be an item which constitutes this network. based SSDs (solid-state drives).
15. (b) Mobile Commerce, also known as 31. (c) The first step the CPU carries out is to fetch some data
M-Commerce or mCommerce, is the ability to conduct instructions from main memory then store them in its
commerce using a mobile device, such as a mobile own internal temporary memory areas. The next step is
phone, a Personal Digital Assistant (PDA), a for the CPU to make sense of the instruction it has just
smartphone, or other emerging mobile equipment. fetched this process is called Decode. Execute is the
16. (d) 17. (c) 18. (a) 19. (c) 80. (b) part of the cycle when data processing actually takes
21. (a) Daisy wheel printing is an impact printing technology place.
invented in 1969 by David S. Lee at Diablo Data 32. (c) · Bit (Binary digit) = a bit can only have one of two
Systems. It uses interchangeable pre-formed type possible values.
elements, each with typically 96 glyphs, to generate · Byte = 1 byte = 8 bits
high-quality output comparable to premium typewriters · Field = one or more bits/bytes holding a single
such as the IBM Selectric, but two to three times faster. piece of information about a single entity
22. (a) A television channel is a physical or virtual channel · Record (Logical) = all the fields which apply to a
over which a television station or television network is single entity (except for "Inverted" files)
distributed. Channel numbers represent actual · File = all the records which apply to a collection
of entities
frequencies used to broadcast the television signal.
· Database = a related collection of Files.
For example, in North America. "Channel 2" refers to
33. (b) The data in the database at a particular point of time is
the broadcast or cable band of 54 to 60 MHz, with
known as database instance or database sate or
carrier frequencies of 55.25 MHz for NTSC analog video
snapshot. The database state is also called an
(VSB) and 59.75 MHz for analog audio (FM), or 55.31 extension of the schema.
MHz for digital ATSC (8VSB). 34. (c) The short-term scheduler (also known as the CPU
23. (b) Mark Elliot Zuckerburg is an American computer scheduler) decides which of the ready, in-memory
programmer and internet entrepreneur. He is best processes is to be executed (allocated a CPU) after a
known as one of five co-founders of the social clock interrupt, an I/O interrupt, an operating system
networking website Facebook. call or another form of signal.
24. (a) The IBM 650 Magnetic Drum Data-Processing 35. (d) 36. (c) 37. (b) 38. (c) 39. (b)
Machine was one of IBM's early computers, and the 40. (c) A nibble is equal to 4 bits. In computing, a nibble (often
world's fir st mass-produced computer. It was nybble or even nyble to match the vowels of byte) is a
announced in 1953 and almost 2000 systems were four-bit aggregation, or half an octet. As a nibble
produced, the last in 1962. Support for the 650 and its contains 4 bits, there are sixteen possible values, so a
component units was withdrawn in 1969. nibble corresponds to a single hexadecimal digit (thus,
25. (a) stack 26. (c) 27. (b) it is often referred to as a "hex digit" or "hexit").
w
w
w
.y
ou
94 Computer

rs
m
41. (b) In html, tags consist of keywords enclosed within of a central node, to which all other nodes are

ah
bo
angular brackets. In the HTML syntax, most elements connected; this central node provides a common

ob
are written with a start tag and an end tag, with the connection point for all nodes through a hub.

.w
content in between. An HTML tag is composed of the 53. (c) 54. (a)

or
dp
name of the element, surrounded by angle brackets. 55. (b) A modem turns the digital data of a personal computer

re
An end tag also has a slash after the opening angle into modulated electrical signals in the voice frequency

ss
bracket, to distinguish it from the start tag. range of a telephone channel. These Signals can be

.c
om
42. (b) Daisy wheel printer cannot print more than one transmitted over telephone lines and demodulated by
character at a time. The daisy - wheel is named after its another modem at the receiver side to recover the digital
shape. The individual characters are arranged on data.
flexible petals (or spokes) 'radiating' from the centre of 56. (c) A moderator is a medium that reduces the speed of fast
the wheel. Each petal has a raised character at its end. neutrons, thereby turning them into thermal neutrons
The spoke hits the ribbon when it is struck by the capable of sustaining a nuclear chain reaction involving
hammer. uranium-235. Commonly used moderators include
43 (a) The waste generated by end life personal computers regular (light) water (roughly 75% of the world's
are known as e-waste. E-waste or "Electronic waste" reactors), solid graphite (20% of reactors) and heavy
may be defined as discarded computers, office water (5% of reactors).
electronic equipment, entertainment device electronics, 57. (d) .mdb is not a picture file. It is a database file used by
mobile phones, television sets, and refrigerators. Microsoft Access. Though Access is tied into the
44. (a) The acronym TELNET stands for Telephone Network. Microsoft Office, MDB is Access's own format, which
45. (b) is based on the Access Jet Database Engine..jpeg; .gif;
46. (b) FORTRAN is derived from Formula Translator. and .png files are graphic files. The PNG, JPEG, and GIF
47. (b) Firmware is the permanent software programmed into formats are most often used to display images on the
a read-only memory. Internet. JPEG stands for Joint Photographic Experts
48. (c) Group. PNG stands for Portable Network Graphics. GIF
49. (b) charts stands for Graphics Interchange Format.
50. (a) Image files can be sent along with the e-mail using 58. (c) Terrabyte (TB) is the largest measure of memory. 1 Bit
attachments.An e-mail attachment is a file that is = Binary Digit; 8 Bits = 1 Byte;
attached to an e-mail message. For example, you may 1024 Bytes = 1 KB (Kilo Byte);1024 KB = 1 MB (Mega
attach a graphic, a spreadsheet, or a word processing Byte); 1024 MB = 1 GB(Giga Byte)
document. 1024 GB = 1 TB(Terra Byte). Thus TB > GB> MB > KB
51. (d) The invention of integrated chips led to the third > Byte > Bit
generation of computers. During the period of 1964 to 59. (c) Maximum number of electrons present in M-shell are
1971 third generation computers were developed. The 18. The maximum number of electrons that can occupy
third generation computers emerged with the a specific energy level can be found using the following
development of IC (Integrated Circuits). IC is a single formula.
component containing a number of transistors. It made Electron Capacity = 2n2, the variable n represents the
the computers more fast and reliable. Principal Quantum Number. Shell M has principle
52. (d) Star networks are one of the most common computer quantum number 3.
network topologies. In its simplest form, a star network 60. (c) 61. (d) 62. (a) 63. (b)
consists of one central switch, hub or computer, which 64. (a) 65. (d) 66. (d) 67. (c)
acts as a conduit to transmit messages. This consists
w
w
w
.y
ou
rs
m
7

ah
bo
ob
.w
GENERAL KNOWLEDGE

or
dp
re
ss
CHAPTER

.c
om
1 The book titled 'The Indian War of Independence' was 11. The 'Project Snow Leopard' to conserve the endangered
written by (SSC CGL 1st Sit. 2010) species, launched by the Union Ministry of Environment
(a) Krishna Verma (b) Madame Cama and Forests covers the states of: (SSC CGL 1st Sit. 2011)
(c) B.G. Tilak (d) V.D. Savarkar (a) Jammu & Kashmir and Himachal Pradesh only
2. Who among the following captured his third consecutive (b) Jammu & Kashmir, Himachal Pradesh and Uttarakhand
National Billiards title in the year 2009 ? only
(SSC CGL 1st Sit. 2010) (c) Jammu & Kashmir, Himachal Pradesh, Uttarakhand and
(a) Pankaj Advani (b) Devendra Joshi Arunachal Pradesh only
(c) Geet Sethi (d) Dhruv Sitawala (d) Jammu & Kashmir, Himachal Pradesh, Uttarakhand,
3. What was the name of the ship that sank near the Paradip Arunachal Pradesh and Sikkim
Port in September, 2009 causing an oil spill ? 12. The 2010 FIFA World Cup Final was held at:
(SSC CGL 1st Sit. 2010) (SSC CGL 1st Sit. 2011)
(a) Red Rose (b) Black Rose (a) Paris (b) Berlin
(c) White Rose (d) Green Rose (c) Johannesburg (d) London
4. Who among the following has been honoured with the 13. Who of the following received– Sangeet Natak Akademi's
prestigious Dadasaheb Phalke Award for 2007 ? Ustad Bismillah Khan Yuva Puruskar for 2009 in 'Hindustani
(SSC CGL 1st Sit. 2010) Vocal Music'? (SSC CGL 1st Sit. 2011)
(a) Yash Chopra (b) Ustad Amjad Ali Khan (a) Ornkar Shrikant Dadarkar
(c) Manna Dey (d) A. Nageshwara Rao (b) Ragini Chander Shekar
5. Who among the following won the 'ICC Cricketer of the (c) Abanti Chakravorty and Sukracharya Rabha
Year Award' for the year 2009 ? (SSC CGL 1st Sit. 2010) (d) K. Nellai Manikandan
(a) M.S. Dhoni (b) Gautam Gambhir 14. Which of the following folk/tribal dances is associated with
(c) Mitchell Johnson (d) Tillakaratne Dilshan Uttar Pradesh? (SSC CGL 1st Sit. 2011)
6. NIS stands for (SSC CGL 2nd Sit. 2010) (a) Veedhi (b) Thora
(a) National Infectious diseases seminar (c) Tamasha (d) Rauf
(b) National Irrigation Schedule 15. Which of the following books has been written by Atiq
(c) National Immunisation Schedule Rahimi? (SSC CGL 1st Sit. 2011)
(d) National Information Sector (a) Earth and Ashes (b) This Savage Rite
7. Who discovered cement ? (SSC CGL 2nd Sit. 2010) (c) The Red Devil (d) Witness the Night
(a) Agassit (b) Albertus Magnus 16. Who is the recepient of the Sahitya Akademi Award 2010 in
(c) Joseph Aspdin (d) Janseen Hindi Literature category? (SSC CGL 1st Sit. 2011)
8. Red data book gives information about species which are: (a) Uday Prakash (b) Laxman Dubey
(SSC CGL 1st Sit. 2011) (c) Nanjil Nandan (d) Mangat Badal
(a) extinct (b) endangered 17. First Indian Prime Minister to visit Siachen has been
(c) dangerous (d) rare (SSC CGL 2nd Sit. 2011)
9. Which of the following is the smallest bird? (a) Rajiv Gandhi (b) Inder Kumar Gujaral
(SSC CGL 1st Sit. 2011) (c) Man Mohan Singh (d) None of the above
(a) Pigeon (b) Parrot 18. Which of the following books has been written by Kishwar
(c) Humming bird (d) House sparrow Desai? (SSC CGL 2nd Sit. 2011)
10. The world's only floating national park is situated in: (a) The Red Devil
(SSC CGL 1st Sit. 2011) (b) Witness the Night
(a) Manipur (b) Kuala Lumpur (c) Tonight This Savage Rite
(c) Bilaspur (d) Dispur (d) Earth and Ashes
w
w
w
.y
ou
96 GENERAL KNOWLEDGE

rs
m
19. Which of the following folk/tribal dances is associated with 33. The most endangered Asiatic top predator on the edge of

ah
(SSC CGL 2nd Sit. 2011) (SSC CGL 1st Sit. 2012)

bo
Karnataka? extinction is

ob
(a) Yakshagana (b) Veedhi (a) Black Bear (b) Asiatic Lion

.w
(c) Jatra (d) Jhora (c) Siberian Tiger (d) Dhole

or
dp
20. The Headquarters of International Atomic Energy Agency 34. Analects is the sacred book of (SSC CGL 1st Sit. 2012)

re
is in (SSC CGL 2nd Sit. 2011) (a) Confucianism (b) Judaism

ss
.c
(a) Geneva (b) Paris (c) Shintoism (d) Taoism

om
(c) Vienna (d) Washington 35. Ram Sharan Sharma, who died in 2011 was an eminent
21. Vasundhara Summit was held in (SSC CGL 2nd Sit. 2011) (SSC CGL 2nd Sit. 2012)
(a) USA (b) UK (a) Chemist (b) Economist
(c) Brazil (d) Australia (c) Indologist (d) Archaeologist
22. The first Indian who was chosen as the Secretary General of 36. The 34th National Games were held in 2011 in:
Commonwealth is (SSC CGL 1st Sit. 2012) (SSC CGL 2nd Sit. 2012)
(a) Rakesh Verma (b) Gopalaswami (a) Uttarakhand (b) Kerala
(c) Krishna Murthy (d) Kamalesh Sharma (c) Karnataka (d) Jharkhand
23. Kuldip Nayer, a journalist, was appointed as a High 37. Torah is the scared book of: (SSC CGL 2nd Sit. 2012)
Commissioner in (SSC CGL 1st Sit. 2012) (a) Zoroastrianism (b) Confucianism
(a) Sri Lanka (b) Australia (c) Taoism (d) Judaism
(c) UK (d) Pakistan 38. The 98th Indian Science Congress was held in 2011 at:
24. The National Policy for Empowerment of Women was (SSC CGL 2nd Sit. 2012)
adopted in the year (SSC CGL 1st Sit. 2012) (a) Bengaluru (b) Bhopal
(a) 2001 (b) 2005 (c) Chennai (d) Bhubaneshwar
(c) 1991 (d) 1995 39. Which bank was the first to introduce ATMs to the world?
25. Ballots were first used in (SSC CGL 1st Sit. 2012) (SSC CGL 2nd Sit. 2012)
(a) Australia (b) USA (a) Hong Kong Bank
(c) Ancient Greece (d) England (b) Standard Chartered Bank
26. Which of the following criteria is not used for the (c) Bank of America
classification of human races? (SSC CGL 1st Sit. 2012) (d) Citi Bank
(a) Nose (b) Hair 40. Mamta Sharma was appointed in 2011 as the chairperson
of: (SSC CGL 2nd Sit. 2012)
(c) Eyes (d) Ear
(a) National Commission for Minorities
27. The study of lake is called (SSC CGL 1st Sit. 2012)
(b) National Commission for Protection of Child Right
(a) Topology (b) Hydrology
(c) National Commission for women
(c) Limnology (d) Potomology
(d) National Commission for BCs
28. The Central Drug Research Institute of India is located at
41. Human Development index was formulated by:
(SSC CGL 1st Sit. 2012)
(SSC CGL 2nd Sit. 2012)
(a) Madras (b) Lucknow
(a) ASEAN (b) IBRD
(c) Delhi (d) Bangalore
(c) UNDP (d) UNCTAD
29. How many spokes are there in the Dharmachakra of the 42. Who is the author of the book ‘Pakistan: Beyond the Crisis
National Flag? (SSC CGL 1st Sit. 2012) State’? (SSC CGL 2nd Sit. 2012)
(a) 14 (b) 18 (a) Khuram Iqbal (b) Maleeha Lodhi
(c) 22 (d) 24 (c) Amir Mir (d) M. J. Akbar
30. The latest official language of the U.N. is 43. What is the name of the currency of Bangladesh ?
(SSC CGL 1st Sit. 2012) (SSC CGL 1st Sit. 2012)
(a) Russian (b) Arabic (a) Taka (b) Lek
(c) Chinese (d) Spanish (c) Dinar (d) Peso
31. The latest book 'Kurukshetra to Kargil' is written by 44. The famous car (chariot) festival is held at
(SSC CGL 1st Sit. 2012) (SSC CGL 1st Sit. 2012)
(a) Suryanath Singh (b) Kunal Bhardwaj (a) Puri (b) Gwalior
(c) Karan Singh (d) Kuldip Singh (c) Bhopal (d) Konark
32. The organisation involved primarily with environmental 45. The most literate Union Territory in India is
planning is (SSC CGL 1st Sit. 2012) (SSC CGL 1st Sit. 2012)
(a) CIFRI (b) ICAR (a) Chandigarh (b) Lakshadweep
(c) CSIR (d) NEERI (c) Delhi (d) Pondicherry
w
w
w
.y
ou
GENERAL KNOWLEDGE 97

rs
m
46. The first Secretary-General of the United Nations was 56. The 'One Straw Revolution" was written by:

ah
bo
(SSC CGL 1st Sit. 2012) (SSC CGL 1st Sit. 2013)

ob
(a) U. Thant (a) Masanobu Fukuoka

.w
(b) Dr. Kurt Waldheim (b) Richael Carlson

or
dp
(c) Dag Hammarskjold (c) M.S. Swaminathan

re
(d) Trygve Lie (d) Norman Borlaug

ss
.c
47. The book “Tinderbox : The Past and Future of Pakistan” is 57. Aung San Suu Kyi is a native to : (SSC CGL 1st Sit. 2013)

om
written by (SSC CGL 1st Sit. 2012) (a) Tibet
(a) Jamil Ahmad (b) Khatija Akbar (b) China
(c) Khurram Iqbal (d) M.J. Akbar (c) Myanmar
48. Which of the following cities in India is considered the (d) Arunachal Pradesh
greenest ? (SSC CGL 1st Sit. 2012) 58. Non Residential Indians (NRI) Day is marked on:
(a) Delhi (SSC CGL 1st Sit. 2013)
(b) Thiruvananthapuram (a) January 7 (b) January 9
(c) Bangalore (c) January 17 (d) January 19
(d) Chandigarh 59. "Whether I earned your vote or not, I have listened to you,
49. Synagogue is the place of worship of I have learned from you. You have made me a better
(SSC CGL 1st Sit. 2012) President", was said by : (SSC CGL 1st Sit. 2013)
(a) Zoroastrianism (b) Taoism (a) Pranab Mukherjee (b) Barack Obama
(c) Judaism (d) Shintoism (c) George Bush (d) APJ Abdul Kalam
50. One of the following Chief Ministers received the Highest 60. First Hindu American to enter the US House of
Civilian Award “Bharat Ratna” (SSC CGL 2nd Sit. 2012) Representative as senator is : (SSC CGL 1st Sit. 2013)
(a) Lalu Prasad Yadav of Bihar (a) Tulsi Gabbard (b) Sunita Williams
(b) Late M. G. Ramachandran of Tamilnadu (c) Jyoti Sengupta (d) Ami bera
(c) Jyothi Basu of West Bengal 61. Who was not a politician ? (SSC CGL 1st Sit. 2013)
(d) Late N. T. Rama Rao of Andhra Pradesh (a) H.N. Bahuguna (b) I.K. Gujral
51. Which one of the following film-actors has been conferred (c) S.L. Bahuguna (d) J.Jayalalitha
the Honorary Doctorate in Arts and Culture by the Bedford 62. Which of the following crop cultivation is banned by the
University, London? (SSC CGL 2nd Sit. 2012) Hon’ble Supreme Court of India?
(a) Shahrukh Khan (b) Amir Khan (SSC CGL 2nd Sit. 2013)
(c) Saif Ali Khan (d) Anil Kapoor (a) Lathyrus (Khesari)
52. Who advocated the adoption of ‘PURA’ model to eradicate (b) Genetically modified brinjal
rural poverty? (SSC CGL 2nd Sit. 2012) (c) Bt cotton for export
(a) Dr. A. P. J. Abdul Kalam (d) Bt cotton for local use
(b) Sri Abhijit Sen 63. Who is known for establishing-the “Anand Van”?
(c) Maulana Abdul Kalam Azad (SSC CGL 2nd Sit. 2013)
(d) Prof. A. M. Patha (a) Jubilant Buddha (b) H. N. Bahuguna
53. Which one of the following though called a garden is infact, (c) Baba Amte (d) Motilal Nehru
not a garden? (SSC CGL 2nd Sit. 2012) 64. The civilian Airport of highest altitude is in
(a) Vrindavan Garden of Mysore (SSC CGL 2nd Sit. 2013)
(b) Hanging Garden of Mumbai (a) Tibet (d) Nepal
(c) Eden Garden of Kolkata (c) India (d) China
(d) Shalimar Garden of Kashmir 65. Greenpark Stadium is in (SSC CGL 2nd Sit. 2013)
54. Who wrote the book "Why Socialism"? (a) Bengaluru (b) Dehradun
(SSC CGL 1st Sit. 2013) (c) Chandigarh (d) Kanpur
(a) Jayaprakash Narayan 66. Human Environment Conference-1972 was held at
(b) Mahatma Gandhi (SSC CGL 2nd Sit. 2013)
(c) Acharya Narendra Dev (a) Stockholm (b) Paris
(d) M.N. Roy (c) Geneva (d) Australia
55. Multi purpose river valley projects are the "New temples of 67. Who said “Truth is the ultimate reality and it is God”?
modern India ". (SSC CGL 1st Sit. 2013) (SSC CGL 2nd Sit. 2013)
(a) Jawaharlal Nehru (b) Motilal Nehru (a) Swamy Vivekananda (b) Rabindra Nath Tagore
(c) Mahatma Gandhi (d) Rajiv Gandhi (c) M. K. Gandhi (d) Radhakrishnan
w
w
w
.y
ou
98 GENERAL KNOWLEDGE

rs
m
68. Who is the founder of “Facebook” which is currently the 78. The first non-Englishman elected as Chairman of the

ah
bo
No. 1 social networking website in India? International Cricket Council was

ob
(SSC CGL 1st Sit. 2013) (SSC CGL 2nd Sit. 2013)

.w
(a) Cydle Walcott (b) Gary Sobers

or
(a) Orkut Buyukkokten (b) Mark Zuckerberg

dp
(c) Bill Gates (d) Martin Cooper (c) Imran Khan (d) Sunil Gavaskar

re
79. For which language included in the Indian Constitution, the

ss
69. Who is the author of the book “No Full Stops in India”?

.c
(SSC CGL 1st Sit. 2013) Jnanpith Award has not been given upto 2011?

om
(a) Nirad C. Choudhuri (b) Mark Tully (SSC CGL 2nd Sit. 2013)
(a) Sanskrit (b) Sindhi
(c) R. K. Narayan (d) Ved Mehta
(c) Kashmiri (d) Konkani
70. Which one of the following pairs is wrongly matched?
80. Who is the first British Author to win the Man Booker Prize
(SSC CGL 1st Sit. 2013)
for fiction twice ? (SSC CGL 2nd Sit. 2013)
Place Location
(a) Peter Carey (b) J.M. Coetzee
(a) Tiananmen Square - Beijing
(c) Hilary Mantel (d) None of the above
(b) Tahrir Square - Abu Dhabi 81. Which one of the following is wrongly paired?
(c) Trafalgar Square - London (SSC CGL 2nd Sit. 2013)
(d) Red Square - Moscow Country Currency
71. The term of a non-permanent member of the U.N. Security (a) Japan Yen
Council is (SSC CGL 1st Sit. 2013) (b) Iran Rand
(a) 1 year (b) 2 years (c) Bangladesh Taka
(c) 3 years (d) 6 months (d) Bhutan Ngultrum
72. Julia Gillard is the Prime Minister of 82. The Red Data Books published by the International Union
(SSC CGL 1st Sit. 2013) for Conservation of Nature and Natural Resources
(a) Canada (b) Australia enumerate (SSC CGL 2nd Sit. 2013)
(c) New Zealand (d) Belgium (a) Biodiversity parks and wild life sanctuaries in different
73. Which is the first state in India to pass the Food Security countries.
Law? (SSC CGL 1st Sit. 2013) (b) Centres of origin of cultivated plants.
(a) Chhattisgarh (b) Gujarat (c) Centres of origin of all economically important plants.
(c) Punjab (d) Kerala (d) Threatened species of plants and animals.
74. Who among the following Presidents of MCC was a non- 83. The two South American countries, which are members of
professional cricketer? (SSC CGL 1st Sit. 2013) the Organisation of Petroleum Exporting Countries (OPEC)
are (SSC CGL 2nd Sit. 2013)
(a) Peter May
(a) Ecuador and Brazil
(b) Christopher Martin Jenkins
(b) Ecuador and Bolivia
(c) Ted Dexter
(c) Ecuador and Venezuela
(d) Colin Cowdrey
(d) Venezuela and Brazil
75. Who won the Nobel Prize for Peace in 2012 ?
84. Who is the Chairperson of the National Green Tribunal ?
(SSC CGL 1st Sit. 2013)
SSC CGL 2nd Sit. 2013)
(a) International Atomic Energy Agency (a) Markandey Katju (b) Swatantra Kumar
(b) Liu Xiaobo (c) Satyananda Mishra (d) K.G. Balakrishnan
(c) European Union 85. Which is the capital of Mali ? (SSC CGL 2nd Sit. 2013)
(d) U. N. Intergovernmental Panel on Climate Change (a) Mopti (b) Barmako
76. Who won the Jnanpith Award for 2011? (c) Cairo (d) Nairobi
(SSC CGL 1st Sit. 2013) 86. Which was the first film of Jaspal Bhatti ?
(a) Sachidananda Routray (SSC CGL 2nd Sit. 2013)
(b) Pratibha Ray (a) Power Cut
(c) Gopinath Mohanty (b) Mahaul Theek Hai
(d) Sitakant Mahapatra (c) Thank You Jijaji
77. Environmental impact assessment was first formally (d) None of the three above
established in 1969 in which country ? 87. Who was the first recipient of the "Bharat Ratna" Award
(SSC CGL 2nd Sit. 2013) posthumously ? (SSC CGL 2nd Sit. 2013)
(a) United Kingdom (b) United States (a) K. Kamaraj (b) Lal Bahadur Shastri
(c) France (d) Netherlands (c) M.G. Ramachandran (d) B.R. Ambedkar
w
w
w
.y
ou
GENERAL KNOWLEDGE 99

rs
m
88. Sarus crane is the state bird of (SSC CGL 1st Sit. 2013) 101. 'Agha Khan Cup' is related with which of the following sport

ah
bo
(a) Rajasthan (b) Uttar Pradesh event ? (SSC CGL 2015)

ob
(c) Madhya Pradesh (d) West Bengal (a) Cricket (b) Hockey

.w
or
89. The Daocheng Yading Airport is located in (c) Table Tennis (d) Football

dp
(SSC CGL 1st Sit. 2013) 102. Where was the first conference of SAARC (South Asian

re
ss
(a) Thailand (b) Philippines Association for Regional Cooperation) held?

.c
(SSC CGL 2015)

om
(c) China (d) Tibet
90. "Martyr's Day" is marked on (SSC CGL 1st Sit. 2013) (a) Dhaka (b) New Delhi
(a) January 1 (b) January 15 (c) Colombo (d) Kathmandu
(c) January 30 (d) January 9 103. First Nobel Prize to India was given for :
91. Which of the following is only domestic Airport? (SSC CGL 2015)
(SSC CGL 1st Sit. 2013) (a) Physics (b) Literature
(a) Dabolin Airport, Goa (c) Medicine (d) Chemistry
(b) Srinagar Airport 104. Who was the Indian women president of the United Nations
General Assembly ? (SSC CGL 2015)
(c) Devi Ahilyabai Holkar Airport
(a) Margret Thatcher (b) Golda Mayer
(d) None of the above
(c) Sarojini Naidu
92. Second India–Africa Forum Summit–2011 was held in
(d) Vijya Lakshmi Pandit
(SSC CGL 1st Sit. 2013)
105. What is the currency of Saudi Arabia ? (SSC CGL 2015)
(a) Eretrea (b) Ethiopia
(a) Riyal (b) Pound
(c) Sudan (d) Nigeria
(c) Lira (d) Dinar
93. NIN (National Institute of Nutrition) Central Office is located
106. Reserve Bank of India was nationalised in :
at (SSC CGL 2014)
(SSC CGL 2015)
(a) Hyderabad (b) Mumbai
(a) 1951 (b) 1947
(c) Bengaluru (d) Kolkata
(c) 1935 (d) 1949
94. "Bull's eye" is used in the game of (SSC CGL 2014)
107. Which among the following is a folk dance of India.
(a) Boxing (b) Basketball
(SSC CGL 2015)
(c) Polo (d) Shooting
(a) Kathakali (b) Mohiniattam
95. Pablo Picasso, the famous painter was (SSC CGL 2014)
(c) Manipuri (d) Garba
(a) French (b) Italian
108. Radio activity was discovered by : (SSC CGL 2015)
(c) Flemish (d) Spanish
(a) Curie (b) Beequeral
96. To whom the line 'A thing of beauty is a joy for ever' is
(c) Soddy (d) Rutherford
attributed ? (SSC CGL 2015)
109. ’Mission Indradhanush’ Campaign in India is associated
(a) John Keats (b) Dr. Charles Dickens
with? (SSC CGL 2016)
(c) Dr. Jonathan Swift (d) William Wordsworth
(a) Nutrition to Pregnant Women
97. The birthday of which of the following leaders is celebrated
(b) Awareness of Diabetes
as 'Teachers Day' in India? (SSC CGL 2015)
(c) Eradication of blindness
(a) Dr. Rajendra Prasad (b) S. Radhakrishnan
(d) Vaccination of children
(c) C. Rajgopalachari (d) Lala Lajpat Rai
110. Who is the recipient of Man Booker International Prize 2016
98. The award given for outstanding performance in sports is for the novel ‘The Vegetarian’? (SSC CGL 2016)
(SSC CGL 2015) (a) Orhan Pamuk (b) Han Kang
(a) Bharat Ratna (b) Padma Shri Award (c) Elena Ferrante (d) Tonkin
(c) Arjuna Award (d) Dronacharya Award 111. The novelist, poet and critic of which language has been
99. Which hill station's name means place of the thunderbolt'? conferred with Jnanpith Award for 2015?
(SSC CGL 2015) (SSC CGL 1st Sit. 2016)
(a) Shillong (b) Oottacamand (a) Bengali (b) Telugu
(c) Darjeeling (d) Gangtok (c) Odia (d) Gujarati
100. The ship building yard––Mazgaon Dock is located at – 112. Which one of the following countries is not a member of the
(SSC CGL 2015) "BRICS" group? (SSC CGL 1st Sit. 2016)
(a) Kochi (b) Kolkata (a) Brazil (b) Russia
(c) Mumbai (d) Vishakhapatnam (c) China (d) Indonesia
w
w
w
.y
ou
100 GENERAL KNOWLEDGE

rs
m
113. Which pair of the following Indians has been selected for 124. Lakshmibai National Institute of Physical Education (LNIPE)

ah
bo
grant of Raman Magasaysay Award 2016? is in: (SSC CHSL 2012)

ob
(SSC CGL 1st Sit. 2016) (a) Patiala (b) Gwalior

.w
(c) Indore (d) Jhansi

or
(a) Bezwada Wilson and T.M. Krishna

dp
(b) T.M. Krishna and Arvind Adiga 125. The concept of 'Carbon credit' originated from :

re
(SSC CHSL 2012)

ss
(c) Arvind Adiga and Bezwada Wilson

.c
(d) T.M. Krishna and Satish Gujral (a) Earth Summit, Rio-de-Janerio

om
114. At Rio Olympics, what was the final position of Dipa (b) Kyoto Protocol
Karmakar in her category? (SSC CGL 1st Sit. 2016) (c) Montreal Protocol
(a) 4 (b) 5 (d) None of the above
126. Minorities Rights Day is observed in India on
(c) 6 (d) 7
(SSC CHSL 2013)
115. The study of mountains is known as
(a) 18th December (B 23rd December
(SSC CGL 1st Sit. 2016)
(c) 5th September (d) 1st December
(a) Oncology (b) Lithology
127. Which one of the following institutes in List I is wrongly
(c) Orology (d) Ornithology matched with its location indicated in List II ?
116. The famous Vishnu temple at Angkor Wat in Cambodia was (SSC CHSL 2013)
built by ? (SSC CGL 1st Sit. 2016) List-I List-II
(a) Shrutavarman (b) Suryavarman II (a) Central Institute of Mining Dhanbad
(c) Indravarman (d) Aniruddha and Fuel Research
117. The new symbol of Indian currency is designed by? (b) Central Building Research Roorkee
(SSC CGL 1st Sit. 2016) Institute
(a) Santosh Kumar (b) Y.V. Reddy (c) Central Arid Zone Jodhpur
(c) Udayakumar (d) Dr. Rangarajan Research Institute
118. What is MUDRA? (SSC CGL 1st Sit. 2016) (d) Central Drug Research Kanpur
(a) Development and Refinance Agency Institute
(b) Scheme for Agricultural Insurance 128. Match the two lists given below: (SSC CHSL 2013)
(c) New Planet Discovered List-I List-II
(d) Development and Regulatory Authority for Urban a. London 1. St. Peter's Square
Township b. Vatican City 2. Times Square
119. Which day is observed as “International Day of Non- c. Moscow 3. Trafalgar Square
Violence” (SSC CHSL 2012) d. New York 4. Red Square
(a) 1 st May (b) 2 nd October (a) a-1, b-3, c-2, d-4 (b) a-2, b-4, c-3, d-1
(c) 24 th October (d) 30 th January (c) a-3, b-1, c-4, d-2 (d) a-4, b-2, c-1, d-3
120. Which country is the largest producer of wool ? 129. Who among the following women tennis players has won
the Grand Slam titles for the maximum number of times ?
(SSC CHSL 2012)
(SSC CHSL 2013)
(a) U.S.A. (b) Australia
(a) Serena Williams (b) Venus Williams
(a) Britain (d) Canada
(c) Steffi Graf (d) Margaret Court
121. Which one of the following African countries in not a
130. In which one of the following films did Pran act as a character
member of OPEC ? (SSC CHSL 2012)
artist and not in the role of a villain? (SSC CHSL 2013)
(a) Angola (b) Libya (a) Himalaya Ki Godh Mein
(c) South Africa (d) Algeria (b) Ram aur Shyam
122. Which one of the following National Park/Sanctuary is not (c) Zanjeer
in Rajasthan ? (SSC CHSL 2012) (d) Madhumati
(a) Sariska National Park 131. Who was the first posthumous recipient of Bharat Ratna?
(b) Sambar Wildlife Sanctuary (SSC CHSL 2013)
(c) Rajaji National Park (a) M.G. Ramachandran (b) B.R. Ambedkar
(d) Rhanthambore National Park (c) K. Kamraj (d) Lal Bahadur Shastri
123. Which one of following is called the primary pace maker of 132. Which country won the ICC Women’s World Cup held in
the heart ? (SSC CHSL 2012) February 2013 ? (SSC CHSL 2013)
(a) A. V. Node (b) Chordae tendinae (a) India (b) New Zealand
(c) A.V. Septum (d) S.A. Node (c) England (d) Australia
w
w
w
.y
ou
GENERAL KNOWLEDGE 101

rs
m
133. Which country’s currency is Ngultrum ? 146. "Life Divine" is a book written by (SSC CHSL 2014)

ah
bo
(SSC CHSL 2013) (a) M. K. Gandhi (b) Rabindranath Tagore

ob
(a) Bhutan (b) Laos (c) S. Radhakrishnan (d) Shri Aurobindo

.w
or
(c) Bangladesh (d) Nepal 147. The Oscar Award was won 36 times by

dp
134. "Sattriya Nritya” recognised as a classical dance form of (SSC CHSL 2014)

re
ss
India by the Sangeet Natak Akademi only in 2000, originated (a) Charlie Chaplin (b) Alfred Hitchcock

.c
from (SSC CHSL 2013) (c) Walt Disney (d) Akiro Kurosawa

om
(a) Tripura (b) Assam 148. Who among the following is not a Bharatanatyam dancer ?
(c) Karnataka (d) Gujarat (SSC CHSL 2015)
135. Grammy Award is given in the field of (SSC CHSL 2014) (a) Sitara Devi (b) Leela Samson
(a) Acting (b) Music (c) Geeta Ramachandran (d) Sonal Mansingh
(c) Singing (d) Boxing 149. Ryder Cup is a famous tournament of :
136. The first woman to get the Bharat Ratan Award is (SSC CHSL 2015)
(SSC CHSL 2014) (a) Badminton (b) Golf
(a) Mother Teresa (b) Indira Gandhi (c) Cricket (d) Lawn Tennis
(c) Lata Mangeshker (d) Sarojini Naidu 150. Kanha National Park is located in : (SSC CHSL 2015)
137. Karl Marx wrote (SSC CHSL 2014) (a) Tamil Nadu (b) Bihar
(a) Asian Drama (b) Em ma (c) Andhra Pradesh (d) Madhya Pradesh
(c) Das Kapital (d) Good Earth 151. Who wrote 'Discovery of India'? (SSC CHSL 2015)
138. The religious text of the Jews is named as (a) Mahatma Gandhi (b) Jawahar Lal Nehru
(SSC CHSL 2014) (c) Bal Gangadhar Tilak (d) APJ Abdul Kalam
(a) The Analectus (b) Torah 152. Who is the first woman IPS officer in India ?
(c) Tripitaka (d) Zend-Avesta (SSC CHSL 2015)
139. "Meghdoot" was written by (SSC CHSL 2014) (a) Sarojini Naidu (b) Kiran Bedi
(a) Humayun Kabir (b) Khushwant Singh (c) Bachendri Pal (d) Indira Gandhi
(c) Banabhatta (d) Kalidasa 153. Which of the following is the right expansion of ILO?
140. Who among the following is a famous English writer ? (SSC CHSL 2015)
(SSC CHSL 2014) (a) International Labour Organization
(a) Amrita Pritam (b) Mahadevi Verma (b) Indian Legal Orientation
(c) Ashapurna Devi (d) Mulk Raj Anand (c) Internatioanl Law and Order
141. In which year were the States reorganized on a linguistic (d) Inter–State Lawful Ordinance
basis ? (SSC CHSL 2014) 154. In 2010 a newspaper published its 70,000th issue. Which
(a) 1951 (b) 1947 was the newspaper ? (SSC CHSL 2015)
(c) 1950 (d) 1956 (a) The Oxford Gazette
142. The "Mein Kampf" was written by (SSC CHSL 2014) (b) The Washington Post
(a) Hitler (b) Mussolini (c) The Times of London
(c) Bismarck (d) Mazzini (d) The Hindustan Times
143. It was decided to observe Mahatma Gandhi's birthday 155. Project tiger programme was launched in :
October 2 as the International Nonviolence Day at (SSC CHSL 2015)
(SSC CHSL 2014) (a) 1994 (b) 1973
(a) International lndology Conference (c) 1975 (d) 1971
(b) Setyagraha Centenary Conference 156. Which Indian newspaper has the largest readership ?
(c) Congress Foundation Day Celebration (SSC CHSL 2015)
(d) None of these (a) The Malayala Manorama
144. ISRO's Master Control Facility is in (SSC CHSL 2014) (b) Indian Express
(a) Andhra Pradesh (b) Orissa (c) The Hindu
(c) Gujarat (d) Karantaka (d) The Dainik Jagran
145. Which place is called as "Silicon Valley" of India ? 157. Which day is celebrated as International Yoga Day ?
(SSC CHSL 2014) (SSC CHSL 2015)
(a) Delhi (b) Pune (a) April 23 (b) September 21
(c) Bengaluru (d) Hyderabad (c) July 21 (d) June 21
w
w
w
.y
ou
102 GENERAL KNOWLEDGE

rs
m
158. December 1 is celebrated as : (SSC CHSL 2015) 172. Iron filings can be separated from a heterogenous mixture

ah
bo
(a) Indian Navy Day (b) UNICEF Day using the technique of (SSC CHSL 2015)

ob
(c) Children's Day (d) World AIDS Day (a) Magnetization (b) Sedimentation

.w
(c) Evaporation (d) Sublimation

or
159. Who was the first Secretary General of UNO?

dp
(SSC CHSL 2015) 173. In which year did Amartya Kumar Sen receive the Nobel

re
Prize in Economics? (SSC Multi Tasking 2013)

ss
(a) Kurt Waldheim (b) Dag Hammarskjold

.c
(a) 1990 (b) 1998
(c) Trygve Lie (d) U–Thant

om
(c) 1995 (d) 2000
160. Earth day is celebrated on : (SSC CHSL 2015)
174. The worldwide Great Depression took place in
(a) April 22 (b) September 17
(SSC Multi Tasking 2013)
(c) February 16 (d) April 4 (a) 1930 (b) 1936
161. What is the number of players on each side in Rugby (c) 1929 (d) 1928
Football? (SSC CHSL 2015) 175. Which one of the following is not an All India Service?
(a) 16 (b) 12 (SSC Multi Tasking 2013)
(c) 11 (d) 15 (a) Indian Police Service
162. Which was the first linguistic state to be created ? (b) Indian Foreign Service
(SSC CHSL 2015) (c) Indian Forest Service
(a) Tamil Nadu (b) Andhra Pradesh (d) Indian Administrative Service
(c) Maharashtra (d) Kerala 176. The Sethusamudram Ship Canal Project (SSCP) is supposed
163. The headquarters of the Survey of India Dept. (department) to reduce the distance between Chennai and Tuticorin by
is located at : (SSC CHSL 2015) _________ nautical miles. (SSC Multi Tasking 2013)
(a) Dehradun (b) Hyderabad (a) 305 (b) 361
(c) New Delhi (d) Jaipur (c) 434 (d) 243
164. The Lalit Kala Academy is devoted to the promotion of : 177. Cities with population from one to five million are called
(SSC Multi Tasking 2013)
(SSC CHSL 2015)
(a) Cosmopolitan (b) Conurbation
(a) Literature (b) Music
(c) Million City (d) Metropolitan
(c) Dance and Drama (d) Fine Arts
178. ‘Kyoto Protocol’, an agreement signed by various countries,
165. World wild life fund was founded in : (SSC CHSL 2015) is associated with (SSC Multi Tasking 2013)
(a) 1969 (b) 1992 (a) Deep Sea Oil and Mineral Exploration
(c) 1961 (d) 1965 (b) Clean Environment and Climate Change
166. Arundhati Roy is the author of : (SSC CHSL 2015) (c) Building common food stock to save human beings
(a) Disgrace (b) The Tin Drum from any natural disaster
(c) My Childhood Days (d) God of Small Things (d) International Trade
167. The first Bio-sphere Reserve in India has been established 179. Sanjukta Panigrahi was famous for the dance
in : (SSC CHSL 2015) (SSC Multi Tasking 2013)
(a) Nilgiri (b) Nanda Devi (a) Odissi (b) Bharatnatyam
(c) Hazaribagh (d) Kanha (c) Kathak (d) Manipuri
168. Kimono is a dress style of which Asian Country ? 180. ‘White Revolution’ is related to
(SSC CHSL 2015) (SSC Multi Tasking 2013)
(a) Korea (b) Laos (a) Fish production (b) Wheat production
(c) China (d) Japan (c) Milk production (d) Flood control
169. Dr. P. Rama Rao Committee is related to which of the 181. Who proposed the adage “Survival of the fittest”?
following ? (SSC CHSL 2015) (SSC Multi Tasking 2013)
(a) Lamarck (b) Darwin
(a) Defence (b) Industry
(c) William (d) Huxley
(c) Agriculture (d) Taxes
182. What is the name given to Moon Mission in India?
170. Kathakali is a dance prevalent in which state ?
(SSC Multi Tasking 2014)
(SSC CHSL 2015)
(a) Vikram I (b) Chandrayaan I
(a) Andhra Pradesh (b) Tamil Nadu
(c) Kalpana II (d) Astrosat
(c) Orissa (d) Kerala 183. Lira was the currency of which country?
171. Rand is the currency of : (SSC CHSL 2015) (SSC Multi Tasking 2014)
(a) Iran (b) Romania (a) China (b) Australia
(c) Norway (d) Namibia (c) Japan (d) Italy
w
w
w
.y
ou
GENERAL KNOWLEDGE 103

rs
m
184. Who is the Supreme Commander-in-Chief of the armed 196. Who was the first woman winner of Kaun Banega Crorepati

ah
bo
forces? (SSC Multi Tasking 2014) 6? (SSC Sub. Ins. 2014)

ob
(a) Prime Minister (b) Defence Minister (a) Kiran Bedi (b) Lata Mangeshkar

.w
(c) President (d) Vice-President

or
(c) Sumeet Kaur Sawhney (d) Preity Zinta

dp
185. Which is The second most populous State of India as per 197. Who among the following was named as “Haryana

re
Census 2011? (SSC Multi Tasking 2014)

ss
Hurricane” ? (SSC Sub. Ins. 2015)

.c
(a) West Bengal (b) Rajasthan (a) Ajay Ratsa (b) Nawab Pataudi

om
(c) Maharashtra (d) Bihar (c) Kapil Dev (d) Ajay Jadeja
186. United Nations Conference on Environment and 198. What does the word ‘amphibian’ mean ?
Development is called (SSC Sub. Ins. 2012)
(SSC Sub. Ins. 2015)
(a) Earth Summit (b) Water Summit
(a) Two lives (b) Four lives
(c) Air Summit (d) Resource Summit
(c) Three lives (d) One life
187. The headquarters of International Atomic Energy Agency
is located at (SSC Sub. Ins. 2012) 199. The founder of the ‘Arya Samaj’ was:
(a) Geneva (b) Washington (SSC Sub. Ins. 2015)
(c) Vienna (d) Sydney (a) Dayananda Saraswati (b) Raja Ram Mohan Roy
188. The first woman to conquer Mount Everest twice is (c) Vivekananda (d) Annie Besant
(SSC Sub. Ins. 2012) 200. The book "Globalization, Democratization and Distributive
(a) Surja Lata Devi (b) Jyoti Randhawa Justice" has been authored by (SSC Steno 2013)
(c) Santosh Yadav (d) Suma Shirur (a) Mool Chand Sharma (b) Karan Bajaj
189. Which Indian hockey player has road named after him in (c) Salman Rushdies (d) Anita Nair
Germany? (SSC Sub. Ins. 2012) 201. Which of the following has become the first airport in the
(a) Dhyan Chand (b) Zafar Iqbal world to fully operation all on solar power?
(c) Roop Singh (d) Dhanraj Pillai (SSC Steno 2013)
190. What is the name of the first research station established (a) Lokpriya Gopinath Bardoli International Airport.
by India in the Antarctica? (SSC Sub. Ins. 2012) (b) Amritsar International Airport.
(a) Dakshin Gangotri (b) Maitri (c) Netaji Subash Chandra Bose International Airport.
(c) Agnihotri (d) Aryabhatta (d) Cochin International Airport Ltd. (Kerla).
191. The five permanent members of the U.N. Security Council 202. Which of the following countries has become the first
are (SSC Sub. Ins. 2012)
country in the world to receive funds from United Nations
(a) China, France, Russia, U.K. and U.S.A. for its fast growing Solar Home Systems?
(b) China, Canada, France, U.S.A and Germany
(SSC Steno 2013)
(c) China, Germany, Russia, U.K. and U.S.A.
(a) Afghanistan (b) Burma
(d) China, Germany, U.S.A., U.K and Canada
(c) Bangladesh (d) Switzerland
192. Which of the following is not correctly paired?
203. Who of the following is not one of the recipients of Arjuna
(SSC Sub. Ins. 2012)
Award 2015? (SSC Steno 2013)
(a) Jwala Gutta — Tennis
(a) Anoop Singh (Wrestling)
(b) Virat Kohli — Cricket
(c) Harbhajan Singh — Kabaddi (b) Dipa Karmakar (Gymnastics)
(d) Saina Nehwal — Badminton (c) Jitu Rai (Shooting)
193. Which of the following States has the largest gap in male (d) P. R. Sreejesh (Hockey)
and female literacy as per 2011 Census (provisional data)? 204. Who wrote 'Communist Manifesto'? (SSC Steno 2013)
(SSC Sub. Ins. 2012) (a) Marx and Engels (b) Mao
(a) Uttar Pradesh (b) Madhya Pradesh (c) Lenin (d) Che - Cuevara
(c) Rajasthan (d) Kerala 205. Where is the permanent headquarters of SAARC?
194. Among the following states, which one has adopted me (SSC Steno 2013)
"Neem" tree as the state tree? (SSC Sub. Ins. 2013) (a) New Delhi (b) Kathmandu
(a) Maharashtra (b) Tamil Nadu (c) Thimphu (d) Dhaka
(c) Kerala (d) Andhra Pradesh 206. The only foreign national to receive the highest Bangladesh
195. When was the Geological Survey of India established ? Award ''Bangladesh Swadhinata Sammanona'' is
(SSC Sub. Ins. 2014) (SSC Steno 2013)
(a) 1841 (b) 1851 (a) P.V. Narasimha Rao (b) Indira Gandhi
(c) 1941 (d) 1951 (c) Sonia Gandhi (d) Rajiv Gandhi
w
w
w
.y
ou
104 GENERAL KNOWLEDGE

rs
m
207. The headquarters of the Organization of petroleum 218. Where is the permanent secretarial of the SAARC?

ah
bo
Exporting Countries (OPEC) is in (SSC Steno 2013) (SSC Steno 2014)

ob
(a) Baghdad (b) Abu Dhabi (a) New Delhi (b) Islamabad

.w
(c) Teheran (d) Vienna

or
(c) Colombo (d) Kathmandu

dp
208. Who was the Captain of Kolkata Knight Riders in IPL - V? 219. 'Statue of Liberty' is the National Emblem of

re
(SSC Steno 2013)

ss
(SSC Steno 2014)

.c
(a) M.S.Dhoni (b) Rahul Dravid

om
(a) UK (b) USA
(c) Gautam Gambhir (d) Saurav Ganguly
(c) Germany (d) Russia
209. Which of the following pairs is correctly matched?
220. Ozone Day is celebrated on (SSC Steno 2014)
(SSC Steno 2013)
(a) April 21 (b) September 16
(a) Kuchipudi - Andhra Pradesh
(c) September 25 (d) June 5
(b) Kathakali - Tamil Nadu
221. The book 'Gokhale, My Political Guru' was written by
(c) Kathak - Manipur
(SSC Steno 2014)
(d) Bharatnatyam - Kerala
210. The term of a non-permanent member og the U. N. Security (a) Shaukat Ali (b) C.R. Das
Council is (SSC Steno 2013) (c) M.A. Jinnah (d) M.K. Gandhi
(a) 6 months (b) 1 year 222. Who amongst the following is the author of the book 'Name
(c) 2 years (d) 3 years Sake'? (SSC Steno 2014)
211. Free and compulsory education for all children is provided (a) Kiran Desai (b) Shobha De
until they attain the age of (SSC Steno 2014) (c) Vikram Seth (d) Jhumpa Lahiri
(a) 16 years (b) 18 years 223. National Science Day is observed on (SSC Steno 2014)
(c) 21 years (d) 14 years (a) 14th March (b) 2nd June
212. The Indian recipient of Noble Peace Prize is (c) 5th January (d) 28th February
(SSC Steno 2014) 224. Which was the first National news Agency of free India ?
(a) Hargovind Khurana (b) Rabindra Nath Tagore (SSC Steno 2016)
(c) Mother Teresa (d) Amartya Sen (a) The Associated Press of India
213. London is situated on the banks of river (b) None of these
(SSC Steno 2014) (c) The Indian Review
(a) Tiber (b) Danube (d) The Free press of India
(c) Tigris (d) Thames
225. The book 'Problem of Human Geography' was written by
214. Identify the odd sportman among the following
(SSC Steno 2016)
(SSC Steno 2014)
(a) Jean Brunches (b) None of these
(a) Narain karthikeyan (b) Vijay Hazare
(c) Albert Demangeon (d) De Mortonne
(c) Sachin Tendulkar (d) Sunil Gavaskar
226. Philadelphia is famous for (SSC Steno 2016)
215. First Indian to make a speech in Hindi before the UN General
Assembly was (SSC Steno 2014) (a) Locomotives (b) Dairy industry
(a) AB Vajpayee (b) Lal Bahadur Shastri (c) Ship building (d) Silk textiles
(c) Lal Krishna Advani (d) Morarji Desai 227. Which one among the following book is centered around
216. Which of the following awards is given for excellence in 'environment'? (SSC Sub. Ins. 2016)
sports coaching? (SSC Steno 2014) (a) Here I Stand
(a) Sahitya Academi Award (b) And Then One Day
(b) Dronacharya Award (c) The Late, Great Planet Earth
(c) Arjun Award (d) Silent Spring
(d) Dada Saheb Phalke Award 228. An international movement with its motto to save the world
217. Who is the winner of the prestigious Man Booker Prize by involving itself with environmental problems is
2013? (SSC Steno 2014) (SSC Sub. Ins. 2016)
(a) Haruki Murakami (b) Eleanor Catton (a) Clean En (b) Eco-friend
(c) Malala Yousufzai (d) Vikram Seth (c) Green-field (d) Green-peace
w
w
w
.y
ou
GENERAL KNOWLEDGE 105

rs
m
ah
HINTS & SOLUTIONS

bo
ob
.w
or
dp
1. (d) 2. (a) 3. (a)

re
32. (d) The National Environmental Engineering Research

ss
4. (c) Prabodh Chandra Dey known by his stage name Manna Insititute (NEERI) is a research institute created and

.c
om
Dey, was an Indian playback singer. He debuted in the funded by Government of India. It was established in
film Tamanna in 1942, by this song “Upar Gagan Bishal” Nagpur in 1958.
and went on to record more than 4,000 songs from 1942 33. (d) The Dhole is on the edge of extinction. Also called the
to 2013. The Government of India honoured him with Asiatic wild dog or Indian wild dog, it is a species of
the Padma Shri in 1971, the Padma Bhushan in 2005 canid native to South and Southeast Asia.
and the Dadasaheb Phalke Award in 2007. 34. (a) The Analects of Confucius is an anthology of brief
5. (b) 6. (d) passages that present the words of Confucius and his
7. (c) Joseph Aspdin was an English cement manufacturer disciples, describe Confucius as a man, and recount
who obtained the patent for Portland cement on 21 some of the events of his life. The Analects includes
October 1824. twenty books, each generally featuring a series of
8. (b) 9. (c) 10. (a) 11. (d) 12. (c) chapter that encompass quotes from Confucius, which
13. (a) 14. (b) 15. (a) 16. (a) 17. (c) were compiled by his disciples after his death.
18. (b) 19. (a) 20. (c) 21. (c) 35. (c) Ram Sharan Sharma was an eminent historian of Ancient
22. (d) Mr. Kamlesh Sharma, an Indian diplomat, became and early Medieval India.
Commonwealth Secretary-General on 1 April 2008. He 36. (d) The 2011 National Games, also known as the 34th
was appointed to the post by Commonwealth Heads National Games of India, was held from 12 February
of Government at their meeting in Kampala, Uganda, in 2011 to 26 February 2011 in Ranchi, Jharkhand.
November 2007. 37. (c) The Torah is Judaism's most important text. It is
23. (c) He was appointed High Commissioner to Great Britain composed of the Five Books of Moses. The term
in 1990. "Torah" means instruction and offers a way of life for
24. (a) The National Policy for Empowerment of Women 2001 those who follow if.
has as its goal bringing about advancement, 38. (c) Indian Science congress Association (ISCA) is a
development and empowerment of women in all spheres premier scientific organisation of India with
of life through creation of a more responsive judicial headquarters at Kolkata, West Bengal. The 98th session
and legal system sensitive to women and was held at SRM University, near Chennai in January
mainstreaming a gender perspective in the development 2011.
process.
39. (d) The Citibank of New York was the first such bank.
25. (a) First used in the Australian state of Victoria in 1857, the
40. (c) Former Rajasthan Mahila Congress President chief
paper ballot listing all the candidates was first known
Mamta Shrma was appointed as National Commission
as "the Australian ballot. "In 1889, New York became
for Women (NCW) chairperson. The appointment was
the first American state to use these ballots.
cleared by the Prime Minister's Office.
26. (d) The term race or racial group usually refers to the
concept of dividing humans into populations or groups 41. (c) The origins of the HDI are found in the annual Human
on the basis of various sets of characteristics. The Development Reports of the United Nations
most widely used human racial categories are based Development Programme (UNDP). These were devised
on visible traits (especially skin colour, cranial or facial and launched by Pakistani economist Mehboob ul Haq
features and hair texture), and self-identification. in 1990.
27. (c) Limnology is the story of lakes and other freshwater 42. (b) Maleeha Lodhi is a journalist, academic and diplomat
basins. from Pakistan. She was the High Commissioner of
28. (b) CDRI, is located at Lucknow was inugurated in 1951 Pakistan to the United Kingdom and is a former
by the then Prime Minister of India, Jawahar lal Nehru. Pakistani Ambassador to the United States.
29. (d) The spokes in the Ashok Chakra represent 24 states as 43. (a) 44. (a) 45. (b) 46. (d) 47. (d)
24 were the number of states at that time. 48. (d) 49. (c) 50. (b) 51. (a) 52. (a)
30. (b) In 1980, the General Assembly made Arabic an official 53. (c) 54. (a) 55. (a) 56. (a)
and working language of all its committees and 57. (c) Aung San Suu Kyi, also called Daw Aung San Suu Kyi
subcommittees. (born June 19, 1945, Rangoon, Burma [now Yangon,
31. (d) The author is Brigadier (Retired) Kuldip Singh. This Myanmar]) politician and opposition leader of
book is an effort to understand now the future will Myanmar, daughter of Aung San (a martyred national
unfold in Asia 2030. hero of independent Burma) and Khin Kyi (a prominent
w
w
w
.y
ou
106 GENERAL KNOWLEDGE

rs
m
Burmese diplomat), and winner of the Nobel Prize for 72. (b) Julia Eileen Gillard is the 27th and current Prime

ah
bo
Peace in 1991. She has held multiple governmental Minister of Australia and the Leader of the Australian

ob
posts since 2016. Labor Party since 24 June 2010. She is the first women

.w
58. (b) to hold the office.

or
dp
59. (b) Barack Hussein Obama II who is the 44th and current 73. (a) Chhattisgarh was the first state in the country to

re
President of the United States. He is the first African introduce the Food Security Act, a unique law so that

ss
it could become a right of the people to get sufficient

.c
American to hold the office and the first president born

om
outside the continental United States. quantity of food at subsidized rates.
60. (a) 74. (b) Christopher Martin - Jen - kins was a British cricket
61. (c) Sunderlal Bahuguna (born 9 January 1927) is a noted journalist and a past President of the MCC. He was
Garhwali environmentalist, Chipko movement leader also a commentator for Test Match Special (TMS) on
and a follower of Mahatma Gandhi's philosophy of BBC Radio from 1973 until diagnosed with terminal
Non-violence and Satyagraha. cancer in January 2012.
He was awarded the Padma Vibhushan, India's second 75. (c) The Nobel Peace Prize 2012 was awarded to European
highest civilian honour, on 26 January 2009. Union (EU) “for over six decades contributed to the
62. (a) Grass pea also known as khesari dal was banned for advancement of peace and reconciliation, democracy
consumption in 1961 as it was believed that Beta–N– and human rights in Europe”.
Oxalylaminoalanine, a neuro–toxic amino–acid in the 76. (b) Pratibha Ray is an Indian academic and writer who
legume, caused neurolathyrim or a paralysis of the won the Jnanpith Award in 2011. She was the first
lower limbs, woman to win the Moortidevi Awards in 1991.
63. (c) Founded in 1948 by noted social activist, Baba Amte, 77. (b) Environmental impact assessments commenced in the
Anandwan, located near Arora in Chandrapur district 1960s, as part of increasing environmental awareness.
in the state of Maharashtra. India, is an ashram, and a EIAs involved a technical evaluation intended to
community rehabilitation centre for leprosy patients contribute to more objective decision making. In the
and the disabled from downtrodden sections of United States, environmental impact assessments
society. obtained formal status in 1969, with enactment of the
64. (d) Daocheng Yading Airport in A southwest China's National Environmental Policy Act.
Sichuan Province is the world's highest–altitude 78. (a) Sir Clyde Leopold Walcott, KA, GCM (17 January 1926
civilian airport. It is located at a height of 4,411 meters - 26 August 2006) was a West Indian cricketer. Walcott
above sea level was a member of the "three W's", the other two being
65. (d) Green Park Stadium is a 60,000 capacity floodlit multi– Everton Weekes and Frank Worrell: all were very
purpose stadium located in Kanpur, India, and the successful batsmen from Barbados, born within a short
home of the Uttar Pradesh cricket team. distance of each other in Bridgetown, Barbados in a
66. (a) The conference was held in Stockholm. the capital of period of 18 months from August 1924 to January 1926;
Sweden. in 1972. The conference agreed upon a all made their Test cricket debut against England in
declaration containing 26 principles concerning the 1948. In the mid-1950s, Walcott was arguably the best
environment and development. batsman in the world. In later life, he had an active
67. (c) Gandhi was known to have considered truth something career as a cricket administrator, and was the first non-
similar to that. Baruch Spinoza considered ultimate truth English and non-white chairman of the International
as the ultimate reality of a rationally ordered system Cricket Council.
that is God. 79. (b) The Jnanpith Award is a literary award in India. Along
68. (b) Mark Elliot Zuckerburg is an American computer with the Sahitya Akademi Fellowship. It is one of the
programmer and internet entrepreneur. He is best two most prestigious literary honours in the country.
known as one of five co-founders of the social The award was instituted in 1961. Any Indian citizen
networking website Facebook. who writes in any of the official languages of India is
69. (b) Mark Tully is the author of No Full Stops in India eligible for the honour. It is presented by the Bharatiya
(1998). Its collection of journalistic essays, was Jnanpith, a trust founded by the Sahu Jain family, the
published in the US as The Defeat of a Congress-man. publishers of the The Times of India newspaper.
70. (b) Tahrir Square, also known as “ Martyr Square”, is a 80. (c) Hilary Mary Mantel is an English writer whose work
major public town square in Down town Cairo. Tahrir ranges in subject from personal memoir and short story
Square was the focal point of the 2011 Egyptain to historical fiction and essay. She has twice been
Revolution against former president Hosni Mubarak. awarded the Booker Prize. She won her first Booker
71. (b) There are 10 non-permanent members in the United Prize for the 2009 novel, Wolf Hall, a fictional account
Nations Security Council, with five elected each year of Thomas Cromwell's rise to power in the court of
Henry VIII. She won her second Booker Prize for the
to serve two-year terms.
w
w
w
.y
ou
GENERAL KNOWLEDGE 107

rs
m
2012 novel, Bring Up the Bodies, the second instalment 97. (b) The birthday of Sarvepalli Radhakrishnan is celebrated

ah
bo
of the Thomas Cromwell trilogy. Mantel was the first as Teacher's Day. The day commemorates the birthday

ob
woman to receive the award twice. of Dr Sarvepalli Radhakhrishnan, a philosopher and a

.w
81. (b) The currency of Iran is Rial. teacher par excellence, and his contribution towards

or
dp
82. (d) The Red Data Book is the state document established Indian education system. As a tribute to this great

re
for documenting rare and endangered species of teacher, his birthday is observed as Teacher's Day

ss
across India on 5th September.

.c
animals, plants and fungi as well as some local sub-

om
species that exist within the territory of the state or 98. (c) The Arjuna Awards are given by the Ministry of Youth
country. This book provides central information for Affairs and Sports, government of India to recognize
studies and monitoring programmes on rare and outstanding achievement in National sports.
endangered species and their habits. 99. (c) "Place of the Thunderbolt" is associated with
83. (c) OPEC is an intergovernmental organization that was Darjeeling(west Bengal). The word Darjeeling is a
created at the Baghdad Conference on September 10- combination of two words 'dorje,' which means
14, 1960, by Iraq, Kuwait, Iran, Saudi Arabia and 'thunderbolt' and 'ling', which means 'place'. Hence the
Venezuela. Later it was joined by nine more word Darjeeling means 'the Land of Thunderbolt'.
governments: Libya, United Arab Emirates, Qatar, 100. (c) The Ship building yard Mazagon Dock Limited (MDL)
Indonesia, Algeria, Nigeria, Ecuador, Angola, and is located in Mumbai(Maharashtra).It is India's prime
Gabon. OPEC was headquartered in Geneva, Switzerland shipyard. It manufactures warships and submarines
before moving to Vienna, Austria, on September 1, 1965. for the Indian Navy and offshore platforms and
84. (b) Swatantra Kumar is the Chairperson of National Green associated support vessels for offshore oil drilling.
Tribunal. 101 (d) Aga Khan Cup is related to Football. When Prince
85. (b) Barmako is the capital of Mali. Aga Khan IV of Iran visit in 1958, expressed his interest
86. (b) Jaspal Singh Bhatti (3 March 1955 - 25 October 2012) to start a major international football club tournament
was an Indian television personality famous for his in the region. The football authorities of East Pakistan
satirical take on the problems of the common man. He in collaboration with Asian Football Confederation
is most well known for his television series Flop Show (AFC) decided to start this event.
and mini capsules Ulta Pulta which ran on Doordarshan, 102. (a) The first summit was held in Dhaka, Bangladesh on 7-
India's national television network, in the late 1980s 8 December 1985 and was attended by president of
and early 1990s. In 2013, he was honoured with the Bangladesh, Maldives, Pakistan and Sri Lanka, the
Padma Bhushan (posthumously), India's third highest kings of Bhutan and Nepal, and the prime minister of
civilian award India.
87. (b) Lal Bahadur Shastri was the second Prime Minister of 103. (b) Rabindranath Tagore was the first Indian ever to
the Republic of India and a leader of the Indian National receive a Nobel Prize. He was awarded the Nobel Prize
Congress party. Shastri joined the Indian independence for Literature in recognition of his work Geetanjali, a
movement in the 1920s. collection of poems, in 1913.
88. (b) 89. (d) 90. (c) 91. (d) 92. (b) 104. (d) Vijaya Lakshmi Pandit became the first woman to be
93. (a) The National Institute of Nutrition (NIN) is an Indian elected president of the UN General Assembly in 1953.
Public health, Biotechnology and Translational She was an Indian diplomat, politician, and a sister of
research center located in Hyderabad, India. The India's first prime-minister, Jawaharlal Nehru. She was
institute is one of the oldest research centers in India, active in the Indian freedom movement and held high
and the largest center, under the Indian Council of national and international positions.
Medical Research, located in the vicinity of Osmania 105. (a) The currency of Saudi Arabia is Saudi Riyal.
University. 106. (d) After independence, the government passed Reserve
94. (d) Bullseye, also known as conventional pistol, is a Bank (Transfer to Public Ownership) Act, 1948 and
shooting sport in which participants shoot handguns took over RBI from private shareholders after paying
at paper targets at fixed distances and time limits. appropriate compensation. Thus, nationalisation of RBI
95. (d) Pablo Ruiz y Picasso, also known as Pablo Picasso, took place in 1949 and from January 1, 1949, RBI started
was a Spanish painter, sculptor, printmaker, ceramicist, working as a government owned central bank of India.
stage designer, poet and playwright who spent most 107. (d) Garba is a folk dance of state of Gujarat.
of his adult life in France. 108. (b) Antoine-Henri Becquerel (1852-1908) is known for his
96. (a) Endymion is a poem,written by John Keats. It begins discovery of radioactivity, for which he received the
with the line "A thing of beauty is a joy forever". The Nobel Prize for Physics jointly with Marie Curie) and
poem tells about how nature and its wonder mesmerize Pierre in 1903 and the contributions he made to that
us and take away all the sorrow that surrounds us from field. Henri Becquerel discovered radioactivity, and
time to time. Mari Curie coined the term.
w
w
w
.y
ou
108 GENERAL KNOWLEDGE

rs
m
109. (d) Mission Indradhanush was launched by Union Health title in singles category. She is a retired World No. 1

ah
bo
Minister J.P Nadda on 25 December 2014. It aims to professional tennis player and Christian minister from

ob
immunize all children under the age of 2 years and Australia.

.w
pregnant women against seven vaccine preventable 130. (c) Pran acted as a character artist in the film Zanjeer. It

or
dp
diseases namely diphtheria, whooping cough, tetanus, made him stand up in defiance as Sher Khan in Zanjeer

re
poliomyelitis, tuberculosis, measles and Hepatitis B (1973) against the six-feet tall, long-legged Amitabh

ss
by 2020. In addition to this, vaccines for Japanese Bachchan.Pran had a successful career spanning over

.c
om
Encephalitis (JE), rotavac and Haemophilus influenzae six decades, and his contribution to Hindi cinema
type B (HIB) are also being provided in selected states. extends beyond his role as an actor.
110. (b) 111. (d) 131. (d) Lal Bahadur Shastri was the first posthumous recipient
112. (d) BRICS is the acronym for an association of five major of Bharat Ratna in 1966. Lal Bahadur Shastri was the
emerging national economies: Brazil, Russia, India, third Prime Minister of the Republic of India and a
China and South Africa. Originally the first four were leader of the Indian National Congress party. Shastri
grouped as “BRIC” (or “the BRICs”), before the joined the Indian independence movement in the 1920s.
induction of South Africa in 2010. 132. (d) Australia won the ICC Women's World Cup held in
113. (a) Bezwada Wilson and TM Krishna are included in the February 2013. The 2013 Women's Cricket World Cup
list of the prestigious Ramon Magsaysay Award was the tenth Women's Cricket World Cup, which was
winners for the year 2016. hosted by India for the third time. India previously
hosted the World Cup in 1978 and 1997. Australia won
114. (a) Dipa Karmakar returned empty handed from the Vaults
the tournament for the sixth time, beating West Indies
final at the Gymnastics event of Rio 2016 Olympics
by 114 runs in the final.
after she finished the event at fourth rank.
133. (a) Bhutan's currency is known as Ngultrum. n 1974, the
115. (c) Orology is the study of mountains.An example of ngultrum was introduced, replacing the rupee at par.
orology is research on how a particular mountain came The ngultrum is equal in value to the Indian rupee and
to be formed. it does not exchange independently with other nations'
116. (b) It was built by the Khmer King Suryavarman II in the currencies but is interchangeable with the Indian rupee.
early 12th century. 134. (b) "Sattriya Nritya" is a classical dance form of India and
117. (d) The Indian rupee sign is the currency sign for the Indian has originated in Assam. Sattriya or Sattriya Nritya , is
rupee, the official currency of India. Designed by D. one among the eight principal classical Indian dance
Udaya Kumar, it was presented to the public by the traditions. In the year 2000, the Sattriya dances of
Government of India on 15 July 2010, following its Assam received recognition as one of the eight
selection through an "open" competition among Indian classical dance forms of India.
residents. 135. (b) A Grammy Award is an accolade by the National
118. (a) Micro Units Development and Refinance Agency Bank Academy of Recording Arts and Sciences (NARAS)
(or MUDRA Bank) is a public sector financial of the United States to recognize outstanding
institution in India. achievement in the music industry.
119. (b) 120. (b) 121. (c) 122. (c) 136. (b) Indira Gandhi was the first woman of India who received
123. (d) 124. (b) 125. (b) the Bharat Ratna, the highest civilian award of the
126. (a) Minorities Rights Day is observed in India in 18th Republic of India, in 1971.
December. National Commission for Minorities 137. (c) Das Kapital by Karl Marx, is a critical analysis of
celebrated Minorities Rights Day on 18 December 2012. political economy, intended to reveal the economic laws
Minorities Rights Day is celebrated on 18th December of the capitalist mode of production.
every year. The day is celebrated to protect rights of 138. (b) The Torah is written on scrolls and kept in a special
the minorities communities as well as bringing the better cabinet called the aron hakodish, the holy ark, in
understanding among religious minorities in India. synagogues. The Torah is read with a pointer called a
127. (d) Central Drug Research Institute is wrongly matched yad (hand) to keep it from being spoiled. Each week,
with Kanpur. The Central Drug Research Institute is a one section is read until the entire Torah is completed
and the reading begins again.
multidisciplinary research laboratory in Lucknow, India,
employing scientific personnel from various areas of 139. (d) Meghdoot is a lyric poem written by Kalidas,
biomedical sciences. considered to be one of the greatest Sanskrit poets.
128. (c) Trafalgar Square is associated with London, St.Peter's 140. (d) Mulk Raj Anand was an Indian writer in English, notable
for his depiction of the lives of the poorer castes in
Square is associated with Vatican City, Red Square is
traditional Indian society. One of the pioneers of Indo-
associated with Moscow and Times Square is
Anglian fiction, he together with R. K. Narayan, Ahmad
associated with New York.
Aliand Raja Rao, was one of the first India-based writers
129. (d) Margaret Court has won the Grand Slams title for the in English to gain an international readership.
maximum number of times. She has won 24 Grand Slams
w
w
w
.y
ou
GENERAL KNOWLEDGE 109

rs
m
141. (d) The States Reorganisation Act, 1956 was a major reform 160. (a) Earth Day is celebrated of 22nd April all over the world

ah
bo
of the boundaries of India's states and territories, to demostrate support for environmental protection.

ob
organising them along linguistic lines. 161. (d) In Rugby football each team starts the match with 15

.w
142. (a) Mein Kampf is an autobiographical manifesto by Nazi players on the field and seven or eight substitutes.

or
dp
leader Adolf Hitler, in which he outlines his political 162. (b) The first state to be formed on linguistic basis in India

re
ideology and future plans for Germany. was Andhra Pradesh, which was divided into two

ss
.c
143. (d) On 15 June 2007, the United Nations General Assembly Telugu-speaking states.

om
voted to establish 2 October as the International Day 163. (a) The Survey of India Dept. is headquartered at
of Non-Violence. The resolution by the General Dehradun. It has 18 civil engineering divisions ranging
Assembly asks all members of the UN system to from the prediction of tides to aerial survey.
commemorate 2 October in "an appropriate manner and 164. (d) The Lalit Kala Academi or National Academy of Art is
disseminate the message of non-violence, including India’s national academy of fine arts. Its headquarters
through education and public awareness are at Ravindra Bhawan, Ferozshah Road, New Delhi.
144. (d) The Master Control Facility (MCF) is a facility set up 165. (c) The World Wildlife Fund is an international non-
by the Indian Space Research Organisation (ISRO) in governmental organization founded on 29th April, 1961.
the city of Hassan in the Indian state of Karnataka. 166. (d) The God of Small Things (1997) is the debut novel of
145. (c) The Silicon Valley of India is a nickname of the Indian Indian writer Arundhati Roy.
city of Bangalore. The name signifies Bangalore's status 167. (a) Nilgiri Biosphere Reserve is India’s first and foremost
as a hub for information technology (IT) companies in biosphere reserves with a heritage rich in flora and
India and is a comparative reference to the original fauna.
Silicon Valley, based around Santa Clara Valley, 168. (d) The Kimono is a traditional Japanese garment.
California, a major hub for IT companies in the United 169. (a) Dr. P. Rama Rao committee is related to defence.
States.
170. (d) Kathakali is a traditional dance from the south Indian
146. (d) Shri Aurbindo has written 'Life Divine' which deals with state of Kerala.
theoretical aspects of Integral Yoga.
171. (d)
147. (c) Walt Disney has won the most Oscar awards.
172. (a) Magnetization technique can be used to seperate the
148. (b) Leela Samson is a Bharatanatyam dancer, iron fillings from the heterogenous mixture assuming
choreographer instructor and writer from India. the other element of the mixture are non-magnetic.
149. (b) Ryder Cup is a biennial men’s gold competition 173. (b) Amartya Kumar Sen, (born 3 November 1933) is an
between teams from Europe and the United States. Indian economist and a Nobel laureate. He has made
150. (d) Kanha Tiger Reserve, also called “Kanha National contributions to welfare economics, social choice
Park” is one of the tiger reserve of India and the largest theory, economic and social justice, economic theories
national Park of Madhya Pradesh. of famines, and indexes of the measure of well-being of
151. (b) The Discovery of India was written by India’s first citizens of developing countries. He was awarded the
Prime Minister Jawaharlal Nehru during his imprisoment Nobel Memorial Prize in Economic Sciences in 1998 for
in 1942-46 at Ahmednagar fort in Maharashtra. his work in welfare economics.
152. (b) Kiran Bedi was the first woman IPS officer in India. 174. (a) The Great Depression was a severe worldwide
Bedi joined the Indian Police Service (IPS) in 1972. economic depression in the decade preceding World
153. (a) ILO stands for International Labour Organisation which War II. The timing of the Great Depression varied
is a United Nations agency dealing with Labour issues. across nations, but in most countries it started in 1930
154. (c) The Times of London in 2010 Published its 70,000th and lasted until the late 1930s or middle 1940s. It was
issue. the longest, most widespread, and deepest depression
155. (b) Project Tiger was launched in 1973 by the Government of the 20th century.
of India during Prime Minister Indira Gandhi’s tenure. 175. (b) The Indian Foreign Service is the foreign service under
156. (d) According to the figures compiled by Media Research Group A and Group B of the Central Civil Services of
Users Council (MRUC) in the Indian Readership the executive branch of the Government of India. It is
Survey (IRS) 2014, the Indian newspaper with the the body of career diplomats of India. The Indian
largest readership is the Dainik Jagran. Foreign Service is part of the Central Civil Services of
157. (d) International Yoga day is celebrated annually on June the Government of India. The Foreign Secretary of India
21. is the administrative head of the Indian Foreign Service.
158. (d) World AIDS Day is observed on 1st December, to raise 176. (c) Sethusamudram Shipping Canal Project is a proposed
awareness of the AIDS pandemic caused by the spread project that would link Palk Bay and the Gulf of Mannar
of HIV infection. between India and Sri Lanka by creating a shipping
159. (c) Trygve Lie was the first secretary General of UNO. channel through the shallow sea called Sethusamudram
and through a chain of islands collectively called
w
w
w
.y
ou
110 GENERAL KNOWLEDGE

rs
m
Adam's Bridge, Ramar Palam , Ram Sethu and similar 185. (c) Maharashtra is the second most populous state as per

ah
bo
names. This would provide a continuously navigable census 2011 after Uttar Pradesh.

ob
sea route in and around the Indian Peninsula. 186. (a) 187. (c)

.w
177. (d) A metropolitan area, sometimes referred to as a metro 188. (c) Santosh Yadav is an Indian mountaineer. She is the

or
dp
area or metro, is a region consisting of a densely first woman in the world to climb Mount Everest twice

re
populated urban core and its less-populated in less than a year. She first climbed the peak in May

ss
surrounding territories, sharing industry, infrastructure,

.c
1992 and then did it again in May 1993.

om
and housing. A metropolitan area usually comprises 189. (c) Roop Singh Bais was an Indian hockey player. He was
multiple jurisdictions and municipalities: part of the celebrated Indian field hockey team which
neighborhoods, townships, cities, exurbs, countries, won gold medals for India at 1932 and 1936 Olympic
and even states. Games. After the final of the 1936 Summer Olympics,
178. (b) The Kyoto Protocol to the United Nations Framework Germans were so impressed with him that they named
Convention on Climate Change (UNFCCC) is an a street after him.
international treaty that sets binding obligations on 190. (a) In 1981 with the first Indian expedition to Antarctica.]
industrialised countries to reduce emissions of The program gained global acceptance with India's
greenhouse gases. The UNFCCC is an environmental signing of the Antarctic Treaty and subsequent
treaty with the goal of preventing "dangerous" construction of the Dakshin Gangotri Antarctic
anthropogenic (i.e., human-induced) interference of the research base in 1983 superseded by the Maitri base
climate system. from 1990.
179. (a) Sanjukta Panigrahi (24 August 1944 - 24 June 1997) [1] 191. (a) 192. (c)
was a dancer of India, who was the foremost exponent 193. (c) Rajasthan has the dubious distinction of having the
of Indian classical dance Odissi. Sanjukta was the first highest difference in male-female literacy in the country
Oriya girl to embrace this ancient classical dance at an at 28%, against the national average of 16.7%.
early age and ensure its grand revival.
194. (d) Azadirachta indica, also known as Neem, is the state
180. (c) The father of the White Revolution in India was tree of Andhra Pradesh. Products made from Neem trees
Verghese Kurien. The white mentioned is milk have been used in India for over two millennia for their
production. He was able to implement programs that medicinal properties: Neem products are believed to
took India from a country with very little dairy be anthelmintic, antifungal, antidiabetic. antibacterial,
production to the world's largest producer. antiviral, contraceptive and sedative.
181. (b) Charles Robert Darwin, FRS (12 February 1809 - 19 195. (b) The Geological Survey of India (GSI), established in
April 1882) was an English naturalist and geologist, 1851, is a government organization in India which is an
best known for his contributions to evolutionary theory. office attached to the Ministry of Mines of Union
He established that all species of life have descended Government of India for conducting geological surveys
over time from common ancestors, and in a joint and studies.
publication with Alfred Russel Wallace introduced his
196. (c) Sunmeet Kaur Sawhney, a Punjabi woman, is the winner
scientific theory that this branching pattern of evolution
of Rs 5 crore on TV game show Kaun Banega
resulted from a process that he called natural selection,
Crorepati 6.
in which the struggle for existence has a similar effect
197. (c) Kapil Dev's fast pace earned him the nickname,
to the artificial selection involved in selective breeding.
'Haryana Hurricane'.
182. (b) Chandrayaan-1 was India's first lunar probe. It was
198. (a)
launched by the Indian Space Research Organisation
in October 2008, and operated until August 2009. 199. (a) Arya Samaj is an Indian religious reform movement
founded by Swami Dayananda on 7th April 1875.
183. (d) Lira was the currency of Italy between 1861 and 2002.
Between 1999 and 2002, the Italian lira was officially a 200. (a) 201. (d) 202. (c) 203. (a) 204. (a)
national subunit of the euro. 205. (b) 206. (b) 207. (d) 208. (c) 209. (a)
184. (c) The President of India is the Supreme Commander of 210. (c) 211. (d) 212. (c) 213. (d) 214. (a)
the Indian Armed Forces. The Indian Armed Forces 215. (a) 216. (b) 217. (b) 218. (d) 219. (b)
are under the management of the Ministry of Defence 220. (b) 221. (d) 222. (d) 223. (d) 224. (a)
(MoD), which is led by the Union Cabinet Minister of 225. (c) 226. (d) 227. (d) 228. (d)
Defence.
w
w
w
.y
ou
rs
m
ah
bo
ob
PRACTICE SET - 1

.w
or
dp
re
ss
.c
om
1. Which of the following symbiotic associations forms a 9. Which of the following is not a laid down principle of the
lichen? Panchsheel ?
(a) An algae and a fungus (a) Mutual respect for each other's territorial
(b) An algae and a bryophyte integrity
(c) A bacterium and a fungus (b) Mutual non-aggression
(d) A bacterium and a gymnosperm (c) Mutual support for each other in world
2. The headquaters of which one of the following organizations forum
(d) Mutual non-interference in each other's
is not in Geneva?
internal affairs
(a) Food and Agricultural Organisation
10. Denatured alcohol
(b) World Meteorological Organisation (a) is a form of alcohol
(c) World Health Organisation (b) is unfit for drinking as it contains poisonous
(d) World Trade Organisation substances
3. The opportunity cost of a factor of production is (c) contains coloured impurities
(a) what it earns in its present use. (d) is sweet to taste
(b) what it can earn in the long period. 11. The city of Prayag was named Allahabad - the city of Allah
(c) what it can earn in some other use. by
(d) the cost of production. (a) Aurangzeb
4. Which Amendment Act is referred as mini constitution? (b) Akbar
(a) 7th Constitutional Amendment Act, 1956 (c) Shahjahan
(b) 24th Constitutional Amendment Act, 1971 (d) Bahadur Shah Zafar
(c) 42nd Constitutional Amendment Act, 1976 12. Arrange the following historical events chronologically
(d) 44th Consitutional Amendment Act, 1978 choosing the correct response:
I. French Revolution
5. Inflation is caused by
II. Glorious Revolution
(a) decrease in production
III. American War of Independence
(b) increase in money supply and decrease in
IV. Russian Revolution
production (a) I II III IV
(c) increase in money supply (b) II III I IV
(d) increase in production (c) II I IV III
6. The equilibrium of a firm under perfect competition will be (d) III II I IV
determined when 13. Chromosomes are made up of
(a) Marginal Cost > Average Cost (a) DNA (b) Protein
(b) Marginal Revenue > Average Cost (c) DNA and Protein (d) RNA
(c) Marginal Revenue > Average Revenue 14. While the computer executes a program, the program is held
(d) Marginal Revenue = Marginal Cost in
7. Which one of the following cities and the personalities (a) RAM (b) ROM
associated with their establishment is wrongly matched? (c) Hard Disk (d) Floppy Disk
(a) Calcutta – Robert Clive 15. Presidential form of government consists of the following?
(b) Pondicherry – Francis Martin (a) Popular election of the President
(c) Ahmedabad – Ahmad Shah I (b) No overlap in membership between the executive and
(d) Madras – Francis Day the legislature
8. Arihant is a (c) Fixed term of office
(a) Multi barrel rocket launcher (d) All of the above
(b) Airborne Early Warning and Control 16. Which of the following places of Sikh religious heritage is
System not in India?
(c) Unmarmed Combat Aerial Vehicle (a) Nankana Sahib (b) Nanded
(d) Nuclear-powered ballistic missile submarine (c) Paonta Sahib (d) Keshgarh Sahib
w
w
w
.y
ou
PS-2 Practice Set-1

rs
m
17. The total population divided by available arable land area is 22. In human body, vitamin A is stored in the –

ah
bo
referred to as (a) liver (b) skin

ob
(a) Population density (c) lung (d) kidney

.w
(b) Nutritional density 23. Ondometer is a –

or
dp
(c) Agricultural density (a) Measuring instrument for distance covered by motor

re
(d) Industrial density wheels

ss
.c
18. The danger signals are red while the eye is more sensitive to (b) Measuring instrument for frequency of

om
yellow because electromagnetic waves
(a) absorption in red is less than yellow and hence red is (c) Device for measuring sound intensity
visible from a distance (d) Measuring instrument for electric power
(b) scattering in yellow light is less than red 24. Which acid is used in rubber, textile, leather and
(c) the wavelength of red light is more than yellow light electroplating industries ?
(d) none of the above reasons (a) Ethanoic acid
19. Who was the author of "India of My Dreams" ? (b) Methanoic acid
(a) J.B. Kripalani (b) M.K. Gandhi (c) Malanic acid
(c) G.K. Gokhale (d) Jawaharlal Nehru (d) Butairic acid
20. How many players are there in a Polo team ? 25. What is the theme of 2016 National Statistics Day?
(a) 4 (b) 7 (a) Social Development
(c) 8 (d) 6 (b) Trees and their calcualtive lives
21. Hemophilia is – (c) Human empowerment
(a) caused by bacteria (b) caused by virus
(d) Agriculture and Farmers’ welfare
(c) caused by pollutants (d) a hereditary defect
w
w
w
.y
ou
Practice Set-1 PS-3

rs
m
ah
HINTS & EXPLANATIONS

bo
ob
.w
or
dp
re
1. (a) 2. (a) 3. (c) 4. (c) 5. (b) 12. (b) 13. (c) 14. (a) 15. (d) 16. (a)

ss
.c
6. (d) 7. (a) 17. (a)

om
8. (d) Arihant is a Nuclear powered ballistic missile submarine. 18. (c) This is because the scattering in red light is less than
9. (c) Option (c) is not in the list of principles of panchsheel. that of yellow colour. The longest visible wavelength
This agreement stated the five principles as: is red and the shortest is violet. The wavelength of red
1. Mutual respect for each other's territorial integrity
light is more than yellow light.
and sovereignty.
19. (b)
2. Mutual non-aggression.
20. (a) There are 4 players in a polo team.
3. Mutual non-interference in each other's internal
affairs. 21. (d) Hemophilia is a hereditary defect.
4. Equality and cooperation for mutual benefit. 22. (a) In human body, vitamin A is stored in the liver.
5. Peaceful co-existence. 23. (b) Ondometer is a measuring instrument for frequency
10. (b) of electromagnetic waves.
11. (b) Emperor Akbar named Prayag as Allahabad - City of 24. (b) Methanoic acid is a colorless, pungent smelling liquid
God- also called Allahabad in 1575 AD. The city of with a boiling point 373.5 K. Due to the presence of
Allahabad is situated at the confluence of three rivers aldehyde-like hydrogen, it is powerful reducing
- Ganga, Yamuna and the invisible Saraswati. Every agent.It reduces Tollen's reagent and Fehling's solution.
12th year when the waters are felt to be especially It is used in rubber, textile, dyeing, leather and
purifying, Allahabad holds a much greater festival called electroplating industries.
Kumbh Mela. Built by Emperor Akbar in 1583 AD, the 25. (d)
Allahbad fort stands on the banks of the river Yamuna
near the confluence site i.e SANGAM.
w
w
w
.y
ou
PS-4 Practice Set-2

rs
m
ah
bo
ob
PRACTICE SET - 2

.w
or
dp
re
ss
.c
om
1. Which Article of the Indian Constitution guarantees rights 12. Tattvabodhini Sabha was founded by ………....... In 1839
to arrested persons ? (a) Swami Vivekanand
(a) Article 22 (b) Article 35 (b) Keshav Chandra Sen
(c) Article 20 (d) Article 42
(c) Dabendranath Tagore
2. For a person having hypermetropia, the near point is
(d) Swami Sahajanamd
…………………..
13. After the revolt of 1857, British pursued the policy of
(a) Greater than 20 cm (b) Lesser than 25cm
(c) Greater than 25cm (d) Lesser than 30cm ……………
3. Cryogenic is a science deals with (a) Divide and Policies
(a) High Temperatures(b) Low Pressure (b) Rules and Regulation
(c) High Pressure (d) Low Temperature (c) Divide and Rule
4. ..................... is an active factor of production (d) Unity and Poliy
(a) Product (b) Labour 14. Prithvi-I missile was inducted into the ..................in 1994
(c) Wages (d) Price (a) Indian Army (b) Indian Air Force
5. When total utility becomes maximum, then marginal utility (c) Indian Navy (d) All of these
will be 15. 2018 FIFA World Cup to be held in …………….
(a) Maximum
(a) China (b) Russia
(b) Minimum
(c) India (d) Brazil
(c) Either maximum or minimum
16. .…………………is issued by the court in case of illegal
(d) Zero
detention of a person
6. Revealed Preference Theory was propounded by
……………….. (a) Quo Warranto (b) Habeas Corpus
(a) Robbins (b) Smith (c) Mandamus (d) Certiorari
(c) Samuelson (d) Schumpter 17. At the time of Emergency, the Indian State become unitary
7. One Carat of diamond is equal to ..…………… from …………………….
(a) 200m (b) 100m (a) Semi Federal (b) Federal
(c) 150m (d) 300m (c) Unitary (d) Quasi-federal
8. Wood Spirit is which of the following ? 18. The book titled 'The Life and Death of Adolf Hitler' is penned
(a) Ethyl Alcohol (b) Propanol by
(c) Methyl Alcohol (d) Butanol (a) Z.A. Bhutto
9. Which of the following is chief source of Napthalene ? (b) James Cross Giblin
(a) Moth balls (b) Mothflakes
(c) J.M. Barrie
(c) Tar Camphor (d) Coal tar
(d) Gunnar Myrdal
10. Study of crop production is ………………
19. Bos Taurus is a scientific name of ………………..
(a) Entology (b) Ecology
(c) Botany (d) Agronomy (a) Buffalo (b) Horse
11. Who was the last guru of the Sikhs ? (c) Cow (d) Cat
(a) Guru Granth Sahib 20. …………….. includes all prokaryotic organism likes bacteria,
(b) Guru Gobind Singh cynobacterioa and archiobacteria
(c) Guru Angad (a) Animalia (b) Protista
(d) Guru Amar Das (c) Monera (d) Planatae
w
w
w
.y
ou
Practice Set-2 PS-5

rs
m
ah
21. …………………… is the Kuchipudi dancer 24. World's largest Charkha (spinning wheels) that was unveiled

bo
(a) Anupama Mohan at Terminal 3 of the Indira Gandhi International Airport (IGI),

ob
(b) Bimbavati Devi New Delhi is made of the teak wood of which country?

.w
or
(c) Arush Mudgal (a) Burma (b) Sri Lanka

dp
(c) Nepal (d) Ukraine

re
(d) Swapnasundari

ss
22. Which of the following is not a chief organ of the United 25. The Union Cabinet recently approved The High Courts

.c
om
Nations Organisations? (Alteration of Names) Bill, 2016 to be introduced in the
(a) International Labour Organisation Parliament. The bill will facilitate the changing of the names
(b) Security Council of which two high courts?
(c) International Court of Justice (a) Bombay High Court and Calcutta High Court
(d) General Assembly (b) Bombay High Court and Madras High Court
23. The treaty of Mangalore was signed between (c) Calcutta High Court and Madras High Court
(a) the English East India Company and Haidar Ali (d) Bombay High Court and Gauhati High Court
(b) the English East India Company and Tipu Sultan
(c) Haidar Ali and the Zamorin of Calicut
(d) the French East India Company and Tipu Sultan
w
w
w
.y
ou
PS-6 Practice Set-2

rs
m
ah
HINTS & EXPLANATIONS

bo
ob
.w
or
dp
re
1. (a) Article 22 proceeds to guarantee certain fundamental 13. (c) After the revolt, the British pursued the policy of divide

ss
.c
rights to every arrested person. and rule, towards the general populace.

om
2. (c) Hypermetropia Myopia is corrected by spectacles 14. (a) Prithvi (Sanskrit: prthvi "Earth") is a tactical surface-
having concave lens.Near point of aperson suffering to-surface short-range ballistic missile .This class of
from hypermetropia is more than 25cm. Prithvi missile was inducted into the Indian Army in
3. (d) Cryogenics is the study of the production and 1994
behaviour of materials at very low temperatures. 15. (b) FIFA's decision to award Russia the right to host the
2018 World Cup surprised many - including some of
4. (b) Some of the important factors of production are: (i) Land
the country's leaders
(ii) Labour (iii) Capital (iv) Enterprnuer. Land is a passive
factor whereas labour is an active factor of production 16. (b) Habeas corpus ("You may have the body") is a recourse
in law whereby a person can report an unlawful
5. (d) When total utility is maximum at the 5th unit, marginal
detention or imprisonment before a court, usually
utility is zero
through a prison official
6. (c) Revealed preference theory, pioneered by American
17. (d) Professor K.C. Wheare, who regards the American
economist Paul Samuelson, is a method of analyzing
constitution as the model of a true federation has
choices made by individuals
described the Indian constitution as 'quasi federal', that
7. (a) One carat is equal to 0.2 grams is 'a unitary state with subsidiary federal features rather
8. (c) Wood spirit is a poisonous colorless liquid used as a than a federal state with subsidiary unitary features
solvent and fuel; ingestion may cause blindness or
18. (b) Many people believe Hitler was the personification of
death. Called also methyl or wood alcohol.
evil. In this Sibert Medal-winning biography, James
9. (d) Naphthalene is an organic compound with formula C Cross Giblin penetrates this façade and presents a picture
10H 8. It is the simplest polycyclic. Naphthalene is the of a complex person-at once a brilliant, influential
most abundant single component of coal tar. politician and a deeply disturbed man. Giblin explores
10. (d) Agronomy is the science and technology of producing the forces that shaped the man as well as the social
and using plants for food, fuel, fiber, and land conditions that furthered his rapid rise to power.
reclamation 19. (c) Cows are raised in many different countries around
11. (b) Guru Gobind Singh was The Tenth Nanak or the last of the world, mainly for the cowsnatural resources such
the Sikhpreachers to live. as milk, meat
12. (c) The Tattwabodhinl Sabha ("Truth Propagating/ 20. (c) Monera Kingdom- All the organisms of this kingdom
Searching Society") was a group started in Calcutta on 6 are prokaryotes
October 1839 as a splinter group of the Brahmo Samaj,
21. (a) Anupama Mohan is one of the best-known disciples
reformers of Hinduism and Indian Society. The founding
of Kuchipudi.
member was Debendranath Tagore
22. (a) 23. (b) 24. (a) 25. (b)
w
w
w
.y
ou
Practice Set-3 PS-7

rs
m
ah
bo
ob
PRACTICE SET - 3

.w
or
dp
re
ss
.c
om
1. Which one of the following is not a computer language? 11. Which one of the following Schedules of the Constitution
(a) Cobol (b) Visual Basic of India includes the disqualification of a Legislator on
(c) HTML (d) Netscape grounds of defection?
2. Who among the following was the first Governor General of (a) 8th Schedule (b) 7th Schedule
India? (c) 9th Schedule (d) 10th Schedule
(a) Lord Amherst 12. Which one of the following causes the chikungunia
(b) Lord William Bentinck disease?
(c) Sir Charles Metcalfe (a) Bacteria (b) Helminthic worm
(d) Robert Clive (c) Protozoan (d) Virus
3. Which one of the following is not a constituent of biogas? 13. Who among the following recommends to the Parliament
(a) Methane (b) Carbon dioxide for the abolition of the Legislative Council in a State?
(c) Hydrogen (d) Nitrogen dioxide (a) The President of India
4. In which one of the following sessions was the Indian (b) The Governor of the concerned State
National Congress split into moderates and extremists? (c) The Legislative Council of the concerned State
(a) Nagpur (b) Allahabad (d) The Legislative Assembly of the concerned State
(c) Surat (d) Calcutta 14. Which one of the following vitamins helps in clotting of
5. Bar is a unit of which one of the following? blood?
(a) Force (b) Energy (a) Vitamin-A (b) Vitamin-B6
(c) Pressure (d) Frequency (c) Vitamin-D (d) Vitamin-K
6. Which of the following metals are present in 15. The ‘Thomas Cup is associated with
haemoglobin and chlorophyll, respectively? (a) Table Tennis (b) Lawn Tennis
(a) Fe and Mg (b) Fe and Zn (c) Badminton (d) Billiards
(c) Mg and Zn (d) Zn and Mg 16. Which one among the following pairs is correctly matched?
7. A mother of blood group O has a group O child. What could (a) The Second Battle : Defeat of Jaichand
be the blood group of father of the child? of Tarain of Kannauj
(a) Only O (b) A or B or O by Muhammad of Ghori
(c) A or B (d) Only AB (b) The First Battle of : Defeat of Sikander
8. Who among the following was the founder of the Muslim Panipat Lodhi by
League? Babur
(a) Muhammad Ali Jinnah (c) The Battle of : Defeat of Humayun
(b) Shaukat Ali Chausa by Sher Shah
(c) Nawab Salimullah (d) The Battle of : Defeat of Rana
(d) Aga Khan Khanwa Pratap by Akbar
9. Which one among the following is not a source of tax 17. What is the purpose of adding baking soda to dough?
revenue for the Central Government in India ? (a) To generate moisture
(a) Income tax (b) Customs duuties (b) To give a good flavour
(c) Service tax (d) Motor Vehicle tax (c) To give good colour
10. Which of the following does not form part of current account (d) To generate carbon dioxide
of Balance of Payments? 18. The ‘Arthasastra’ is a treatise on which one of the following?
(a) Export and import of goods (a) Economics
(b) Export and import of services (b) Environment
(c) Income receipts and payments (c) Political Philosophy
(d) Capital receipts and payments (d) Religion in Administration
w
w
w
.y
ou
PS-8 Practice Set-3

rs
m
19. Which one of the following glands in the human body stores 23. What is the recently extended deadline for states to join

ah
bo
iodine? Union Government's Ujwal Discom Assurance Yojana

ob
(a) Parathyroid (b) Thyroid (UDAY) scheme?

.w
(c) Pituitary (d) Adrenal

or
(a) March 2017

dp
20. India's first integrated Defense Communication Network
(b) January 2018

re
(DCN) has been launched in which of the following cities?

ss
(c) March 2018

.c
(a) New Delhi (b) Lucknow

om
(c) Ahmedabad (d) Kochi (d) January 2017
21. The book "The mind of the terrorist: the Psychology of 24. As of now, how many countries are members of Nuclear
terrorism from the IRA to al-Qaeda" has been authored by Suppliers Group (NSG)?
whom? (a) 48 (b) 56
(a) Amitav Ghosh (c) 64 (d) 96
(b) Ashwin Sanghi 25. India's first AYUSH university will be set up in which state
(c) Kunal Basu of India?
(d) Jerrold M. Post (a) Sikkim
22. The Government of India (GoI) has extended the ban on the
(b) Haryana
import of milk and milk products of China till which year?
(c) Karnataka
(a) June 2017 (b) March 2017
(d) West Bengal
(c) December 2017 (d) January 2017
w
w
w
.y
ou
Practice Set-3 PS-9

rs
m
ah
HINTS & EXPLANATIONS

bo
ob
.w
or
dp
1. (d) Netscape is an Internet browser that was popular 14. (d) Vitamin-K adds in blood clotting. Vitamin-K acts as an

re
ss
during the early 1990's. essential cofactor for factor-II, VII, IX, X and also for

.c
proteins Z, C and S.

om
2. (a)
3. (d) Nitrogen dioxide (NO2) is not a component of biogas. 15. (c) Thomas Cup is associated with Badminton.
4. (c) The 23rd Session (1907) of the Congress was held at 16. (b) First Battle of Panipat (1526) was fough between two
Surat.In the session, there was an open clash between mega-powers- Babur, then ruler of Kabul and Ibrahim
the Moderates and the Extremists and ultimately it led Lodhi, king of Delhi Sultanate. It was fought near
to a split in the Congress. Panipat (present day Haryana). Babur won the battle
5. (c) 1 Bar = 105 Pa. Both bar and Pa are the unit of pressure. and established the Mughal Empire.Second Battle of
6. (a) Fe and Mg metals are present in haemoglobin and Panipat (1556) was fought between Akbar (Ruler of
chlorophyll respectively. Mughal Dynasty) and Muhammad Adil Shah (ruler of
7. (b) The blood group of father of the child could be A or B Pashtan Suri Dynasty), along with his Prime Minister
or O. Hemu. Third Battle of Panipat (1761) was fought
8. (c) The All India Muslim League, a political organization between the Afghans and the Marathas. The battle
was founded in 1906 by Aga Khan under the Nawab of lasted for two months which ultimately resulted in the
Dhaka Salimullah. Its main purpose was to safeguard defeat of Marathas and end of their dominance in
the political rights of Muslims in India. India.
9. (d) Motor Vehicle tax is not a source of tax revenue for the 17. (d) Baking soda has sodium bicarbonate as the chief
central government in India. constituent. It decomposes on heating giving carbon
10. (d) Capital receipts and payments do not form part of dioxide. This causes dough, cakes, biscuits etc. to
current account of Balance of Payment. expand and become light.
11. (d) The 10th Schedule to the Indian Constitution is known 18. (c) The Arthasastra is a treatise on Political philosophy.
as Anti-Defection Law. It was inserted by the 52nd The book, written in Sanskrit, discusses theories and
Amendment Act 1985 to the Constitution. It sets the principles of governing a state. The meaning
provisions for disqualification of elected members on ofArthashastrais 'Science of Polity'. It is written by
the grounds of defection to another political party. Kautilya.
12. (d) Chikungunia is caused by chikenguniya virus which 19. (b) Thyroid gland in human body contains iodine.
is an insect borne virus of genus Alphavirus. Deficiency of iodine creates goitre disease. Which is
Symptoms show high fever, maculopapular rash, observed by the enlargement of larynx.
headache, etc. 20. (a) 21. (d) 22. (b) 23. (a)
13. (d) 24. (a) 25. (b)
w
w
w
.y
ou
PS-10 Practice Set-4

rs
m
ah
bo
ob
PRACTICE SET - 4

.w
or
dp
re
ss
.c
om
1. Which one of the following is a programme that converts 10. In the human body, Cowper's glands form a part of which
high level language to machine language? one of the following system?
(a) Linker (b) Assembler (a) Digestive system
(c) Interpreter (d) Compiler (b) Endocrine system
2. Classification of an enterprise into public or private sector (c) Reproductive system
is based on (d) Nervous system
(a) number of employees in the enterprise 11. Mist is a result of which one of the following
(b) ownership of assets of the enterprise (a) Condensation
(c) employment conditions for workers in the enterprise (b) Evaporation
(d) nature of products manufactured by the enterprise (c) Sublimation
3. Which one of the following glands produces the growth (d) Saturation
hormone (somatotrophin)? 12. ‘Dyarchy’ was first introduced in India under
(a) Adrenal (b) Pancreas (a) Morley-Minto reforms
(c) Pituitary (d) Thyroid (b) Montford reforms
4. Who among the following was elected as the President of (c) Simon Commision plan
All India Khilafat Conference met at Delhi in 1919? (d) Government of India Act, 1935
(a) Motilal Nehru (b) Mahatma Gandhi 13. When Lord Mountbatten became the first Governor-General
(c) M A Jinnah (d) Shaukat Ali of India, who among the following became the Governor-
5. Which one of the following Indian states does not have a General for Pakistan?
common international border with Bangladesh? (a) Lord Mountbatten
(a) Manipur (b) Paschim Banga (b) Muhammad Ali Jinnah
(c) Tripura (d) Asom (c) Liaquat Ali Khan
6. Who among the following is the author of the book. ‘The (d) Shaukat Ali
Namesake’? 14. Fiscal Policy in India is formulated by
(a) Arundhati Roy (b) Amitava Ghosh (a) the Reserve Bank of India
(c) Jhumpa Lahiri (d) Kiran Desai (b) the Planning Commission
7. Who among the following was not a member of the (c) the Finance Ministry
Constituent Assembly? (d) the Securities and Exchange Board of India
(a) Sardar Vallabhbhai Patel 15. Fat can be separated from milk in a cream separation because
(b) Acharya JB Kriplani of
(c) Lok Nayak Jayprakash (a) cohesive force
(d) K M Munshi (b) gravitational force
8. Carbon dioxide is called a greenhouse gas because (c) centrifugal force
(a) its concentration remains always higher than other (d) centripetal force
gases 16. The average fixed cost curve will always be
(b) it is used in photosynthesis (a) a rectangular hyperbola
(c) it absorbs infrared radiation . (b) a downward sloping convex to the origin curve
(d) it emits visible radiation (c) a downward sloping straight line
9. Laser is a device to produce (d) a U-shaped curve
(a) a beam of white light
17. Malaria in the human body is caused by which one of the
(b) coherent light following organisms?
(c) microwaves (a) Bacteria (b) Virus
(d) X-rays (c) Mosquito (d) Protozoan
w
w
w
.y
ou
Practice Set-4 PS-11

rs
m
18. The focal length of convex lens is 22. Mohammad Shahid is associated with which sports?

ah
bo
(a) the same for all colours (a) Hockey (b) Badminton

ob
(b) shorter for blue light than for red (c) Wrestling (d) Boxing

.w
(c) shorter for red light than for blue

or
23. KG Subramanyan, who passes away recently, was a famous

dp
(d) maximum for yellow light personality of which field?

re
19. The Name of Ram Prasad Bismil is associated with

ss
(a) Art (b) Journalism

.c
(a) Kanpur Conspiracy Case (c) Sports (d) Politics

om
(b) Alipore Conspiracy Case 24. Who has won the 2016 wins Iceland's presidential election?
(c) Kakori Conspiracy Case (a) Davíd Oddsson
(d) Meerut Conspiracy Case
(b) Olafur Ragnar Grimsson
20. The Indian Research Station 'Himadri' is located at
(c) Gudni Johannesson
(a) Siachen (b) Darjeeling
(d) Andri Snaer Magnason
(c) Arctic Region (d) Antarctica
25. Which of the following countries has become the newest
21. What is the theme of the first-ever "National Yoga Olympiad",
member of the Missile Technology Control Regime (MTCR)?
which has been organized by the NCERT?
(a) Canada (b) India
(a) Yoga for Peace and Dhyana
(c) Brazil (d) Poland
(b) Yoga for Health and Harmony
(c) Yoga fo r Kriya and Harmony
(d) Yoga for Emotional and Mental development
w
w
w
.y
ou
PS-12 Practice Set-4

rs
m
ah
HINTS & EXPLANATIONS

bo
ob
.w
or
dp
re
1. (d) A compiler is a special programme that processes 13. (b) Mohammed Ali Jinnah was Indian Muslim politician,

ss
statements written in a particular programming founder and first governor-general (1947-48) of

.c
om
language and turns them into machine language or Pakistan. As the first Governor-General of Pakistan,
"code" that a computer's processor uses. Jinnah worked to establish the new nation's
2. (b) A company organized for commercial purposes is government and policies, and to help settle the millions
called an enterprise. Classification of an enterprise into of Muslim migrants who had emigrated from the new
public or private sector is based on ownership of nation of India to Pakistan after the partition. He is
assets of the enterprise. revered in Pakistan as Quaid-i-Azam.
3. (c) Somatotrophin is produced by the anterior pituitary. 14. (c) The Department of Economic Affairs (DEA) under
It is a peptide hormone that induces growth, cell Ministry of Finance is the nodal agency of the Union
reproduction and regeneration. Government to formulate and monitor country's
4. (b) Gandhiji was elected President of the All-India Khilafat
economic policies and programmes having a bearing
Conference which met at Delhi on November 23, 1919.
on domestic and international aspects of economic
They decided to withdraw all cooperation from the
management.
government if their demands were not met.
5. (a) Manipur does not have boundary with Bangladesh. 15. (c) Centrifugal force separates fat from milk.
6. (c) The Namesake (2004) is the first novel by Jhumpa 16. (a) Total fixed costs are constant, so the average fixed
Lahiri. cost curve diminishes with the output. Thus, the
7. (c) The Constitution of India was drafted by the average fixed cost curve is a rectangular hyperbola.
constituent assembly and it was set up under the 17. (d) Malaria is a mosquito borne disease of humans and
cabinet Mission plan on 16 May 1946. The members other animals caused by Plasmodium protozoan.
of the constituent assembly were elected by the Severe disease is largely caused by Plasmodium
Provincial assemblies by method of single transferable falciparum whereas mild forms are due to P vivax, P
vote system of proportional representations.Members oval and P malariae.
of the committee: Sardar Vallabhbhai Patel, K. M. 18. (b) The focal length of a convex lens is shorter for blue
Munshi,Acharya J.B kriplani . Lok Nayak Jai Prakash light than for red.
was not the member of the constituent assembly. 19. (c) Ram Prasad Bismil was the famous freedom fighter
8. (c) Greenhouse gases catch the sun's radiation on its way who was involved in the historic Kakori train robbery.
back into space and reflect some of that warmth back He was born in 1897 at Shahjahanpur,Uttar Pradesh.
to Earth, increasing temperatures. Carbon dioxide is On 9th August, 1925, Ram Prasad Bismil along with
known as greenhouse gas because of their ability to his fellow followers looted the money of the British
trap and reflect the sun's radiation back to Earth. government from the train while it was passing through
9. (b) A laser is a device that emits coherent light through a
Kakori, Lucknow. Except Chandrashekhar Azad, all
process called stimulated emission.
other members of the group were arrested. Ram Prasad
10. (c) Cowper’s gland is related to reproductive system.
Bismil along with others was given capital punishment.
Cowper’s gland is the bulbourethal gland found in
human males. They are found in pair and secrete This great freedom fighter of India was executed on
viscous secretion called pre ejaculate that helps in 19th December, 1927.
coitus. 20. (c) Himadri Station is India's first Arctic research station
11. (a) Mist is a thin fog resulting from condensation in the located at Spitsbergen, Svalbard, Norway. It was
air near to the earth's surface. inaugurated on the 1st of July, 2008 by the Minister of
12. (a) Dyarchy was a system of double government Earth Sciences.
introduced by British India. 21. (b) 22. (a) 23. (a) 24. (c) 25. (a)
w
w
w
.y
ou
Practice Set-5 PS-13

rs
m
ah
bo
ob
PRACTICE SET - 5

.w
or
dp
re
ss
.c
om
1. When had Muslim league passed the resolution "Divide 10. According to the Constitution of India, the Right to
and Quit" movement ? Property is a –
(a) 1945 (b) 1943 (a) Fundamental Right
(c) 1944 (d) None of these (b) Directive Principle
2. What is the ratio of money held by the public in currency to (c) Legal Right
that they held in deposit ? (d) Social Right
(a) The currency deposit ratio 11. Babar declared himself as an emperor first at –
(b) The reserve deposit ratio (a) Samarqand
(c) Cash reserve ratio
(b) Farghana
(d) Cash deposit ratio
(c) Kabul
3. The chemical behavior of an atom depends upon –
(d) Panipat
(a) the number of Neutrons in the nucleus
(b) the number of Nucleons in the nucleus 12. How many times has financial emergency been declared in
(c) the number of Protons in its nucleus India, so far?
(d) the number of Electrons orbiting around the nucleus (a) Five times (b) Four times
4. 88th amendement of the Indian Constitution is related to – (c) Once (d) Never
(a) The demarcation of new boundaries between states 13. Economy is in the "Liquidity Trap" when –
(b) The Constitution of the National Judicial Commission (a) Rate of interest on bonds is minimum
(c) Empowering the Centre to levy and appropriate Service (b) Rate of interest on bonds is maximum
tax (c) Transaction demand for money is maximum
(d) Readjustment of electroal constituencies on the basis (d) None of the above
of the population census 2001 14. Who is the author of "The Unseen Indira Gandhi"?
5. The joint sitting of both Houses of Indian Parliament is held (a) K.P. Mathur
in connection with – (b) Bilal Siddique
(a) Constitutional amendment bill (c) Anurag Mathur
(b) Ordinary bill (d) N.R. Narayana Murthy
15. What is 'biomagnification'?
(c) Money bill
(a) blowing up of environmental issues by man
(d) Election of the Vice – President of India
(b) growth of organisms due to food consumption
6. Many Fungi belonging to the genera Microporum (c) reduction of dissolved O2 caused by microbial
Trichophyton and Epidermophyton are responsible for – organisms
(a) Filarial (b) Cancer (d) increase in the concentration of nondegradable
(c) Ringworms (d) AIDS pollutants as they pass through food chain
7. A boat will submerge when it displaces water equal to its 16. Subhash Chandra Bose formed the government for
own – independent India in Singapore, on –
(a) volume (b) weight (a) 22nd September, 1943
(c) surface area (d) density (b) 20th October, 1943
8. Which organ of Human body is affected by Alzheimer (c) 21st October, 1943
(d) 22nd October, 1943
disease ?
17. 'Laffer Curve" shows the relationship between –
(a) Brain
(a) Government Revenue and Government Expenditure.
(b) Bone Marrow (b) Tax Rates and Tax Revenue.
(c) Lung (c) Direct Taxes and GDP.
(d) Intestine (d) None of the above
9. What is the chemical name of vitamin E ? 18. 'Cerebral palsy' is a brain disorder found generally in –
(a) Calciferol (a) Old people
(b) Tocopherol (b) Drug addicts
(c) Riboflavin (c) Small children
(d) Phylloquinone (d) Only in ladies
w
w
w
.y
ou
PS-14 Practice Set-5

rs
m
19. India is not a member of which of the following constituent

ah
23. Which of the following statements are NOT correct?

bo
organizations of the World Bank Group? (a) Prithvi-II is a Surface-to-Surface Missile

ob
(a) International Centre for Settlement of Investment (b) Prithvi-IIcan carry a 500 kg nuclear warhead

.w
Disputes (ICSID)

or
(c) Prithvi-IIhas a range of 350 KM

dp
(b) International Development Association (IDA)
(d) Akash is the naval variant of the Prithvi missile

re
(c) International Finance Corporation (IFC)

ss
(d) Multilateral Investment Guarantee Agency (MIGA) 24. The 2016 Copa America Football tournament has been won

.c
om
20. The mirror used in search light is – by which of the following countries ?
(a) Concave Mirror (b) Convex Mirror (a) Argentina
(c) Plane Mirror (d) None of these (b) Colombia
21. A.T.F. is related to – (c) Chile
(a) Civil Aviation (b) Railways (d) Peru
(c) Road transport (d) None of these
25. Which historical site has been declared as the SARRC
22. A hybrid computer is the one having the combined properties
cultural capital for 2016-17?
of
(a) Super and micro computers (a) Bamiyan
(b) Mini and micro computers (b) Maynamati
(c) Analog and digital computers (c) Shilaidah
(d) Super and mini computers (d) Mahashangarh
w
w
w
.y
ou
Practice Set-5 PS-15

rs
m
ah
HINTS & EXPLANATIONS

bo
ob
.w
or
dp
re
1. (b) The communal question had become a baffling one as 14. (a) The book “The Unseen Indira Gandhi” has been

ss
.c
the Muslim League tightened its demand for Pakistan. authored by Dr. KP Mathur, who was the personal

om
Against the congress demand of “quit India”, the physician of the former Prime Minister Indira Gandhi
Muslim League’s new slogan was “Divide and quit”. for nearly 20 years till her assassination in 1984. The
On March 21, 1943, Muslim League observed as foreword of the book was written by her granddaughter
Pakistan Day. Priyanka Gandhi Vadra. The book provides some
2. (a) The currency deposit ratio shows the amount of interesting peeps into the responses of Mrs. Gandhi’s
currency that people hold as a proportion of aggregate to challenges both personal and political.
deposits. An increase in cash deposit ratio leads to a 15. (d) Biomagnification, also known as bioamplification or
decrease in money multiplier. An increase in deposit biological magnification, is the increasing concentration
rates will induce depositors to deposit more, thereby of a substance, such as a toxic chemical, in the tissues
leading to a decrease in cash to Aggregate Deposit of organisms at successively higher levels in a food
ratio. chain.
This will in turn lead to a rise in Money Multiplier. 16. (c) On 21st October, 1943, Subhas Chandra Bose
3. (d) The chemical behaviour of an atom depends upon the proclaimed the formation of the Provisional
number of Electrons orbiting around the nucleus. Government of Free India at the Cathay Cinema Hall.
4. (c) 5. (b) 6. (c) Two days later, he declared war on Britain and the
7. (b) A boat will float when the weight of the water displaces United States. With help from the Japanese, he
equals the weight of the boat and anything will float if reorganised and rejuvenated the Azad Hind Fauj (also
it is shaped to displace its own weight of water before called the Indian National Army). He lobbied
it reaches the point where it will submerge. Floating of aggressively for funds in Malaya and other parts of
the boat works on the principle of buoyancy force which Southeast Asia and launched a recruitment drive for
is an upward force exerted by a liquid, gas or other the Azad Hind Fauj.
fluid, that opposes the weight of an immersed object.
17. (b) The Laffer curve, invented by Arthur Laffer, shows
8. (a) Alzheimer’s disease affects the brain. The disease
the relationship between tax rates and tax revenue
causes degeneration of brain tissues and nerve cells.
collected by governments. The chart below shows
9. (b) Chemical namee of Vitamin E is Tocopherols. the Laffer Curve:
10. (c) The Indian Constitution does not recognize the
property right as a fundamental right. In the year 1977,
the 44th amendment eliminated the right to acquire, hold
and dispose of property as a fundamental right.
However, in another part of the Constitution, Article
300 (a) was inserted to affirm that no person shall be
deprived of his property by the authority of law.
11. (d) Babur declared himself as the emperor at Panipat.
12. (d) Financial emergency in India has never been declared
so far.
13. (a) A liquidity trap is a situation, described in the 18. (c) Cerebral palsy (CP) is a group of permanent movement
Keynesian Economics, in which injections of cash into disorders that appear in early childhood. Signs and
the private banking system by a central bank fail to symptoms vary between people. Often, symptoms
dec rease interest rates and hence make monetary include poor coordination, stif f muscles, weak
policy ineffective. A liquidity trap is caused when muscles, and tremors. There may be problems with
people hoard cash because they expect an adverse sensation, vision, hearing, swallowing, and speaking.
event s uch as defl ati on, insufficient aggregate Often babies with cerebral palsy do not roll over, sit,
demand, or war. Common characteristics of a liquidity crawl, or walk as early as other children their age.
trap are interest rates that are close to zero and Difficulty with the a bi lity to think or reason and
fluctuations in the money supply that fail to translate seizures each occurs in about one third of people with
into fluctuations in price levels. CP.
w
w
w
.y
ou
PS-16 Practice Set-5

rs
m
19. (a) The World Bank Group consists of – India is a member of four of the five constituents of

ah
bo
- Internation al Bank for Reconstr uction and the World Bank Group viz., International Bank for

ob
Development (IBRD), established in 1945, which Reconstruction and Developmen t (IBRD),

.w
provides debt financing on the basis of sovereign International Development Association (IDA),

or
dp
guarantees; - International Finance Corporation (IFC), International Finance Corporation (IFC) and

re
established in 1956, provides various forms of Multilateral Investment Guarantee Agency (MIGA).

ss
.c
financing without sovereign guarantees, primarily to India is not a member of ICSID (International Centre

om
the private sector; for Settlement of Investment Disputes).
- International Development Association (IDA), 20. (d) A search light produces an intense parallel beam of
established in 1960, provides concessional financing light. This requires a reflector of large aperture. When
(interest-free loans or grants), usually with sovereign a source is placed at the focus of a large concave
guarantees; mirror only the paraxial rays (not the marginal rays)are
- International Centre for Settlement of Investment reflected as parallel beam, but when a source is placed
Disputes (CSID), established in 1965, which works at the focus of parabolic mirror all the rays are reflected
with governments to reduce investment risk; as an intense parallel beam.
- Multilateral Investment Guarantee Agency (MIGA), 21. (a) ATF is Aviation Turbine Fuel related to Civil Aviation.
established in 1988, which provides insurance against 22. (c)
certain types of risk, including political risk, primarily 23. (d) Dhanush is the naval variant of the Prithvi missile.
to the private sector. 24. (c) 25. (d)
om
.c
ss
re
dp
or
.w
ob
bo
ah
m
rs
ou
.y
Current Affairs & GK Update

w
w
w
w
w
w
2 Current Affairs & GK Update

.y
GK-

ou
rs
m
ah
bo
ob
.w
or
dp
re
ss
.c
om
w
w
w
Current Affairs & GK Update 3

.y
GK-

ou
rs
m
ah
bo
ob
.w
or
dp
re
ss
.c
om
w
w
w
4 Current Affairs & GK Update

.y
GK-

ou
rs
m
ah
bo
ob
.w
or
dp
re
ss
.c
om
w
w
w
Current Affairs & GK Update 5

.y
GK-

ou
BOOKS & AUTHORS

rs
m
The Legend of Lakshmi Prasad– Twinkle Khanna The Unseen Indira Gandhi–Dr KP Mathur

ah
bo
Driven: The Virat Kohli Story–Vijay Lokapally A Life in Diplomacy–Maharajakrishna Rasgotra

ob
Democrats and Dissenters–Ramachandra Guha Blood on my Hands: Confessions of Staged Encounters–

.w
One Indian girl–Chetan Bhagat Kishalay Bhattacharjee

or
dp
Six Machine (I Don’t Like Cricket I Love It)–Chris Gayle The Making of India: The Untold Story of British Enterprises–

re
His Bloody Project–Graeme Macrae Brunet Kartar Lalvani

ss
A State in Denial–BG Verghese

.c
AB The autobiography–AB De Villiers

om
Citizen and Society–Hamid Ansari The Kiss of Life–Emraan Hashmi
Anything But Khamosh: The Shatrughan Sinha Biography–
The Ocean of Churn: How the Indian Ocean Shaped Human
Bharathi S Pradhan
History–Sanjeev Sanyal
Gandhi: An Illustrated Biography–Pramod Kapoor
R D Burmania: Panchamemoirs–Chaitanya Padukone
Fixed! Cash and Corruption in Cricket–Journalist Shantanu
India Rising: Fresh Hope, New Fears–Ravi Velloor Guha Ray
The Great Derangement: Climate Change and the “Who was Shivaji?”–Govind Pansare
Unthinkable–Amitav Ghosh A Kingdom for His Love–Vani Mahesh, Shinie Antony
Courage & Commitment–Margaret Alva Nathuram Godse: The Story of an Assassin–Anup Ashok
India vs Pakistan: Why Can’t we just be Friends?–Husain Sardesai
Haqqani The Turbulent Years, Volume II–President Pranab Mukherjee
Chaos and Caliphate: Jihadis and the West in the Struggle for The Z Factor–Subhash Chandra
the Middle East–Patrick Cockburn Jawaharlal Nehru and The Indian Polity in Perspective–
A Call to Mercy: Hearts to Love, Hands to Serve–Mother Hamid Ansari
Teresa Maru Bharat Saru Bharat–Jain Acharya Maharaj

IMPORTANT APPOINTMENTS 2016


JANUARY Mar 15, Htin Kyaw elected as first civilian President of
Jan 1, Amitabh Kant appointed CEO of NITI (National 2016 Myanmar.
2016 Institution for Transforming India) Aayog. Mar 23, Justice Permod Kohli appointed as Chairman of
Jan 4, RK Mathur sworn in as 8th Chief Information 2016 Central Administrative Tribunal.
2016 Commissioner (CIC). APRIL
Jan 25, K Durga Prasad appointed as DG of CRPF April 7, CP Gurnani appointed as Chairman of
2016 (Central Reserve Police Force). 2016 NASSCOM (National Association of Software
FEBRUARY and Services Companies).
MAY
Feb 2, Archana Ramasundram becomes 1st woman DG
2016 of Sashastra Seema Bal. May 3, Bhupendra Kainthola takes charge as FTII (Film
2016 and Television Institute of India) Director.
Feb 3, DJ Pandian appointed AIIB (Asian Infrastructure
May 31, Admiral Sunil Lanba takes charge as Navy chief.
2016 Investment Bank) Vice-President.
2016
Feb 13, Ashok Chawla appointed as new Chairman of
JUNE
2016 TERI (The Energy and Resources Institute).
June 3, Navin Agarwal appointed as DG of NADA
Feb 23, KN Vyas appointed as Director of Bhabha Atomic
2016 (National Anti Doping Agency).
2016 Research Centre.
June 15, Peter Thomson elected as President of 71st session
Feb 25, 2016 of United Nations General Assembly.
Rajendra Singh appointed as DG of Coast Guard.
2016
June 15, Hollywood actress Anne Hathaway appointed as
Feb 29, Former CJI HL Dattu takes over as Chairman of 2016 UN Women Goodwill Ambassador.
2016 NHRC (National Human Rights Commission).
June 22, Ken Miyauchi appointed as President and COO of
MARCH 2016 SoftBank Group.
Mar 2, Vice Admiral Atul Kumar Jain appointed as Chief June 23, BCCI appoints Anil Kumble as the head coach of
2016 of Staff of Eastern Naval Command. 2016 Indian Cricket Team.
Mar 11, Justice Balbir Singh Chauhan appointed as June 27, Sujoy Bose appointed as first CEO of NIIF
2016 Chairman of 21st Law Commission of India. 2016 (National Investment and Infrastructure Fund).
Mar 12, Lt Gen N P S Hira appointed Deputy Chief of June 28, NS Vishwanathan appointed as deputy governor
2016 Army. 2016 of RBI.
w
w
w
6 Current Affairs & GK Update

.y
GK-

ou
JULY OCTOBER

rs
m
July 4, Sudarshan Sen appointed Executive Director of Oct 3, S Sivakumar appointed as full-time member of 21st

ah
2016 RBI. 2016 Law Commission of India.

bo
ob
July 11, D Rajkumar appointed as CMD of BPCL (Bharat Oct 6, Sushil Chandra appointed CBDT Chairman.

.w
2016 Petroleum Corporation Ltd). 2016

or
dp
July 13, Theresa May appointed as PM of the United Oct 14, Portugal’s Antonio Guterres appointed UN

re
2016 Kingdom. 2016 Secretary-General.

ss
Rakesh Kumar Chaturvedi appointed as Chairman

.c
July 14, Amandeep Singh Gill appointed as India’s
Oct 18,

om
2016 of CBSE. Ambassador to UN Conference on Disarmament,
2016
July 19, Guruprasad Mohapatra appointed as Chairman of Geneva.
2016 Airports Authority of India. Oct 21, Vice Admiral SV Bhokare assumes office as the
July 21, Ajay Bhushan Pandey appointed as CEO of 2016 Commandant of Indian Naval Academy.
2016 UIDAI (Unique Identification Authority of India). Oct 27, Karnal Singh appointed as Director of Enforcement
AUGUST 2016 Directorate.
Aug 1, Rani Singh Nair appointed as Chairman of CBDT NOVEMBER
2016 (Central Board of Direct Taxes). Nov. 7, M Rajeshwar Rao new RBI Executive Director.
SEPTEMBER 2016
Sep 4, Urjit Patel assumes charge as 24th Governor of Nov. 24, Yudhvir Singh Malik appointed as new Chairman
2016 Reserve Bank of India. 2016 of NHA.
Sep 19, DECEMBER
Alka Sirohi appointed UPSC chairman.
2016 Dec 5, Senior AIADMK leader O Panneerselvam
Sep 28, World Bank reappoints Jim Yong Kim for second 2016 appointed as the new AIADMK chief and the new
2016 term as President. Tamil Nadu Chief Minister.

OBITUARY IN 2016
Mufti Mohammad Sayeed Former-Jammu Jun 22, 2016 Amjad Sabri–a famed Pakistani Qawwal.
Jan 7, 2016
and Kashmir Chief Minister. Jun 29, 2016 KG Subramanyan–Renowned painter.
Jan 10, 2016 David Bowie–Legendary artist. Ramchandra Chintaman Dhere–Eminent
Jul 1, 2016
Jan 19, 2016 Ravindra Kalia–Noted Hindi writer. Marathi literary scholar.
Jan 21, 2016 Mrinalini Sarabhai–Legendary dancer. Jul 20, 2016 Mohammed Shahid – Indian Hockey legend.
Kalpana Ranjani–Noted Malayalam film Jul 28, 2016 Mahasweta Devi–Writer and social activist.
Jan 25, 2016
actress. Pandit Lacchu Maharaj–Veteran tabla
Jul 28, 2016
Feb 2, 2016 S S Tarapore–Former RBI Deputy Governor. maestro.
Feb 2, 2016 Intizar Hussain–Veteran Urdu writer. Aug 14, 2016 Na Muthukumar–Tamil Lyricist.
Sushil Koirala–Former Nepal Prime Aug 8, 2016 Mahim Bora–Eminent Assamese litterateur.
Feb 9, 2016 Kalikho Pul–Former Chief Minister of
Minister. Aug 9, 2016
Ustad Abdul Rashid Khan–Veteran singer of Arunachal Pradesh.
Feb 18, 2016 Sellapan Ramanathan–Singapore’s longest
Gwalior Gharana. Aug 22, 2016
Feb 19, 2016 serving president.
Bhubaneswari Mishra–Renowned singer.
Sep 2, 2016 Islam Karimov–President of Uzbekistan.
Purno Agitok Sangma–Former Lok Sabha
Mar 4, 2016 Sep 5, 2016 Lindsay Tuckett–World’s oldest cricketer.
Speaker.
Sep 24, 2016 Reoti Saran Sharma–Hindi and Urdu writer.
Mar 5, 2016 Ray Tomlinson–inventor of E-mail.
Syed Shamsul Haq–Renowned Bangla
Ustad Ali Ahmad Hussain–Famous Shehnai Sep 27, 2016
Mar 16, 2016 writer.
exponent. Shimon Peres–Former Israeli Prime
Mar 26, 2016 Jim Harrison–Renowned American author. Sep 28, 2016
Minister.
Barbara Turner–Hollywood actress and Oct 13, 2016 Bhumibol Adulyadej–King of Thailand.
Apr 5, 2016
screenwriter. M. Balamuralikrishna–Indian Carnatic
Apr 14, 2016 Satyanand Munjal–Hero Group co-founder. Nov 22, 2016
musician and composer
Apr 21, 2016 Prince Rogers Nelson–Music legend. Fidel Castro - Cuban Politician, Prime
Apr 27, 2016 V T Thomas–Kerala cartoonist. Nov 25, 2016
Minister.
May 11, 2016 Tony Cozier–Renowned commentator. Dilip Padgaonkar–Journalist, Editor in
Nov 25, 2016
May 27, 2016 RG Jadhav – Noted Marathi writer. Chief.
Jun 3, 2016 Muhammad Ali–American Boxer. Jayalalithaa Jayaraman- Indian actor and
Dec 5, 2016
Jun 12, 2016 Achyut Lahkar–Noted Playwright and Actor. Chief Minister of Tamil Nadu.
w
w
w
Current Affairs & GK Update 7

.y
GK-

ou
BILLS AND ACTS 2016

rs
m
ah
Maternity Benefits (Amendment) Bill, 2016 • The fine for driving without license goes up ten times to `

bo
5,000 while penalty for drunken driving increases fivefold to

ob
• Rajya Sabha has passed this bill on August 11, 2016 which
` 10,000.

.w
aims to raise maternity leave for women from 12 to 26 weeks.

or
• The Maternity Benefit Act, 1961, protects the employment • The significant provisions include increase in compensation

dp
of women during the time of maternity and entitles them for Hit & Run cases from ` 25000 to ` 2 lakhs. It also has

re
provision for payment of compensation up to ` 10 lakh in road

ss
of a full paid absence from work to take care for the child.

.c
The amendment bill seeks to increase maternity leave to 26 accidents fatalities.

om
weeks in all establishments, including private sector. • For offences by juveniles, the guardian/owner shall be deemed
• The act is applicable to all establishments employing to be responsible. They will have to pay a fine of ` 25,000
10 or more persons. The bill also provides 12 weeks leave apart from three-year imprisonment. The juvenile will be
for commissioning and adopting mothers and makes it tried under the Juvenile Justice Act and registration of his or
compulsory to provide crèche facility for establishment where her motor vehicle will be cancelled.
the number of workers is 50 and above. Adopting mother is • In the area of road safety, bill proposes to raise penalties to
the woman who legally adopts a child below 3 months of age. act as restraint against traffic violations. Stricter provisions
A commissioning mother is a biological mother who uses her are being proposed in respect of offences like juvenile
egg to create an embryo implanted in another woman. driving, drunken driving, driving without license, dangerous
• There is already a provision of 26-week or 6-month driving, over-speeding, overloading, etc. Stricter provisions
maternity leave for the government employees, most private for helmets have been introduced along with provisions for
sector firms offer maximum three months of such leave. The electronic detection of violations.
bill may also have an enabling provisions to allow working • To bring agreement of the registration and licensing process,
mothers to exercise work-from-home option. it is proposed to create National Register for Driving Licence
and National Register for Vehicle registration through
The Constitution (122nd Amendment) (GST) Bill, 2014 “Vahan” & “Sarathi” platforms. This will assist uniformity
• Rajya Sabha gave its approval on August 3, 2016 and Lok of the process across the country.
Sabha on August 8, 2016 for the crucial 122nd Constitutional
Amendment to turn the bill into law. The Lok Sabha had Benami Transaction Bill 2016
already passed the key provisions of the bill on May 06, • The Benami Transactions (Prohibition) Amendment Bill,
2015.  2016 was passed by the Rajya Sabha. It was already passed
• The Bill amends the Constitution to introduce the goods and by Lok Sabha in the last week of July 2016.
services tax (GST). • The bill has provision for confiscating ‘benami’ (proxy)
• Parliament and state legislatures will have concurrent assets.
powers to make laws on GST. Only the Centre may levy an • The bill seeks to establish adjudicating authorities and an
integrated GST (IGST) on the interstate supply of goods and appellate tribunal to deal with benami transactions as also
services, and imports. specify penalty for entering into benami transactions.
• Alcohol for human consumption has been exempted from the • The bill is part of measures planned by the Government to
purview of GST. GST will apply to five petroleum products tackle the problem of black money.
at a later date. • A transaction is considered as benami (meaning nameless)
• The GST Council will recommend rates of tax, period of levy where a property is transferred to or is held by a person and
of additional tax, principles of supply, special provisions to
the consideration for such property has been provided or paid
certain states, etc. The GST Council will consist of the Union
by another person.
Finance Minister, Union Minister of State for Revenue, and
• People keeping ‘benami’ properties to evade taxes will get
state Finance Ministers.
up to seven years rigorous imprisonment and fine, as per a
• The Bill empowers the Centre to impose an additional tax of
new law aimed at checking black money in real estate sector.
up to 1%, on the inter-state supply of goods for two years or
more. This tax will accrue to states from where the supply Lokpal and Lokayukta (Amendment) Bill, 2016
originates. • The Lokpal and Lokayukta (Amendment) Bill, 2016 was
• Parliament may, by law, provide compensation to states for passed by the Lok Sabha on July 27, 2016.
any loss of revenue from the introduction of GST, up to a five • The bill seeks to amend the Lokpal and Lokayukta Act, 2013
year period. which provides Lokpal (at Centre) and Lokayukta (at States)
• India’s 29 states currently levy their own sales taxes, statutory backing to inquire into allegations of corruption
dividing the Indian market. GST aims to provide a uniform against certain public functionaries and for related matters.
tax structure across the country.
Key Provisions
Motor Vehicle (Amendment) Bill, 2016 • The bill enables the leader of the single largest opposition
• The Union Cabinet chaired by the Prime Minister Narendra party in the Lok Sabha in the absence of a recognised Leader
Modi has given its approval on August 03, 2016 for Motor of Opposition to be a member of the selection committee that
Vehicle (Amendment) Bill 2016. would select the ombudsman.
w
w
w
8 Current Affairs & GK Update

.y
GK-

ou
• It amends section 44 of parent Act that deals with provision • It will lead to greater inclusiveness and will make the

rs
of furnishing of details of assets and liabilities of public transgender persons productive members of the society.

m
ah
servants within 30 days of joining the government service. • The Bill will make all the stakeholders responsive and

bo
• The amendment has removed the period of 30 days. Now accountable for upholding the principles underlying the Bill.

ob
the public servants will make declaration of their assets • It will bring greater accountability on the part of the Union

.w
Government and State Governments/UT administrations for

or
and liabilities in the form and manner as prescribed by

dp
government. issues concerning Transgender persons.

re
• It gives extension of the time given to public servants • Background Transgender community is among one of the

ss
most marginalized communities in the country because they

.c
and trustees and board members of Non-Governmental

om
Organisations (NGOs) to declare their assets and those of don’t fit into the stereotypical categories of gender of ‘men’
their spouses. or ‘women’.
• In this case public servants and trustees and board members • They face problems ranging from social exclusion, ostracism
of NGOs must be receiving government funds of more than and discrimination, as well as verbal, physical and sexual
` 1 crore or foreign funding of more than ` 10 lakh.
abuse.
• They also lack of access to education and employment
Child Labour (Prohibition and Regulation) opportunities, ending up into organised groups, forced
Amendment Bill, 2016 begging or demanding money.
• Lok Sabha has passed the Child Labour (Prohibition and Insolvency and Bankruptcy Code Bill, 2016
Regulation) Amendment Bill, 2016, despite opposition by
• Rajya Sabha passed the major economic reform Bill moved
several MPs.
by the Government i.e. ‘Insolvency and Bankruptcy Code,
• The Bill, passed by the Rajya Sabha on July 19, 2016. 2016’ on 11th May 2016.
• The Bill seeks to improve the Child Labour (Prohibition and • The Lok Sabha had earlier passed the Bill on 5th May, 2016.
Regulation) Act, 1986, which prohibits the employment of • The objective of the new law is to promote entrepreneurship,
children below 14 years in 83 hazardous occupations and availability of credit, and balance the interests of all
processes. The amendment extends this ban on employment stakeholders by consolidating and amending the laws
of children under 14 across all sectors, prohibits the relating to reorganization and insolvency resolution of
employment of adolescents aged 14-18 years in risky corporate persons, partnership firms and individuals in a time
occupations and introduces more severe jail term and fines bound manner and for maximization of value of assets of
for offenders: a jail term of six months to two years and a fine such persons and matters connected therewith or incidental
of ` 20,000 to ` 50,000. thereto.
• The opposition by members was mainly to the fact that • The essential idea of the new law is that when a firm defaults
children under 14 years will be allowed to work in family on its debt, control shifts from the shareholders/promoters
businesses, outside of school hours and during holidays, and to a Committee of Creditors, who have 180 days in which to
in entertainment and sports. MPs objected to the definition of evaluate proposals from various players about resuscitating
family which includes not only parents and siblings but also the company or taking it into liquidation.
the siblings of either parent. • The Insolvency and Bankruptcy Code is a comprehensive
• The Bill brings down the list of hazardous and systemic reform, which will give a quantum leap to the
occupations from the earlier 83 to just three: mining, functioning of the credit market.
inflammable substances, and hazardous processes under the
Mines and Minerals Amendment Bill, 2016
Factories Act. Read together, with the family enterprises
clause, critics say, children can be made to work in carpet, • Parliament passed the Mines and Minerals (Development
zari and bidi units, mica or diamond cutting, scavenging, and Regulation) Amendment Bill 2016. The Bill amends
the Mines and Minerals (Development and Regulation) Act,
brick kilns, slaughterhouses, handling e-waste or as domestic
1957.
help. Several MPs objected to the clause that also gives the
• The Bill was passed in the Lok Sabha on 16th March 2016
Centre the right to add to the list of non-hazardous work.
and in the Rajya Sabha on 2nd May 2016.
Transgender Persons (Protection of Rights) Bill, 2016 • The Bill allows for the transfer of mining leases which have
• The Transgender Persons (Protection of Rights) Bill, 2016 been granted through procedures other than auction, and
was approved by the Union Cabinet on July 21 to protect where the minerals are used for captive purpose.
transgenders from discrimination. Through this Bill the • The Bill adds a definition of leased area, as the area within
Union Government has evolved a holistic mechanism for which mining operations can be undertaken. This will also
the social, economic and educational empowerment of include the non-mineralised area required for the activities
marginalized transgenders community. defined under mine in the Mines Act, 1952.
Key Facts • The Mines and Minerals (Development and Regulation) Act,
• The Bill will benefit a large number of transgender persons, 1957 regulates the mining sector in India and specifies the
mitigate the stigma, discrimination and abuse against them requirement for obtaining and granting leases for mining
and also bring them into the mainstream of society. operations.
w
w
w
Current Affairs & GK Update 9

.y
GK-

ou
Real Estate (Regulation and Development) Act, 2016 • It provides to give 10% reservation to these castes for class

rs
III and class IV posts and 6% in class 1 and class 2 posts.

m
• Rajya Sabha passed the Real Estate (Regulation and

ah
Development) Bill, 2016 which aims to guard the interests • These castes will be given 10% reservation for admission

bo
in educational institutions. State Legislative Assembly also

ob
of numerous aspiring house buyers alongside enhancing
passed Haryana Backward Classes Commission Bill, 2016

.w
the credibility of construction industry by promoting

or
transparency, accountability and efficiency in execution of for institution of a permanent Backward Classes Commission.

dp
projects.

re
Aadhaar Act, 2016

ss
• The Bill seeks to set up an effective regulatory mechanism

.c
for orderly growth of the sector which is the second largest The Aadhaar (Targeted Delivery of Financial and other

om
seasonal employer after agriculture. Subsidies, benefits and services) Act, 2016 has been notified
• It provides that 70% of sale proceeds will have to be kept by the Union Government on March 28. The Act will provide
aside by the developer in an escrow account which is a statutory backing for transfer of subsidies and benefits to eligible
financial instrument held by a third party on behalf of the people having Aadhaar (UID) number.
other two parties in a transaction. Key facts
• The Bill will set up state-level real estate regulatory authorities • The act seeks to provide efficient, transparent and targeted
and appellate tribunals to monitor transactions relating to delivery of subsidies, benefits and services to individuals
both residential and commercial projects and their timely residing in India by assigning them unique identity numbers
completion and handover. It will also look after complaints (UID) or Aadhaar number.
at both appellate tribunals and regulatory authorities within • It will be used for all benefit that will be linked to consolidated
60 days ensuring timely resolution of disputes. fund of India or the expenditure incurred from it.
• The Real Estate Bill enables the people meet their genuine • Both Central and state governments can use Aadhaar for
aspirations of owning a house including those of urban poor disbursal for benefits and subsidies. However the Aadhaar
by giving a fillip to affordable housing initiative under which number cannot be used as a proof of citizenship or domicile.
the Government intends to enable construction of 2 crore by • The Act gives statutory for establishment of the Unique
the year 2022 under Prime Minister’s Awas Yojana (Urban). Identification Authority of India consisting of a Chairperson
Industries (Development and Regulation) (part time or full time) and two Members (part time).
Amendment Bill, 2015 • Those individuals not having Aadhaar number shall be
• The Industries (Development and Regulation) Amendment offered alternate and viable means of identification for
Bill, 2015 was passed by the Parliament on April 29, delivery of the subsidies, benefits or service.
2016. The Bill amends the Industries (Development and Scheduled Tribes (Prevention of Atrocities)
Regulation) Act, 1951. It was first was passed in Lok Sabha
Amendment Act, 2015
in December 2015 and later in Rajya Sabha in April 2016.
Now it will go for Presidential assent as per Article 111 of the • The amended law for prevention of atrocities against the
Indian Constitution. Scheduled Castes and the Scheduled Tribes was enforced
Key facts from January 26, 2016 after the Central Government’s
• The bill seeks to bring industries engaged in the manufacture approval.
of potable alcohol under the exclusive control of States in all • The SC/ST Bill was unanimously passed by the Rajya Sabha
respects. on December 21, 2015 without a debate, and had been
• The bill amends 1st schedule of the parent Act to bring awaiting a formal notification. The Lok Sabha had passed
required changes for regulating industries manufacturing the bill on 4thAugust, 2015.
potable alcohol. However, Union Government will continue • After the enforcement, there will be stricter prosecution
to be responsible for formulating policy and regulating for new offences of atrocities, such as tonsuring of head,
foreign collaboration for all products of fermentation moustache, or similar acts, which are derogatory to the
industries, including industrial and potable alcohol. dignity of members of the SC/ST community, garlanding
Jat Reservation Bill with chappals, denying access to irrigation facilities or
forest rights, dispose or carry human or animal carcasses,
The Haryana Backward Classes (Reservation in services
or to dig graves, using or permitting manual scavenging,
and admission in educational institutions) Bill, 2016 was
dedicating an SC/ST woman as devadasi, abusing in caste
unanimously passed by the Haryana Legislative Assembly in
name, perpetrating witchcraft atrocities, imposing social or
May 2016.
economic boycott, preventing SC/ST candidates from filing
Key Provisions of the bill
of nomination to contest elections, hurting an SC/ST woman
• It seeks to provide reservation in services and admission in
by removing her garments, forcing a member of these
educational institutions to persons belonging to backward
classes including Jats and five other castes. communities to leave house, village or residence, defiling
• The backward classes have been divided into three blocks as objects sacred to members of SC/ST, touching or using
Block A, Block B and Block C. Jats, Jat Sikhs, Ror, Bishnoi, words, acts or gestures of a sexual nature against members of
Tyagi and Mulla/Muslim Jat have been included in the Block C. the community.
w
w
w
10 Current Affairs & GK Update

.y
GK-

ou
The Juvenile Justice (Care and Protection of • The 228th report of the Law Commission of India has also

rs
recommended for prohibiting commercial surrogacy and

m
Children) Act, 2015

ah
allowing ethical altruistic surrogacy to the needy Indian

bo
• The Juvenile Justice (Care and Protection of Children) Act, citizens by enacting a suitable legislation.

ob
2015 has come into force from January 15, 2016 and repeals the

.w
Juvenile Justice (Care and Protection of Children) Act, 2000. HIV and AIDS (Prevention and Control) Bill, 2014

or
• The Bill was passed by Lok Sabha on 7th May, 2015 The Union Cabinet has given its approval to introduce

dp
re
and Rajya Sabha on 22nd December, 2015 and received amendments to the HIV and AIDS (Prevention and Control)

ss
Presidential assent on 31st December, 2015. Bill, 2014.

.c
om
• Under Section 15, special provisions have been made to • Prevent and control the spread of HIV and AIDS. It prohibits
tackle child offenders committing heinous offences in the discrimination against persons with HIV and AIDS. No
age group of 16-18 years. person will be compelled to disclose his HIV status except
President assent to the Sugar Cess (Amendment) with his informed consent, and if required by a court order
establishments keeping records of information of PLHIV
Bill, 2015
must adopt data protection measures.
• President Pranab Mukherjee gave his assent to the Sugar • Obligations on establishments to safeguard rights of persons
Cess (Amendment) Bill, 2015 on 8th January, 2016. living with HIV arid create mechanisms for redressing
• This will raise the ceiling of the impost from ` 25 to ` 200 per complaints. Lists various grounds on which discrimination
quintal, a measure that will help sugarcane growers and cash against HIV-positive persons and those living with them is
starved mills.
prohibited.
• The Sugar Cess (Amendment) Bill, 2015 was introduced
• These include the denial, discontinuation, termination or
in the Lok Sabha by Mr. Ram Vilas Paswan, Minister
unfair treatment with regard to employment, educational
of Consumer Affairs, Food and Public Distribution on
establishments, health care services, renting property etc.
December 11, 2015. The Bill proposes to amend the Sugar
Prohibits, requirement for HIV testing as a pre-requisite for
Cess Act, 1982.
obtaining employment or accessing health care or education.
Surrogacy (Regulation) Bill, 2016  • Prohibits any individual from publishing information or
• Union Cabinet chaired by the Prime Minister Narendra advocating feelings of hatred against HIV positive persons
Modi gave its approval for introduction of the “Surrogacy and PLHIV.
(Regulation) Bill, 2016” on 24th August.  National Water Framework Bill, 2016
• The Bill will regulate surrogacy in India by establishing The Central Government has brought final draft of the National
National Surrogacy Board at the Central level and State Water Framework Bill, 2016 to provide uniform national legal
Surrogacy Boards and Appropriate Authorities in the State framework to manage water in a better and efficient way. The
and Union Territories. The legislation will ensure effective comprehensive draft Bill proposes model law for all states.
regulation of surrogacy, prohibit commercial surrogacy and However, water being a State subject under VII Schedule of
allow ethical surrogacy to the needy infertile couples.  Constitution the law will be not binding on States for adoption.
• All infertile Indian married couple who want to avail ethical
Key Features of Bill
surrogacy will be benefited. Further the rights of surrogate
mother and children born out of surrogacy will be protected. • Every person has a right to sufficient quantity of safe water
The Bill shall apply to whole of India, except the state of for life within easy reach of the household regardless of his/
Jammu and Kashmir.  her socio-economic factors.
• The major benefits of the Act would be that it will regulate the • All basin states have equitable rights over the use of river
surrogacy services in the country. While commercial surrogacy water provided such use does not violate the right to water
will be prohibited including sale and purchase of human for life of any person in the river basin. States must recognise
embryo and gametes, ethical surrogacy to the needy infertile the principle that the rivers are public trustees and not owned
couples will be allowed on fulfillment of certain conditions by the basin-States.
and for specific purposes. As such, it will control the unethical • All the basin States are equal in rights and status, and there
practices in surrogacy, prevent commercialization of surrogacy is no hierarchy of rights among them. Here equality of
and will prohibit potential exploitation of surrogate mothers rights means not equal but equitable shares in river waters.
and children born through surrogacy.  Managing water at river basin-level and right measurement
• No permanent structure is proposed to be created in the Draft of State’s contribution to river system in order to resolve
Bill. Neither there are proposals for creating new posts. The conflicts.
proposed legislation, while covering an important area is • Establishing River Basin Authority (RBA) for each inter-
framed in such a manner that it ensures effective regulation State basin to ensure optimum and sustainable development
but does not add much vertically to the current regulatory of rivers and valleys. Establishing institutional arrangements
structure already in place at the Central as well as states. to deal with inter-state water disputes in order to “obviate”
Accordingly, there will not be any financial implications disputes through negotiations, mediation or conciliation.
except for the meetings of the National and State surrogacy • Proposes other mechanisms such as National water quality
Boards and Appropriate Authorities which will be met out of and footprint standards, Integrated river basin development
the regular budget of Central and State governments.  and management plan and graded pricing system.
w
w
w
Current Affairs & GK Update 11

.y
GK-

ou
Merchant Shipping Bill, 2016 • It also provides for survey, inspection and certification of

rs
vessels to enable simplified regime for convenience of Indian

m
Union Cabinet approved the introduction of Merchant Shipping

ah
shipping industry.
Bill, 2016 in the Parliament. Simultaneously, the Cabinet also

bo
ob
approved repealing of the Merchant Shipping Act, 1958 and the Taxation Laws (Second Amendment) Bill, 2016

.w
Coasting Vessels Act, 1838. The Union Government has introduced the Taxation Laws

or
The Merchant Shipping Act, 1958 has been amended 17 times

dp
(Second Amendment) Bill, 2016 in the Parliament. The bill

re
between 1966 and 2014 resulting in an increase in the number of amends existing tax laws to impose a higher rate of tax and

ss
sections to more than 560 sections. However, these provisions penalty in respect of undisclosed incomes. It proposes, Taxation

.c
om
have been shortened to 280 sections in the Bill. and Investment Regime for Pradhan Mantri Garib Kalyan
Salient Features Yojana, 2016 (PMGKY), an anti-poverty scheme. It seeks to
• It allows substantially-owned vessels and vessels on Bare attract more people to disclose their unaccounted cash and also
Boat-cum-Demise (BBCD) to be registered as Indian flag to put in a framework in place to use that for the welfare of
vessels. the people especially in the rural areas. It also ensures that tax
defaulting assessees and black money holders are subjected to
• It recognises Indian controlled tonnage as a separate category.
tax at a higher rate and stringent penalty provision
• It provides for issuing the licences to Indian flag vessels
Key features of Bill
for coastal operation and for port clearance by the Customs
• Mandatory for black money declarants to deposit 25% of
authorities.
amount disclosed in Pradhan Mantri Garibi Kalyan Yojana
• It calls for making separate rules for coastal vessels to (PMGKY) 2016 for a 4 year lock-in period without interest.
develop and promote coastal shipping. • Those who choose to declare their ill-gotten wealth stashed
• It introduces welfare measures for seafarers by providing till now in banned `1,000 and Rs. 500 currency notes under
them with the wages till they are released from hostage the PMGKY scheme will have to pay a tax at the rate of
captivity of pirates and reach back home safely. 30% of the undisclosed income.
• It directs owners of vessels to take insurance of crew engaged • Additionally, 10% penalty will be levied on the undisclosed
on vessels including fishing, sailing, etc. This applies for the income and PMGK Cess (Surcharge) at the rate of 33% of
vessels whose net tonnage is less than 15 tons. tax (33% of 30%).
• It says that the requirement of signing of articles of agreement • Further, the declarants will have to deposit 25% of the
by the crew will no longer be necessary. undisclosed income in a scheme to be notified by the
• It calls for registration of few residuary category of vessels government in consultation with the Reserve Bank of India
not covered under any statute and makes provisions for (RBI).
security-related aspects. • The money from PMGK will be used for projects in
• It adds provisions relating to seven different conventions- irrigation, infrastructure, primary education, primary
health, housing, toilets and livelihood so that there is justice
Intervention Convention 1969, Search and Rescue
and equality.
Convention 1979, Protocol for Prevention of Pollution from
• For those who continue to hold onto undisclosed cash and
Ships Annex VI to Marine Pollution Convention, Convention
are caught they will be levied with flat 60% tax plus a
for Control and Management of Ships Ballast Water and surcharge of 25% of tax (15%), which will amount a levy
Sediments 2004, Nairobi Wreck Removal Convention 2007, of 75%.
Salvage Convention 1989 and International Convention for • Besides, if the assessing officer can charge a 10% penalty in
Bunker Oil Pollution Damage, 2001. addition to the 75% tax.
POLICIES AND SCHEMES 2016
Pashudhan Bima Yojana • Under it, the insurance companies would give compensation
Haryana Government launched the Pashudhan Bima Yojana in case of death of animal (cattle). The scheme is free of cost
in Jhajjar on 29th July. The Yojna was launched by the state’s for cattle breeders from the scheduled castes.
• The state Animal Husbandry and Dairying Department has
Animal Husbandry and Dairying Minister, O P Dhankar. The
set a target to cover one lakh cattle under Pashudhan Bima
Yojana was started in accordance to the recommendations
Yojana.
made in Swaminathan Report. Animal Husbandry and Dairying
Department has set a target to cover one lakh cattle under Sewage & Waste Water Policy in Rajasthan
Pashudhan Bima Yojana. Rajasthan has become the first state to give go-ahead to the
Key Facts sewage and waste water policy, in the country. The state cabinet
• Pashudhan Bima Yojana will provide insurance cover to meeting chaired by the Chief Minister Vasundhara Raje in Jaipur
cattle breeders in the state at different premium rates for approved the first-of-its-kind policy.
different animals. Key Facts
• Insurance cover will be provided at a premium of ` 100 for • Under this policy, all district headquarters, heritage cities, and
cows, buffaloes, bulls, camels and at ` 25 for sheep, goat and cities in the state which have more than one lakh population
pig for a period of three years. will be included.
w
w
w
12 Current Affairs & GK Update

.y
GK-

ou
• Waste water treated under it will be used in irrigation and National Apprenticeship Promotion Scheme

rs
agricultural work. The requirement of next 30 years will be

m
The Union Cabinet chaired by the Prime Minister Narendra

ah
covered in this poilcy.
Modi approved a National Apprenticeship Promotion Scheme

bo
• It will be compulsory for every house in the state to connect

ob
on 5th July. The Scheme has an outlay of ` 10,000 crore with a
with sewage system in those cities which have 100% sewage

.w
target of 50 lakh apprentices to be trained by 2019-20.

or
system in next five years.

dp
Features of the scheme

re
KVK Portal • The Scheme would be implemented by Director General of

ss
The Union Minister of Agriculture and Farmers Welfare, Radha Training (DGT) under Ministry of Skill Development and

.c
om
Mohan Singh formally launched Krishi Vigyan Kendra (KVK) Entrepreneurship (MSDE). It provides for incentivizing
portal (http://kvk.icar.gov.in) on 8th July in New Delhi. employers to engage apprentices. 25% of the total stipend
Features of the Portal payable to an apprentice would be shared with employers
• The portal provides provisions for online monitoring of directly by Government of India. It is for the first time a
KVKs which will include reporting of major events on scheme has been designed to offer financial incentives
regular basis and submission of monthly reports online. to employers to engage apprentices. In addition, it also
• The portal will also provide information on different services supports basic training, which is an essential component
being provided by different KVKs. of apprenticeship training. 50% of the total expenditure
• Weather and Market related informations can also be incurred on providing basic training would be supported by
accessed by the farmers and others. Government of India.
• The forthcoming programmes will also be available on the • The Scheme will catalyze the entire apprenticeship
website which will benefit farmers and youth in joining ecosystem in the country and it will offer a win-win situation
different training programmes being organized by KVKs. for all stakeholders. It is expected to become one of the most
• Question and answer facility will also be available for the powerful skill-delivery vehicle in the country.
farmers. • The National Policy of Skill Development and
• Agriculture related information of the districts will also be Entrepreneurship, 2015 launched by the Prime Minister
available on the portal. focuses on apprenticeship as one of the key components
• The farmers and the Agricultural Officers may register for creating skilled manpower in India. The present scheme
themselves and seek different informations related to KVKs. also aims to achieve the objective as stated in the National
‘Namami Gange’ programme Policy, 2015. The policy proposes to work pro-actively with
the industry including MSME to facilitate ten fold increase
Union Minister Uma Bharti on 7th July launched 43 projects worth
opportunities in the country by 2020.
` 250 crore under Namami Gange programme in Uttarakhand.
The Government has lined up 1,000 projects in different states National Green Highways Mission
including Delhi, Uttarakhand, Uttar Pradesh, Bihar, Jharkhand
Union Minister of Road Transport & Highways and Shipping
and West Bengal as part of the National Mission for Clean
Nitin Gadkari launched the initial plantation drive on 1,500 km
Ganga. Bharati said the second phase of the project will begin
of National Highways at a cost of about ` 300 crore under the
by October 2018.
Key Facts National Green Highways Mission in New Delhi on 1st July.
Key features
• The Namami Gange Programme aims at cleaning the holy
• NGHM was launched under the Green Highways Policy,
river and ensuring its unfettered flow. Union Government
has allocated ` 20,000 crore budget for it. 2015 to provide a holistic vision of developing eco-friendly
• Under it, projects such as modernization and redevelopment and green National Highways.
of ghats, crematoriums and other development activities will • The mission aims to provide a green canopy along 100,000
be undertaken. km of highways and create jobs for 1 million youth.
• Besides it will also undertake project for development • It has been initiated to engage corporates, Public Sector
of sewage infrastructure and treatment, tree plantation, units, Government organizations and other institutions for
afforestation, pilot drain project, interceptor drain project, trash developing green corridor along National Highways through
skimmers and conservation of biodiversity. plantation and allied activity on median, avenue and other
• The projects were launched simultaneously in five basin states available nearby land patches .
of the river Ganga viz. Uttarakhand, Uttar Pradesh, Bihar, • The funds for the mission will be met by Green Fund corpus
Jharkhand and West Bengal. Besides, projects were also which is 1% of the total project cost set aside for plantation
launched in Delhi and Haryana through which Ganga’s purposes. The afforestation across National Highways is
largest tributary Yamuna passes. expected to help in sequestering approximately 12 lakh
• In Uttar Pradesh alone total 112 projects were launched metric ton carbon annually.
through this ambitious scheme. 47 projects were inaugurated • It will also generate jobs and can be a game-changer for
at various locations in Uttrakhand. 20 projects were agriculture and rural economy. In future, the mission will be
inaugurated at various locations in West Bengal. 26 projects linked with the Mahatma Gandhi National Rural Employment
were inaugurated in Bihar. Guarantee (MNREG) scheme.
w
w
w
Current Affairs & GK Update 13

.y
GK-

ou
Chandranna Bima Yojana • The benefits will also be extended to family pensioners

rs
including war widows and disabled pensioners.

m
Andhra Pradesh Chief Minister N Chandrababu Naidu has launched

ah
• All arrears will be paid in four half-yearly instalments.
Chandranna Bima Yojana in Vijayawada on 1st May. The Yojana

bo
However all widows, family pensioners including those
launched on the occasion of May Day is an insurance scheme

ob
in receipt of Gallantry award will be paid arrears in one

.w
for the working class people.
instalment.

or
Salient features of the scheme

dp
• Pension will be re-fixed every five years and future
• All unorganised workers in the State in the age group of

re
enhancements in rates of pension would be automatically

ss
18 to 70 years are eligible to be registered as unorganised
passed to the past pensioners.

.c
workers and enrolled as beneficiaries of the Chandranna

om
• Armed forces personnel who retire voluntarily would be not
Bima Scheme. covered under OROP scheme. However, it will be applicable
• All unorganised workers will be registered under the to personnel who have already retired prematurely.
Unorganised Workers Social Security Act, 2008 and enrolled
as beneficiaries under Chandranna Bima Scheme. Stand up India scheme for SC/ST & Women
• SERP shall facilitate registration of unorganised workers and The Stand-up India scheme was launched on April 6 by
their enrollment in the Chandranna Bima Scheme, data entry, Prime Minister Narendra Modi to provide credit to Scheduled
maintenance of data base, hard copies of signed applications Caste (SC), Scheduled Tribe (ST), and women borrowers
and other connected matters. in the non-farm sector. The scheme is intended to promote
• Assistant Labour Officer of the Labour Department shall be entrepreneurship among SC/ST and women and further gives a
the Registering Authority. push to government’s financial inclusion programme.
• The registered unorganised workers will be enrolled as Main features of the scheme
members under State Accident Death and Disability Scheme • Composite loan between 10 lakh rupees and up to 1 crore
and under Aam Admi Bima Yojana (AABY) and will also rupees will be provided to SC/ST and women and also
be covered under the Pradhan Mantri Suraksha Bima Yojana inclusive of working capital component for setting up any
(PMSBY). new enterprise.
UJALA Scheme • Debit Card (RuPay) for withdrawal of working capital:
Credit history of borrower to be developed of these under-
The National LED programme – Unnat Jyoti by Affordable banked sections of society.
LEDs for All (UJALA) was launched by the Union Government • Refinance window through Small Industries Development
on May 1. The scheme was launched by Union Minister for Bank of India (SIDBI) with an initial amount of 10,000 crore
State (IC) for Power, Coal and Renewable Energy Piyush Goyal rupees.
in Bhopal, Madhya Pradesh. • Creation of a corpus of 5,000 crore rupees for credit guarantee
Key facts through National Credit Guarantee Trustee Company
• The UJALA scheme is being implemented by Energy (NCGTC).
Efficiency Services Limited (EESL), a joint venture of PSUs • Handholding support for borrowers with comprehensive
under the Union Ministry of Power. support for pre loan training needs, facilitating loan,
• It is LED based Domestic Efficient Lighting Programme factoring, marketing, etc.
(DELP). • Web Portal for online registration and support services will
• Under the scheme, 3 crore LED Bulbs will be distributed in be provided.
Madhya Pradesh in the next 6 months. • Under the scheme, 1.25 lakh bank branches will provide loans
• People of Madhya Pradesh will get subsidised 9W energy up to 1 crore rupees to SC/ST and women entrepreneurs. Thus
efficient LED bulbs by paying just 85 rupees per LED bulb. it will help in creating 2.5 lakh entrepreneurs throughout the
• The scheme will help reduce electricity bills of consumers, country.
contribute to the energy security of India and also help in Pradhan Mantri Awaas Yojana – Gramin
environment protection.
The implementation of the rural housing scheme of Pradhan
• It will help in saving energy around 24 crore units every year. Mantri Awaas Yojana- Gramin was given approval by the Union
One Rank One Pension Scheme Cabinet on March 25. Under the scheme, financial assistance
will be provided for construction of pucca houses to all houseless
One Rank One Pension (OROP) scheme was given ex-post facto
and households living in dilapidated houses.
approval on 7 April by the Union Cabinet for implementation of
Key facts
retrospectively with effect from 1st July, 2014. Decision in this • The expenditure involved in implementing the project is `
regard was taken by Union Cabinet meeting chaired by Prime 81975 crore in a span of 3 years from 2016-17 to 2018-19.
Minister Narendra Modi in New Delhi. OROP will provide ex- • Under the scheme, it is proposed to provide assistance to one
servicemen of the same rank and same length of service uniform crore households for construction of pucca house.
pension regardless of date of retirement. • It will be implemented in rural areas throughout the country
Key facts except in Delhi and Chandigarh.
• The benefits of OROP will be given with effect from 1st • The beneficiaries will be identified using Socio Economic and
of July 2014 and it will benefit ex-servicemen of all three Caste Census (SECC)-2011 data ensuring total transparency
services- Army, Navy and Air Force. and objectivity.
w
w
w
14 Current Affairs & GK Update

.y
GK-

ou
• The cost of houses would be shared between Centre and • To improve technology depth in Indian capital goods from

rs
States Governments in the ratio 60:40 in plain areas and the current basic and intermediate levels to advanced levels.

m
ah
90:10 for North Eastern and hilly states. • Objectives: Increase total production to achieve total

bo
• Under this scheme, annual list of beneficiaries will be production in excess of 5 lakh crore rupees by 2025 from the

ob
identified from the total list through participatory process by current 2.2 lakh crore rupees.

.w
the Gram Sabha.

or
• To increase domestic employment from the current 15

dp
• Funds will be transferred electronically directly to the
lakhs to at least 50 lakhs by 2025 thus providing additional

re
account of the beneficiary.

ss
• He will be able to track the progress of his payments through employment to over 35 lakhs people.

.c
• To increase the share of domestic production in India’s

om
the app. Inspection will be done through a mobile app.
• Locally appropriate house designs, will be mainly practised capital goods demand from 56% to 80% by 2025 and in the
to address the natural calamities. process improve domestic capacity utilization to 80-90%.
• The beneficiary is entitled to 90 days of unskilled labour • To improve skill availability by training 50 lakhs people by
from MGNREGA. This will be ensured through a server 2025.
linkage between PMAY and MGNREGA. • To improve ‘technology depth’ in capital goods sub-sectors
• The additional requirement of building materials, bricks
by increasing research intensity in India from 0.9% to at least
using cement stabilised earth or fly ash will be taken up
2.8% of GDP.
under MGNREGA.
• National Technical Support Agency will be established Pradhan Mantri Ujjwala Yojana
to provide technical support to the Centre and On May 1, the Pradhan Mantri Ujjwala Yojana (PMUY) was
States to ensure their quality of construction of the houses.
launched. The scheme will benefit the poor, especially the
Udyam Aakansha scheme women. The scheme aims to provide cooking gas connections
Chhattisgarh Government has launched ‘Udyam Aakansha’ to five crore below-poverty-line beneficiaries over the next three
Scheme for promoting micro, small and medium enterprises in years.
Salient features of the scheme
the state on February 9.
• Cabinet Committee on Economic Affairs (CCEA) approved
Highlights of the scheme
8000 crore rupees for the next 3 years starting from financial
• The scheme would facilitate free online registration and year 2016-17.
self-certification to the investors interested in investment on • PM Ujjwala Yojana will provide 5 crore LPG connections to
micro, small and medium enterprises and service businesses. BPL Families.
• The investors can enrol themselves online through website • A budgetary provision of the scheme is 2000 crore rupees.
of the Industries department which will be based on self- • 5 crore LPG connections will be provided to BPL families
certification process. with a support of 1600 rupees.
• For the online registration, investors need not submit any • Connections will be given in the name of women beneficiaries.
documents nor have to pay any fee. • Identification of BPL families will be made in consultation
• Chhattisgarh is among some leading states where this scheme with State Governments and Union territories.
has been initiated. • EMI Facility for meeting the cost of Stove and Refill cost.
• Keeping in view the objective of promoting the ease of doing
business, the process of registration and self-certification for Pradhan Mantri Yuva Yojana
entrepreneurs has been simplified. The Union Ministry of Skill Development and Entrepreneurship
(MSDE) launched the Pradhan Mantri YUVA Yojana to boost up
National Capital Goods Policy 2016
an ecosystem of entrepreneurship for young people.
National Capital Goods Policy 2016 was unveiled by Union
Ministry of Heavy Industries on 15 February 2016. The policy, Highlights of the Yojana
with the theme Building India of Tomorrow, seeks to achieve • The scheme was launched for the time period of over five
the vision of Make in India initiative and establishes India as a years (2016-17 to 2020-21) with a project cost of 499.94
global manufacturing hub by unlocking the potential for capital crore rupees.
goods sector. • It will offer entrepreneurship education and training to
Main Features around 7 lakh students in 5 years through 3050 Institutes.
• Vision: To increase the share of capital goods contribution • It will also offer easy access to information and mentor
from present 12 to 20% of total manufacturing activity by network, credit and advocacy to build a way-out for the
2025. youngsters.
• Mission: To become one of the top capital goods producing • The institutes under the Yojana include 2200 Institutes
nations of the world by raising the total production to over of Higher Learning (colleges, universities, and premier
twice the current level. institutes), 300 schools, 500 ITIs and 50 Entrepreneurship
• To raise exports to a significant level of at least 40% of total Development Centres.
production and thus gain 2.5% share in global exports of • These institutes will carry out the training through Massive
capital goods. Open Online Courses (MOOCs).
w
w
w
Current Affairs & GK Update 15

.y
GK-

ou
Other Launches The scheme had also found its mention in Prime Minister

rs
Narendra Modi’s monthly radio address Mann Ki Baat in June

m
• The Ministry also unveiled the guidelines for State

ah
Engagement under Pradhan Mantri Kaushal Vikas Yojana 2016.

bo
2.0 (2016-2020). It allocated around 3000 crore rupees for Salient Features

ob
.w
the States to achieve its target of training 10 million people • To provide healthy life to pregnant women

or
in over 4 years. • To lower the maternity mortality rate

dp
• To make pregnant women aware of their health issues and

re
• It also launched the Lab Guidelines towards standardisation

ss
diseases
of lab equipment across skill development training centres in

.c
• To ensure safe delivery and healthy life of the baby

om
India. • The Pradhan Mantri Surakshit Matritva Abhiyan is applicable
• MSDE announced the institutionalisation of National for all pregnant women. It will provide medical checkups
Entrepreneurship awards for first generation achievers below completely free to pregnant women.
30 years for the very first time. The Entrepreneurship Awards • The checkups will take place at the medical centres,
will be given on 16 January 2017. government and private hospitals and private clinics across
the country.
Pradhan Mantri Surakshit Matritva Abhiyan • These free of cost tests will include blood pressure, sugar
level, weight, haemoglobin test, blood test and screening.
The scheme aims at providing pregnant women ante-natal
• Women will be marked differently using coloured stickers
services and required treatment for free at government health based on their health problems so that doctors can easily
centres and hospitals on 9th of every month. detect the problem.

INDIAN STATES AND UNION TERRITORIES


INDIAN STATES & UNION TERRITORIES
India/State Capital Area Language Establishment Sex Literacy Pop. Festivals Dance Tribes
(Sq.Km) Year Ratio Average Density
/1000 % (Sq. Km)
INDIA New Delhi 3.3 No National 15-08-1947 940 73% 382 G.Jayanti, – –
Million Language I.Day. &
R. Day
1. Andhra Hyderabad 160205 Telugu, 01-10-1953 992 67.7% 308 Sankranti, Kuchipudi Andh, Bagata,
Pradesh Urdu Ugadi Bhil, Konda
2. Arunachal Itanagar 83,743 English 20-02-1987 920/ 66.95% 17 Losar” or The Bardo Chham Abor, Aka,
Pradesh 1000 New Year Apatani
3. Assam Dispur 78,550 Assamese, 1st April 1912 – – 397 Bihu Ankia Naat Mikirs, Khasis,
(Assom) Bengali, (Onkeeya) Nagas, Boro
4. Bihar Patna 99,200 Bhojpuri, 1st April 1936 916 63.4% 1,102 Chhath Bidesia Kajari Gonda,
Maithili Mundas, Gaur
5. Chhattisgarh Raipur 135,194 Chattisgarhi, 1-11-2000 991 71.04% 189 Bastar Panthi, Rawat Agariya, Andh,
Hindi Dussere, Nacha Baiga, Bhaina,
Bhoramdeo
6. Goa Panaji 3,702 Konkani 30-05- 1987 968 88.70% 394 per Ganesh Dekhnni, Fugdi Dhodia, Dubla
sq km. Chaturthi (Halpati),
7. Gujarat Gandhinagar 196,204 Gujarati 01-05-1960 918 79.31% 310 Makar Rass-garba Bhils, Barda,
Sankranti Bavacha
8. Haryana Chandigarh 44,212 Punjabi, 01-11-1966 877 76.64% 573 Haryali Teej, Saang, Dhamal Meo, Ror
Haryanvi Lohri,
9. Himachal Shimla 55,673 Pahari, 25-01-1971 974 83.78% 123 Kullu, Losar Shona Bhot, Bodh,
Pradesh Kangri Shoolini Chuksam Gaddi, Gujjar
10. J & K Srinagar 222,236 Kashmiri, 26-10-1947 883 66.7% 56 Hemis, Urs Dumhal, Rouff Balti, Beda,
Urdu Bot, Boto
11. Jharkhand Ranchi 79,714 Santhali, 15-11-2000 947 67.6% 414 Jhumar, Paika, Karam, Vat Asur, Agaria,
Mundari, Ho Chau, Agni savitri Baiga, Banjara
12. Karnataka Bengaluru 191,791 Kannada 01-11-1956 968 75.60% 320 Mysore Bharatanatyam, Adiyan, Barda,
Dasara, Ugadi Bolak-aat Bavacha, Bhil
w
w
w
16 Current Affairs & GK Update

.y
GK-

ou
13. Kerala Thiruvanantha- 38,863 Malayalam, 01-07-1949 1,084 93.91% 860 Onam Kathakali Adiyan,

rs
m
puram English Arandan

ah
bo
14. Madhya Bhopal 308,245 Hindi 01-11-1956 930 70.60% 236 Shivratri, Badhai, Rai, Bhil, Bhunjia,

ob
Pradesh Bahgoriya Saira Biar, Binjhwar

.w
15. Maharashtra Mumbai 307,713 Marathi 01-05-1960 929 82.9% 370 Vijayadashami Lavani, Koli Andh, Baiga,

or
dp
or Dasara Barda

re
16. Manipur Imphal 22,327 Meeteilon 21-01-1972 987 79.21% 120 Lui-ngai- Manipuri Aimol, Anal,

ss
.c
ni Ningol Angami

om
Chakouba,
Yaoshang
17. Meghalaya Shillong 22,429 Khasi,Garo 21-01-1972 986 75.84% 130 Nongkrem, Nongkrem` Chakma,
Wangala Dimasa, Garo
18. Mizoram Aizawl 21,087 Mizo 20-02-1987 975 91.58% 52 Chapchar Kut, Cheraw, Chakma,
Thalfavang Khuallam Dimasa, Garo
Kut,
19. Nagaland Kohima 16,579 English 01-12-1963 931 80.11% 119 Hornbill, Zeliang Naga, Kuki,
Sekrenyi Mikir, Garo
20. Odisha Bhubaneshwar 155,820 Odia, Eng- 01-04-1936 978 73.45% 270 Ganesh Odissi Agata, Bathudi,
lish Chaturthi Birhor
21. Punjab Chandigarh 50,362 Punjabi 15-08-1947 893 76.68% 550 Bandi Chhor, Bhangra, –
Vaisakhi, Giddha
Lohri
22. Rajasthan Jaipur 342,239 Hindi, 01-11-1956 926 67.68% 201 Gangaur, Teej, Ghoomar –
Rajasthani Gogaji
23. Sikkim Gangtok 7,096 Nepali, 16-5-1975 889 82.2% 86 Maghe, Losar Singhi Chham Bhutia, Lepcha,
Bhutia Limboo
24. Tamil Nadu Chennai 130,058 Tamil 26-01-1950 995 80.33 % 550 Pongal Bharata-natyam Adiyan,
Aranadan,
25. Telangana Hyderabad 114,840 Telugu, 02-06-2014 – 66.50% 310 Ugadi Kuchipudi Andh, Konda
Urdu
26. Tripura Agartala 10,491.69 Bengali , 21-01-1972 961 94.65% 350 – Goria, Jhum Bhil, Bhutia,
Kokborok Chaimal
27. Uttarakhand Dehradun 53,483 Garhwali, 9-11-2000 963 79.63% 189 Kandali, Langvir Nritya Bhotia, Buksa,
Kumaoni Ramman Jaunsari, Raji
28. Uttar Pradesh Lucknow 243,286 Hindi, Urdu 01-04-1937 908 69.7% 820 Makar Sankranti, Kathak Bhotia, Buksa,
Chhath Tharu, Baiga
29. West Bengal Kolkata 88,752 Bengali and 15-08-1947 947 77.08% 1,000 Durga Puja, Chau dance Asur, Baiga,
English Kali Puja Bedia, Chero
Union
Territory
1. Andaman and Port Blair 8,073 English, 01-11-1956 878 86.27% 46 – – Andamanese,
Nicobar Islands Hindi Chariar, Chari
2. Chandigarh Chandigarh 114 Punjabi 01-11-1966 818 81.9% 9,300/ Lohri Bhangra –
3. Dadra and Silvassa 102 English, 11-08-1961 775 77.65% 698 Pongal Tarpa, Bhavada Warlis, dublas
Nagar Haveli Gujarati
4. Daman and Daman 102 Gujarati, 30-05-1987 618 87.07% 2169 Garba Mando, Vira Dhodia, Dubla
Diu Marathi (Halpati)
5. Lakshadweep Kavaratti 32 English, 01-11-1956 946 92.28% 2013 Eid-Ul-Fitr, Lava, Kolkali Koya, Malmi
Malayalam Muharram
6. NCT of Delhi New Delhi 1,484.0 Hindi 01-02-1992 866 86.34% 11,297 Diwali, Eid – –
ul-Fitr
7. Puducherry Pondicherry 492 Malayalam, 07-01-1963 1,038 86.34% 2,500 Pongal Garadi Grulas, Villi
Tamil
w
w
w
Current Affairs & GK Update 17

.y
GK-

ou
STATE ANIMALS AND BIRDS

rs
m
ah
State Animal Birds

bo
Andhra Pradesh Blackback (Antilope cervicapra) Indian Roller (Coracias benghalensis)

ob
Arunachal Pradesh Gayal (Bos frontalis) Great Hornbill (Buceros bicornis)

.w
Asom (Assam) One-horned rhino (Rhinoceros unicornis) White-winged wood duck (Cairina scutulata)

or
dp
Bihar Gaur (Bos gaurue) House Sparrow (Passer domesticus)

re
Chhattisgarh Wild buffalo (Bubalis arnee) Bastar Hill myna (Gracula religiosa)

ss
.c
Goa Gaur (Bos gaurus) Black-crested bulbul (Pycnonotus gularis)

om
Gujarat Asiatic lison (Panthera leo persica) Greater Flamingo (Phoenicopterus roseus)
Haryana Blackbuck (Antilope cervicapra) Black Francolin (Francolinus francolinus)
Himachal Pradesh Snow Leopard (Uncia uncia or Panthna uncia) Jujurana Western Irogapa (Trogopan melanocephalus)
Jammu and Kashmir Kashmir Stag (Cervus elaphus hanglin) Black-necked crane (Grus nigricollis)
Jharkhand Indian Elephant (Elephas maximus indicus) Asian koel (Eudynamys scolopacea)
Karnataka Indian Elephant (Elephas maximus indicus) Indian Roller (Coracias benghalensis)
Kerala Indian Elephant (Elephas maximus indicus) Great Hornbill (Buceros bicornis)
Madhya Pradesh Barasingha (Rucervus duvaucelii) Asian Paradise Flycatcher (Trepsiphone paradisi)
Maharashtra Indian Giant Squirrel (Ratufa indica) Yellow footed green pigeon (Treron phoenicoptera)
Manipur Sangai (Cervus eldi) Mrs. Humes Pheasant (Syrmaticus humiae)
Meghalaya Clouded Leopard (Neofelis nebulosa) Hill Myna (Gracula religiosa)
Mizoram Hoolock gibbon (Hoolock hoolock) Mrs. Humes pheasant (Syrmaticus humaie)
Nagaland Gaur (Bos gaurus) Blyth’s Tragopan (Tragopan blythii)
Odisha Sambar (Rusa unicolor) Indian Roller (Coracias benghalensis)
Punjab Blackbuck (Antilope cervicapra) Northern Goshawk (Accipiter gentilis)
Rajasthan Chinkara (Gazella bennettii) Great Indian Bustard (Ardeotis nigps)
Sikkim Red panda (Ailurus fulgens) Blood pheasant (Ithaginis cruentus)
Tamil Nadu Nilgiri Tahr (Nilgiritragus hylocrius) Emerald Dove (Chalcophaps indica)
Telangana Chital (Zinka) Pala Pitta (Coracias benghalensis)
Tripura Phayre’s Langur (Trachypithecus phayrel) Green Imperial Pigeon (Dacula genea)
Uttarakhand Musk deer (Moschus cupreus) Himalayan Monal (Lophophorus impejanus)
Uttar Pradesh Swamp Deer (Rucervus duvaucelii) Sarus Crane (Grus antigone)
West Bengal Fishing cat (Prionailurus viverrinus) White-breasted Kingfisher (Halcyon smyrnensis)
Lakshadweep House Sparrow (Passer Domesticus)
National Capital House Sparrow (Passer domesticus)
Territory of Delhi
Puducherry Squirrel Sciuridae ratufinae Asian Koel (Eudynamys scolopaceus)

Four Ends of India Foundation day of States


Easternmost point of India is known as Kibithu; situated on right 1st January Nagaland Day
bank of river Lohit separating India from China-Tibet region. 21st January Manipur, Meghalaya and Tripura Day
It is a small village at the altitude of 3,350 metre in Arunachal 6th February Jammu and Kashmir Day
Pradesh. Westernmost point is situated in Kuch area of Gujarat
20th February Mizoram and Arunachal Pradesh day
called as Ghuar Mota. The region is famous for its harsh climate
11th March Andman and Nicobar Islands Day
with 45°C in summer and 20°C in winter. During monsoon
22nd March Bihar Day (Bihar Diwas)
season this region looks like a tortoise surrounded by sea water.
30th March Rajasthan Day
Northernmost point of India has been in controversies ever since
India’s independence. The Siachen Glacier in the state of J&K is 1st April Utkal (Odisha) Day
the northern boundary of India according to the official division 14th April Tamil Nadu Day
of India during the time of independence. The Southernmost 15th April Himachal Pradesh Day
point of the mainland of India is Kanyakumari District in the state 1st May Gujarat and Maharashtra Day
of Tamil Nadu. Kannyakumari, formerly was known as Cape 16th May Sikkim Day
Comorin. It is the second largest and urbanized of Tamil Nadu. 1st November Chattisgarh
Indira Point is a village in the Nicobar district of Andaman and 9th November Uttaranchal (Now Uttarakhand) Day
Nicobar Islands, India. It is located in the Great Nicobar tehsil. 15th November Jharkhand Day (Jharkhand Diwas)
It is the location of the southernmost point of India’s territory. 2nd June (2014) Telangana Day
w
w
w
18 Current Affairs & GK Update

.y
GK-

ou
AWARDS AND HONOURS

rs
m
Gallantry Awards Gandhi Peace Prize

ah
bo
Param Vir Chakra: The highest Gallantry Award • Established in 2 October, 1994. It carries a cash prize of ` 1

ob
Mahavir Chakra: The second highest Gallantry Award crore.

.w
Vir Chakra: The third highest Gallantry Award

or
Indira Gandhi Prize for Peace, Disarmament and

dp
Ashok Chakra: The highest peacetime Gallantry award
Development

re
Kirti Chakra: For conspicuous Gallantry

ss
Shaurya Chakra: For an act of Gallantry • Instituted in 1985, this prestigious award is regarded as

.c
om
Bharat Ratna ‘Nobel’ and over the years it has been awarded to those
The highest civilian award of India. persons who have done outstanding work for international
• The first three recipients of Bharat Ratna: C. Rajagopalchari, peace, disarmament and development.
Dr. S. Radhakrishnan and DR. C.V.Raman (1954) Dadasaheb Phalke Award
Padma Awards
• It is India’s highest award in cinema.
• Padma Vibhushan : The second highest civilian award given • It is presented annually at the National Film Awards
for exceptional and distinguished service.
ceremony by the Directorate of film festival.
• Padma Bhushan : The third highest civilian award given for
distinguished service of a high order. Borlaug Award
• Padma Shri : The fourth highest civilian award given for • Instituted in 1973, carries a cash prize of ` 1 lakh.
distinguished service. • Instituted to honour outstanding agricultural scientists.
Bharatiya Jnanpith Award
Sahitya Akademi Award
• Instituted in 22 May, 1961, carries a cash prize of ` 5 lakh, a
• Awarded for outstanding literary work and carries a cash
citation and a bronze replica of Vagdevi (Saraswati).
• This award is given for the best literary writing by an Indian prize of ` 1 lakh.
citizen in a language listed in eight schedule of the Indian • Sahitya Academi gives 22 awards for literary works in the
Constitution. languages which has recognized works.

INDIA’S POLITICAL JOURNEY 70 YEARS


Date/ Period Events
1946 The Constituent assembly elected in 1946 served as the Parliament
15 Aug. 1947 • India became independent
• Jawaharlal Nehru becomes the 1st P. M. of Independent India
30 Jan. 1948 • Mahatma Gandhi was shot dead by Nathuram Godse
• India sends troops and took over the State of Hyderabad 
• Sheikh Abdulla became the Prime Minister of Jammu and Kashmir.
• Pakistan adopted Guerrilla War in J & K but India recaptured Dras, Kargil and Poonch.
26 Nov. 1949 Constitution of India was adopted with a Preamble, 395 Articles & 8 Schedules.
26 Jan. 1950 • Constitution of India was enforced & India was declared a Republic State
• Rajendra Prasad, 1st President of India
• Home Minister Vallabhbhai Patel died on 15 Dec. 1950.
1951 • India’s first general/Lok Sabha election ( 25 Oct. 1951 to 21 Feb. 1952) & Indian National Congress won
with full majority (75%)
1953 • Sheikh Abdulla was dismissed, arrested and Ghulam Mohammed became the Prime Minister of Jammu and
Kashmir.
1954 • French finally gave up its territory Pondicherry to India.
1956 Death of B. R. Ambedkar on 6 Dec. 1956
1957 • Second general elections held in India & Congress won  371 seats
1959 • Mrs. Indira Gandhi was elected the president of Indian National Congress.
1960 • The states of Gujarat and Maharashtra came into being on May 1, 1960 & Bombay got allocated to
Maharashtra.
1961 • Indian troops liberated Goa from Portuguese colony by Operation Vijay
1962 • Congress retained its majority in the general elections (361 seats out of 494).
w
w
w
Current Affairs & GK Update 19

.y
GK-

ou
1964 • Jawahar Lal Nehru passed away. Gulzarilal Nanda was sworn in as acting prime minister.

rs
m
• Congress Working Committee finalized on Lal Bahadur Shastri as Nehru’s successor. (Other contenders were

ah
bo
Tamilian K. Kamraj & Morarji Desai)

ob
1965 • Pakistan invaded India over a salt marsh in Kachchh desert forcing Indians to withdraw some 40 miles.

.w
or
1966 • L.B. Shastri & Pakistani President Ayub Khan met in Taskent along with the Soviet Union P.M. Kosygin, and

dp
signed “Taskent Agrement”.

re
ss
• Lal Bahadur Shashtri dies in sleep, of cardiac arrest.

.c
om
• Gulzarilal Nanda was (once again) sworn in as acting prime minister.
• Indira Gandhi became PM on Congress president Kamraj endorsement for which Morarji Desai was interested.
• Punjab and Haryana divided into separate states.
1967 • 4th general elections held in February & INC got 283 seats.
1969 14 banks were nationalized which was opposed by then Finance Minister Morarji Desai
1971 • The Indo-Pakistan War of 1971  over Bangladesh liberation. Pakistani troops surrendered & Provisional
Government of the People’s Republic of Bangladesh came into being.
• 5th general elections of India was held 14 months before scheduled time and Congress returned with 362
seats out of 520
1975 • On 26th June 1975 national emergency was declared.
• Opposition leaders, including, Jayprakash Narayan, Morarji Desai, were sent to jail. Sanjay Gandhi became
leading political executive overshading Indira Gandhi
1977 • Mrs. Gandhi dissolved Lok Sabha and general elections were held.
• The Congress lost around 200 seats. Both Indira Gandhi and Sanjay Gandhi lost.
• Janata Party came with majority & Morarji Desai became PM of India.
1979 • Morarji Desai resigned after Janata Party splits into three parts: headed by Morarji Desai, Jagjivan Ram and
Charan Singh.
• Charan Singh was sworn as Prime Minster, but a month later Indira’s Congress (I) withdrew support, which
forced a mid-term election.
1980 General elections of India was held & INC won 353 seats as a result Mrs.I. Gandhi became PM. Sanjay Gandhi
died in a plane accident.
1984 • Indira Gandhi shot dead by two of her Sikh personal bodyguards.
• Rajiv Gandhi became Prime Minister of India.
• 8th general elections of India was held & INC won 404 seats.
1989 General elections of India was held & INC won 197 seats. The Janata Dal led by V P Singh secured 143 seats,
BJP 85 & Left Parties 45 seats. V.P. Singh became PM with support of BJP & Left parties.
1991 • Rajiv Gandhi assassinated by suicide bomber sympathetic to Sri Lanka's Tamil Tigers(LTTE).
• 10th General elections were held. INC+ got 244, BJP+ 120 & NF got 69 seats. P.V. Narasimha Rao became
PM with the support of Left parties.
1992 Babri Mosque in Ayodhya was demolished, triggering widespread Hindu-Muslim violence.
1996 • General elections were held. UF 192(Janata Dal+) won 192, BJP 187, & INC 140 seats.
• AB Vajpayee became PM only for 13 days followed by H.D. Deve Gowda & IK Gujral (Janata Dal). The 11th
Lok Sabha produced three Prime Ministers in two years.
1999 General elections were held. NDA(BJP+) won 254(182+), INC+ 144 & UF 64 seats. AB Vajpayee became PM.
1999 Kargil war between India & Pakistan. India came as victorious.
2004 General elections were held. UPA got 218, NDA 181 seats & Manmohan Singh became PM of India.
2009 General elections were held. UPA got 262, NDA 159 seats & Manmohan Singh became PM of India.
2014 General elections were held. BJP got 282, INC 44. Narendra Modi became PM of India.
2016 India got entry into Missile Technology Control Regime (MTCR) group.
GK-
20

INDIA'S ECONOMIC JOURNEY 70 YEARS

w
w
w
.y
ou
rs
m
ah
bo
ob
.w
Current Affairs & GK Update

or
dp
re
ss
.c
om
w
w
w
Current Affairs & GK Update 21

.y
GK-

ou
IMPORTANT DAYS OF THE YEAR

rs
m
ah
9th January: NRI Day 28th July: World Hepatitis Day

bo
10th January: World Laughter Day 29th July: International Tiger Day

ob
12th January: National Youth Day 30th July: International Day of Friendship

.w
or
15th January: Indian Army Day 6th August: Hiroshima Day

dp
23rd January: Netaji Subhash Chandra Bose’s birth anniversary 12th August: International Youth Day

re
ss
24th January: National Girl Child Day 15th August: India’s Independence Day

.c
25th January: National Tourism Day

om
19th August: World Humanitarian Day
26th January: Republic Day 29th August: National Sports Day
29th January: National Newspaper Day 5th September: Teacher’s Day
30th January: Martyr’s Day 8th September: International Literacy Day
2nd February: World Wetlands Day 14th September: Hindi Day
4th February: World Cancer Day 15th September: World Engineer’s Day
13th February: World Radio Day 16th September: World Ozone Day
14th February: St. Valentine’s Day 18th September: International Day of Peace
21st February: International Mother Language Day 21st September: Biosphere Day
28th February: National Science Day 24th September: Girl Child Day
8th March: International Women’s Day and Mother’s day 25th September: Social Justice Day, World Maritime Day
13th March: No Smoking Day 27th September: World Tourism Day
15th March: World Consumer Rights Day 1st October: International Music Day
22nd March: World Water Day 2nd October: Gandhi Jayanti, International Non-Violence Day
23rd March: World Meteorological Day 3rd October: World Habitat Day
24th March: World Tuberculosis Day
8th October: National Air force Day
27th March: World Theatre Day
9th October: World Postal Day National Postal Week (9th
2nd April: World Autism Awareness Day October to 14th October)
7th April: World Health Day 12th October: World Sight Day
18th April: World Heritage Day 16th October: World Food Day
22nd April: World Earth Day 31st October: National Integration Day
25th April: World Malaria Day 7th November: World Cancer Awareness Day
30th April: World Jazz Day 11th November: National Education Day
1st May: International Labour Day 14th November: Children’s Day
3rd May: World Asthma Day 16th November: National Press Day
5th May: World Athletics Day
21st November: World Fisheries Day, World Hello Day, World
8th May: International Red Cross Day Television Day
11th May: National Technology Day 25th November: International Day for the Elimination of
17th May: World Telecommunications Day Violence against Women
20th May: World Refugee Day 26th November: National Constitution day
21st May: Anti-terrorism Day 26th November: National Milk day
24th May: Commonwealth Day 30th November: Flag Day
31st May: World No Tobacco Day 3rd December: World Conservation Day
1st June: World Milk Day 5th December: World Soil Day
5th June: World Environment Day 7th December: Armed Forces Flag Day
8th June: World Ocean Day 9th December: International Day against Corruption, National
20th June: International Refugee Day Immunization Day
21st June: International Yoga Day 10th December: World Human Rights Day, International
23rd June: International Olympic Day Broadcasting Day
27th June: World Diabetes Day 11th December: UNICEF Day
1st July: World Doctor’s Day Van Mahotsav Week (1st July to 14th December: National Energy Conservation Day
7th July) 16th December: Vijay Diwas
11th July: World Population Day 22nd December: National Mathematics Day
26th July: Kargil Victory Day 23rd December: Kisan Diwas (Farmers’ Day)
w
w
w
22 Current Affairs & GK Update

.y
GK-

ou
SPACE SCIENCE

rs
m
ah
Space mission 1975-2016 EDUSAT 20-Oct-04 GSLV-F01

bo
Satellite Launch Date Launch Vehicle HAMSAT 5-May-05 PSLV-C6

ob
Aryabhata 19-Apr-75 u-11 Interkosmos CARTOSAT-1 5-May-05 PSLV-C6

.w
Bhaskara-I 7-Jun-79 C-1 Interkosmos

or
INSAT-4A 22-Dec-05 Ariane-5GS

dp
Rohini Technology 10-Aug-79 SLV-3 INSAT-4C 10-Jul-06 GSLV-F02

re
Payload CARTOSAT-2 10-Jan-07 PSLV-C7

ss
.c
Rohini RS-1 18-Jul-80 SLV-3 Space Capsule Recovery 10-Jan-07 PSLV-C7

om
Rohini RS-D1 31-May-81 SLV-3
Experiment (SRE-1)
Ariane Passenger 19-Jun-81 Ariane-1 (V-3)
INSAT-4B 12-Mar-07 Ariane-5ECA
Payload Experiment
INSAT-4CR 2-Sep-07 GSLV-F04
Bhaskara -II 20-Nov-81 C-1 Intercosmos
INSAT-1A 10-Apr-82 Delta 3910 PAM-D CARTOSAT-2A 28-Apr-08 PSLV-C9
Rohini RS-D2 17-Apr-83 SLV-3 IMS-1 (Third World 28-Apr-08 PSLV-C9
INSAT-1B 30-Aug-83 Shuttle [PAM-D] Satellite – TWsat)
Stretched Rohini 24-Mar-87 ASLV Chandrayaan-1 22-Oct-08 PSLV-C11
Satellite Series RISAT-2 20-Apr-09 PSLV-C12
(SROSS-1) ANUSAT 20-Apr-09 PSLV-C12
IRS-1A 17-Mar-88 Vostok Oceansat-2 (IRS-P4) 23-Sep-09 PSLV-C14
Stretched Rohini 13-Jul-88 ASLV GSAT-4 15-Apr-10 GSLV-D3
Satellite Series CARTOSAT-2B 12-Jul-10 PSLV-C15
(SROSS-2)
StudSat 12-Jul-10 PSLV-C15
INSAT-1C 21-Jul-88 Ariane-3
GSAT-5P / INSAT-4D 25-Dec-10 GSLV-F06
INSAT-1D 12-Jun-90 Delta 4925
RESOURCESAT-2 20-Apr-11 PSLV-C16
IRS-1B 29-Aug-91 Vostok
INSAT-2DT 26-Feb-92 Ariane-44L H10 Youthsat 20-Apr-11 PSLV-C16
Stretched Rohini 20-May-92 ASLV GSAT-8 / INSAT-4G 21-May-11 Ariane-5 VA-202
Satellite Series GSAT-12 15-Jul-11 PSLV-C17
(SROSS-C) Megha-Tropiques 12-Oct-11 PSLV-C18
INSAT-2A 10-Jul-92 Ariane-44L H10 Jugnu 12-Oct-11 PSLV-C18
INSAT-2B 23-Jul-93 Ariane-44L H10+ RISAT-1 26-Apr-12 PSLV-C19
IRS-1E 20-Sep-93 PSLV-D1 SRMSAT 26-Apr-12 PSLV-C18
Stretched Rohini 4-May-94 ASLV GSAT-10 29-Sep-12 Ariane-5 VA-209
Satellite Series SARAL 25-Feb-13 PSLV-C20
(SROSS-C2) IRNSS-1A 1-Jul-13 PSLV-C22
IRS-P2 15-Oct-94 PSLV-D2 INSAT-3D 26-Jul-13 Ariane-5
INSAT-2C 7-Dec-95 Ariane-44L H10-3
GSAT-7 30-Aug-13 Ariane-5
IRS-1C 29-Dec-95 Molniya
Mars Orbiter Mission 5-Nov-13 PSLV-C25
IRS-P3 21-Mar-96 PSLV-D3
(MOM)
INSAT-2D 4-Jun-97 Ariane-44L H10-3
GSAT-14 5-Jan-14 GSLV-D5
IRS-1D 29-Sep-97 PSLV-C1
IRNSS-1B 4-Apr-14 PSLV-C24
INSAT-2E 3-Apr-99 Ariane-42P H10-3
IRNSS-1C 10-Nov-14 PSLV-C26
Oceansat-1 (IRS-P4) 26-May-99 PSLV-C2
GSAT-16 7-Dec-14 Ariane-5
INSAT-3B 22-Mar-00 Ariane-5G
IRNSS-1D 28-Mar-15 PSLV-C27
GSAT-1 18-Apr-01 GSLV-D1
GSAT-6 27-Aug-15 GSLV-D6
Technology 22-Oct-01 PSLV-C3
Astrosat 28-Sep-15 PSLV-C30
Experiment Satellite
(TES) GSAT-15 11-Nov-15 Ariane 5 VA-227
INSAT-3C 24-Jan-02 Ariane-42L H10-3 IRNSS-1E 20-Jan-16 PSLV-C31
Kalpana-1 (METSAT) 12-Sep-02 PSLV-C4 IRNSS-1F 10-Mar-16 PSLV-C32
INSAT-3A 10-Apr-03 Ariane-5G IRNSS-1G 28-Apr-16 PSLV-C33
GSAT-2 8-May-03 GSLV-D2 Corbo Set-2C 22-Jun-16 PSLV-C34
INSAT-3E 28-Sep-03 Ariane-5G Insat 3DR 8-Sep-16 GSLV-F05
RESOURCESAT-1 17-Oct-03 PSLV-C5 SCATSAT-1 26-Sep-16 PSLV-C35
(IRS-P6) GSAT-18 5-Oct-16 Ariane 5 ECA VA-231
w
w
w
Current Affairs & GK Update 23

.y
GK-

ou
FIRST IN MALE

rs
m
ah
First Governor of Bengal Lord Clive (1757-60) The first Prime Minister of India who Charan Singh

bo
Last Governor of Bengal Warren Hastings did not face the Parliament

ob
(1772-74) The first Field Marshal of India S.H.F. Manekshaw

.w
The first Indian to get Nobel Prize in C.V.Raman

or
The first British Governor General of Lord Warren Hasting

dp
Bengal (1774-1885) Physics

re
The first British Governor General of Lord William The first Indian to receive Bharat Ratna Dr. Radhakrishnan

ss
award

.c
India Bentinck

om
(1833-1835) The first Indian to cross English Mihir Sen
The first British Viceroy of India Lord Canning (1856- Channel
62) The first Person to receive Jnanpith Sri Shankar Kurup
The first Governor General of free Lord Mountbatten award
India (1947-1948) The first Speaker of the Lok Sabha Ganesh Vasudeva
Mavalankar
The first and the last Indian to be C. Rajgopalachari
Governor General of free India (1948-1950) The first Vice-President of India Dr. Radhakrishnan
The first President of Indian Republic Dr. Rajendra Prasad The first Education Minister Abdul Kalam Azad
The first Prime Minister of free India Pt. Jawahar Lal The first Home Minister of India Sardar Vallabh Bhai
Patel
Nehru
The first Indian Air Chief Marshal S. Mukherjee
The first Indian to win Nobel Prize Rabindranath Tagore
The first Indian Naval Chief Vice Admiral R.D.
The first President of Indian National W.C. Banerjee
Katari
Congress
The first Judge of International Court Dr. Nagendra Singh
The first Muslim President of Indian Badruddin Tayyabji of Justice
National Congress
The first person to reach Mt. Everest Sherpa Anga Dorjee
The first Muslim President of India Dr. Zakir Hussain without oxygen
The first man who introduced printing James Hicky The first person to get Param Vir Major Somnath
press in India Chakra Sharma
The first Indian to join the I.C.S Satyendra Nath The first Chief Election Commissioner Sukumar Sen
Tagore The first person to receive Magsaysay Acharya Vinoba
India’s first man in Space Rakesh Sharma Award Bhave
The first Prime Minister of India who Morarji Desai The first person of Indian origin to Hargovind Khurana
resigned without completing the full receive Nobel Prize in Medicine
term The first Chinese traveller to visit India Fa-hein
The first Indian Commander-in-Chief General Cariappa The first person to receive Stalin Prize Saifuddin Kitchlu
of India The first person to resign from the Shyama Prasad
The first Chief of Army Staff Gen. Maharaj Central Cabinet Mukherjee
Rajendra Singhji The first person to receive Nobel Prize Amartya Sen
The first Indian Member of the S.P.Sinha in Economics
Viceroy’s Executive Council The first Chief Justice of Supreme Justice Hirala J.
The first President of India who died Dr. Zakir Hussain Court Kania
while in office The first Indian Pilot J.R.D. Tata (1929)
FIRST IN FEMALE
The first lady to become Miss World Rita Faria The first woman President of Indian Mrs Annie Besant
The first woman judge in Supreme Mrs. Meera Sahib National Congress
Court Fatima Bibi The first woman pilot in Indian Air Harita Kaur Dayal
The first woman Ambassador Miss C.B. Muthamma Force
The first woman Governor of a state in Mrs Sarojini Naidu The first woman Graduates Kadambini Ganguly
free India and Chandramukhi
The first woman Speaker of a State Shanno Devi Basu, 1883
Assembly The first woman Airline Pilot Durga Banerjee
The first woman Prime Minister Mrs Indira Gandhi The first woman Honours Graduate Kamini Roy, 1886
The first woman Minister in a Rajkumari Amrit The first woman Olympic medal Winner Karnam Malleswari,
Government Kaur 2000
The first woman to climb Mount Everest Bachhendri Pal The first woman Asian Games Gold Kamlijit Sandhu
The first woman to climb Mount Everest Santosh Yadav Medal Winner
twice The first woman Lawyer Cornelia Sorabjee
w
w
w
24 Current Affairs & GK Update

.y
GK-

ou
The first woman President of United Mrs Vijaya Laxmi The first woman Lieutenant General Puneeta Arora

rs
m
Nations General Assembly Pandit The first woman Air Vice Marshal P. Bandopadhyaya

ah
The first woman Chief Minister of an Mrs Sucheta The first woman chairperson of Indian Sushma Chawla

bo
Indian State Kripalani Airlines

ob
The first woman Chairman of Union Roze Millian Bethew The first woman IPS officer Mrs. Kiran Bedi

.w
or
Public Service Commission The first and last Muslim woman ruler Razia Sultan

dp
The first woman Director General of Kanchan Chaudhary of India

re
Police Bhattacharya The first woman to receive Ashoka Chakra Neerja Bhanot

ss
.c
The first woman Judge Anna Chandy (She The first woman to receive Jnanpith Ashapurna Devi

om
became judge in a Award
district court in 1937) The first woman to cross English Aarti Saha
The first woman Chief Justice of High Mrs Leela Seth Channel
Court (Himachal Pradesh The first woman to receive Nobel Prize Mother Teresa
High Court) The first woman to receive Bharat Ratna Mrs Indira Gandhi
The first woman Judge in Supreme Kumari Justice M. The first woman to receive Jnanpith Ashapurna Devi
Court of India Fathima Beevi Award

FIRST IN OTHERS
First Wax statue of a Living Indian Mahatma Gandhi at Madame Tussaud’s in 1939
First Exclusive internet magazine Bharat Samachar
First Miss India to participate in Miss Universe Indrani Rehman
First Judge in International Court of Justice Dr. Nagender Singh
First Graduate in Medicine Soorjo Coomar Goodeve Chukerbutty
India’s First University Nalanda University
India’s First Open University Andhra Pradesh Open University
India’s First Lok Sabha Member to be elected with a record P.V.Narasimha Rao
maximum number of votes
First Indian to reach Antarctica Lt. Ram Charan
First Test tube baby of India Indira (Baby Harsha)
First Post Office opened in India Kolkata(1727)

SUPERLATIVES
Structures •• Longest Railway Tunnel- Pir Panjal Railway Tunnel (11 km)
•• Longest Road Tunnel - 9.2 km long tunnel on Jammu-
•• Highest Tower ( Minaret ) – Qutub Minar
Srinagar National Highway
•• Higher Gateway – Buland Darwaza at Fatehpur Sikri near
•• Largest Public Sector Bank- State Bank of India
Agra. Built by Akbar (53.5 m /175 ft High) •• Largest Botanical Garden - National Botanical Garden in
•• Highest Dam – Bhakra Dam Kolkata
•• Highest Bridge – Chenab Bridge •• Largest Church- Se Cathedral at Old Goa, 10 km from Panaji.
•• Highest Airport- Leh Air Port in Ladakh (3256 m/ 16080 ft •• Largest Delta- Sunderbans (75,000 sq km) formed by the Ganga
high) and Brahmaputra in West Bengal and Bangladesh
•• Highest Hydel Power Station- Rongtong Hydel Project in •• Largest Stupa- Kesariya Stupa in Bihar
Kinnaur district of Himachal Pradesh. •• Largest Library- National Library, Kolkata
•• Highest Mountain Peak- Kanchenjunga •• Largest Planetarium- Birla Planetarium, Kolkata.
•• Highest Road- Road at Khardungla in the Leh-Manali Sector •• Largest Prison- Tihar Jail, Delhi
•• Highest Waterfall- Jog Waterfall, Karnataka •• Largest Concentration of Scheduled Tribes- Madhya Pradesh
•• Largest Residence – Antilia Bhawan built by Mukesh •• Largest Scheduled Caste- Community Santhal
Ambani •• Longest River Bridge – Bandra-Worli sea link which is 5.6
•• Largest Cinema Hall – Prasad Max, Hyderabad km.
•• Largest Museum – National Museum Delhi •• Largest Corridor – Rameshwaram Temple Corridor
•• Largest River Barrage – Farakka Barrage •• Largest irrigation Canal-Indira Gandhi Canal or Rajasthan
•• Biggest Auditorium (Mumbai) – Sri Shanmukhanand Hall Canal (959 km long)
•• Largest zoo – Arignar Anna Zoological Park •• Longest Dam-Hirakund Dam on Mahanadi river in Odisha
•• Largest Cave Temple – Ellora (24.4 km long)
•• Largest Gurudwara – Golden Temple, Amritsar •• Longest Glacier-Siachen Glacier on the Indo-Pakistan
•• Largest Mosque – Jama Masjid, Delhi (built by Shah Jahan border (75.6 km long and 2.8 km wide)
in 1644-58) •• Longest Railway Bridge Nehru Setu Bridge (4.62 km) long
•• Largest Man-made Lake – Govind Sagar (Bhakra) •• Fastest Train-Gatiman Express between New Delhi and
•• Largest Dome – Gol Gumbaz (Karnataka) Agra at a speed of 160 kmph
•• Largest Cantilever Bridge – Howrah Bridge •• Tallest Light House – Jakhau, light hour, Gujarat
w
w
w
Current Affairs & GK Update 25

.y
GK-

ou
•• Tallest Statues – Statue of Jain Saint Gomateswara at Leader of Nations-Famous Father

rs
Sravanabelagola in Karnataka

m
•• America—George Washington

ah
•• Tallest Chimber – Hanuman Swami statue with 135ft. tall.
•• Bangladesh—Mujibur Rehuman

bo
•• Oldest Church- St. Thomas Church at Palayar in Trichur
•• China—Sunyatsen

ob
district in Kerala built in 52 AD.
•• India—Gandhiji

.w
•• Oldest Monastery- Buddhist Monastery, (situated at an

or
•• Indonesia—Sukarno
altitude of 3,048 m /10,000 ft) at Tawang in Arunachal Pradesh.

dp
•• Mauritius—Ramgoolam

re
•• Largest mall- Lulu Mall Kochi
•• Namibia—Sam Nujoma

ss
•• Most Populous City- Mumbai

.c
•• Pakistan—Muhammad Ali Jinna

om
Natural •• Sri Lanka—D.S.Senanayeke
•• Longest River – Ganges •• Tanzania—Julius Nerera
•• Largest Desert – Thar ( Rajasthan) •• Turkey—Musthafa Kamal
•• Largest Lake – Wular Rajasthan
•• Largest Fresh Water Lake-Kolleru in Andhra Pradesh
Founders of Towns in India
•• Largest Cave- Amarnath (about 44 km from Pahalgam in •• Agra- Sikandar Lodhi
Jammu and Kashmir) •• Ahmedabad - Ahmed Shah
•• Ajmer- Ajaypal Chauhan
Founders of Indian Institutions •• Allahabad- Akbar
•• Arya Samaj-Swami Dayanand Saraswati •• Calcutta- Job Charnock
•• Athmiya Sabha-Raja Ram Mohan Roy •• Delhi- Anankapalan
•• Brahma Samaj-Raja Ram Mohan Roy •• Fatehpur Sikri - Akbar
•• Deccan Education Society-G.G.Agarkar, M.G.Ranade, •• Hisar- Ferozshah Tuglaq
V.G.Gibhongar •• Hyderabad - Quli Qutabshah
•• Dharma Sabha-Radhakanthadev •• Jodhpur- Rao Jodha
•• Indian Brahma Samaj-Keshav Chandra Sen •• Mahabalipuram - Narasimhawarman
•• Manavadharma Sabha-Durgaram Manjaram •• Siri- Alauddin Khilji
•• Prarthana Samaj-Athmaram Pandurang •• Vijayanagaram - Hariharan 1
•• Pune Sewa Sadan-Smt.Remabhai Ranade, G.K.Devdhar
•• Ramakrishna Mission-Swami Vivekananda Agricultural Revolution
•• Sadharan Brahma Samaj-Shivananda Sashtri, Anand Mohan •• Blue Revolution - Fisheries Development
Bose •• Brown Revolution - Leather Production
•• Servants of India Society-Gopalakrishna Gokhale •• Grey Revolution - Housing Development
•• Sewa Sadan-Bahuramji M.Malabari •• Green Revolution - Agriculture Production
•• Sewa Samithi-H.N.Kunsru •• Pink Revolution - Drugs & Pharmaceuticals
•• Social Service League-N.M.Joshi •• Silver Revolution - Egg Production
•• Thathwabodhini Sabha-Debendranatha Tagore •• White Revolution - Dairy Development
•• Theosophical Society-Madam H.P.Blavadski, Col.H.L.Olkott •• Yellow Revolution - Oil Seed Production

SOBRIQUETS
A sobriquet is a nickname, occasionally assumed and often Manitas de Plate Flamenco guitarist Ricardo
given by another. Baliardo
Person Primary Names Netaji Subhash Chandra Bose
Anna C N Annadurai Nightingale of India Sarojini Naidu
Badshah Khan/ Frontier Gandhi Abdul Ghaffar Khan Panditji Jawahar Lal Nehru
Buddha Siddhartha Gautama Punjab Kesari Lala Lajpat Rai
Chacha Jawahar Lal Nehru Rajaji C Rajagopalachari
Deenabandhu C F Andrews Saint of the Gutters Mother Teresa
Deshbandhu C. R. Das Father of the Nation Mohandas Karamchand
Father of the Nation Mohandas Karamchand Gandhi
Gandhi Haryana Hurricane Kapil Dev
Grand Old Man of India Dadabhai Naoroji
Prince of Kolkata Saurav Ganguly
Gurudev Rabindranath Tagore
Places Primary Names
Guruji M S Gohlwalkar
Bengal’s Sorrow Damodar River
Kaviguru Rabindranath Tagore
Blue Mountain Niligiri Hills
Lokmanya Bal Gangadhar Tilak
Loknayak Jayaprakash Narayan City of Golden Temple Amritsar
Mahatma Gandhi Mohandas K. Gandhi City of Palaces Kolkata
Man of Peace Lal Bahadur Shastri Diamond City in India Surat, Gujarat
w
w
w
26 Current Affairs & GK Update

.y
GK-

ou
Garden City of India Bengaluru States & its Slogans

rs
m
Garden of India Kashmir •• Rice bowl of India, Egg bowl of Asia – Andhra Pradesh.

ah
Gateway of India Mumbai •• The Orchid state of India or the paradise of the Botanists-

bo
Arunachal Pradesh.

ob
God’s Own Country Kerala
•• Rice bowl of Central India – Chhattisgarh.

.w
Land of Five Rivers Punjab •• Jewel of the Western part of India– Gujarat.

or
dp
Pink City Jaipur •• State of Apples, Dev Bhoomi (Adobe of Gods) – Himachal

re
Queen of Arabian Sea Kochi Pradesh

ss
•• Heaven on Earth – Jammu and Kashmir.

.c
Spice Garden of India Kerala

om
The City of Joy Kolkata •• The Land of Jungles & Jharis (Bushes) – Jharkhand
•• God’s own country – Kerala
The City of Palaces Kolkata
•• Heart of India – Madhya Pradesh
Venice of East Alleppey •• Gateway of the East – Manipur
Queen of Arabian Sea Kochi •• The adobe of the clouds – Meghalaya
Garden City of India Bangalore •• Land of the hill people – Mizoram
Blue Mountains Niligiri Hills •• Falcon capital of the world-Nagaland
Silicon Valley of India Bengaluru •• Granary of India, India’s bread-basket – Punjab
•• Land of the Gods – Uttarakhand
Planned City of India Chandigarh
•• Hindi Heartland of India – Uttar Pradesh

HISTORY
LITERATURE OF MUGHAL PERIOD
Book Author Contents Nuriyya-i- Abdul Haq Theory of Kinship during
Sultaniyya Mughal Period
Tuzuk-i-Baburi Babur Describes military tactics and
administrative organisation Waqt-i- Nimat Khan Aurangzeb's Golconda conquest
during Babur’s reign Hyderabad Ali
Qanun-i- Khwand Describes Humayun’s Futuhat-i- Ishwar Das Aurangzeb's history
Humayu Amair administration, festivities and Alamgiri
buildings of that period Nuskha-i- Bhimsen Analysis of Aurangzeb's rule
Humayun Nama Gulbadan Biography of Humayun Dilkusha Saxena and character
Begum Khulasat-ul- Sujan Raj History of Aurangzeb's rule
Tawarikh Khatri
Akbar Nama Abul Fazl Gives a history of Akbar’s
reign Padshah Namah Abdul Hamid History of Shah Jahan's reign
Lahori
Tobaqat-i- Khwajah -do-
Akbari Nizamuddin Padshah Namah Muhammad -do-
Ahmad Baksh Waris
Shahjahan Muhammad -do-
Tuzuk-i- Jahangir Memoirs of his own reign
Jahangiri Namah Salih
Shahjahan Inyat Khan -do-
Iqbalnama-i- Muhammad History of Jahangir's reign Namah
Jahangiri Khan
Hamlai-Haidri Muhammad History of Aurangzeb's rule
Chahar Chaman Chandra History of Shah Jahan's rule Rafi Khan
Bhan
Brahman Namah-e- Aquil Khan -do-
Alamgiri Zafar
Alamgir-nama Munshi Mirza Gives an account of
Sirr-i-Akbar Dara Shikoh Urdu translation of
Muhamma Aurangzeb's first 10 years of
Upanishad
Kazin rule
Safinat-ul- -do- Biographies of Sufi Saints
Massir-i- Saqi Mustaid Official history of Auliya
Alamgiri Khan Aurangzeb’s reign written
after his death Majma-ul- -do- Philosophical ideas discussed
Bahrain
Ain-i-Akbari Abul Fazl History of Akbar's reign
Raqqat-e- Aurangzeb A compendium of his letters
Muntakhab-ul- Badauni History of Akbar's rule Alamgiri
Tawarikh
Hasmat-ul- Dara Shikoh Religious ideas discussed
Tawarikh-i-Alfi Mulla Daud -do- Arifin
w
w
w
Current Affairs & GK Update 27

.y
GK-

ou
THE FREEDOM STRUGGLE TIME LINE

rs
m
1885 Pherozeshah Mehta, K.T. Telang, Badruddin Tyabji formed Bombay Presidency Association.

ah
28 Dec. 1885 Indian National Congress was formed by Allan Octavian Hume.

bo
ob
28-31 Dec. 1885 First session of Indian National Congress was attended by 72 delegates under the presidency of W.C. Bonnerjee.

.w
1896-97 Bal Gangadhar Tilak initiated a no-tax campaign in Maharashtra.

or
dp
20 July, 1905 Partition of Bengal order was passed by Lord Curzon.

re
Dec. 1905 Gokhale then the president of Congress condemned the partition of Bengal and supported Swadeshi and

ss
Boycott movement.

.c
om
1906 Dadabhai Naoroji became the president of National Congress and clearly declared their goal to be self-
government or Swaraj like the other colonies.
30 Dec. 1906 All India Muslim League was formed by Aga Khan III and the founding meeting was hosted by Nawab Sir
Khwaja Salimullah.
1909 The Indian Councils Act or Morley-Minto Reform was announced.
1911 Government announced the withdrawal of Partition of Bengal.
1913 Ghadar Party founded by Punjabi Indians in the United States and Canada aiming at securing India’s
independence.
April, 1915 First session of Hindu Mahasabha was held under the presidentship of Maharaja of Kasim Bazar.
26 Dec. 1916 Lucknow Pact was signed dealing with the structure of the government of India and with relation to the Hindu
and Muslim communities.
1917 Indigo Satyagraha started by M.K. Gandhi in Champaran, Bihar.
1918 Edwin Montagu, then the Secretary of State and Lord Chelmsford, the Viceroy produced a scheme of
constitutional reform which was called as the Montague-Chelmsford reforms.
1919 Enactment of the Government of India Act.
March, 1919 Rowlatt Act was passed which enabled government to imprison people without trial.
13 April, 1919 Unarmed crowd gathered at Jallianwala Bagh to protest against the arrest of Dr. Saifuddin Kitchlew and Dr.
Satyapal was attacked by the British army as commanded by General Dyer.
31 August, 1920 Khilafat Committee launched a non-cooperation Movement.
1 February, 1922 M.K. Gandhi announced mass Civil Disobedience movement.
5 Feb. 1922 Protesters participating in the Non-cooperation Movement turned violent, leading to police opening fire in
Chauri Chaura. Congress as a result halted the non-cooperation Movement.
1925 Communist Party came into existence.
Nov. 1927 Simon headed commission was set up to submit report on working of Indian constitution established by
Government of India Act, 1919.
17 Nov. 1928 Lala Lajpat Rai died due to the injuries by the beating of local police during a protest demonstration at Lahore.
Dec. 1928 Gandhi joined back the active politics at Calcutta session.
26 Jan. 1930 Was fixed as the First Independence Day and since then was celebrated every year up to 1947.
Feb. 1930 Chandra Shekhar Azad was shot dead in a park called Azad Park at Allahabad, in an encounter with British police.
12 March, 1930 Dandi March lead by M.K. Gandhi took place. Together with 78 companions he walked 375 km from
Sabarmati Ashram to Dandi.
6 April, 1930 Gandhi reached Dandi and broke the Salt law.
12 Nov. 1930 First round table conference was held in London, was chaired by British Prime Minister Ramsay MacDonald.
5 March, 1931 Gandhi-Irwin Pact was signed between Gandhi and then viceroy of India Lord Irwin. According to which
British agreed to withdraw all ordinances and end prosecutions and release all political prisoners.
24 August, 1932 Poona Pact was signed between Gandhi and Dr. B.R. Ambedkar at Yerwada Central Jail.
1935 Government of India Act was passed according to which All India Federation was established including
British India and Princely States (representative were appointed by the rulers) forming a bicameral federal
legislature.
October, 1940 Gandhi gave an order for limited satyagraha (for few individuals only).
8 August, 1942 Quit India Movement was launched by M.K.Gandhi.
1945 Congress working committee adopted a resolution to abolish landlordism.
2 Sept. 1946 Interim government of India formed the newly elected Constituent Assembly of India. This idea was rejected
by Muslim league.
9 Dec. 1946 The Constituent Assembly met for the first time.
GK-
28

Can be Suspended FRs available only to citizens


Magna Carta Part III (Article Borrowed from during Emergency & Not to Foreigners- Art. 15,
of India 12 to 35) USA & France except Art. 20 & 21 16, 19, 29, 30

Fundamental Rights

Right to Equality Right to Freedom Right Against Exploitation Right to Freedom of Cultural and Right to Constitutional
Religion Educational Rights Remedies

Articles Articles Articles Articles Articles Articles


14-18 19-22 23-24 25-28 29-30 32
(14) Equality before Law (19) 6 Rights: Freedom of (23) Prohibition of traffic in (25) Freedom of conscience & (29) Protection of Includes writs:
& Equal protection speech & expression, human beings & forced free profession, practice & interests of (1) Habeas Corpus
of law. Assembly, Association, labour. propagation of religion. minorities.
POLITY

(2) Mandamus
Movement, Residence & (3) Prohibition
Fundamental Rights

Profession. (4) Certiorari


(5) Quo–Warranto
(15) Prohibition of (20) Protection in respect of (24) Prohibition of (26 ) Manage religious affairs. (30) Right of minorities
discrimination on conviction for offences. employment of to establish
grounds of religion, children in factories & administer
race, sex etc. etc. educational
institutions.
(16) Equality of (21) Protection of life & (27) Freedom from payment of
opportunity in personal liberty. taxes for promotion of any
public employment religion.

w
(17) Abolition of (22) Protection against (28) Freedom from attending
w
w
untouchability. arrest & detention in religious instruction

.y
certain cases. or worship in certain
educational institutions.
ou
(18) Abolition of titles.
rs
m
ah
bo
ob
.w
Current Affairs & GK Update

or
dp
re
ss
.c
om
w
w
w
Current Affairs & GK Update 29

.y
GK-

ou
Indian Legislature

rs
m
ah
bo
Union Leg

ob
State Leg
(Parliament)

.w
or
dp
re
President Rajya Sabha Lok Sabha Bicameral Unicameral

ss
.c
om
Legislative Assembly
Upper House Lower House

Also known as House of People


council of states Maximum Leg. Council or Leg. Assembly or
Total members-250 Strength - 552 Vidhan Parishad Vidhan Sabha

238 12 530 20 2 (Nominated Lower House


Indirectly Nominated (States) (UTs) members from Upper House
Anglo-Indian Maximum
elected Maximum
Community) Strength - 500
Permanent Body Strength - 1/3 rd
Members are of total strength Minimum
directly elected of Assembly. Strength - 60
Term of individual
member - 6 years Term - 5 years Minimum Members are
Strength - 40 directly elected
1/3 rd members Presiding Officers by people
Members are
retires at expiration indirectly Term - 5 years
of every 2nd year. elected
Speaker Deputy
Speaker Permanent body
Presiding officers
Qualification 1/3rd of its members
retire every second
Chairman Deputy year.
Chairman Citizen Not less Term - 6 years
of India than 25
Qualification years.

Citizen of Not less than


India 30 years

GEOGRAPHY
MAJOR STRAIT OF THE WORLD Taurus Strait Arafura Sea & Gulf of Papua New Guinea -
Name Joins Location Papua Australia
Bass strait Tasman Sea & South Australia
Malacca Strait Andaman Sea & South Indonesia - Malaysia
Sea
China Sea
Bering Strait Bering Sea & Chukchi Alaska-Russia
Palk Strait Palk Bay & Bay of India-Sri Lanka
Sea
Bengal
Bonne-Fasio Mediterranean Sea Corsika-Sardinia
Sunda Strait Java Sea & Indian Indonesia
Strait
Ocean
Bosphorus Black Sea and Marma- Turkey
Yucatan Strait Gulf of Mexico and Mexico-Cuba
Strait ra Sea
Caribbean Sea
Dardanelle Marmara Sea and Ae- Turkey
Messina Strait Mediterranean Sea Italy-Sicily
Strait gean Sea
Otranto Strait Adriatic Sea & Ionian Italy-Albania Davis strait Baffin Bay & Atlantic Greenland-Canada
Sea Ocean
Bab-el-Man- Red Sea & Gulf of Yemen-Djibouti Denmark North Atlantic and Greenland-Iceland
deb Strait Aden strait Arctic Ocean
Cook Strait South Pacific Ocean New Zealand (N & S Dover strait English Channel & England-France
islands) North Sea
Mozambique Indian Ocean Mozambique - Mala- Florida Strait Gulf of Mexico and USA-Cuba
Strait gasy Atlantic Ocean
North Chan- Irish Sea & Atlantic Ireland-England Hormuz strait Gulf of Persia & Gulf Oman-Iran
nel Ocean of Oman
w
w
w
30 Current Affairs & GK Update

.y
GK-

ou
Hudson strait Gulf of Hudson & At- Canada Gulf of Mannar, between India and Sri Lanka

rs
m
lantic Ocean
Gulf of Mexico, between Mexico, the United States, and Cuba

ah
Gibraltar Mediterranean Sea & Spain-Morocco

bo
Strait Atlantic Ocean Gulf of Morbihan, a natural harbour on the coast of the

ob
Département of Morbihan in the south of Brittany

.w
Magellan Pacific and South At- Chile

or
strait lantic Ocean Gulf of Nicoya, in Costa Rica, Central America.

dp
Makassar Java Sea & Celebeze Indonesia

re
Gulf of Oman, between the south eastern Arabian Peninsula,

ss
Strait Sea Iran, Pakistan and Arabian Sea.

.c
om
Tsugaru Strait Japan Sea and Pacific Japan (Hokkaido-Hon- Gulf of Oristano, near Oristano on the Western Sardinian
Ocean shu island) coast
Tatar Strait Japan Sea & Okhotsk Russia (E Russia-
Gulf of Panama in the Pacific Ocean south of Panama
Sea Sakhalin Island)
Persian Gulf between Iran and the Arabian Peninsula
MAJOR GULFS AND THEIR LOCATION Gulf of Roses, the most northeastern bay on the Catalan coast
Gulf of Aden of the southwestern corner of the Arabian Gulf of Saint Lawrence, the world’s largest estuary and the
Peninsula outlet of the Saint Lawrence River into the Atlantic Ocean
Gulf of Alaska in the Pacific Ocean south of the state of Alaska Gulf St Vincent, separated from Spencer Gulf by the Yorke
Amundsen Gulf in the Arctic Ocean northwest of Canada Peninsula
Gulf of Aqaba in the northern end of the Red Sea, leading to Gulf of Sidra, just north of Libya in the Mediterranean
Israel and Jordan Spencer Gulf, near Port Lincoln, South Australia
Gulf of Bahrain, part of the Persian Gulf Gulf of Suez, in the northern end of the Red Sea, leading to
Gulf of Bothnia, part of the Baltic Sea between Sweden and the Suez Canal
Finland Gulf of Thailand, just south of Thailand in the Indian Ocean
Gulf of Cádiz, part of the Atlantic Ocean off the southern Gulf of Tonkin, just east of North Vietnam in the Pacific
border of Spain and Portugal
Ocean
Gulf of California in the Pacific Ocean in northwestern
Gulf of Tunis, in the Mediterranean off the coast of Tunisia
Mexico
Gulf of Carpentaria a large bay off northern Australia Tribal Groups of India
Gulf of Cazones a large gulf in southern Cuba Tribal Region Tribal Region
Group Group
Gulf of Corinth, which extends into Greece from the
Mediterranean Abhor Arunachal Pradesh Kharia Jharkhand,
Odisha
Davao Gulf in the Philippines
Adivasi A.P, Bihar, Odisha, Kol Madhya Pradesh
Gulf of the Farallones, westward from the opening of the San Jharkhand, Madhya
Francisco Bay and Drakes Bay to the Farallon Islands Pradesh, Maharashtra,
Gulf of Finland, between the southern coast of Finland and Rajasthan, Tamil Nadu,
the northern coast of Estonia in the Baltic Sea. Some Northeastern
States, West Bengal, An-
Gulf of Genoa inside the Ligurian Sea on the northwestern
daman and Nicobar
coast of Italy
Ahgani Manipur Kolam Maharashtra,
Gulf of Guinea in the Atlantic Ocean off the coast of Equatorial Andhra Pradesh,
Africa Telengana, M.P
Gulf of Izmir in the Aegean Sea between Turkey and Greece. Apatani Arunachal Pradesh Kota Karnataka
It was formerly called the Gulf of Smyrna.
Baiga Madhya Pradesh Kuki Mizoram
Gulf of Khambhat in the Arabian Sea, formerly known as the Bakar- Jammu and Kashmir Lahaula Himachal
Gulf of Cambay wal Pradesh
Gulf of Kutch in the Arabian Sea Bhil M.P and Rajasthan Lepcha Sikkim
Lingayen Gulf of western Luzon, the Philippines, in the South Birhor M.P and Bihar Lushai Mizoram,
China Sea Manipur
Gulf of Lion, a bay on the Mediterranean coastline of Chang Nagaland Muria Chhattisgarh
Languedoc-Roussillon and Provence in France Chench- Telengana, Karnataka Miha Rajasthan
Gulf of Maine, off the State of Maine, New Brunswick, and uas
Nova Scotia in the Atlantic Ocean Sutiya Assam Moplah Malabar
w
w
w
Current Affairs & GK Update 31

.y
GK-

ou
Gaddis Himachal Pradesh Munda West Bengal, Ghana Bird Rajasthan Water Bird, Black-buck, Cheetal,

rs
m
Jharkhand, Sanctuary Sambar

ah
Odisha,
Panchmarhi Madhya Tiger, Panther, Sambhar, Nilgai,

bo
Chhattishgarh

ob
Pradesh Baskeng, Deer
Gallong Arunachal Pradesh Nishi Assam

.w
Dandeli Karnataka Tiger, Panther, Elephant, Cheetal,

or
Garo Meghalaya Naga Nagaland

dp
Sanctuary Sanbhar, Wild Boar

re
Gond M.P and Bihar Oraon MP, Bihar and
Kutch Near Great Indian Bustard, Lesser Florican,

ss
Odisha, Chhota

.c
Bustard Naliya, Houbara bustard, Chinkara, Blue
Nagpur, W.B,

om
Sanctuary Kachchh bull, Herpetofauna
Gujjar Rajasthan Onges Andaman &
Nicobar BIOSPHERE RESERVES IN INDIA AREA-WISE
Irula Tamil Nadu Singpho Assam, Arunachal Name State Key Fauna
Pradesh
Nilgiri Tamil Nadu, Nilgiri tahr, lion-tailed macaque
Jaintia Meghalaya Santhal WB, Odisha,
Bihar, Jharkhand, Biosphere Kerala and
Assam Reserve Karnataka
Jarawa Andaman, Islands Sangtam Nagaland Nanda Devi Uttarakhand Himalayan musk deer, mainland
National Park serow, Himalayan tahr
Kanikar Tamil Nadu and Kerala Sema Nagaland & Biosphere
Kalkari Maharashtra Sentine- Andaman & Reserve
lese Nicobar Gulf of Tamil Nadu Dugong or sea cow
Kharia Maharashtra Shompen Andaman & Mannar
Nicobar
Nokrek Meghalaya Red panda
Khond Jharkhand Toda Tamil Nadu
Sundarbans West Bengal Royal Bengal tiger
Khasi Meghalaya Uralis Kerala
Manas Assam Golden langur, red panda
Wancho Arunachal
Pradesh Simlipal Odisha Gaur, Royal Bengal tiger, elephant
Warli Maharashtra, Dihang- Arunachal Mishmi takin, red goral, musk
Daman and Diu, Dibang Pradesh deer
Bihar, Madhaya Pachmarhi Madhya Giant squirrel, flying squirrel
Pradesh, West Biosphere Pradesh
Bengal Dadra and Reserve
Nagar Haveli Achanakmar- Madhya Four horned antelope (Tetracerus
WILDLIFE SANCTUARIES IN INDIA Amarkantak Pradesh, quadricornis), Indian wild dog (Cuon
Biosphere Chhattisgarh alpinus), Saras crane (Grus antigone),
Name of the Location Major Species Reserve Asian white-backed vulture (Gyps
Sanctuary bengalensis), Sacred grove bush frog
Gir Wildlife Sasan Gir, Lion, Leopard, Chausinga, Chital, (Philautus sanctisilvaticus)
Sanctuary Junagadh, Hyena, Sambar, Chinkara, Great Rann Gujarat Indian wild ass
Amreli Herpetofauna, Crocodiles and birds of Kutch
Wild Ass Little Rann Wild Ass, Chinkara, Blue Cold Desert Himachal Snow leopard
Sanctuary of Kachchh bull, Houbara bustard, Wolf, Pradesh
Waterfowls, Herpetofauna Khangchendzonga Sikkim Snow leopard, red panda
Hingolgadh Hingolgadh, Chinkara, Blue bull, Wolf, AgasthyamalI Kerala, Nilgiri tahr, elephants
Sanctuary Rajkot Hyena, Fox, Birds, Herpetofauna Biosphere Tamil Nadu
Marine Gulf of Sponges, Corals, Jellyfish, Reserve
Sanctuary Kachchh, Sea horse, Octopus,Oyster, Great Andaman Saltwater crocodile
Jamnagar Pearloyster, Starfish, Lobster, Nicobar and Nicobar
Dolphin, Dugong, waterfowls Biosphere Islands
Simlipal Odisha Elephant, Tiger, Leopard, Gaur, Reserve
Sactuary Cheetal Dibru- Assam Golden langur
Kutch Great Rann Chinkara, Hyena, Fox, Flamingo, Saikhowa
Desert of Kachchh Pelicans & other waterfowls, Seshachalam Andhra Yellow-throated bulbul
Sanctuary Herpetofauna Hills Pradesh
Rampara Rampara, Blue bull, Chinkara, Wolf, Fox, Panna Madhya Tiger, chital, chinkara, sambhar
Sanctuary Rajkot Jackal, Birds, Herpetofauna Pradesh and sloth bear
w
w
w
32 Current Affairs & GK Update

.y
GK-

ou
BIOLOGY

rs
m
ah
bo
DIGESTION OF FOOD

ob
.w
Name of the Name of the enzymes Substrate End product

or
dp
Digestive juice

re
Saliva Ptyalin (Salivary amylase) Starch Maltose

ss
Pancreatic Juice Amylopsin (pancreatic amylase) Starch, Glycogen Maltose and Glucose

.c
om
Intestinal juice Sucrase (invertase), Maltase, Lactase Sucrose; Maltose, Lactose Glucose, fructose and galactose
Gastric Juice Pepsin, Rennin Proteins, Casein Proteoses and peptones, Calcium caseinate
Pancreatic Juice Trypsin, Chymotrypsin, Carboxyl Proteins, Proteins Peptides Proteoses and peptides Peptides Amino
peptidases acid.
Intestinal juice Amino peptidase, Dipeptidase Peptides Amino acids
Vitamin Required by the Body
Vitamin Chemical Name Function in Body Deficiency Disease Sources
B1 Thiamine Part of coenzyme for respirationBeri-beri: nerve and heart disorders Cereals, legumes, beans,
pyrophosphate nuts.
B2 Riboflavin Part of coenzyme FAD needed for Ariboflavinosis: skin and eye Milk, yogurt, meat, leafy
respiration disorders greens, whole grains.
B12 Cyanoco-balamin Coenzyme needed for making red Pernicious anaemia Meat, fish, poultry,
blood cells, bone, blood and nerve shellfish, eggs, cheese,
changes milk.
B5 Nicotinic Part of coenzymes NAD, NADP Pellagra: skin, gut and nerve Widespread in foods.
acid (‘niacin’) used in respiration disorders
C Ascorbic acid Not precisely known Scurvy: degeneration of skin teeth Citrus fruits, e.g. lemon.
and blood vessels.
A Retinol Not fully known but forms part of Xeropthalmia: ‘dry eyes’ Milk, eggs, meat, fish liver
visual pigment, rhodopsin oils, Green leafy vegetables.
D Cholecalciferol Stimulates calcium absorption by Rickets: bone deformity Exposed to sunlight, Dairy
small intestine, needed for proper products, egg yolk, fish
bone growth liver oils, oysters, yeast.
E Tocopherol Not precisely known Infertility Plant oils, green, leafy
vegetables, egg yolk, nuts,
seeds, and liver.
K Phylloquinone Involved in blood clotting Possible haemorrhage Bacterial synthesis in the
digestive tract. Green
leafy vegetables, cabbage
and milk

DISEASE AND DEFENCE MECHANISM


Pratozoan Diseases Bacteria Diseases
Disease Pathogen Disease Pathogen
Malaria Plasmodium
Amoebiasis Enta moeba histolylica Dysentery Shigella
Giardiasis Giardia lambia Plague Pasteurellapestis
Sleeping sickness Trypanosoma
Leshmanis Leishmania Diphtheria Corynebacterium diphtheriae
Trichomoniasis Trichomonas vaginalis Cholera Vibrio cholerae
Fungal diseases Tuberculosis Mycobacterium tuberculosis
Disease Pathogen Tetanus Clostridium tetani
Aspergillosis Aspergillus fumigatuo Whooping cough Bordetella pertussis
Candidiasis Candida albicens
Leprosy Mycobacterium leprae
Ringworm Trichoplyton
Blastomycosis Blasto myces dermatitidis Anthrax Bacillus anthrasis
Sporotnichosis Sporothrix Schenckii Weil’s disease Leptospira
w
w
w
Current Affairs & GK Update 33

.y
GK-

ou
Viral Diseases Important Vaccines

rs
m
Vaccine Developed by Country Year

ah
Disease Pathogen

bo
Rabies Rabies virus Small Pox Edward Jenner England 1796

ob
Cholera Louis Pasteur France 1880

.w
Dengue Dengue virus

or
Influenza Influenza virus Diphtheria and Emil Adolf Von Behring Germany/ 1891

dp
Tetanus and Shibasaburo Kitasato Japan

re
Measles Rubeola virus

ss
TB Vaccine Albert Calmette and France 1922
German measles Rubella virus

.c
Camille Guerin

om
Mumps Mumps virus
Polio Vaccine Jonas E. Salk US 1952
Chicken pox Varicella zoster
Oral Polio Albert Bruce Sabin US 1955
Small pox Variola virus Vaccine
Polio Polio virus Measles Vaccine John F. Enders, Thomas US 1953
Chikungunya Chikungunya (CHIK) Virus peeble
Avian flu H5N1 virus Rabies Vaccine Louis Pasteur France 1885
H1N1(Swine flu) H1N1 virus Typhus Vaccine Charles Nicolle France 1909

SPORTS
TROPHIES ASSOCIATED WITH SPORTS
NATIONAL Merdeka Cup Football (Asia)
Name of the Trophy Related game Thomas Cup World Badminton (Men)
Aga Khan Cup Hockey Uber Cup World Badminton (women)
Barna Bellack Cup Table Tennis US-Open Lawn Tennis
French-Open Lawn Tennis
Beighton Cup Hockey
Australian Open Lawn Tennis
Bombay Gold Cup Hockey
Wimbledon Lawn Tennis
Burdwan Trophy Weight Lifting Masters Champions Trophy Hockey
D.C.M. Trophy Football British Open Golf
Dhyan chand Trophy Hockey Malaysian Open Badminton
Dr. B.C. Roy Trophy Football Tata Open Lawn Tennis
Duleep Trophy Cricket
Durand Cup Football
Number of Players on Each Side
Ezra Cup Polo Badminton 1 or 2 Polo 4
I.F.A Shield Football Baseball 9 Rugby Football 15
Lady Ratan Tata Trophy Hockey Basketball 5 Tennis and Table tennis 1 or 2
Moin ud daula Gold Cup Cricket Cricket 11 Water Polo 7
Rangaswami Cup Hockey Football 11 Volleyball 6
Hockey 11 Kabaddi 7
Ranji Trophy Cricket
Chess 1
Santosh Trophy Football
Scindia Gold Cup Hockey National Sports of Famous Countries
Subroto Mukherjee Cup Football (Inter-School) Name National game
Wellington Trophy Rowing Australia Cricket
INTERNATIONAL Brazil Football
Canada Ice Hockey
Name of the Trophy Related game China Table Tennis
Nehru Trophy Hockey England Cricket
American Cup Yatch Racing India No Game
Ashes Cup Cricket (Australia-England) Japan Judo or Ju Jitsu
Azlan Shah Hockey Malaysia Badminton
US Masters Golf Pakistan Hockey
Hopman Cup Lawn Tennis Russia Chess, Football
Colombo Cup Trophy Football Scotland Rugby, Football
Davis Cup Lawn Tennis Spain Bull Fighting
Kings Cup Race Air Races (England) United States of America Baseball
w
w
w
34 Current Affairs & GK Update

.y
GK-

ou
TERMS USED IN SPORTS AND GAMES Hockey Bull, Carry, Centre Forward, Carried, Dribble,

rs
Goal, Hat trick, Penalty corner, Scoop, Short corner,

m
ah
Badminton Deuce, Double, Drop, Fault, Game, Let, Love, Sticks, Striking circle, Under cutting.

bo
Smash. Horse Racing Jockey, Place, Protest, Punter, Win.

ob
Baseball Bunt, Diamond, Home, Pitcher, Put out, Strike. Lawn Tennis Back-hand-drive, Service, Smash, Volley, Deuce,

.w
Billiards Break, Cannons, Cue, In off, Jigger, Scratch, Game, Set, Love.

or
dp
Boat Race Cox Polo Bunder, Chuckker, Mallet.

re
Boxing Hook, Jab, Knock-out, Punch, Upper cut. Rifle Shooting Bull’s eye.

ss
Rugby Drop kick, Screen.

.c
Bridge Diamonds, Dummy, Grand slam, Little slam,

om
Revoke, Ruff, Tricks, Trump. Swimming Stroke.
Volley ball Booster, Deuce, Love, Service, Spikers.
Chess Check, Checkmate, Gambit, Stalemate
Wrestling Half Nelson, Heave.
Cricket Bowling, Bouncer, Crease, Cover point, Drive,
Duck, Follow on, Googly, Gulley, Hat Trick, Hit SPECIFIC NAMES OF PLAYING AREAS
wicket, L.B.W. (Leg Before Wicket), Leg Break, Sport Name of Sport Name of Playing
Leg spinner, Leg bye, Maiden over, No ball, Pitch, Playing Area Area
Run, Silly point, Stumped, Wicket keeper. Badminton Court Golf Link, Green**
Football Dribble, Drop Kick, Foul, Hattrick, Off-side, Penalty, Baseball Diamond Lawn Tennis Court
Throw in, Touch Down. Boxing Ring Ice Skating Rink
Golf Bogey, Caddie, Hole, Links, Put, Putting the green, Cricket Pitch* Wrestling Ring, Arena
Stymie, Tee. * Pitch in fact is the space between the wickets and not the entire cricket field.
** It is the area around the hole only.

S-ar putea să vă placă și